Download Test 26 – Obstetrics A 29 year-old G1P0 at 41 weeks gestation

Document related concepts

Maternal health wikipedia , lookup

Medical ethics wikipedia , lookup

Women's medicine in antiquity wikipedia , lookup

Patient safety wikipedia , lookup

Menstruation wikipedia , lookup

Electronic prescribing wikipedia , lookup

Adherence (medicine) wikipedia , lookup

Prenatal nutrition wikipedia , lookup

Dysprosody wikipedia , lookup

Childbirth wikipedia , lookup

Prenatal development wikipedia , lookup

Maternal physiological changes in pregnancy wikipedia , lookup

Fetal origins hypothesis wikipedia , lookup

Prenatal testing wikipedia , lookup

Transcript
Test 26 – Obstetrics
A 29 year-old G1P0 at 41 weeks gestation presents for a prenatal visit. Her prenatal course is complicated by
tobacco abuse and intermittent prenatal care. Her last visit was at 35 weeks. Prenatal labs are unremarkable except
cervical culture positive for Chlamydia, which was treated, and a Pap smear with low-grade squamous intraepithelial
lesion. Ultrasound at 21 weeks was consistent with gestational age. Her vitals reveal a blood pressure of 128/76;
pulse 74; and temperature 98° F, 36.7°C . Fundal height is 39 cm with estimated fetal weight of 2700 gm. Cervix is
dilated to 1 cm, 50% effaced, -2 station. What is the next best step in the management of this patient?
a) Non-stress test
b) Vibroacoustic stimulation test
c) Oxytocin challenge test
d) Return visit in 1 week
e) Cesarean section
Correct!!! The non-stress test is an assessment of fetal well-being that measures the fetal heart rate response
to fetal movement. The normal or reactive non-stress test occurs when there are two fetal heart rate
accelerations of 15 beats/minute for 15 seconds within 20 minutes. Vibroacoustic stimulation is not
indicated unless the NST is non-reactive. Contraction stress test assesses uteroplacental insufficiency and
looks for persistent late decelerations after contractions (3/10 minutes); however, it is not necessary to
perform, as the non-stress test will assess fetal well being, also. Observation only would not be proper care
as the patient is post-term. In the presence of abnormal testing, labor would be induced or a Cesarean
section performed.
A 29 year-old G1P0 at 41 weeks gestation presents in early labor. The prenatal course was uncomplicated.
Ultrasound at 21 weeks was consistent with gestational age. Her vitals reveal a blood pressure of 128/76; pulse 74;
and she is afebrile. Fundal height is 36 cm with estimated fetal weight of 2700 gm. Cervix is dilated to 1 cm, 50%
effaced and the fetal vertex is at -2 station. The nurse calls you to evaluate the fetal tracing. Which statement best
describes the tracing seen below?
a) Normal fetal heart rate with good variability and regular contractions
b) Fetal tachycardia with good variability and regular contractions
c) Normal fetal heart rate with poor variability and regular contractions
d) Fetal tachycardia with poor variability and irregular contractions
e) Normal fetal heart rate with poor variability and irregular contractions
Correct!!! The baseline fetal heart rate is normal with good accelerations and regular contractions. There is
no tachycardia. This is a reassuring tracing.
A 19 year-old G3P0 with spontaneous rupture of membranes for 13 hours presented to labor delivery. She had no
prenatal care. Her vital signs are: blood pressure 120/70; pulse 72; afebrile; fundal height 36 cm; and estimated fetal
weight of 2700 gm. Cervix is dilated to 1 cm, 50% effaced, -2 station. Which statement best describes the tracing
seen below?
a) Normal fetal heart rate with good variability and regular contractions
b) Fetal tachycardia with good variability and regular contractions
c) Normal fetal heart rate with poor variability and irregular contractions
d) Fetal tachycardia with poor variability and regular contractions
e) Normal fetal heart rate with good variability and irregular contractions
Correct!!! The baseline fetal heart rate is >160 with no accelerations or variability. There are regular
contractions. Prolonged periods of fetal tachycardia are frequently found with maternal fever or
chorioamnionitis.
-2-2
A 19 year-old G3P0020 with spontaneous rupture of membranes for 19 hours presents to labor delivery. She had no
prenatal care. Her vital signs are: blood pressure 120/70; pulse 72; febrile to 102° F , 38.9°C); fundal height 36 cm;
and estimated fetal weight of 2700 gm. Cervix is dilated to 1 cm, 50% effaced and -2 station. The fetal heart rate
tracing demonstrates regular contractions and fetal tachycardia of 180 beats/minute with poor variability. In this
patient, what is the most likely etiology of this fetal heart rate pattern?
a) Nuchal cord
b) Prematurity
c) Fetal anomalies
d) Chorioamnionitis
e) Oligohydramnios
Correct!!! Regular contractions with fetal tachycardia in the presence of prolonged ruptured membranes are
most likely due to infection (i.e. chorioamnionitis). In the presence of a nuchal cord and oligohydramnios,
the fetal heart rate tracing may show variable decelerations. There is no classic fetal heart rate pattern in
pregnancies complicated by prematurity or fetal anomalies.
-2-2
A 19 year-old G1P0 at 41 weeks with spontaneous rupture of membranes for 13 hours presented to labor and
delivery. She had an uncomplicated prenatal course. Her vital signs are: blood pressure 120/70; pulse 72; afebrile;
fundal height 36 cm; and estimated fetal weight of 2700 gm. Cervix is dilated to 4 cm, 100% effaced, + 1 station.
What does the fetal heart rate tracing seen below show?
a) Late deceleration
b) Variable decelerations
c) Early decelerations
d) Sinusoidal rhythm
e) Normal fetal heart rate pattern
Correct!!! Early decelerations are physiologic caused by fetal head compression during uterine
contractions, resulting in vagal stimulation and slowing of the heart rate. This type of deceleration has a
uniform shape, with a slow onset that coincides with the start of the contraction and a slow return to the
baseline that coincides with the end of the contraction. Thus, it has the characteristic mirror image of the
contraction. A late deceleration is a symmetric fall in the fetal heart rate, beginning at or after the peak of
the uterine contraction and returning to baseline only after the contraction has ended. Late decelerations are
associated with uteroplacental insufficiency. Variable decelerations show an acute fall in the FHR with a
rapid down slope and a variable recovery phase. They are characteristically variable in duration, intensity,
and timing and may not bear a constant relationship to uterine contractions. The true sinusoidal pattern is a
regular, smooth, undulating form typical of a sine wave that occurs with a frequency of two to five
cycles/minute and an amplitude range of five to 15 beats/minute. It is also characterized by a stable
baseline heart rate of 120 to 160 beats/minute and absent beat-to-beat variability.
A 19 year-old G1P0 at 42 weeks presents to labor and delivery with spontaneous rupture of membranes for 13 hours
and spontaneous onset of labor. Her vital signs are: blood pressure 120/70; pulse 72; afebrile; fundal height 36 cm;
and estimated fetal weight of 2700 gm. Cervix is dilated to 4 cm, 100% effaced, + 1 station. Which statement best
describes the tracing seen below?
a) Normal fetal heart rate pattern
b) Sinusoidal rhythm
c) Late deceleration
d) Variable decelerations
e) Early decelerations
Correct!!! Variable decelerations show an acute fall in the FHR, with a rapid down slope and a variable
recovery phase. They are characteristically variable in duration, intensity, and timing, and may not bear a
constant relationship to uterine contractions. Early decelerations are physiologic caused by fetal head
compression during uterine contraction, resulting in vagal stimulation and slowing of the heart rate. This
type of deceleration has a uniform shape, with a slow onset that coincides with the start of the contraction
and a slow return to the baseline that coincides with the end of the contraction. Thus, it has the
characteristic mirror image of the contraction. A late deceleration is a symmetric fall in the fetal heart rate,
beginning at or after the peak of the uterine contraction and returning to baseline only after the contraction
has ended. Late decelerations are associated with uteroplacental insufficiency. The true sinusoidal pattern is
a regular, smooth, undulating form typical of a sine wave that occurs with a frequency of two to five
cycles/minute and an amplitude range of five to 15 beats/minute. It is also characterized by a stable
baseline heart rate of 120 to 160 beats/minute and absent beat-to-beat variability.
A 19 year-old G1P0 at 39 weeks presents in labor. She denies ruptured membranes. Her prenatal course was
uncomplicated and ultrasound at 18 weeks revealed no fetal abnormalities. Her vital signs are: blood pressure
120/70; pulse 72; temperature 101.0° F, 38.3° C; fundal height 36 cm; and estimated fetal weight of 2900 gm.
Cervix is dilated to 4 cm, 100% effaced and at +1 station. At spontaneous rupture of the membranes, light
meconium-stained fluid was noted. Soon after the fetal heart rate tracing revealed variable decelerations with good
beat-to-beat variability. What is the most likely cause for the variable decelerations?
a) Umbilical cord compression
b) Meconium
c) Maternal fever
d) Uteroplacental insufficiency
e) Maternal drugs
Correct!!! Variable decelerations are reflex mediated usually associated with umbilical cord compression as
a result of cord wrapped around fetal parts, fetal anomalies or oligohydramnios. The presence of light
meconium-stained fluid is not associated with a specific fetal heart rate tracing. Uteroplacental
insufficiency is associated with late decelerations. Maternal drugs may cause loss of variability.
A 29 year-old G1P0 at 42 weeks presents in labor. She denies ruptured membranes. Her prenatal course was
complicated by chronic hypertension. Her vital signs are: blood pressure 130/80; pulse 72; afebrile; fundal height 36
cm; and estimated fetal weight of 2100 gm. Cervix is dilated to 4 cm, 100% effaced, +1 station. The fetal heart rate
tracing is shown below. What is the most likely diagnosis?
a) Normal fetal heart rate pattern
b) Sinusoidal rhythm
c) Late deceleration
d) Variable decelerations
e) Early decelerations
Correct!!! Late decelerations are a symmetric fall in the fetal heart rate, beginning at or after the peak of the
uterine contraction and returning to baseline only after the contraction has ended. Late decelerations are
associated with uteroplacental insufficiency. Variable decelerations show an acute fall in the FHR with a
rapid down slope and a variable recovery phase. They are characteristically variable in duration, intensity,
and timing and may not bear a constant relationship to uterine contractions. Early decelerations are
physiologic caused by fetal head compression during uterine contraction, resulting in vagal stimulation and
slowing of the heart rate. This type of deceleration has a uniform shape, with a slow onset that coincides
with the start of the contraction and a slow return to the baseline that coincides with the end of the
contraction. Thus, it has the characteristic mirror image of the contraction. The true sinusoidal pattern is a
regular, smooth, undulating form typical of a sine wave that occurs with a frequency of two to five
cycles/minute and an amplitude range of five to 15 beats/minute. It is also characterized by a stable
baseline heart rate of 120 to 160 beats/minute and absent beat-to-beat variability.
A 29 year-old G1P0 at 42 weeks presents in labor. She denies ruptured membranes. Her prenatal course was
complicated by chronic hypertension. Her vital signs are: blood pressure 130/80; pulse 72; afebrile; fundal height 36
cm; and estimated fetal weight of 2400 gm. Cervix is dilated to 4 cm, 100% effaced, -1 station, and bulging bag of
water. The fetal heart rate tracing reveals 5 contractions in 10 minutes and repetitive late decelerations. What is the
most likely cause of her late decelerations?
a) Uteroplacental insufficiency
b) Umbilical cord compression
c) Uterine hyperstimulation
d) Occiput posterior position
e) Fetal head compression
Correct!!! Late decelerations when viewed as repetitive and/or with decreased variability are an ominous
sign. The can be associated with uteroplacental insufficiency as a result of decreased uterine perfusion or
placental function, thus leading to fetal hypoxia and acidemia. Common causes include chronic
hypertension and postdate pregnancies. Variable decelerations are associated with cord compression.
Uterine hyperstimulation may cause prolonged bradycardia. Occiput posterior position should not be the
cause of late decelerations. Fetal head compression may be associated with early decelerations.
A 29 year-old G1P0 at 42 weeks presents to labor and delivery because of intermittent contractions. She denies
ruptured membranes. Her prenatal course was uncomplicated. Her vital signs are: blood pressure 140/96; pulse 72;
afebrile; fundal height 32 cm; and estimated fetal weight of 2900 gm. Cervix is closed, 25% effaced, -2 station. The
fetal heart rate tracing shows occasional late decelerations. Of the following, what is the next best step in
management?
a) Maternal left lateral position
b) Intrauterine resuscitation with terbutaline
c) Start an amnioinfusion
d) Begin magnesium sulfate
e) Augment labor with oxytocin
Correct!!! Initial measures to evaluate and treat fetal hypoperfusion include a change in maternal position
to left lateral position which increases perfusion to the uterus, maternal supplemental oxygenation,
treatment of maternal hypotension, discontinue oxytocin, consider intrauterine resuscitation with tocolytics
and intravenous fluids, fetal acid-base assessment with fetal scalp capillary blood gas or pH measurement.
An amnioinfusion may be used to treat patients with variable decelerations. Measures to improve
uteroplacental blood flow should be attempted prior to proceeding with Cesarean delivery. Magnesium
sulfate is not yet indicated in this patient with one slightly elevated blood pressure. Augmentation of labor
may accentuate the late decelerations
A 17 year-old woman comes to your office for her first gynecologic visit. She has been sexually active for the last
year and always uses condoms. What is the most appropriate management regarding Pap smear screening for this
patient?
a) Pap smear at age of 21
b) Pap smear at this visit and then anually
c) Pap smear now and every 3 years
d) Pap smear now and then every other year
e) Pap smear at age 18
Correct!!! The 2009 ACOG recommendation is for patients to have an annual Pap smear starting at 21
years of age regardless of history of sexual activity. Cervical neoplasia develops in susceptible individuals
in response to a sexually transmitted infection with a high-risk type of HPV (Human papillomavirus). HPV
causes carcinogenesis in the transformation zone of the cervix, where the process of squamous metaplasia
replaces columnar with squamous epithelium. Squamous metaplasia is active in the cervix during
adolescence and early adulthood. Human papillomavirus infections are commonly acquired by young
women shortly after the initiation of vaginal intercourse but, in most, they are cleared by the immune
system within 1–2 years without producing neoplastic changes. The recommendation to start screening at
age 21 years regardless of the age of onset of sexual intercourse is based in part on the very low incidence
of cancer in younger women. It is also based on the potential for adverse effects associated with follow-up
of young women with abnormal cytology screening results.
An 18 year-old woman comes to your office for her first gynecologic visit. Her 43 year-old stepmother was just
diagnosed with breast cancer. She wants to discuss breast cancer screening and wants to have a mammogram.
According to the American College of Obstetricians and Gynecologists (ACOG), which of the following is the most
appropriate recommendation for this patient?
a) Yearly staring at age 35
b) Every 1-2 years from 35-49,then yearly starting at age 50
c) Every 1-2 years from 40-49,then yearly starting at age 50
d) Yearly breast ultrasound at age 35, then yearly mammograms at age 50
e) Yearly breast ultrasound now, then yearly mammograms at age 40
Correct!!! Although the recommendation to initiate breast cancer screening timing with mammograms
varies among different professional societies (American Cancer Society: yearly at age 40; ACOG: every 12 years at age 40, then yearly at age 50; United States Preventive Services Task Force [USPSTF]: every 2
years at age 50). None recommend routine screening prior to age 40 in a patient with no family history of
breast cancer.
A 40 year-old patient presents for her first annual visit. She denies any new complaints or symptoms. She has no
history of any gynecologic problems. Family history is significant for a father with hypertension and a mother,
deceased, with breast cancer diagnosed at age 36. A maternal aunt has ovarian cancer. A physical exam is
unremarkable. What screening test should be offered to this patient next?
a) Breast MRI
b) Mammogram
c) Transvaginal pelvic ultrasound
d) Breast ultrasound
e) BRCA-1/BRCA-2 testing
Correct!!! Presently, there are limitations in the ability to screen for cancer. Although the patient is young,
her family history of a first-degree relative with premenopausal breast cancer means that she is considered
high-risk. Given the patient’s family history of both breast and ovarian cancer, genetic counseling and a
thorough pedigree analysis should be suggested for evaluation for BRCA testing and possible enrollment in
a specialized ovarian cancer screening trial. At present, though, transvaginal sonography and CA-125
testing are not recommended as first-line screening tools for the general population for ovarian cancer. The
patient should, however, have a mammogram at this earlier than routine age, due to the family history.
Ultrasound is a helpful adjunct for a young patient. Ultrasound and MRI are generally used as adjunctive
studies when a patient has an abnormal or unsatisfactory mammogram.
A 19 year-old patient has just started a new sexual relationship and comes in to discuss contraception. She is most
concerned about preventing a sexually transmitted infection. The contraceptive method most likely to prevent
sexually transmitted infections is:
a) The male condom
b) The female condom
c) Cervical cap
d) Cervical cap and Spermicide
e) Diaphragm
Correct!!! The male condom protects against sexually transmitted infections. Spermicide and a diaphragm
with spermicide protect against bacterial sexually transmitted infections. The female condom may protect
against sexually transmitted infections. Hormonal methods such as Depo-Provera, oral contraceptives, the
contraceptive patch and the contraceptive ring do not protect against sexually transmitted infections.
A 16 year-old patient has a new boyfriend and comes in to discuss contraception. She is well aware of the
importance of preventing sexually transmitted infections and specifically wants to know about prevention of
pregnancy. Other than abstinence, the most effective method of birth control in this patient is:
a) The male condom
b) The diaphragm with spermicide
c) Oral contraceptives
d) Depo-Provera
e) The contraceptive ring
Correct!!! Contraceptive methods with <1% pregnancy rates (typical use) are Depo-Provera, IUD,
sterilization (male or female), and Implanon. Oral contraceptives have a 3% pregnancy rate, with typical
use, and the male condom has a 12% pregnancy rate. 8% of women will experience an unintended
pregnancy after 1 year of typical use with a contraceptive ring. Of the methods listed, the diaphragm with
spermicide has the highest pregnancy rate, with typical use, being 18%.
A 65 year-old postmenopausal woman has an appointment for her annual visit. She wants to discuss the use of bone
density screening for osteoporosis. Initial bone mineral density testing should be recommended to all the following
EXCEPT:
a) All postmenopausal women aged 65 years or older
b) A postmenopausal woman under 65 with hyperparathyroidism
c) A postmenopausal woman under 65 with hypertension
d) All postmenopausal women who present with fractures
e) Women under the age of 50 who have been rendered surgically menopausal and who have other risk factors for
developing osteoporosis
Correct!!! Risk factors for osteoporosis are early menopause, glucocorticoid therapy, sedentary lifestyle,
alcohol consumption, hyperthyroidism, hyperparathyroidism, anticonvulsant therapy, vitamin D deficiency,
family history of early or severe osteoporosis, or chronic liver or renal disease. These would institute early
screening in a patient for osteoporosis. Hypothyroidism is not a risk factor for osteoporosis, but you should
consider early screening if the patient had difficulty attaining proper treatment since over-repletion puts her
at increased risk. A postmenopausal patient presenting with fractures should alert you to suspect
osteoporosis.
A 36 year-old health worker presents for her annual visit. She is sexually active and not using contraception. She
doesn’t think she is pregnant, but would be happy if she were. As part of her general preventive care, you discuss
immunizations. Which vaccination is contraindicated if this patient gets pregnant now?
a) Measles-Mumps-Rubella (MMR)
b) Pneumococcus
c) Hepatitis B
d) Polio
e) Influenza
Correct!!! Pregnancy or the possibility of pregnancy within 4 weeks is a contraindication to the MMR and
varicella vaccinations. Tetanus, Hepatitis B, Polio and Pneumococcal vaccinations would not be
contraindicated.
A 42 year-old patient presents for an annual visit. Her past medical history, physical exam and labs are normal. Her
body mass index (BMI) is normal. Her family history is significant for hypertension and hypercholesterolemia in her
father and diabetes mellitus in her mother. What lifestyle modification is most important for this patient?
a) Starting a weight loss diet
b) Starting a sugar-free and cholesterol-free diet
c) Recording a daily blood pressure
d) Starting an aerobic exercise program
e) Recording a weekly blood pressure
Correct!!! Heart disease is the number one killer of women. Lifestyle modifications to reduce her risk,
especially considering her family history, are important proactive changes that she can make. Studies show
an inverse relationship between the level of physical activity and incidence of death from coronary disease.
Exercise would be an appropriate first step with this patient. She does not need to lose weight (normal
BMI) and does not need to be on a special diet (normal labs). Decreasing her work hours is not indicated.
A 28 year-old patient presents to the office for her first visit. She is currently on oral contraceptive pills, but reports
a history of irregular menses prior to starting them. Her past medical history is otherwise non-contributory. On
physical examination, she is 5’2” tall and weighs 180 pounds. She has an area of velvety, hyperpigmented skin on
the back of her neck and under her arms. What is the next best step in the management of this patient?
a) Complete blood count
b) Diabetes screen
c) Biopsy pigmented area
d) Pelvic ultrasound
e) Lipid profile
Correct!!! The patient is obese, with a BMI = 33. In addition, the skin changes are consistent with
acanthosis nigricans, which is closely associated with insulin resistance. Given these risk factors, she
should be tested for diabetes.
A 24 year-old patient is considering pregnancy in the next year. Her medical history and physical examination are
normal. She is unaware of any significant family history. She exercises regularly and does not smoke or drink
alcohol. She asks if her diet has adequate folate and what you recommend to your patients. Which of the following
statements about folate is correct?
a) Folate supplementation is recommended for this patient
b) Most patients get an adequate intake by diet alone
c) She is not currently planning a pregnancy and does not need to worry about folate
d) Current grain supplementation adequately prevents neural tube defects
e) Folate does not need to be started until after a pregnancy is documented
Correct!!! Folate lowers homocysteine levels. The Nurses Health Study showed fewer nonfatal MIs and
fatal coronary events in women with adequate intake doses of folate and vitamin B6. Folate can also help
prevent neural tube defects. Studies have shown that diet alone is not effective in achieving adequate levels
and routine folate supplementation is therefore recommended. Women of reproductive age should take a
daily 400-microgram supplement. Adequate levels are especially important prior to pregnancy and during
the first 4 weeks of fetal development.
Obstetrics – Exam 8
An 18-year-old G1P0 woman is seen in the clinic for a routine prenatal visit at 28 weeks gestation. She is Caucasian,
not of Mediterranean, Asian or Pacific descent. Her prenatal course has been unremarkable. She has not been taking
prenatal vitamins. Her pre-pregnancy weight was 120 lb. Initial hemoglobin at the first visit at 8 weeks gestation
was 12.3 g/dL. Current weight is 138 lb. No bruising or petechiae are present. After performing a screening
complete blood count (CBC), the results are notable for a white blood count 9,700/mL, hemoglobin 10.6 g/dL, mean
corpuscular volume 88.2 (80.8 – 96.4,) and platelets 215,000/mL. The patient denies vaginal or rectal bleeding. The
best explanation for the patient’s anemia is:
a) Folate deficiency
b) Relative hemodilution of pregnancy
c) Iron deficiency
d) Underlying hemoglobinopathy
e) Chronic medical condition
Correct!!! There is normally a 36% increase in maternal blood volume; the maximum is reached around 34
weeks. The plasma volume increases 47% and the RBC mass increases only 17%. This relative dilutional
effect lowers the hemoglobin, but causes no change in the MCV. Folate deficiency results in a macrocytic
anemia. Iron deficiency and hemoglobinopathies cause a microcytic anemia. This is a normocytic anemia,
so A and C are not correct. In addition, the fact that she is Caucasian, and not of Mediterranean, Asian, or
Pacific descent, makes a hemoglobinopathy unlikely. Not much information is provided about her diet, but
dietary deficiencies usually cause either a microcytic or a macrocytic anemia. Anemia due to chronic
disease is characterized by a normal MCV and a normal or low reticulocyte count as there is impaired RBC
production. While this patient could have anemia due to a chronic medical condition, the most likely
explanation remains B.
A 34-year-old G3P1 woman reports “difficulty catching her breath,” especially after exertion for the last two
months. She is 26 weeks gestation. She is a non-smoker. She does not have any history of pulmonary or cardiac
disease. She denies fever, sputum, cough and any recent illnesses. On physical exam, her vital signs are: blood
pressure 108/64, pulse 88, respiratory rate 15, and she is afebrile. Lungs are clear to auscultation. Heart: regular rate
and rhythm with II/VI systolic murmur heard at the upper left sternal border. Normal extremities. A complete blood
count reveals a hemoglobin of 9.9g/dL. What is the most likely explanation for this woman’s symptoms?
a) Pulmonary embolism
b) Mitral valve stenosis secondary to rheumatic heart disease
c) Physiologic dyspnea of pregnancy
d) Reactive airway disease
e) Anemia
Correct!!! Physical exam findings are not consistent with pulmonary embolus (tachycardia, tachypnea,
chest pain, signs of a DVT, etc.,) mitral stenosis (diastolic murmur, signs of heart failure,) or reactive
airway disease (tachypnea, wheezing.) Physiologic dyspnea of pregnancy is present in up to 75% of women
by the third trimester. The history and physical do not suggest a pathologic process, nor does her
hemoglobin level.
A 24-year-old G4P2 woman at 34 weeks gestation complains of a cough and whitish sputum for the last three days.
She reports that “everyone in the family has been sick.” Her temperature last night was 101.5°F. She denies chest
pain. She smokes a half-pack of cigarettes per day. She has a history of asthma with no previous intubations. She
uses an albuterol inhaler, although she has not used it this week. Pertinent physical exam findings: temperature
99.3°F, respiratory rate 14, pulse 93, blood pressure 113/77, peak expiratory flow rate 430 L/min (baseline
documented in the outpatient chart = 425 L/min) Pharyngeal mucosa is erythematous and injected. Lungs are clear
to auscultation. Pertinent laboratory studies: white blood cell count 8,700; arterial blood gases on room air (normal
ranges in parentheses): pH 7.44 (7.36 – 7.44); PO2 103 mm Hg (>100), PCO2 26 mm Hg (28 – 32), HCO3 19 mm
Hg (22 – 26.) Chest x-ray is normal. What is the correct interpretation of this arterial blood gas?
a) The woman is hypoventilating
b) This is a compensated respiratory alkalosis
c) This is a compensated metabolic alkalosis.
d) This is an acute metabolic acidosis
e) She should use her albuterol inhaler
Correct!!! B is the correct answer because the increased minute ventilation during pregnancy causes a
compensated respiratory alkalosis. The PO2 is normal and the PCO2 is decreased, so answers A and C are
not correct (hypoventilation results in increased PCO2 and the PO2 would be decreased if she was
hypoxic.) A metabolic acidosis would have a decreased pH and a low HCO3, so answer D is incorrect. The
patient does not have symptoms or signs of an asthmatic exacerbation. The findings are most consistent
with a viral upper respiratory syndrome. With a normal peak flow, the albuterol is not indicated, so answer
E is incorrect.
A 28-year-old G1 woman internal medicine resident at 18 weeks wants to discuss the values on her pulmonary
function tests performed two days ago. She is asymptomatic. She is currently on a pulmonary rotation. As an
exercise during the rotation, residents undergo spirometry in the pulmonary lab. She is concerned about the results ,
and is wondering if they are “abnormal.” She is a non-smoker, and has no personal or family history of cardiac or
respiratory disease. Pertinent physical exam findings: respiratory rate 12, lungs clear, gravid abdomen with fundal
height 2 cm below the umbilicus; all other findings normal. The results of the spirometry are:
Inspiratory Capacity (IC) increased
Tidal volume (TV) increased
Functional reserve capacity (FRC) decreased
Expiratory reserve capacity (ERC) decreased
Residual volume (RV) decreased
Total lung capacity (TLC) decreased
Which of the following pulmonary changes will NOT be seen in this patient?
a) The diaphragmatic excursion is increased
b) Minute ventilation is increased due to the increased respiratory rate
c) The level of the diaphragm rises by about 4 cm
d) The subcostal angle is increased from about 68 to 103 degrees
e) The mechanical changes are not due to pressure from the enlarging uterus
Correct!!! The statements in A, C, D, and E are all correct. The minute ventilation is the product of the
respiratory rate and the tidal volume (TV.) The respiratory rate does not change during pregnancy, but the
TV is increased which increases the minute ventilation, which is responsible for the respiratory alkalosis.
A 24-year-old G1P0 woman at 30 weeks gestation reports difficulty breathing, cough and frothy sputum. She was
admitted for preterm labor 24 hours ago. She has had worsening dyspnea for the past 8 hours, with new onset cough
and frothy sputum. She is a non-smoker. She has received 6 liters of lactated Ringers solution since admission.
Urine output has not been calculated. She is receiving terbutaline. Pertinent physical exam findings: Temperature
101.4°F, respiratory rate 24, heart rate 110, blood pressure 132/85, pulse oximetry on 100% non-rebreather mask
97%. She appears in distress. Lungs reveal bibasilar crackles. Uterine contractions are regular every 3 minutes.
Uterus is tender to palpation between contractions. The fetal heart rate is 140 beats per minute. Labs: White blood
cell count 17,500/mL with 94% segmented neutrophils. Potassium and sodium are normal. The patient is correctly
diagnosed with pulmonary edema. What is the most likely cause of pulmonary edema in this patient?
a) Increased plasma osmolality
b) Tocolysis with terbutaline
c) Chorioamnionitis
d) Preterm labor
e) Increased systemic vascular resistance
Incorrect!!! Plasma osmolality is decreased during pregnancy; this increases the susceptibility to pulmonary
edema. Tocolysis with terbutaline or other alpha agonists increase the susceptibility to pulmonary edema,
especially with the use of isotonic fluids. Systemic vascular resistance is decreased during pregnancy.
Women with chorioamnionitis are also more likely to develop pulmonary edema, but this is not usually the
main cause unless patient is in septic shock.
A 25-year-old G1P0 woman is seen for an initial obstetrical appointment at 8 weeks gestation. She has had a small
muscular ventricular septal defect (VSD) since birth. She has no surgical history and no limitations on her activity.
Pertinent examination findings: respiratory rate 12, heart rate 88, blood pressure 112/68. Skin acyanotic. Lungs clear
to auscultation. Heart regular rate and rhythm. There is a IV/VI course pansystolic murmur at the left sternal border,
accompanied by a thrill. Chest x-ray and ECG are normal. Which of the following is the correct statement regarding
cardiovascular adaptation in this patient?
a) Approximately 2% of women will normally have a diastolic murmur
b) Maternal pulmonary vascular resistance is normally greater than systemic vascular resistance.
c) The maternal cardiac output will increase by 33% by the twentieth week.
d) Maternal systemic vascular resistance increases throughout pregnancy.
e) The increase in cardiac output is only due to the increase in the maternal stroke volume
Incorrect!!! The cardiac output increases due to increases in both the heart rate and the in stroke volume, so
answer E is incorrect. The SVR falls during pregnancy, so answer D is incorrect. Up to 95% of women will
have a systolic murmur due to the increased volume. Diastolic murmurs are always abnormal, so A is
incorrect. Answer B is incorrect: the systemic vascular resistance (SVR) is normally greater than the
pulmonary vascular resistance. If the pulmonary vascular resistance exceeds the SVR, right to left shunt
will develop in the setting of a VSD, and cyanosis will develop.
17-year-old G1P0 woman at 32 weeks gestation complains of right flank pain that is “colicky” in nature and has
been present for two weeks. She denies fever, dysuria and hematuria. Physical examination is notable for moderate
right costovertebral angle tenderness. Pertinent labs: White blood cell count 8,800/mL, urine analysis negative for
infection and blood. A renal ultrasound reveals no signs of urinary calculi, but there is moderate (15 mm) right
hydronephrosis. Which of the following statements is true regarding urinary tract changes during pregnancy?
a) The urinary collecting system is dilated in only 20% of women by mid-gestation
b) Caliceal and ureteral dilatation is more common on the left side
c) Compression by the uterus and right ovarian vein, and smooth muscle relaxation due to elevated
progesterone have all been proposed as etiologies for urinary system dilatation.
d) Urinary tract dilatation decreases the risk for urinary tract infection during pregnancy.
e) The glomerular filtration rate normally decreases producing a concomitant decrease in the serum creatinine.
c) Compression by the uterus and right ovarian vein, and smooth muscle relaxation due to elevated
progesterone have all been proposed as etiologies for urinary system dilatation. would be awarded 10
points
A 34-year-old G4P2 woman at 18 weeks gestation reports nausea, vomiting and weight loss. These symptoms have
persisted since 4 weeks gestation. Family history is positive for a mother and sister with Grave’s disease. Pertinent
physical examination findings: pre-pregnancy weight 140 lb, current weight 130 lb. Afebrile, heart rate 80; nontender, no thyroid enlargement. Pertinent labs: thyroid stimulating hormone (TSH) 0.01 mU/L (normal 0.5 – 5.0).
Which of the following would be found in this patient?
a) Increased total thyroxine
b) Decreased total thyroxine
c) Normal thyroxine levels
d) Decreased thyronine binding globulin (TBG)
e) Decreased free thyroxine
Incorrect!!! TBG is increased due to increased circulating estrogens with a concomitant increase in the total
thyroxine. The free thyroxine remains relatively constant, so A is the correct answer
An 18-year-old G1P0 woman at an unknown gestational age reports nausea, vomiting, vaginal bleeding and a
“racing heart.” These symptoms have been present on and off for the past 8 weeks. Patient has not had prenatal care.
The patient has no significant past medical, surgical or family history. Physical examination findings: afebrile, heart
rate 120, blood pressure 127/77, uterine fundus 4 cm below the umbilicus, no fetal heart tones obtained by fetal
Doppler device, cervix is 1 cm dilated with pinkish/purple “fleshy” tissue protruding through the os. Pertinent labs:
hemoglobin 8.2 gm/dL, Quantitative beta human chorionic gonadotropin (Beta HCG) 1.5 Million IU/mL, thyroidstimulating hormone (TSH) undetectable, free T4 3.2 (normal 0.7 – 2.5.) An ultrasound is performed, which reveals
heterogeneous cystic tissue in the uterus (snowstorm pattern.) Which of the following is the most appropriate next
step in the management of this patient?
a) Repeat quantitative HCG
b) Repeat transvaginal ultrasound
c) PET scan
d) Chest x-ray
e) CBC
Incorrect!!! A chest x-ray would be the most appropriate step, as the lungs are the most common site of
metastatic disease in patients with gestational trophoblastic disease. Though a repeat quantitative HCG will
be required on a weekly basis, an immediate post-operative value will be of little clinical utility. A PET
scan is not indicated and the patient already had a CBC done.
A 42-year-old G5P4 woman at 14 weeks gestation presents for her first prenatal appointment. She has glycosuria
noted on urine dipstick in the office. Pertinent history: Four prior vaginal deliveries at full-term with birth weights
ranging from 9 to 10.5 lb. Family history is positive for Type II diabetes in mother and two siblings. Weight 265 lb.
Height 64 inches. A one-hour glucose challenge test (50 grams) is 234 mg/dL. Glycosylated hemoglobin is 6.4
percent. The most correct statement regarding this patient’s diabetes is?
a) Because this woman was diagnosed at 14 weeks, she is most likely a Type I diabetic.
b) Chorionic somatomammotropin induces a state of insulin resistance by inhibiting gluconeogenesis and
glucose uptake in this patient
c) Glycosuria is always an abnormal finding
d) Insulin readily crosses the placenta and can produce fetal hypoglycemia
e) This patient should have a 3 hour GTT to confirm a diagnosis of gestational diabetes
Correct!!! Chorionic somatomammotropin (previously called human placental lactogen) induces a state of
insulin resistance, so answer B is correct. This woman likely is a Type II diabetic. She has genetic
predisposition, has had four macrosomic infants (birth weight > 4,000 grams,) is 42-years-old and is obese.
These are all risk factors for Type II diabetes and for gestational diabetes. Answer C is incorrect.
Glycosuria during pregnancy is due to the increased glomerular filtration rate and increased renal filtering
of glucose, but impaired tubular reabsorption. Glycosuria is common during pregnancy and does not
necessarily reflect hyperglycemia. Answers D and E are incorrect. Insulin is a peptide hormone and does
not cross the placenta. Glucose crosses the placenta via carrier-mediated diffusion. This patient is likely a
Type II diabetic as she has a significantly elevated glucose at 14 weeks. A 3-hour GTT is not necessary.
Obstetrics – Exam 9
A 26-year-old African-American G1P0 woman presents to your office at 7 weeks gestation with her husband, who is
also African-American. The patient’s brother has sickle cell anemia, and has been hospitalized on numerous
occasions with painful crises requiring narcotic pain medication and blood transfusions. What are the odds that this
couple will have a child with sickle cell anemia, if the carrier rate for sickle cell disease in the African American
population is 1/10?
a) 1 in 15
b) 1 in 60
c) 1 in 160
d) 1 in 400
e) 1 in 100
Incorrect!!! Sickle cell anemia is an autosomal recessive condition that occurs in 1/500 births in the
African-American population. The carrier state, or sickle-cell trait, is found in approximately 1/10 AfricanAmericans. Since the patient’s brother is affected, both of their parents have to be carriers. Each time two
carrier parents for an autosomal recessive condition mate there is a 1/4 chance of having either an affected
or an unaffected child and a 1/2 chance of having a child who is a carrier. Since the patient is unaffected,
she has a 1/3 chance of not being a carrier and a 2/3 chance of being a carrier. The patient’s husband has a
1/10 chance of being a carrier (the general population risk for African-Americans). Thus, the chance that
this couple will have a child with sickle cell anemia is: 2/3 X 1/10 X 1/4 = 1/60.
An African-American couple comes to you for preconception counseling. Neither one of them has any significant
family or genetic history. Based on their African-American descent, you recommend the following blood tests to
screen for hemoglobin abnormalities:
a) MCV and CBC
b) Sickle cell preparation and CBC
c) Hemoglobin F level and CBC
d) Hemoglobin electrophoresis and CBC
e) Sickle cell preparation with a hemoglobin electrophoresis, if the sickle preparation is abnormal
Incorrect!!! Screening for carriers of both alpha and beta thalassemia is possible by evaluation of red cell
indices. Although solubility tests for hemoglobin S or sickle cell preparations can be used for screening,
hemoglobin electrophoresis is definitive and preferable because other hemoglobinopathies can also be
detected including hemoglobin C trait and thalassemia minor.
A couple presents to you for preconception counseling. The woman is a 28-year-old G0 woman whose family is
Ashkenazi Jewish from Poland. Her husband is 30-years-old and is also Jewish. They seek information about
preconception and prenatal screening. You should offer carrier screening to your patient for all of the following
conditions except:
a) Fanconi anemia
b) Tay-Sachs disease
c) Beta thalassemia anemia
d) Cystic fibrosis
e) Niemann-Pick disease
Correct!!! Fanconi anemia, Tay-Sachs disease, Cystic Fibrosis, and Niemann-Pick disease are all autosomal
recessive conditions that occur at an increased incidence in Jews of Ashkenazi descent. The Beta
thalassemia is seen mainly in Mediterranean populations.
A Jewish couple of Ashkenazi descent comes to you for a first trimester obstetrical visit and asks about their risk for
having a child with an inherited genetic disorder. The 30-year-old patient has a brother who has a child who has
recently been diagnosed with attention deficit hyperactivity disorder. You tell them that the most likely inherited
genetic disease that they would pass onto one of their future children is:
a) Canavan disease
b) Bloom syndrome
c) Tay-Sachs disease
d) Niemann-Pick disease type A
e) Glycogen storage disorder
Correct!!! The carrier frequency for Tay-Sachs disease is estimated at 1/30 for Ashkenazi Jews. The gene
occurs at a much lower frequency (1 in 300) in most other populations. It is the most common recessive
genetic disease among individuals of Eastern European Jewish descent. The frequency of the disease is
1/3000.
A 24-year-old G0 woman presents to you for preconception counseling. Her medical history is complicated by
insulin dependent diabetes mellitus (IDDM, Class C,) hypertension, epilepsy, and hypothyroidism. She is actively
trying to conceive. Her medications include insulin, methyldopa, valproic acid and Synthroid. You counsel her that
her infant has the greatest risk of having which of the following anatomical defects?
a) Duodenal atresia
b) Skeletal anomalies
c) Renal tubular dysgenesis
d) Neural tube defects
e) Omphalocele
Correct!!! Valproic acid is associated with an increased risk for neural tube defects, hydrocephalus and
craniofacial malformations, so D is the correct answer. Insulin and methyldopa are not associated with fetal
defects. Omphalocele is not increased in insulin dependent diabetic patients.
A 30-year-old G1P1 woman with Type 1 diabetes mellitus (White classification, Class B) is at 10 weeks gestation.
Her only other significant history is that she still smokes a half a pack of cigarettes per day. She is concerned that
her child may have birth defects. In this patient, the most likely structural anomaly to be detected by ultrasound on
an 18-week detailed fetal anatomic survey is:
a) Cardiac anomalies
b) Caudal regression malformation
c) Hydrocephalus
d) Microcephaly
e) Limb reductions
Incorrect!!! Women with poorly controlled diabetes immediately prior to conception and during
organogenesis have a four- to eight-fold risk of having a fetus with a structural anomaly. The majority of
lesions involve the central nervous system (neural tube defects) and the cardiovascular system.
Genitourinary and limb defects have also been reported.
A 37-year-old G3P2 woman presents with her husband at 11 weeks gestation for more information regarding genetic
counseling. She has a history of recurrent early pregnancy losses and has been on aspirin for the past six months.
The patient and her husband have heard that chorionic villus sampling (CVS) is available at your institution and are
interested in having it done. CVS can detect which of the following?
a) Spina bifida
b) Fetal omphalocele
c) Trisomy 21
d) Anencephaly
e) Fetal cardiac anomaly
Correct!!! CVS is generally performed at 10-12 weeks gestation. The procedure involves sampling of the
chorionic frondosum, which contains the most mitotically active villi in the placenta. CVS can be
performed using a transabdominal or transcervical approach. The sampled placental tissue may be analyzed
for fetal chromosomal abnormalities, biochemical, or DNA-based studies. CVS cannot be used to detect
neural tube defects. Omphaloceles and neural tube defects are generally diagnosed using prenatal
ultrasound.
A 40-year-old G3P2 woman at 17 weeks gestation would like to have serum screening for Down syndrome. She
asks if she should have a triple screen or a quadruple screen and questions you about what serum marker
distinguishes the quadruple screen from the triple screen. You correctly inform her that the distinguishing serum
marker is which of the following?
a) Alpha fetoprotein (AFP)
b) Pregnancy associated plasma protein A (PAPP A)
c) Progesterone
d) Unconjugated estriol
e) Inhibin A
Correct!!! A fourth maternal serum marker, Inhibin A, is used in the quad screen that is a more sensitive
screening test for Down syndrome. Screening should be offered to all pregnant women between 15 and 18
weeks gestation, although the test may be performed up to 22 weeks gestation. The triple screen is used to
screen for Down syndrome, trisomy 18 and neural tube defects. The three components are AFP, hCG and
unconjugated estriol. PAPP A is an effective marker for screening for Down syndrome in the first trimester.
A 35-year-old G3P0 woman is at 11 weeks gestation. Her body mass index is 38. She desires to have chorionic
villus sampling (CVS); however, she is worried about losing her pregnancy from an elective procedure. After a
thorough discussion of the risks and benefits of CVS, you inform the patient that the risk of miscarriage associated
with CVS is approximately which of the following?
a) 0.1%
b) 1%
c) 5%
d) 10%
e) 15%
Correct!!! The risk of fetal loss associated with CVS is approximately 1.2%.
A 28-year-old G1P0 woman is at 15 weeks gestation. Her husband’s cousin has moderate mental retardation. The
most common cause of inherited mental retardation in this patient’s child would be?
a) Undiagnosed phenylketonuria (PKU
b) Neonatal hypothyroidism
c) Fragile X syndrome
d) Down syndrome
e) Autism
Incorrect!!! Fragile X syndrome is the most common form of inherited mental retardation
Obstetrics – Exam 10
A 35-year-old G3P2 woman presents for her initial prenatal care visit at 15 weeks gestation, according to her last
menstrual period. She reports that a home pregnancy test was positive about 5 weeks ago. Review of her history is
unremarkable and her entire family is in good health. Physical examination reveals a 10-week size uterus. Which of
the following is the most appropriate next step in establishing this pregnancy’s gestational age?
a) Checking fetal heart tones
b) Hysterosonogram
c) Quantitative BHCG
d) Obstetrical ultrasound
e) Quadruple screen
Correct!!! The patient’s reported gestational age and the findings on physical exam are discordant. In this
case, the most reliable method of confirming gestational age is a dating ultrasound. Confirming fetal heart
tones would not help give an accurate gestational age. A hysterosonogram is contraindicated with a
suspected pregnancy. A quantitative BHCG will not reliably predict the gestational age. The uterine size on
physical exam is not the most accurate way to date a pregnancy. Quadruple screen is not used to establish
gestational age.
You are scheduled to see a 34-year-old G2P1 woman at 12 weeks gestation. She did not seek preconception
counseling and is worried about delivering a child with Down syndrome, given her maternal age. She has no
significant medical, surgical, family or social history. Which of the following tests is most effective in screening for
Down syndrome in the second trimester?
a) Quadruple screen
b) Triple screen
c) Amniotic fluid for alpha fetoprotein level
d) Maternal serum alpha fetoprotein level
e) Nuchal translucency measurement with serum PAPP-A and free beta-hCG levels
Incorrect!!! The quadruple test (maternal serum alpha fetoprotein, unconjugated estriol, human chorionic
gonadotropin, and inhibin A) is the most effective screening test for Down syndrome in the second
trimester. Down syndrome occurs in about 1 in 800 births in the absence of prenatal intervention. The
efficacy of screening for Down syndrome is improved when additional components are added to the
maternal serum alpha fetoprotein screening. The addition of unconjugated estriol and human chronic
gonadotropin (the Triple screen) results in a 69% detection rate for Down syndrome. Adding inhibin A to
produce a quadruple screen achieves a detection rate of 80-85%. An amniotic fluid alpha fetoprotein level
is unnecessary. Nuchal translucency measurement with maternal serum PAPP-A and free beta-hCG (known
as the combined test) is a first trimester screen for Down syndrome. It detects approximately 85% of cases
of Down syndrome at a 5% false positive rate.
A 26-year-old G2P1 woman at 26 weeks gestation presents for a routine 50 gram glucose challenge test. After
receiving a one-hour blood glucose value of 144, the patient has a follow up 100 gram 3-hour oral glucose tolerance
test with the following plasma values: fasting blood-sugar 102, 1-hour 181, 2-hours 162, and 3-hours 139 (Normal:
fasting 95, 1-hour 180, 2-hours 155, 3-hours 140). Your next step in this patient’s management is:
a) Repeat the glucose tolerance test in early or mid-third trimester
b) Explain to the patient that she has gestational diabetes and begin diet and blood glucose monitoring
c) Explain to the patient that she has gestational diabetes and begin diet, treatment with oral hypoglycemic agents
and blood glucose monitoring
d) Explain to the patient that she has gestational diabetes and begin diet, treatment with insulin and blood glucose
monitoring
e) Follow the patient as a normal gestation
Correct!!! This patient has three values on the 3-hour glucose tolerance test that were abnormal. The cutoffs
for normal are: Fasting blood sugar less than or equal to 95 1-hour less than or equal to 180 2-hour less than
or equal to 155 3-hour less than or equal to 140 Initial management should include teaching the patient how
to monitor her blood glucose levels at home, on a schedule that would include a fasting blood sugar and 1or 2-hour post-prandial values after all three meals, daily. Goals for blood sugar management would be to
maintain blood sugars when fasting below 90 and 1- and 2-hour post-meal values below 120. A repeat
glucose tolerance test would not add any value, as an abnormal test has already been documented. Oral
hypoglycemic agents and insulin are not indicated at this time, as the patient may achieve adequate glucose
levels with diet modification alone. Gestational diabetes varies in prevalence. The prevalence rate in the
United States has varied from 1.4 to 14% in various studies. Risk factors for high-risk for gestational
diabetes include: a previous large baby (greater than 9 lb,) a history of abnormal glucose tolerance, prepregnancy weight of 110% or more of ideal body weight, and member of an ethnic group with a higher
than normal rate of Type 2 diabetes, such as American Indian or Hispanic descent.
A 29-year-old G2P1 woman at 36 weeks gestation is seen in your clinic for management of her gestational diabetes.
Despite diet modification the patient has required insulin to control her serum glucose levels. She has gained 25
pounds with the pregnancy. She is at risk for all the following complications, EXCEPT:
a) Polyhydramnios
b) Neonatal hypoglycemia
c) Intrauterine growth restriction
d) Preeclampsia
e) Fetal macrosomia
Incorrect!!! Intrauterine growth restriction is typically seen in women with pre-existing diabetes and not
with gestational diabetes. Shoulder dystocia, metabolic disturbances, preeclampsia, polyhydramnios and
fetal macrosomia are all associated risks of gestational diabetes.
A 32-year-old G3P2 woman has delivered a previous child with anencephaly. What is the appropriate recommended
dose of folic acid for this woman?
a) 0.4 mg
b) 0.8 mg
c) 1.0 mg
d) 4 mg
e) 8 mg
Correct!!! In 1991, the Centers for Disease Control and Prevention recommended that all women with a
previous pregnancy complicated by a fetal neural tube defect ingest 4 mg of folic acid daily before
conception and through the first trimester. In one analysis, this dose of folic acid in women at high risk
would reduce the incidence of neural tube defects by 85%.
A 20 year old woman with a history of epilepsy has conceived while taking valproic acid. She is 18 weeks gestation.
She is scheduled for an ultrasound. What is the most common anomaly associated with prenatal exposure to valproic
acid?
a) Ventricular septal defects
b) Omphalocele
c) Neural tube defects
d) Cleft lip and palate
e) Holoprosencephaly
Correct!!! Valproic acid use during pregnancy is associated with a 1 to 2% incidence of neural tube defects,
specifically lumbar meningomyelocele. Fetal ultrasound examination at approximately 16 to 18 weeks
gestation is recommended to detect neural tube defects. Other malformations have been reported in the
offspring of women being treated with valproic acid and a fetal valproate syndrome has been described
which includes spina bifida, cardiac defects, facial clefts, hypospadius, craniosynostosis, and limb defects,
particularly radial aplasia. Case reports have associated prenatal exposure to valproic acid with
omphalocele and lung hypoplasia.
An 18-year-old G1P0 woman presents for prenatal care at 14 weeks gestation. Her medical, surgical, gynecologic,
social and family history is unremarkable. Her dietary history includes high carbohydrate intake with no fresh
vegetables. Her physical examination is within normal limits except that she is pale and has a BMI of 42. Nutritional
counseling should include the following?
a) 25-30 grams of protein in her diet everyday
b) A strict diet to maintain her current weight
c) Folic acid supplementation
d) Intake of 1200 calories a day
e) Initiating at least 1-hour of aerobic exercise a day
Incorrect!!! There should be folic acid supplementation, as well as evaluation for deficiencies in her iron,
protein and other nutrient stores. In general, a patient needs approximately70 grams of protein a day, along
with her other nutrients. It would be prudent to caution her that, though aerobic exercise is recommended
and would be a benefit to her, it is not advisable to initiate a vigorous program in a woman who has not
been routinely working out. Women should gain weight during their pregnancy, and 1200 calories a day is
not sufficient for a pregnant woman.
A 33-year-old G2P1 woman at 10 weeks gestation asks about weight gain during pregnancy. Her body mass index
that day was recorded as 42.The recommendation for weight gain in this woman is:
a) Maintain current weight; no weight gain
b) 5lbs for the entire pregnancy
c) 11-20 lbs for the entire pregnancy
d) 25-35 lbs over the course of the pregnancy
e) Lose up to 10 lbs as long as the fetal growth is appropriate
Correct!!! According to the Institute of Medicine recommendations, obese women with BMI more than 30
should limit their weight gain to 11-20 lbs in pregnancy. The majority of weight gain occurs in the second
half of pregnancy.
A 31-year-old G2P1 woman at 10 weeks gestation presents for prenatal care with an unplanned pregnancy. This
pregnancy was due to a condom accident and the patient was unaware of the availability of emergency
contraception. She receives care from an internist for hypertension, a seizure disorder and a history of depression
and anxiety. She was taking labetalol, phenobarbital, and citalopram. She is concerned about the teratogenic risks
associated with her medications and medical history. Which of the following statements is true with regard to this
patient�s risk of delivering an infant with a birth defect?
a) She is still at risk for teratogenesis if she continues to take her medications
b) The patient can decrease her risk for teratogenesis if she stops all of the medications that may be associated with
birth defects.
c) She has already passed the period of organogenesis
d) Her risk for teratogenesis is present for the entire pregnancy
e) The patient should not be concerned about the fact that she was taking phenobarbital, as it is not associated with
birth defects
Correct!!! The critical period for organogenesis is during the first 8 menstrual weeks. In order for
medications to adversely affect a pregnancy, there must be exposure of the fetus to a noxious stimulus for a
period during a time of active cell differentiation and that the affected tissue is active differentiation.
A 27-year-old G2P1 woman at 31 weeks gestation comes into the labor room after having 24 hours of crampy lower
abdominal discomfort. When reviewing her history, she reports taking methyldopa for chronic hypertension,
phenytoin for a seizure disorder, retinoic acid for cystic acne and low molecular weight heparin for a deep venous
thrombosis that occurred after a recent knee surgery. She also reports that before she knew she was pregnant she had
been taking warfarin and enalapril. Which of her medications do not pose a known risk to her pregnancy?
a) Warfarin
b) Methyldopa
c) Retinoic Acid
d) Phenytoin
e) Enalapril
Obstretrics – Exam 11
A 23-year-old G1P0 woman at 38 weeks gestation, with an uncomplicated pregnancy, presents to labor and delivery
with the complaint of lower abdominal pain and mild nausea for one day. Fetal kick counts are appropriate. Her
review of symptoms is otherwise negative. Vital signs: temperature 97.8; blood pressure 100/60; pulse: 79;
respiratory rate: 13; fetal heart rate: 140s, reactive, with no decelerations; tocometer: irregular contractions every 2
to 8 minutes; fundal height: 36 cm; cervix firm, long, closed and posterior. A urine dipstick is completely negative.
Which of the following is the most likely diagnosis in this patient?
a) Appendicitis
b) Intra-amniotic infection
c) Braxton-Hicks contractions
d) Active labor
e) Placental abruption
Incorrect!!! Braxton Hicks contractions are characterized as short in duration, less intense than true labor,
and the discomfort as being in the lower abdomen and groin areas. A urinary tract infection is unlikely
given the patient’s lack of symptoms and negative urinary dipstick. Preterm labor is not correct, as the
patient is 38 weeks gestation. Active labor is defined by strong, regular uterine contractions that cause
cervical change. This patient’s history does not suggest she is in active labor. Patients with appendicitis
usually present with fever, decreased appetite, nausea and vomiting. Patients with intra-amniotic infection
frequently present with fever, fundal tenderness, maternal and fetal tachycardia.
A 22-year-old G3P0 woman at 38 weeks gestation with an uncomplicated pregnancy presents to labor and delivery
with the complaint of lower abdominal pains for one day. Fetal kick counts are appropriate. She has no other
symptoms. Vital signs: temperature 97.8; blood pressure 100/60; pulse 79; respiratory rate 13; fetal heart rate 140s,
reactive, with no decelerations; fundal height 36 cm; cervix firm, long, closed and posterior. After a full maternal
and fetal evaluation, you tell the patient that she is experiencing Braxton-Hicks contractions and that she can return
home. An appropriate indication for the patient to return to the hospital is:
a) Contractions every 20 minutes
b) Contractions every 10 minutes
c) Fetal movement greater than 10 per two hours
d) Pelvic pressure
e) Vaginal bleeding
Incorrect!!! As part of routine antepartum care counseling, the patient is told to report to the hospital for
suspected labor if any of the following occurs: contractions every five minutes for one hour, rupture of
membranes, fetal movement less than 10 per two hours or vaginal bleeding.
A 38-year-old G1P0 woman presents to labor and delivery at 39 weeks gestation in early labor. She reports good
fetal movement and denies vaginal bleeding and leakage of fluid. Her initial assessment should include which of the
following?
a) Contraction stress test
b) Biophysical profile
c) Physical examination
d) Blood glucose level
e) Nitrazine test
Incorrect!!! The initial evaluation of patients presenting to the hospital for labor includes a review of the
prenatal records with special focus on the antenatal complications and dating criteria, a focus history and a
limited physical examination, especially maternal vital signs and fetal heart rate, with abdominal and pelvic
examination. A speculum exam with nitrazine test may be performed to confirm rupture of membranes if
the patient’s history is suggestive of this condition or if a patient is uncertain as to whether she has
experienced leakage of amniotic fluid.
16-year-old G1P0 woman at 39 weeks gestation presents to the labor and delivery room reporting a gush of bloodtinged fluid approximately 5 hours ago and the onset of uterine contractions shortly thereafter. She says that, over
the past hour, the contractions have become stronger and closer together. After a brief history and physical, the
patient is put on the fetal monitor. The fetal heart rate is 140 to 150 with accelerations and no decelerations. Uterine
contractions are recorded every 2 to 3 minutes. A pelvic exam reveals that the cervix is 4 cm dilated and 100 percent
effaced. Fetal station is 0. After walking around for 30 minutes the patient is put back in bed after complaining of
further discomfort. She requests an epidural. However, obtaining the fetal heart rate externally has become difficult
because the patient cannot lie still. What is the most appropriate next step in the management of this patient?
a) Placing the epidural
b) Applying a fetal scalp electrode
c) Performing a fetal ultrasound to assess the fetal heart rate
d) Place an intrauterine pressure catheter (IUPC)
e) Recommending a cesarean delivery
Correct!!! If the fetal heart rate cannot be confirmed using external methods, then the most reliable way to
document fetal well-being is to apply a fetal scalp electrode. Putting in an epidural without confirming fetal
status might be dangerous. Although ultrasound will provide information regarding the fetal heart rate, it is
not practical to use this to monitor the fetus continuously while the epidural is placed. An intrauterine
pressure catheter will provide information about the strength and frequency of the patient’s contractions. It
will not provide information regarding the fetal status. A cesarean section is not indicated at this point.
Closer fetal monitoring via a fetal scalp electrode should be performed.
A 26-year-old G2P1 woman at 41 weeks is brought to the labor room by ambulance. The emergency medical
technician reports that a pelvic exam performed 20 minutes ago when the patient had a severe urge to push revealed
that the patient was fully dilated and the fetal station was +2. Fetal heart tones were confirmed to be in the 150s,
with no audible decelerations. When the patient is placed on the fetal monitor, the heart rate is noted to be in the 60s.
The maternal heart rate is recorded as 93. Without pushing, the fetal scalp is visible at the introitus. A repeat pelvic
exam shows that the infant is in the left occiput anterior position. What is the most appropriate next step in the
management of this patient?
a) Emergent cesarean section
b) Change the patient's position in an attempt to improve fetal heart tones
c) Assisted operative vaginal delivery
d) Confirm the fetal heart rate with an internal fetal scalp electrode
e) Use ultrasound to assess the fetal heart rate
Incorrect!!! If the patient cannot deliver the infant with one or two pushes, the next best choice given the
fetal station and presentation is to perform an emergent outlet forceps or vacuum-assisted delivery. None of
the other options offer an expedient mode of delivery. Since the patient’s heart rate is distinct from the fetal
heart rate, it is not necessary to check the fetal heart rate with an ultrasound. This will potentially delay the
time until delivery of the fetus.
25-year-old G2P1 woman at 38 weeks gestational age presents to labor and delivery with spontaneous onset of labor
and spontaneous rupture of membranes. Cervical exam was 3 cm at presentation and 5 cm at last check, two hours
ago. Presently, patient is very comfortable and notes mild contractions. You decide to place an intrauterine pressure
catheter (IUPC). On placement, approximately 300 cc of frank blood and amniotic fluid flow out of the vagina.
What is the most appropriate next step in the management of this patient?
a) Emergent cesarean section
b) Withdraw the IUPC, monitor fetus and then replace if tracing reassuring
c) Begin amnioinfusion
d) Begin Pitocin augmentation
e) Keep IUPC in position and connect to tocometer
Incorrect!!! If an intrauterine pressure catheter is placed, and a significant amount of vaginal bleeding is
noted, then it must be assumed that a uterine perforation may have occurred. In this case, withdrawing the
catheter, monitoring the fetus and observing for any signs of fetal compromise would be the most
appropriate management. If the fetal status is found to be reassuring, then another attempt at placing the
catheter may be undertaken.
A 24-year-old G2P1 woman presents at 40 weeks in early labor. Her pregnancy has recently been complicated by
gestational hypertension. Her cervix is long, closed and posterior. The fetal head is at -2 station. There is copious
clear fluid pooling in the posterior vagina. Estimated fetal weight is 3600 grams. She is put on the fetal monitor and
there are frequent variable decelerations. The most common cause of variable decelerations in this patient is?
a) Umbilical cord compression
b) Placental insufficiency
c) Head compression
d) Oligohydramnios
e) Umbilical cord prolapse
Correct!!! Variable decelerations are typically caused by cord compression and are the most common
decelerations seen in labor. Placental insufficiency is usually associated with late decelerations. Head
compression typically causes early decelerations. Oligohydramnios can increase a patient’s risk of having
umbilical cord compression; however, it does not, in itself, cause variable decelerations. Umbilical cord
prolapse occurs in 0.2 to 0.6% of births. Sustained fetal bradycardia is usually observed.
A 34-year-old G1P0 woman at 39 weeks gestation is in active labor. Her cervical exam an hour ago was 5cm
dilated, 90 percent effaced and 0 station. The patient’s nurse asks you to evaluate the fetal heart tracing. The
baseline is 140 beats/minute. There is a deceleration after the onset of each of the last four contractions. You
examine the patient. She is completely dilated and the fetal head is at +3 station. What is the most likely etiology for
these decelerations?
a) Rapid change in cervical dilation
b) Rapid change in descent
c) Umbilical cord compression
d) Uteroplacental insufficiency
e) Head compression
Incorrect!!! This patient is having late decelerations. Late decelerations are associated with uterine
contractions. The onset, nadir, and recovery of the decelerations occur, respectively, after the beginning,
peak and end of the contraction. Late decelerations are associated with uteroplacental insufficiency. A rapid
change in cervical dilation and descent are not associated with late decelerations. Umbilical cord
compression is associated with variable decelerations. Head compression is associated with early
decelerations.
A 19-year-old G1P0 woman comes in for her first prenatal visit at 8 weeks gestation. Her friend had an episiotomy
performed before her forceps-assisted delivery and ended up with a fourth-degree laceration. Concerned about
perineal trauma and fecal incontinence, the patient asks several questions about episiotomies. The most correct
statement regarding episiotomy is:
a) Midline episiotomies cause more long-term discomfort than mediolateral episiotomies.
b) Midline episiotomies are associated with more blood loss than mediolateral episiotomies.
c) An episiotomy enlarges the vaginal outlet.
d) Episiotomies are not associated with an increased incidence of fecal incontinence.
e) The episiotomy is indicated for all forceps deliveries.
Incorrect!!! The episiotomy is used to enlarge the vaginal outlet to facilitate the delivery of the baby. An
episiotomy may be indicated for instrumental deliveries or protracted or arrest of descent; however the role
of prophylactic episiotomy has not been determined. The advantages of the midline episiotomy over the
mediolateral episiotomy are less pain, ease of repair and less blood loss. Midline episiotomy increases the
risk of third- and fourth-degree tears. Therefore, there should be no role for routine episiotomy in modern
obstetrics. Midline episiotomy is associated with a significant increase of third- and fourth-degree
lacerations in spontaneous vaginal delivery in nulliparous women both with spontaneous and operative
vaginal delivery.
A 19-year-old G1P0 woman at 39 weeks gestation has an uncomplicated spontaneous vaginal delivery. Her placenta
delivered spontaneously 10 minutes ago and her estimated blood loss was 250 cc. The immediate postpartum period
that this patient is in is also known as:
a) Third stage of labor
b) Recovery phase
c) Involution
d) Fourth stage of labor
e) Restitution
Incorrect!!! There are four stages of labor. The fourth stage is defined as the immediate postpartum period
of approximately two hours after delivery of the placenta. The first stage is the interval between the onset
of labor and full cervical dilation. The second stage encompasses complete cervical dilation through the
delivery of the infant. The third stage begins after the delivery of the infant and ends with the delivery of
the placenta.
Obstetrics – Exam 13
A 28-year-old G3P3 woman delivered, vaginally, a newborn weighing 4150 grams after an uncomplicated
spontaneous labor. Prior obstetric history was notable for a previous low uterine segment transverse Cesarean
section, secondary to transverse fetal lie. The patient has had no antenatal problems. The placenta delivered
spontaneously without difficulty. There was an immediate rapid vaginal bleeding of 700cc. Which is the most likely
cause of this patient’s hemorrhage?
a) Vaginal or cervical lacerations
b) Uterine inversion
c) Uterine atony
d) Uterine dehiscence
e) Uterine rupture
Correct!!! Postpartum hemorrhage (PPH) is an obstetrical emergency that can follow vaginal or cesarean
delivery. It is best to find and diagnose clinically as excessive bleeding that makes the patient symptomatic
and/or results in signs of hypovolemia, such as hypotension, tachycardia or oliguria. The most common
definition of PPH is an estimated blood loss of greater than or equal to 500ml after vaginal birth, or greater
than or equal to 1000ml after Cesarean delivery. Uterine atony is the most common cause of PPH and is
seen in one in every twenty deliveries.
A 21-year-old G1P1 woman, diagnosed with a postpartum hemorrhage, experienced a period of hypovolemic shock
before her bleeding was controlled and was treated with intravascular volume replacement. An estimated blood loss
of two liters was encountered and there appeared to be full recovery of the patient. On a follow-up office visit, the
patient noted that she was unable to breast feed and had encountered gradual breast atrophy and amenorrhea. The
patient also has slurred speech, moderate, non-pitting edema and constipation. What is her most likely diagnosis?
a) Amenorrhea-galactorrhea
b) Prolactinoma
c) Sheehan Syndrome
d) Asherman Syndrome
e) Pituitary tumor
Correct!!! Sheehan Syndrome is a rare occurrence. When a patient experiences a significant blood loss, this
can uncommonly result in anterior pituitary necrosis, which may lead to loss of gonadotropin, thyroidstimulating hormone (TSH) and adrenocorticotropic hormone (ACTH) production, as they are all produced
by the anterior pituitary.
A 21-year-old G1P0 woman delivered a 4000 gram infant by a low-forceps delivery after a protracted labor course
that included a three-hour second stage. Her prenatal course was notable for development of anemia, poor weight
gain and maternal obesity. Following the delivery, the patient was noted to have a vaginal sulcus laceration and a
third-degree perineal laceration, which required extensive repair. Which of the following factors places this patient
at greatest risk for developing a puerperal infection?
a) Maternal exhaustion
b) Poor nutrition
c) Obesity
d) Iron deficiency
e) Protracted labor
Incorrect!!! Endometritis appearing in a postpartum period is most closely related to the mode of delivery.
Endometritis can be found in less than 3% of vaginal births and this is contrasted by a 5-10 times higher
incidence after Cesarean deliveries. Factors related to increased rates of infection with a vaginal birth
include prolonged labor, prolonged rupture of membranes, multiple vaginal examinations, internal fetal
monitoring, removal of the placenta manually and low socioeconomic status.
A 23-year-old G1P1 woman develops a fever on the third day after an uncomplicated Cesarean section that was
performed secondary to arrest of descent. The only significant finding on physical exam is moderate breast
engorgement and mild uterine fundal tenderness. The most likely diagnosis in this patient is:
a) Urinary tract infection
b) Mastitis
c) Endometritis
d) Wound cellulitis
e) Septic pelvic thrombophlebitis
Incorrect!!! The most common cause of postpartum fever is endometritis. The differential diagnosis
includes urinary tract infection, lower genital tract infection, wound infections, pulmonary infections,
thrombophlebitis, and mastitis. Endometritis appearing in a postpartum period is most closely related to the
mode of delivery and occurs after vaginal delivery in approximately 2 percent of patients and after cesarean
delivery in about 10 to 15 percent. Factors related to increased rates of infection with a vaginal birth
include prolonged labor, prolonged rupture of membranes, multiple vaginal examinations, internal fetal
monitoring, removal of the placenta manually and low socioeconomic status.
A 34-year-old G4P4 woman is diagnosed with endometritis following a Cesarean section performed three days ago.
You perform an endometrial culture. Which of the following is most likely to be found?
a) Escherichia coli
b) Anaerobic streptococcus
c) Anaerobic staphylococcus
d) Clostridium perfringens
e) Aerobic and anaerobic bacteria
Incorrect!!! Bacterial isolates related to postpartum endometritis are usually polymicrobial resulting in a
mix of aerobes and anaerobes in the genital tract. The most causative agents are staph aureus and
Streptococcus.
A 45-year-old G2P2 woman reports crying spells, loss of appetite, difficulty sleeping and a feeling of low self-worth
that began approximately 2 to 3 days after a normal vaginal delivery approximately 10 days ago. The patient noted
that the symptoms had been present for approximately one week prior to her visit to your office. Over the past few
days, there has been a progressive decline in the severity of her symptoms. She denied any suicidal or homicidal
ideations. She has a previous history of anxiety. Which of the following is the most likely diagnosis in this patient?
a) Postpartum depression
b) Manic depression
c) Postpartum psychosis
d) Anxiety disorder
e) Postpartum blues
Correct!!! Signs and symptoms of depression which last for less than two weeks are called postpartum
blues. It occurs in 40-85% of women in the immediate postpartum period. It is a mild disorder that is
usually self-limited. Persistence of symptoms beyond two weeks most likely represents postpartum
depression. The patient does not meet the criteria for postpartum psychosis. She does not have
signs/symptoms of manic depression or anxiety disorder.
A 35-year-old G4P3 woman comes in for a postpartum visit. She had a normal uncomplicated vaginal delivery two
weeks ago. She has a history of postpartum depression, which required treatment with antidepressants with her last
pregnancy. Which of the following signs or symptoms of postpartum depression are most useful to distinguish it
from postpartum blues and normal changes that occur after delivery?
a) Anhedonia
b) Crying Spells
c) Ambivalence toward the newborn
d) Sleeplessness
e) Weight loss
Incorrect!!! In addition to the more common symptoms of depression, the postpartum patient may manifest
a sense of incapability of loving her family and manifest ambivalence toward her infant. Anhedonia is an
inability to experience pleasure from normally pleasurable life events such as eating, exercise, and social or
sexual interaction.
A 20-year-old G2P1 woman, who delivered a healthy baby girl two months ago, is experiencing persistent
depressive symptoms suggestive of postpartum depression. She is recently divorced and has no immediate family or
close friends. She works as a mechanic in a local garage and is planning on going back to school. She denies having
a history of depression. Which of the following is NOT a risk factor for postpartum depression in this patient?
a) Marital conflict
b) Unplanned pregnancy
c) Complicated labor and delivery
d) Personal history of depression
e) Poor relationship with her own mother
Incorrect!!! Risk factors for postpartum depression include: a history of depression, marital conflict, lack of
perceived social support from family and friends, having contemplated terminating the current pregnancy,
stressful life events in the previous twelve months, and a sick leave in the past twelve months related to
hyperemesis, uterine irritability or psychiatric disorder.
A 17-year-old G1P1 woman delivered a term infant two days ago. She is not interested in breastfeeding and she asks
for something to suppress lactation. Which of the following is the safest method of lactation suppression in this
patient?
a) Parlodel
b) Breast binding, ice packs and analgesics
c) Medroxyprogesterone acetate
d) Oral contraceptives
e) Express milk by hand until production stops
Incorrect!!! Hormonal interventions for preventing lactation appear to predispose to thromboembolic
events, as well as a significant risk of rebound engorgement. Parlodel (Bromocriptine), in particular, was
associated with hypertension, stroke and seizures. The safest method still seems to be breast binding, ice
packs and analgesics. The patient should avoid suckling or other means of milk expression, and the natural
inhibition of prolactin secretion will result in breast involution.
A 23-year-old G2P1 woman delivered a healthy male infant two days ago. She comes to your breastfeeding
counseling session for new mothers. Which of the following statements regarding breastfeeding would you include?
a) Breast milk is a major source of immunoglobulin G (IgG)
b) Most ingested drugs that are soluble in maternal blood do not cross into breast milk
c) Breast milk contains a large amount of iron
d) Prolactin stimulates milk production and breast development
e) Breast milk is a good source for Vitamin K
Incorrect!!! Human milk is recognized by the American Academy of Pediatrics as an optimal feeding for
all infants. The American Academy of Pediatrics recommends exclusive breastfeeding for the first six
months after birth. Physicians can influence a patient’s feeding choice, and prenatal education is important
in the initiation and maintenance of breastfeeding. Nationally representative surveys have noted that
women were more likely to initiate breastfeeding if their physicians or nurses encouraged it. Benefits to the
mother include increased uterine contraction due to oxytocin released during milk let down and decreased
blood loss. Breast milk is a major source of immunoglobulin A, the common antibody contained in breast
milk. This appears to prevent the newborn from developing gastrointestinal infections. Breast milk is very
low in iron and supplementation is needed for babies. The majority of drugs will enter breast milk and they
must be considered under counseling in prescribing for breastfeeding mothers.
Obstetrics – Exam 27
A 34-year-old GIP0 woman delivers vaginally at 41 weeks after a difficult and protracted labor. The labor was
The infant weighed 3350 grams at birth. For which of the following is this patient at greatest risk?
a) Uterine atony
b) Vaginal lacerations
c) Amniotic fluid embolism
d) Uterine inversion
e) Retained placenta
Correct!!! Uterine atony is the most common cause of postpartum hemorrhage. Risk factors for uterine
atony include precipitous labor, multiparity, general anesthesia, Oxytocin use in labor, prolonged labor,
macrosomia, hydramnios, twins and chorioamnionitis. Patients at risk for genital tract lacerations are those
who have a precipitous labor, macrosomia or who have an instrument assisted delivery or manipulative
delivery (i.e. breech extraction.) Retained placenta would be more likely if the patient had a prior Cesarean,
uterine leiomyomas, prior uterine curettage, or a succenturiate lobe of placenta. Factors that lead to an overdistended uterus are risk factors for uterine inversion. Grand multiparity, multiple gestation,
polyhydramnios and macrosomia are all risk factors. The most common etiology, however, is excessive
(iatrogenic) traction on the umbilical cord during the 3rd stage of delivery.
A 36- year-old G1P0 woman presents in active labor. Her past medical history and prenatal course were complicated
by chronic hypertension and superimposed preeclampsia. She received magnesium sulfate for seizure prophylaxis
and oxytocin augmentation. She undergoes an uneventful spontaneous vaginal delivery. Postpartum, she has a
1000cc hemorrhage due to uterine atony. Her blood pressure is 130/80; pulse 96; and she is afebrile. Which of the
following uterotonics agents is contraindicated in this patient?
a) Oxytocin
b) Methylergonovine
c) Prostaglandin F2
d) Prostaglandin E2
e) Misoprostol
Correct!!! Methergine, Prostaglandins and Oxytocin are all uterotonics and used to increase uterine
contractions and decrease uterine bleeding. Methylergonovine is an ergot alkaloid, which is a potent
smooth muscle constrictor. It is also a vasoconstrictive agent and should be withheld from women with
hypertension and/or preeclampsia. Misoprostol, non-FDA approved, is used for cervical ripening and labor
induction.
A 19-year-old G1P0 woman at 40 weeks gestational age has an uncomplicated vaginal delivery followed by a brisk
hemorrhage. Her past medical history is significant for steroid-dependent asthma. Her blood pressure is 110/70;
pulse 84; and she is afebrile. Which of the following uterotonic agents should not be used in this patient?
a) Intramuscular Oxytocin
b) Methylergonovine
c) Prostaglandin F2
d) Intravenous Oxytocin
e) Misoprostol
Incorrect!!! Methergine, Prostaglandins and Oxytocin are all uterotonics and used to increase uterine
contractions and decrease uterine bleeding. Prostaglandin F2 (Hemabate) is a potent smooth muscle
constrictor, which also has bronchio-constrictive effect. As such, it should be used with caution in any
patient with reported history of asthma. It is absolutely contra-indicated in patients with poorly controlled
or severe asthma. Misoprostol, non-FDA approved, is used for cervical ripening and labor induction.
A 29-year-old G2 P1 woman presents in early labor after spontaneous rupture of the fetal membranes. Thirty
minutes after arrival, she delivers a 2650 gram male infant. A globular pale mass appears at the introitus when
attempting to deliver the placenta. Her blood pressure is 90/60; pulse 104; and temperature is 37°C. What is the
most likely etiology for this event in this patient?
a) Multiparity
b) Twin gestation
c) Leiomyoma
d) Uterine inversion
e) Rapid labor
Incorrect!!! Uterine inversion is an uncommon etiology of postpartum hemorrhage. Factors that lead to an
over-distended uterus are risk factors for uterine inversion. Grand multiparity, multiple gestation,
polyhydramnios and macrosomia are all risk factors. The most common risk factor, however, is excessive
(iatrogenic) traction on the umbilical cord during the 3rd stage of delivery. Although leiomyomas may
spontaneously prolapsed, it is unlikely during the peripartum period.
A 17-year-old woman presents in labor at term. Her prenatal course was uncomplicated. She delivers a 3500gm
male spontaneously after Oxytocin augmentation of labor. Postpartum, she experiences excessive bleeding. Which
of the following defines postpartum hemorrhage in this patient?
a) Greater than 500 cc
b) Greater than 750 cc
c) Greater than 1000 cc
d) Greater than 1500 cc
e) Any amount of bleeding that leads to hypovolemia
Incorrect!!! Postpartum hemorrhage is defined as bleeding in excess of 500cc after a vaginal delivery or in
excess of 1000 cc after a Cesarean delivery.
A 37-year-old woman with poorly controlled chronic hypertension presents in labor at term. Her prenatal course was
uncomplicated. She delivers a 3500gm male spontaneously after Oxytocin augmentation of labor. Immediately
postpartum, she experiences excessive bleeding. Her blood pressure is 130/90; pulse 84; and she is afebrile. Uterine
fundus is firm; on examination, placenta is intact. Which of the following is the most appropriate next step in the
management of this patient?
a) Exploration for lacerations
b) Intravenous Oxytocin
c) Methylergonovine
d) Uterine artery embolization
e) Intramuscular Oxytocin
Correct!!! The first steps in the management of postpartum hemorrhage are to make sure uterus is wellcontracted, there is no retained placental tissue and to look for lacerations. This patient has a firm fundus,
which indicates a contracted uterus. Her placenta is complete, which typically rules out retained placental
tissue, so it is important to rule out lacerations, which can lead to hemorrhage. Methylergonovine,
Prostaglandins and Oxytocin are all uterotonics and used to increase uterine contractions and decrease
uterine bleeding. Methylergonovine is an ergot alkaloid, which is a potent smooth muscle constrictor. It is
also a vasoconstrictive agent and should be withheld from women with hypertension and/or preeclampsia.
Uterine artery embolization can be considered after other sources of bleeding such as lacerations are ruled
out.
A 30-year-old G5P4 woman at 24 weeks gestation is found to have an anterior placenta with a previa. She has had
three prior Cesarean deliveries. What is the most serious complication that can lead to obstetric hemorrhage in this
woman?
a) Placental abruption
b) Uterine dehiscence prior to labor
c) Uterine inversion
d) Placenta accreta
e) Uterine atony
Correct!!! Placental abruption and uterine atony are both common but, in the presence of a low-lying
anterior placenta in a patient with a history of multiple Cesarean births, the diagnosis of the placenta
accreta must be entertained. Placenta accreta is an abnormally firm attachment of the placenta to the uterine
wall. The incidence of placenta accreta may be increasing because of the increasing number of women with
previous caesarean sections. This is a serious obstetric complication leading to retained placenta and severe
postpartum hemorrhage. Hysterectomy is frequently required due to intractable hemorrhage at delivery.
A 28-year-old G2P1 woman presents in labor at term and delivers a 3500gm male spontaneously after Oxytocin
augmentation of labor. Thirty minutes later, the placenta has not delivered. Her past medical history is significant for
leiomyoma uteri and male factor infertility. Her prenatal course was uncomplicated. What is the most likely risk
factor for retained placenta in this case?
a) Placenta abruption
b) Labor augmentation
c) Leiomyomas
d) Multiparity
e) Circumvallate placenta
Incorrect!!! Placental abruptions, labor augmentation, degree of parity and circumvallate placenta have no
impact on the risk of retained placenta. The following are associated with retained placenta: prior Cesarean,
uterine leiomyomas, prior uterine curettage and succenturiate lobe of placenta.
A 37-year-old G4P3 woman presents in labor at term. Her prenatal course was uncomplicated. She delivers a
3500gm male spontaneously after Oxytocin augmentation of labor. Immediately postpartum, there is excessive
bleeding greater than 2000cc. There are no lacerations and the uterus is found to be floppy. Which of the following
is the most appropriate next step in the management of this patient?
a) Intramuscular methylergonovine
b) Intravenous methylergonovine
c) Intravenous oytocin bolus
d) Intramuscular prostaglandin F2
e) Intravenous prostaglandin F2
Incorrect!!! Prostaglandin F2 should be administered intramuscularly. It could also be injected directly into
the uterine muscle. Neither prostaglandin F2 nor methylergonovine should ever be administered IV, as they
can lead to severe bronchoconstriction and stroke, respectively. Oxytocin is administered as a rapid
infusion of a dilute solution (20-80 units in a liter) and not as an IV bolus.
A 23-year-old G1P0 woman presents in labor at term. Her prenatal course was uncomplicated. She delivers a
3500gm male spontaneously after Oxytocin augmentation of labor. Immediately postpartum, there is excessive
bleeding greater than 2000cc. There are no lacerations and the uterus is found to be floppy. Her blood pressure is
70/30; pulse 144; and hematocrit is 20%. Conservative and medical management have failed and you proceed with
an exploratory laparotomy, which of the following is the most appropriate next step in the management of this
patient?
a) Cervical artery ligation
b) Ovarian artery ligation
c) External iliac artery ligation
d) Hypogastric artery ligation
e) Hysterectomy
Incorrect!!! Ligation of a number of pelvic vessels can lead to reduction in the vascular pressure in the
pelvis thus controlling hemorrhage. This is especially true with internal iliac artery (hypogastirc artery)
ligation. However, ligation of the ovarian arteries should not be undertaken. Ligation of the external iliac
artery results in devascularization of the leg and, therefore, should not be performed. If these more
conservative maneuvers fail, hysterectomy may be necessary but should be last resort considering the age
and the parity of the patient.
Obstetrics - Exam 28
A 20 year-old G1P1 delivered her first baby 24 hours ago. Delivery was uncomplicated and she had an epidural
placed for analgesia at 5 centimeters of cervical dilation. Earlier in the afternoon, she was complaining of a
headache and was given ibuprofen. Three hours later, she complained of increasing headache, photophobia and
nausea. She denies heavy bleeding. Vital signs are pulse 110; respirations 20; temperature 101.5°F, 38.6°C; and
blood pressure 100/50. Physical examination reveals obvious distress, as she has her eyes covered and pain when
she moves her neck. Her lungs are clear and heart has a regular rate. Her abdomen is nontender, and uterine fundus
is easily palpable just below the umbilicus and is nontender. Her extremities reveal no erythema, swelling or
tenderness. Which of the following would be your next diagnostic step?
a) Chest x-ray
b) Urinalysis
c) Lumbar puncture
d) CBC with differential
e) Pelvic ultrasound
Incorrect!!! Epidurals are used commonly for pain relief during labor. Complications of epidural include
spinal headache, localized back pain and meningitis. Symptoms of meningitis progress rapidly and require
aggressive treatment with antibiotics. Diagnosis is made with evaluation of the cerebral spinal fluid from a
lumbar puncture.
Thirty-six hours ago a 23 year-old G1P1 delivered vaginally and sustained a 2nd-degree laceration. She had a
prolonged first stage of labor, ruptured membranes for 26 hours and received penicillin for group B Strep
prophylaxis. She now complains of increasing abdominal pain, cramping and heavy foul smelling lochia. Her vital
signs reveal a temperature of 100.0¢X F, 37.8„a C; pulse 80; blood pressure 120/60; and respirations 18. She has a
tender uterine fundus that measures at the umbilicus. Her extremities reveal mild bilateral edema; no erythema or
tenderness. Blood work reveals a white count of 12.2; hematocrit of 34%; and normal chemistries. Her urinalysis is
positive for blood and negative for WBCs, leukocyte esterase and nitrites. In addition to ampicillin, which of the
following would be the best antibiotic choice?
a) Erythromycin
b) Gentamicin
c) Doxycycline
d) Vancomycin
e) Ciprofloxacin
Correct!!! Endomyometritis is a common complication of prolonged labor, prolonged rupture of
membranes and multiple vaginal examinations. The infection is polymicrobial, mostly anaerobic and
requires broad spectrum antibiotics for treatment until the patient is afebrile for 24 hours. By adding
Gentamicin, you are covering the spectrum of gram-negative organisms. Erythromycin provides good
coverage for upper respiratory infections. Vancomycin provides good coverage for S. aureus and penicillinresistant gram-positive bacteria. Ciprofloxacin provides excellent coverage for gram-negative pathogens,
including Pseudomonas.
A 23 year-old G1P1 delivered vaginally a 42-week infant after a prolonged induction of labor. She had an epidural,
with an indwelling catheter for 36 hours and three IV sites for her intravenous medications. She now complains of
lower abdominal pain, frequency and dysuria. Her vital signs are temperature 98.6°F, 37°C; pulse 70; blood pressure
100/60; and respirations 12. On examination, her lungs are clear, cardiac exam is normal, abdomen is soft, uterine
fundus is firm and nontender, and she has mild suprapubic tenderness. Which of the following organisms is most
likely causing her discomfort?
a) Group A streptococcus
b) Gardnerella vaginalis
c) Chlamydia trachomatis
d) Escherichia coli
e) Group B Streptococcus
Correct!!! Acute cystitis is a common complication after vaginal delivery and the risk increases with the
use of an indwelling catheter. The most common cause of acute cystitis infection is gram-negative bacteria.
The major pathogens are E. coli (75%), P. mirabilis (8%), K. pneumoniae (20%), S. faecalis (<5%), and S.
agalactiae.
A 23 year-old G1P1 delivered her first baby two days ago after an uncomplicated labor and vaginal delivery. She
wants to breast feed and has been working with the lactation team. Prior to discharge, her temperature was 100.4°F,
38°C (other vitals were normal). She denies urinary frequency or dysuria and her lochia is mild without odor. On
examination, her lungs are clear, cardiac exam normal, and abdomen and uterine fundus are nontender. Her breasts
are firm and tender throughout, without erythema and nipples are intact. Which of the following is the most likely
cause of her fever?
a) Endomyometritis
b) Septic pelvic thrombophlebitis
c) Mastitis
d) Breast engorgement
e) Vaginitis
Correct!!! Breast engorgement is an exaggerated response to the lymphatic and venous congestion
associated with lactation. Milk “let-down” generally occurs on postpartum day 2 or 3. If the baby is not
feeding well, the breast can become engorged, which can cause a low-grade fever. Lactating women are
encouraged to feed their baby frequently, and use a breast pump to prevent painful engorgement and
mastitis. Postpartum fever differential includes endometritis, cystitis and mastitis. These are easy
distinguished, based on clinical findings. Vaginitis is not accompanied by fever. Septic pelvic
thrombophlebitis is a rare condition and characterized by high fever not responsive to antibiotics and is a
diagnosis of exclusion.
30 year-old G1P1 is 5 days post-operative from a Cesarean section for arrest of labor at 7 centimeters. She now
complains of minimal abdominal pain and drainage from the right side of the incision. Lochia is normal and she has
no urinary complaints. Her vital signs are normal and she is afebrile. On physical exam, her lung and cardiac
examinations are normal. Her abdomen and uterine fundus are nontender. Her Pfannenstiel incision has erythema
extending 3 centimeters from the incision and there is purulent, bloody drainage coming from the right side. What is
the next best step in the management of this patient?
a) Initiate intravenous antibiotics
b) Initiate oral antibiotics
c) Occlusive dressing to the wound
d) Open drainage of wound
e) Tropical antibiotics to the wound
Correct!!! Mixed bacteria originating from the skin, uterus and vagina cause wound infections after a
Cesarean section. Treatment requires opening the wound, checking for fascia dehiscence and drainage of
the purulent material. Packing the wound until it has healed from the bottom up prevents persistent
infection. Broad spectrum antibiotics are started, but alone will not treat the abscess. Hot packs may relieve
some minor symptoms, but is not adequate treatment alone.
A 32 year-old G2P2 delivered five days ago by uncomplicated vaginal delivery. Her postpartum course thus far has
been unremarkable and she is breast feeding without difficulty. She woke up in the middle of the night with terrible
upper abdominal pain and chills. She admits that she has had pain like this before, but never this severe. Vital signs
reveal blood pressure 120/70; pulse 110; and temperature 101.8°F, 38.8°C. On physical examination, she has
abdominal pain located in the right upper quadrant with rebound. Her uterine fundus is well below the umbilicus and
nontender. Her lochia is normal. Laboratory tests reveal mild anemia, a slightly elevated white count and slightly
elevated liver function tests. What is the most likely etiology of her pain?
a) Endomyometritis
b) Ruptured ovarian abscess
c) Gastric ulcer
d) Cholecystitis
e) Appendicitis
Correct!!! Non-pregnancy related conditions must be considered when evaluating women in the postpartum
period. Pregnancy puts women at risk for cholelithiasis and, therefore, cholecystitis. Classic symptoms
include nausea, vomiting, dyspepsia and upper abdominal pain after fatty foods. Treatment would be
dependent on the severity of symptoms, but often involves cholecystectomy that is usually performed
laparoscopically. Classic clinical findings for endomyometritis include fever and maternal tachycardia,
uterine tenderness and no other localizing signs of infection. This patient is unlikely to have an ovarian
cyst. A gastric ulcer would most likely have caused some symptoms during pregnancy. Appendicitis
presents with nausea, vomiting, anorexia and abdominal pain.
A 22 year-old delivered her first baby five days ago after a prolonged labor and subsequent Cesarean section for
arrest of cervical dilation at 7 centimeters. Fever was noted on postoperative day 2 and, despite broad spectrum
antibiotics, she continues to have temperature spikes above 101.3F, 38.5C. She is eating a normal diet and
ambulating normally. On physical examination, her breasts have no erythema and nipples are intact. Her abdomen is
soft, uterine fundus is firm and nontender, and her incision is healing without induration or erythema. She has
normal lochia and her urinalysis is normal. Pelvic examination reveals a firm nontender uterus and no adnexal
masses or tenderness. Which of the following is the most likely cause of her fevers?
a) Endometritis
b) Cystitis
c) Septic pelvic thrombophlebitis
d) Ovarian abscess
e) Mastitis
Incorrect!!! Septic thrombophlebitis involves thrombosis of the venous system of the pelvis. Diagnosis is
often one of exclusion of other causes, but sometimes a CT scan will reveal thrombosed veins. Treatment
requires addition of anticoagulation to antibiotics and resolution of fevers is rapid. Anticoagulation
treatment is short-term. Classic clinical findings for endometritis include fever and maternal tachycardia,
uterine tenderness and no other localizing signs of infection. The clinical manifestations of cystitis include
lower abdominal pain, frequency, urgency and dysuria. The clinical findings in patients with mastitis
include fever, tenderness, induration and erythema of the affected breast.
A 22 year-old delivered her first baby five days ago after a prolonged labor and subsequent Cesarean section for
arrest of cervical dilation at 7 centimeters. Fever was noted on postoperative day 2 and despite intravenous broad
spectrum antibiotics, she continues to have temperature spikes above 101.3F, 38.5C. She is eating a normal diet and
ambulating normally. On physical examination, her breasts have no erythema and nipples are intact. Her abdomen is
soft, uterine fundus is firm and nontender, and her incision is healing without induration or erythema. She has
normal lochia and her urinalysis is normal. Pelvic examination reveals a firm nontender uterus and no adnexal
masses or tenderness. Which of the following treatments is indicated for this patient?
a) Addition of antifungal therapy
b) Addition of oral antibiotic therapy
c) Addition of antiviral therapy
d) Surgical exploration
e) Heparin anticoagulation
Incorrect!!! Septic thrombophlebitis involves thrombosis of the venous system of the pelvis. Diagnosis is
often one of exclusion of other causes, but sometimes a CT scan will reveal thrombosed veins. Treatment
requires addition of anticoagulation to antibiotics and resolution of fevers is generally rapid.
Anticoagulation treatment is short-term. The addition of oral antibiotics has no extra benefit on a patient
who is already on broad spectrum IV antibiotics. She has no evidence of fungal or viral infections, so
therapy for these is not indicated. There is also no indication she needs surgery.
A 32 year-old delivered a 9-pound baby and sustained a 4th-degree laceration two days ago. The delivery was
complicated by a shoulder dystocia. The laceration was repaired in layers in the customary fashion. She now
complains of increasing perineal pain, fever chills and weakness. Her vital signs are: blood pressure 90/50; pulse
120; and temperature 102.2F 39C. Her abdomen is soft, nontender and her uterine fundus is firm and nontender. Her
perineum is erythematous, swollen and the laceration edges are grey. The laceration site is nontender and without
feeling but there is tenderness of the surrounding tissue. What is the most likely etiology for her fevers?
a) Endomyometritis
b) Necrotizing fascitis
c) Cellulitis
d) Hematoma
e) Proctocolitis
Incorrect!!! Necrotizing fascitis, a dangerous infection caused by gas forming organisms, such as
Clostridium, can quickly cause sepsis and death. Classic clinical manifestations include fever, pain and
induration of the wound. Treatment involves early recognition, antibiotics and debridement of the necrotic
tissue. Classic clinical findings for endomyometritis include fever and maternal tachycardia, uterine
tenderness and no other localizing signs of infection. Cellulitis presents as swollen, erythematous, tender
and warm area, but without grey necrotic edges. Hematoma findings include a swollen, tender and painful
area that may be expanding.
A 32 year-old delivered a 9-pound baby and sustained a 4th-degree laceration two days ago. The delivery was
complicated by a shoulder dystocia. Her laceration was repaired in layers in the customary fashion. She now
complains of increasing pain in her perineal area, fever chills and weakness. Her vital signs are: blood pressure
90/50; pulse 120; and temperature 102.2°F, 39°C. Her abdomen is soft, nontender and her uterine fundus is firm and
nontender. Her perineum is erythematous, swollen, but the laceration edges have separated and are grey. The
laceration site is nontender and without feeling but there is tenderness of the surrounding tissue. In addition to broad
spectrum antibiotics, what is your next step in the management of this patient?
a) Sitz baths
b) Whirlpool therapy
c) Debridement
d) Repair of laceration site
e) Incision and drainage of perineal laceration
Correct!!! Aggressive debridement of the necrotic areas is required to prevent further spread of the
infection. Debridement should extend until vital tissue with good blood supply is encountered. Repair of
the defect should be delayed until the infection has completely resolved. Sitz baths and whirlpool therapy
will provide symptomatic relief for her discomfort, but not adequate treatment. Incision and drainage of
perineal laceration is appropriate for an uncomplicated abscess.
Obstetrics - Exam 15
A 30-year-old G3P2 woman had her last normal menstrual period 8 weeks ago. She began spotting 3 days ago and
developed cramping this morning. She has a history of a chlamydia infection with a previous pregnancy. She has no
history of other sexually transmitted infections and is currently sexually active with one partner. She smokes one
pack of cigarettes per day and denies alcohol or drug use. On physical exam, her blood pressure is 120/70, pulse 82,
respirations 20, and temperature 98.5°F. On abdominal exam, she has suprapubic and diffuse bilateral lower
quadrant tenderness, but no rebound or guarding. On pelvic exam, she has normal external genitalia, old blood in the
vaginal vault, the cervix is without lesions and os closed. Pertinent labs: Quantitative beta hCG = 1000mIU/ml;
urinalysis normal; hematocrit = 32%; transvaginal ultrasound shows no intrauterine pregnancy, no adnexal masses,
no free fluid in pelvis. Which of the following is the most appropriate next step in the management of this patient?
a) Treat with Methotrexate
b) Exploratory surgery
c) Repeat hCG in 48 hours
d) Repeat hCG in one week
e) Admit the patient to the hospital for observation
Incorrect!!! The patient first needs to have an accurate diagnosis before a treatment plan is entertained. She
does not yet have a diagnosis of ectopic pregnancy. Repeating the hCG is the next step in this patient’s
management. Inappropriately rising hCG levels (less than 50% increase in 48 hours) or levels that either do
not fall following diagnostic dilatation and curettage (D&C) would be consistent with the diagnosis of
ectopic pregnancy. Alternatively, a fetal pole must be visualized outside the uterus on ultrasound. The
patient would need an hCG level over the discriminatory zone (the level where an intrauterine pregnancy
can be seen on ultrasound) with an empty uterus. The level commonly used is 2000mIU/ml). Treatment
with methotrexate may be appropriate, but only after a definitive diagnosis is made. The patient does not
yet have this level and is stable. She is, therefore, not a candidate for exploratory surgery. If she had
unstable vital signs or an acute abdomen, a diagnostic laparoscopy/laparotomy would be indicated.
Diagnostic D&C can be done in the instance where the patient has inappropriately rising hCG levels.
Repeating the ultrasound in one week is not recommended because a delay in diagnosis could result in a
ruptured ectopic pregnancy and increased risk to the patient. The patient is stable; therefore, she does not
need to be admitted to the hospital.
A 28-year-old female patient had her last normal menstrual period 4 weeks ago. She began spotting 2 days ago, and
awoke this morning with left lower quadrant pain of intensity 4/10. She has had no urinary complaints, no nausea or
vomiting, and the remainder of the review of systems is negative. She has no history of sexually transmitted
infections. She smokes one pack of cigarettes per day and denies alcohol or drug use. Her vital signs are: blood
pressure 124/68, pulse 76, respirations 18, and temperature 99° F. On exam, she has mild left lower quadrant
tenderness, with no rebound or guarding. Pelvic exam is normal except for mild tenderness on the left side. Pertinent
labs: Quantitative beta hCG = 400 mIU/ml; progesterone = 5 ng/ml; hematocrit = 34%.Ultrasound shows a fluid
collection in the uterus, with no adnexal masses and no free fluid. What is the most likely diagnosis?
a) Ovarian torsion
b) Missed abortion
c) Early intrauterine pregnancy
d) Unable to establish a diagnosis
e) Ectopic pregnancy
Incorrect!!! It is difficult to establish a definitive diagnosis at this time. When the hCG level is below the
discriminatory zone (2000 mIU/ml,) an early pregnancy may not be visualized in the uterus on ultrasound.
Missed abortion, early intrauterine pregnancy and ectopic pregnancy could only be diagnosed by serial
hCG levels (at least every 48 hours until a trend is established, usually 3 levels.) Ovarian torsion is a
possible diagnosis; however, this is more common with an ovarian mass.
A 17-year-old G3P0 woman who had her last normal menstrual period 7 weeks ago has severe right lower quadrant
pain. She notes that last night she began having suprapubic pain that radiated to her right lower quadrant. This
morning, the pain awoke her from sleep. She has had no vaginal bleeding, no nausea or vomiting. The patient’s
history is notable for two first trimester elective abortions and a history of Chlamydia treated twice. Pertinent
physical exam findings include: blood pressure 90/60; pulse 99; respirations 22; and temperature 98.8°F. The patient
is curled on a stretcher in a fetal position and says she hurts too much to move. She has rebound and voluntary
guarding on abdominal examination. She has profound cervical motion tenderness and rectal tenderness. Pertinent
labs: beta hCG level 2500 mIU/ml; hematocrit 24%; and urinalysis negative. Ultrasound shows no intrauterine
pregnancy, a right adnexal mass, 6x 2 cm, and a moderate amount of free fluid. Which of the following is the most
appropriate next step in the management of this patient?
a) Admit as inpatient for serial examinations
b) Exploratory surgery
c) Recheck hCG level in 48 hours
d) Administer methotrexate
e) Dilatation and Curettage
Incorrect!!! This patient has a ruptured ectopic pregnancy until proven otherwise. Her vital signs,
examination and anemia are consistent with an intra-abdominal bleed. Exploratory laparoscopy/laparotomy
is indicated at this point. Conservative management with observation, serial examinations or repeat hCG
testing could be dangerous in a patient suspected of having a ruptured ectopic pregnancy. Medical
management (methotrexate) is not used in a patient with an acute surgical abdomen. Dilatation and
curettage would not be the next step in management and might only be considered in this scenario after the
patient’s abdomen was explored.
A 19-year-old G2P1 woman has vaginal spotting and uterine cramping. She had her last normal menstrual period 6
weeks ago and she began spotting 3 days ago. She denies any history of sexually transmitted infections. Her vital
signs include: blood pressure 120/70; pulse 78; respirations 20; and temperature 98.4°F. On pelvic exam, she has no
cervical motion tenderness, her uterus is nontender and measures 7 x 5 cm; no adnexal masses are palpable.
Pertinent labs: Quantitative BHCG 48 hours ago was 1500, currently BHCG is 3100mIU/ml; progesterone = 26
ng/ml; hematocrit = 38%; and urinalysis is normal. What would you expect to see on transvaginal ultrasound?
a) Debris in uterus
b) Viable intrauterine pregnancy
c) Adnexal mass, empty uterus
d) No adnexal mass, empty uterus
e) Non-viable intrauterine pregnancy
Incorrect!!! Transvaginal ultrasound will most likely show an intrauterine pregnancy. The hCG level is
above the discriminatory zone for ultrasound (2000 mIU/ml), and the level has doubled in 48 hours.
Additionally, the progesterone level is outside the range of the usual failing pregnancy (>25 ng/ml suggests
healthy pregnancy) and up to 30% of all normal pregnancies experience first trimester spotting/bleeding.
The findings of debris in the uterus, an empty uterus, with or without an adnexal mass, or free fluid
(suggesting hemoperitoneum) would not be anticipated.
A 20-year-old G1P0 woman has vaginal spotting and mild cramping for the last 3 days. She had her last normal
menstrual period approximately 6-1/2 weeks ago. She had a positive home pregnancy test. Her medical and
gynecologic histories are negative and non-contributory. On physical exam: blood pressure 120/72; pulse 64;
respirations 18; temperature 98.0°F. On pelvic exam, she has scant old blood in the vagina, with a normal appearing
cervix and no discharge. On bimanual exam, her uterus is nontender and small, and there are no adnexal masses
palpable. Pertinent labs: Quantitative BHCG=750 mIU/ml 48 hours ago; today, the level is 760 mIU/ml;
progesterone level = 3.2 ng/ml; hematocrit is 37%. Transvaginal ultrasound shows a fluid collection in the uterus
and no fetal pole, no masses and no free fluid in the pelvis. Which of the following is the most appropriate next step
in the management of this patient?
a) Exploratory surgery
b) Treat with Methotrexate
c) Treat with Mifepristone
d) Dilatation and Curettage
e) Repeat hCG level in one week
Incorrect!!! The pregnancy is failing or is located outside the uterus based on the abnormal hCG levels. In a
normal pregnancy, the level should rise by at least 50% every 48 hours until the pregnancy is 42 days old
(after that time, the rise in level may not follow the curve.) In addition, a progesterone level of <5 ng/ml
suggests a failing or extrauterine pregnancy. In this instance, a dilatation and curettage can be both
therapeutic and diagnostic. Following the procedure, an hCG level should be drawn in the recovery room
and then 12-24 hours later. If the level does not decrease by 15% or if it increases, the diagnosis of an
ectopic pregnancy is made and can be treated accordingly. Exploratory abdominal surgery is not necessary
at this stage in the evaluation. Treatment with methotrexate or mifepristone (RU486) is not indicated, since
the diagnosis is not yet ascertained. Repeating the hCG level in one week will delay treatment and put the
patient at unnecessary risk for complications.
A 41-year-old G3P2 woman has cramping, vaginal bleeding and right lower quadrant pain for five days has
progressively worsened and localized to the right side. Her last normal menstrual period occurred 7 weeks ago. Her
pain is currently constant, and she is bleeding similar to a normal menstrual period. She reports no other symptoms.
Her surgical history is notable for a bilateral tubal ligation following her last delivery. On physical exam: blood
pressure is 110/74; pulse 82; respirations 18; temperature 98.8°F. On abdominal exam, she has right lower quadrant
tenderness, with rebound and bilateral guarding in the lower quadrants. On pelvic exam, she has scant old blood in
the vagina and a normal appearing cervix. Her uterus is normal size and slightly tender. She has cervical motion
tenderness on bimanual examination, and marked tenderness on rectal examination. Pertinent labs: Quantitative
hCG=4000 mIU/ml; progesterone level =6.2 ng/ml; hematocrit is 34%; and the white blood cell count is 15.4, with
88% segmented neutrophils with no bands. The transvaginal ultrasound shows an empty uterus with endometrial
thickening, a mass in right ovary measuring 3.8x 2 cm, and a small amount of free fluid in the pelvis. What is the
most likely diagnosis in this patient?
a) Pelvic inflammatory disease
b) Ectopic pregnancy
c) Heterotopic pregnancy
d) Missed abortion
e) Ruptured corpus luteum cyst
Correct!!! The diagnosis of ectopic pregnancy is made when either: 1) a fetal pole is visualized outside the
uterus on ultrasound; 2) the patient has an hCG level over the discriminatory zone (the level at which an
intrauterine pregnancy should be seen on ultrasound, usually 2000 mIU/ml) and there is no intrauterine
pregnancy (IUP) seen on ultrasound; or 3) the patient has inappropriately rising BhCG levels (less than 50
% increase in 48 hours) and has levels which do not fall following diagnostic dilatation and curettage. This
patient falls under #2 criteria, as her beta hCG is >2000 with no intrauterine pregnancy seen on ultrasound.
The history, physical exam and lab data are not consistent with pelvic inflammatory disease, ovarian
torsion, appendicitis or a ruptured corpus luteum cyst. With a heterotopic pregnancy, there should be a
visible pregnancy in the uterus. With a missed abortion there should also be some visible tissue or a fetal
pole within the uterus.
A 23-year-old G1P0 woman has cramping, vaginal bleeding and right lower quadrant pain. Her last normal
menstrual period occurred 7 weeks ago. On physical exam, vital signs are: blood pressure 110/74; pulse 82;
respirations 18; and temperature 98.8°F. On abdominal exam, she has very mild right lower quadrant tenderness. On
pelvic exam, she has scant old blood in the vagina and a normal appearing cervix. Her uterus is normal size and
slightly tender. On bimanual exam, there is no cervical motion tenderness, and she has slight tenderness in right
lower quadrant. Pertinent labs: Quantitative beta hCG=2500 mIU/ml; progesterone level = 6.2 ng/ml; hematocrit =
34%. The transvaginal ultrasound shows an empty uterus with endometrial thickening, a mass in the right ovary
measuring 3.0 x 2 cm and a small amount of free fluid in the pelvis. Which of the following is the most appropriate
next step in the management of this patient?
a) Methotrexate
b) Antibiotics, repeat the hCG level in 48 hours
c) Observation, repeat the ultrasound in 48 hours
d) Dilation and curettage
e) Culdocentesis
Incorrect!!! Certain conditions must be met prior to initiating methotrexate therapy for treatment of an
ectopic pregnancy. These include: hemodynamic stability, nonruptured ectopic pregnancy, size of ectopic
mass <4 cm without a fetal heart rate or <3.5 cm in the presence of a fetal heart rate, normal liver enzymes
and renal function, normal white cell count, and the ability of the patient to follow up rapidly (reliable
transportation, etc.,) if her condition changes. There is no indication for antibiotics in this scenario.
Offering observation delays treatment and pain control would not address the underlying cause of the
patient’s problem. Culdocentesis is not indicated and does not change the management of this patient.
A 32-year-old G5P3 woman presents with left sided abdominal pain and pelvic pain. Her last normal menstrual
period was 8 weeks ago. She began having pain early this morning and it has increased to a severity of 8/10. She
denies nausea or vomiting, vaginal bleeding and other symptoms. Her gynecological history is notable for a history
of ectopic pregnancy on the right side 4 years ago. At that time, she had a right salpingectomy and a left tubal
ligation. On physical exam: blood pressure is 90/54; pulse 108; respirations 22; and temperature 98.4°F. On
abdominal examination, she has rebound and guarding in all quadrants, and on pelvic exam, her uterus is very
tender. A transvaginal ultrasound shows an empty uterus, left pelvic mass with a gestational sac and fetal pole, and a
large amount of free fluid in the pelvis. Her hematocrit is 26%. What would be the next best step in the
management?
a) Treat with Methotrexate
b) Admit for observation
c) Repeat hCG level in 48 hours
d) Laparoscopy
e) Dilatation and Curettage
Incorrect!!! This scenario is consistent with the patient having a ruptured ectopic pregnancy. Signs of
hypovolemia (tachycardia, hypotension) with peritoneal signs (rebound, guarding and severe abdominal
tenderness) and a positive pregnancy test lead to the diagnosis of ruptured ectopic pregnancy. Conservative
management, with observation and repeating the hCG level in 48 hours is not indicated since a diagnosis is
clear and waiting can potentially be dangerous to the patient. Dilatation and curettage would only be
considered after laparoscopy, if needed.
A 19-year-old G1P0 woman with a desired pregnancy notes vaginal spotting early this morning and it has slightly
increased. Her last normal menstrual period occurred 6 weeks ago. She has no pain or other symptoms. Her medical
history is noncontributory. On physical exam: blood pressure is 120/68; pulse 68; respirations 20; and temperature
98.6°F. On pelvic exam, her cervix is normal; her uterus is small and nontender; there are no masses palpable. Labs
show quantitative BhCG 750 mIU/ml; progesterone level 3.8 ng/ml; hematocrit is 38%. Which of the following is
the most appropriate next step in the management of this patient?
a) Order a transvaginal ultrasound
b) Repeat BhCG level in 24 hours
c) Repeat BhCG level in 48 hours
d) Dilation and curettage
e) Recommend that the patient remain on bed and pelvic rest of the next 24 hours
Incorrect!!! Repeating the hCG level will show whether the pregnancy is viable or failing. The appropriate
time interval for repeating the initial level is 48 hours, since during the first 42 days of gestation, levels
increase by approximately 50% every 48 hours in most viable pregnancies. Ordering an ultrasound would
not be helpful, since the patient’s hCG level is lower than the discriminatory zone (the level at which an
intrauterine pregnancy should be seen on ultrasound, usually 2000 mIU/ml). There is no need to repeat the
progesterone level. Dilatation and curettage or treatment with methotrexate are both inappropriate without a
diagnosis, since both could interrupt a viable pregnancy. Progesterone suppositories will not help because
progesterone level is below viable level.
A 19-year-old G1P0 woman notes vaginal spotting. Her last normal menstrual period occurred 6 weeks ago. She
began having spotting early this morning and it has increased only slightly. She has no pain and denies other
symptoms. Her medical history is noncontributory. On physical exam: blood pressure is 120/68; pulse 68;
respirations 20; and temperature 98.6°F. On pelvic exam, her cervix is normal; uterus is small and nontender; and no
masses are palpable. Initial labs show quantitative beta hCG 2000 mIU/ml and hematocrit is 38%. A repeat hCG
level 48 hours later is 2100 mIU/ml. A transvaginal ultrasound shows an empty uterus with a thin endometrial stripe
and no adnexal masses. What is your next step in management?
a) Dilatation and Curettage
b) Treat with Methotrexate
c) Exploratory laparotomy
d) Repeat hCG level in 48 hours
e) Repeat ultrasound in 24 hours
Incorrect!!! The patient clearly has an abnormal pregnancy, as demonstrated by the slowly rising hCG
levels. Since the hCG level is above 2000 mIU/ml, and she has a thin endometrial stripe, this rules out an
intrauterine pregnancy and the diagnosis is an ectopic pregnancy. She is a good candidate for medical
treatment with methotrexate. Criteria to consider for medical treatment include hemodynamic stability,
nonruptured ectopic pregnancy, size of ectopic mass <4 cm without a fetal heart rate or <3.5 cm in the
presence of a fetal heart rate, normal liver enzymes and renal function, normal white cell count, the ability
of the patient to follow up rapidly if her condition changes (reliable transportation, etc.) Dilatation and
curettage and exploratory laparotomy are invasive procedures that can be avoided in this patient. She does
not need another hCG level because the diagnosis is clear. There is no indication for an ultrasound in this
case.
Obstetrics - Exam 16
A 25-year-old G2P0 female patient presents to the emergency department at 7-1/2 weeks gestation with vaginal
bleeding. She has not passed any products of conception. On exam, there is blood in the vault and a closed cervical
os. Sonogram confirms an intrauterine pregnancy and fetal heart motion is observed. Which of the following
diagnoses most accurately describes her condition?
a) Incomplete abortion
b) Missed abortion
c) Threatened abortion
d) Complete abortion
e) Recurrent abortion
Correct!!! Threatened abortion refers to vaginal bleeding before 20 weeks gestation without the passage of
any products and a closed cervix. This distinguishes the patient from others experiencing incomplete
abortions, which have passed some, but not all, of the products of conception or complete abortions where
they have passed all the products of conception. Missed abortions refer to those who have experienced fetal
demise without cervical dilatation or passage of products of conception. Recurrent abortion refers to three
successive spontaneous abortions.
A 25-year-old female patient presents to the clinic for follow-up of a first trimester spontaneous abortion. She wants
to discuss the cause of this event. Which of the following general etiologic categories accounts for the majority of
first trimester spontaneous abortions?
a) Immunologic abnormalities
b) Conceptus genetic anomalies
c) Maternal genetic anomalies
d) Maternal chronic diseases
e) Structural/uterine anomalies
Correct!!! Although investigators have implicated all of the listed categories as possible causes of
spontaneous abortion, genetic abnormalities involving the conceptus account for the majority. In fact,
approximately 50 to 60 percent of embryos and early fetuses that are spontaneously aborted contain
chromosomal abnormalities, accounting for most of early pregnancy wastage.
A 35-year-old woman presents to the emergency department with heavy vaginal bleeding at 6 weeks gestation. She
asks you about the potential etiology and you explain that genetic abnormalities of the conceptus are the most
common cause. Which of the following is the chromosomal abnormality most often associated with first trimester
spontaneous pregnancy loss?
a) Autosomal trisomy
b) Triploidy
c) Tetraploidy
d) Monosomy X (45X,0)
e) Fragile X mutation
Correct!!! Autosomal trisomy is the most common abnormal karyotype encountered in spontaneous
abortuses, accounting for approximately 40-50% of cases. Triploidy accounts for approximately 15%, and
tetraploidy 5% of cases. Monosomy X (45X, 0) is seen in 15-25% of losses. The Fragile X mutation
involves an expanded number of trinucleotide repeats in the CGG (cytosine-guanine) sequence.
A 27-year-old G2P0 woman is diagnosed with an early first trimester spontaneous abortion. She has a history of
Type I Diabetes Mellitus, mild chronic hypertension and one prior termination of pregnancy. The patient is very
upset because she thinks she will never have a baby and wants to know what caused this to happen. Which of the
following is the most likely cause of this spontaneous abortion?
a) Prior termination of pregnancy
b) Chronic hypertension
c) Diabetes mellitus
d) Intrauterine adhesions
e) Infection
Correct!!! Systemic diseases such as diabetes mellitus, chronic renal disease and lupus are associated with
early pregnancy loss. In women with insulin-dependent diabetes, the rates of spontaneous abortion and
major congenital malformations are both increased. The risk appears related to the degree of metabolic
control in the first trimester. There are many other causes of spontaneous abortion, including genetic
factors, endocrine abnormalities, reproductive tract abnormalities, immunologic factors and environmental
factors. The patient’s history of mild chronic hypertension and one prior termination of pregnancy do not
increase her risk of a first trimester loss. Additionally, an uncomplicated termination of pregnancy should
not lead to intrauterine adhesions and infection, and are not likely causes in this scenario.
A 25-year-old G2P1 woman at 8 weeks gestation is diagnosed with a spontaneous abortion. Her husband is 40years-old. The patient’s past medical history is noncontributory. She gets some exercise regularly and smokes two
packs of cigarettes a day. Three years ago, she had a full-term delivery that was complicated by mild preeclampsia.
Which of the following factors is most likely the cause of this spontaneous abortion?
a) Infection
b) Advanced paternal age
c) Environmental factors
d) Uterine anomaly (i.e. unicornuate uterus)
e) History of preeclampsia
Incorrect!!! Environmental factors, such as smoking, alcohol and radiation are causes of spontaneous
abortion. Although the risk increases with infections, such as listeria, mycoplasma, ureaplasma,
toxoplasmosis and syphilis, advancing maternal or paternal age, advancing parity and some mullerian
anomalies, the clinical scenario does not support these as possible causes. An isolated history of
preeclampsia confers no increase in risk of spontaneous abortion.
A 22-year-old G1P0 woman presents to the emergency department at eight weeks gestation experiencing heavy
vaginal bleeding. Pelvic exam demonstrates brisk bleeding through a dilated cervical os. The patient’s hemoglobin is
7 (hematocrit 21%.) Which of the following is the most appropriate next step in the management of this patient?
a) Administration of intravaginal Misoprostol
b) Administration of oral Misoprostol
c) Dilatation and suction curettage
d) Endometrial ablation
e) Expectant care to permit spontaneous abortion
Incorrect!!! This patient is actively bleeding and is anemic. She, therefore, requires immediate surgical
treatment consisting of dilatation and suction curettage. Although clinicians increasingly utilize both
expectant management and various drug regimens to treat spontaneous abortion, a prerequisite for either is
that the patient is hemodynamically stable and reliable for follow-up care. Endometrial ablation will not
work in this case, as the products of conception need to be evacuated to control the bleeding.
A 34-year-old female patient originally presented with slight spotting. A sonogram and serial quantitative BhCG
tests have confirmed a 6-week missed abortion. She has no other complaints or relevant gynecologic, medical or
surgical histories. Her partner accompanies her and is supportive. The patient wishes to avoid any unnecessary
medical interventions and asks whether she can safely let nature take its course. What is the best advice for this
patient?
a) She should undergo immediate dilation and suction curettage to prevent severe hemorrhage
b) She should undergo treatment in one week to prevent infection
c) She should undergo immediate medical treatment to lessen surgical risk
d) The patient does not require any treatment over the next couple of weeks
e) The patient should undergo a dilatation and evacuation in one week
Incorrect!!! Patients experiencing early pregnancy loss can safely consider several different treatments,
including expectant management, drug therapy to assist with expulsion of the pregnancy or surgical
evacuation. Provided the patient is hemodynamically stable and reliable for follow-up, expectant
management is appropriate therapy. At the gestational age described, expectant management portends no
increase in risk of either hemorrhage or infection compared with surgical or medical evacuation. A dilation
and evacuation is used for second trimester pregnancies.
A 29-year-old G3P0 woman presents to your office for prenatal care at 8 weeks gestation. Her two prior pregnancies
ended in a spontaneous loss at 19 and 18 weeks, respectively. Records corroborate the patient’s history of an
uncomplicated gestation until the evening of the losses, when she experienced a pink-tinged discharge that prompted
her to call her obstetrician. In both cases, the obstetrician noted that her cervix had dilated completely with the
amnionic sac bulging into the vagina to the level of the introitus. The patient was afebrile without other complaints
and there was no uterine tenderness on exam. She spontaneously delivered the fetus and placenta in both cases. A
sonohysterogram confirmed normal uterine anatomy several weeks later. What is the most reasonable plan of
management for this patient?
a) Begin weekly fetal fibronectin testing
b) Placement of a cervical cerclage at approximately 14 week’s gestation
c) Immediate placement of a cervical cerclage
d) Administration of low dose aspirin and heparin and evaluation for cerclage placement
e) Administration of prophylactic progesterone for a history of preterm labor
Incorrect!!! Although some investigators question the therapeutic efficacy of prophylactic cerclage, this
represents the only reasonable therapeutic choice among those listed for a patient with an incompetent
cervix. Fetal fibronectin is not an indicator for incompetent cervix and is usually used later in pregnancy as
a negative predictor of preterm delivery. Pregnancy loss in the late second trimester is not usually related to
genetic abnormality of the conceptus and most clinicians delay placement of a cerclage until after the first
trimester, given the high background prevalence of first trimester pregnancy wastage. Although some
clinicians strongly support the existence of an antiphospholipid antibody syndrome, the term most
commonly refers to pregnancy loss or demise, rather than the clinical scenario of silent cervical dilatation
with delivery described. The patient would, therefore, not need aspirin or Heparin. The patient’s prior
pregnancy loss occurred earlier in gestation than associated with preterm labor, which is most commonly
diagnosed after 24 to 28 weeks gestation. Although some clinicians use prophylactic progesterone to
prevent recurrent abortion, as well as preterm labor, no controlled trials support the use of prophylactic
progesterone in the treatment of cervical incompetence.
A 32-year-old G3P0 woman presents to the clinic for preconception counseling. Her prior three pregnancies resulted
in first trimester losses. Which of the following tests should be ordered for this patient?
a) Adrenal stimulation test
b) Pelvic MRI (Magnetic Resonance Imaging)
c) Lupus anticoagulant test
d) Clomiphene citrate-FSH challenge test
e) Serum uric acid
Incorrect!!! It is important to rule out systemic disease in a patient with recurrent abortion (three successive
first trimester losses.) Testing for lupus anticoagulant, diabetes mellitus and thyroid disease is commonly
performed. Evaluation of maternal and paternal chromosomes should also be performed. Infectious causes
should also be considered. Uterine imaging to exclude a septum or other anomaly is routinely done using
hysteroscopy or hysterography. There is no role for Clomiphene citrate-FSH challenge or serum uric acid
testing in the evaluation of this patient.
A 26-year-old G2P0 woman presents for counseling following manual vacuum aspiration of an 8-week missed
abortion. The patient asks whether an uncomplicated first trimester pregnancy termination three years ago might
have predisposed her to the subsequent spontaneous abortion. What are the patient’s risks associated with the prior
surgical abortion in the first trimester?
a) Does not predispose the patient to subsequent spontaneous abortion
b) Increases the risk of spontaneous abortion two-fold compared to a control group of pregnant women carrying
pregnancies to term
c) Predisposes the patient to primary infertility, but does not increase the risk of spontaneous abortion once
implantation has occurred
d) Increases the likelihood of subsequent pregnancy loss in both the first and second trimesters
e) Increases the likelihood of spontaneous abortion and future delivery complications
Incorrect!!! Neither controlled trials nor surveillance data support the contention that a single, prior first
trimester surgical abortion increases the risk of subsequent first trimester pregnancy loss. Indeed, first
trimester surgical abortion confers no subsequent obstetric disadvantage, particularly when compared with
an appropriate control population. The clinician should reassure this patient that first trimester spontaneous
abortion is a common occurrence and that she has not caused this missed abortion.
Obstetrics - Exam 17a
A 28-year-old G0 woman presents to your office for preconception counseling. She has a history of Type I diabetes,
diagnosed at age 6, and uses an insulin pump for glycemic control. She has a history of proliferative retinopathy
treated with laser. Her last ophthalmologic examination was 3 months ago. Her last hemoglobin A1C (glycosylated
hemoglobin level) 6 months ago was 9.2%. Which of the following complications is of most concern for her planned
pregnancy?
a) Oligohydramnios
b) Fetal growth restriction
c) Fetal cardiac arrhythmia
d) Group B Streptococcal infection
e) Post term pregnancy
Correct!!! The patient with Type I diabetes is at risk for many pregnancy complications. In women with
insulin-dependent diabetes, the rates of spontaneous abortion and major congenital malformations are both
increased. The risk appears related to the degree of metabolic control in the first trimester. Overt diabetic
patients are also at an increased risk for fetal growth restriction, although fetal macrosomia may also occur.
The latter becomes a greater concern as in this patient, with longer-term diabetes and vascular
complications, such as retinopathy. Diabetics also have increased risk for polyhydramnios, congenital
malformations (cardiovascular, neural tube defects, and caudal regression syndrome), preterm birth and
hypertensive complications.
A 36-year-old G2 P1 woman presents for her first prenatal visit at 11 weeks gestation. She has a 2-year history of
chronic hypertension, for which she is being treated with lisinopril and labetalol. In addition, she has
hypothyroidism treated with levothyroxine, and recurrent herpes, for which she is on chronic acyclovir suppressive
therapy. She takes amitriptyline for migraine headaches. Which of these medications is contraindicated in
pregnancy?
a) Levothyroxine
b) Labetalol
c) Acyclovir
d) Lisinopril
e) Amitriptyline
Correct!!! Amitriptyline, levothyroxine, labetalol and acyclovir are medications that are frequently used in
pregnancy and generally are felt to have acceptable safety profiles. The use of angiotensin converting
enzyme inhibitors, such as Lisinopril, beyond the first trimester of pregnancy has been associated with
oligohydramnios, fetal growth retardation and neonatal renal failure, hypotension, pulmonary hypoplasia,
joint contractures and death. Amitriptyline is used in pregnancy to treat migraine headaches and has a good
safety profile.
A 19-year-old G1 woman at 36 weeks gestation presents for her first prenatal visit, stating she was recently
diagnosed with HIV. She was tested because her former partner recently tested positive as well. The HIV Western
Blot is positive. The CD4 count is 612 cells/µl. The viral load is 9,873 viral particles per ml of patient serum. Which
of the management options would best decrease the risk for perinatal transmission of HIV?
a) Treatment with intravenous Zidovudine at the time of delivery
b) Treatment of the newborn with oral Zidovudine only if HIV-positive
c) One week maternal treatment with Zidovudine now
d) Cesarean section in second stage of labor
e) Single drug therapy to minimize drug resistance
Correct!!! Antiretroviral therapy should be offered to all HIV-infected pregnant women to begin maternal
treatment as well as to reduce the risk of perinatal transmission regardless of CD4+ T-cell count or HIV
RNA level. The baseline transmission rate of HIV to newborns can be reduced from about 25% to 5-8%
with a protocol of oral zidovudine beginning at 14 weeks, continuing through delivery with intravenous
zidovudine in labor, and zidovudine treatment for the neonate. Cesarean section prior to labor can further
reduce this rate to 2% (although the benefit is less clear in women with viral loads <1,000 particles per ml.)
Multiple agents should be used in pregnancy to minimize the development of drug resistance.
Paramedics bring your patient, a 24-year-old G3P0 woman at 26 weeks, to the hospital. Her husband found her
shivering and barely responsive. Two days prior, the patient noted that she was feeling sick, with a slight cough. She
was having back pain at the time, but thought it was probably normal for pregnancy. Her pregnancy has been
uncomplicated except for the recent diagnosis of gestational diabetes. On exam, her temperature is 37.6; pulse is
160; and blood pressure is 68/32; respiratory rate is 32. Oxygen saturation is 82% on room air. There is no apparent
fundal tenderness, although the patient exhibits pain with percussion of the right back. Fetal heart tones are not
audible. There is no evidence of vaginal bleeding. Extremities are cool to touch. Pertinent labs include white blood
cell count of 24,000; hemoglobin 9.5; hematocrit 27%. Urine microscopic analysis shows many white blood cells.
What is the most likely etiology for this patient’s disease?
a) Abruptio placentae
b) Pyelonephritis
c) Diabetic ketoacidosis
d) Chorioamnionitis
e) Pneumonia
Correct!!! This is a patient in septic shock. The most common cause of sepsis in pregnancy is acute
pyelonephritis. Given the absence of bleeding, the clinical picture is not suggestive of placental abruption.
Diabetic ketoacidosis is unusual in gestational diabetic patients. Chorioamnionitis and pneumonia may both
lead to sepsis, but are less important causes than is pyelonephritis, and are not suggested by the clinical
picture.
A 24-year-old G2P1 woman at 18 weeks with a history of asthma presents to the office with worsening symptoms,
needing to use her inhaler more frequently. The symptoms began with the pregnancy and have gradually increased.
She is using her albuterol inhaler as needed, recently 3 times a day. She denies any illness or fever. She has had
asthma since she was a child. On exam, the patient appears comfortable. Her temperature is 37.4 and respiratory rate
is 18. Auscultation of the lungs shows good air movement with mild scattered end expiratory wheezes. There are no
rales or bronchial breath sounds. Which of the following is the most appropriate next step in the management of this
patient?
a) Oral theophylline
b) Subcutaneous terbutaline
c) Inhaled corticosteroids
d) Oral Zafirlukast (leukotriene inhibitor)
e) Antibiotic treatment
Incorrect!!! This patient has worsening asthma in pregnancy. The incidence of asthma in pregnancy is
approximately 1%, with 15% having one or more severe attacks. Asthma generally worsens in 40% of
pregnant patients. One of the indications for moving to the next line of treatment includes the need to use
short-acting beta agonists more than twice a week. The appropriate choice for her treatment would be
inhaled corticosteroids or cromolyn sodium. Theophylline would be used in more refractory patients.
Subcutaneous terbutaline and systemic corticosteroids would be used in acute cases. Zafirlukast, a
leukotriene modifier that inhibits their synthesis, is not effective for acute disease. There is little experience
with their use in pregnancy, thus the safety of zafirlukast in pregnancy is not well established. Antibiotic
treatment is only used when a pulmonary infection is diagnosed.
A 34-year-old G4P3 woman at 19 weeks presents to the emergency department with chest pain, palpitations and
sweating, which began 2-3 hours ago. On further questioning, she states that she has been very anxious lately and is
not sleeping well, which she attributes to the pregnancy. She reports that she has lost 40 pounds in the last year
without trying. She denies significant medical problems. On exam, the patient appears diaphoretic and anxious, her
eyes are wide open, prominent, and you can easily see the sclera surrounding the pupil. Her temperature is 38.1;
pulse is 132; and her blood pressure is 162/84. Height is 1.75 meters (70”) and weight is 58 kg (128 lb.). Her thyroid
is palpably enlarged, with an audible bruit. Electrocardiogram shows sinus tachycardia. Remaining labs are pending.
Which of the following therapies is contraindicated at this time?
a) Radioactive iodine (131I)
b) Propylthiouracil (PTU)
c) Propranolol
d) Inorganic iodide
e) Intravenous fluid replacement
Correct!!! This is a patient in thyroid storm an acute, life-threatening, hypermetabolic state. Radioactive
iodine (I-131) concentrates in the fetal thyroid and may cause congenital hypothyroidism, so it should not
be deliberately used in pregnancy. Acute treatment of thyroid storm may include thioamides, (i.e. PTU)
propranolol, sodium iodide and dexamethasone. Oxygen, digitalis, antipyretics and fluid replacement may
also be indicated. Maternal mortality with thyroid storm exceeds 25%.
An 18-year-old G1 woman presents for prenatal care at 16-weeks gestation without complaints. The patient denies
any history of sexually transmitted disease, although admits to a history of multiple sex partners in the last year, with
irregular use of condoms. She is allergic to penicillin, which causes anaphylaxis. Physical exam is unremarkable.
Pertinent labs: Rapid plasma reagin test (RPR) positive (titer = 32); fluorescent treponemal antibody absorption test
(FTA-ABS) is positive. Which of the following is the correct treatment?
a) Oral Erythromycin
b) Intravenous Erythromycin
c) Desensitization and Penicillin
d) Oral Doxycycline
e) Cefazolin
Correct!!! This patient has syphilis, and the fluorescent treponemal antibody absorption test (FTA-ABS)
confirms the diagnosis. The transmission rates for primary and secondary disease are approximately 5080%. There are no proven alternatives to penicillin therapy during pregnancy and Penicillin G is the
therapy of choice to treat syphilis in pregnancy. Women with a history of penicillin allergy can be skin
tested to confirm the risk of immunoglobulin E (IgE)-mediated anaphylaxis. If skin tests are reactive,
penicillin desensitization is recommended and is followed by benzathine penicillin G treatment.
Erythromycin has an 11% failure rate. Doxycycline is contraindicated in pregnancy. Cefazolin is
commonly used to treat urinary tract infections.
A 34-year-old G1 woman presents for prenatal care. She has a history of diabetes diagnosed at age 12. She has had
no complications (such as retinopathy or cardiovascular disease) from the diabetes. What is her diabetes class using
the White Classification?
a) A1
b) B
c) C
d) D
e) F
Incorrect!!! This patient is a Class D diabetic, having had her disease for greater than 20 years. Patients
with Class A (1 or 2) have gestational diabetes, diagnosed only during pregnancy. Patients with Class B are
diagnosed over 20 years of age, and have been diagnosed for less than 10 years duration. Class C patients
are diagnosed before 20 years, having carried the diagnoses for more than 10 years. Patients with Class F
have nephropathy, with diagnosis at any age.
A 22-year-old G4P1 woman at 26 weeks presents with a postcoital musty odor and increased milky, gray-white
discharge for the last week. This was an unplanned pregnancy. She had her first pregnancy at age 15. She reports
that she has no new sex partners, but the father of the baby may not be monogamous. On exam, there is a profuse
discharge in the vaginal vault, which covers the cervix. Pertinent labs: Wet mount: pH >4.5 and whiff test positive.
Microscopic exam reveals clue cells, but no trichomonads or hyphae. Which of the following is the most appropriate
next step in the management of this patient?
a) Delay treatment until postpartum
b) Treat her partner
c) Treat her now
d) Treat her and her partner
e) No treatment necessary
Correct!!! The patient has bacterial vaginosis. All symptomatic pregnant women should be tested and if
positive treatment should be not be delayed, because treatment has reduced the incidence of preterm
delivery. The optimal regimen for women during pregnancy is not known, but the oral metronidazole
regimens are probably equally effective. The patient’s partner is not usually treated with this condition.
A 33-year-old G2P1woman at 8 weeks presents to clinic. This is an unplanned pregnancy. She had planned a tubal
ligation 6 years ago, when she was diagnosed with pulmonary hypertension, but was unable to have the procedure.
She states her pulmonary hypertension has been stable, but she gets short of breath when climbing stairs. She sleeps
on one pillow at night. What is the concern for her during this pregnancy?
a) There are no additional concerns compared to a normal pregnancy
b) She will need a Cesarean section at delivery
c) Her baby is at increased risk for pulmonary hypoplasia
d) The mother’s mortality rate is above 25%
e) Epidural analgesia is contraindicated.
Incorrect!!! Among women with cardiac disease, patients with pulmonary hypertension are among the
highest risk for mortality with pregnancy. These women have a 25-50% risk for death with pregnancy.
Management of labor and delivery is particularly problematic. These women are at greatest risk when there
is diminished venous return and right ventricular filling, which is associated with most maternal deaths.
Similar mortality rates are seen in aortic coarctation with valve involvement and Marfan syndrome with
aortic involvement. The baby is not at increased risk of pulmonary hypoplasia or Marfan’s due to the
mother’s condition.
Obstetrics - Exam 17b
A 19-year-old G1P0 woman at 18 weeks gestation presents with a 3-month history of palpitations and intermittent
chest pain. Physical examination reveals a pulse of 96 and grade II/VI systolic ejection murmur with a click. The
ECG shows normal rate and rhythm and an echocardiogram is ordered. Which of the following is the best treatment
in the management of this patient?
a) anxiolytics
b) β-blockers
c) calcium-channel blockers
d) digitalis
e) no treatment needed at this time
Incorrect!!! Most women with mitral valve prolapse are asymptomatic and diagnosed by routine physical
examination or as an incidental finding at echocardiography. The small percentages of women with
symptoms have anxiety, palpitations, atypical chest pain, and syncope. For women who are symptomatic,
b-blocking drugs are given to decrease sympathetic tone, relieve chest pain and palpitations, and reduce the
risk of life-threatening arrhythmias. Because she is symptomatic, the option of no treatment is not correct.
A 27-year-old G1P0 woman at 32 weeks gestation presents complaining of cough, fever, chest pain, and dyspnea.
Physical examination reveals the following: Vital Signs – Pulse 108/min, Temperature 38.6°C, RR 22 and right
lower lobe bronchial breathe sounds. Which of the following tests would be most appropriate for making a diagnosis
for this patient?
a) complete blood cell count
b) mycoplasma-specific immunoglobulin G
c) urinalysis for pneumococcal antigen
d) chest x-ray
e) Pulmonary function tests
Incorrect!!! This woman presents with classic symptoms and findings for pneumonia. The typical
symptoms include cough, dyspnea, sputum production, and pleuritic chest pain. Mild upper respiratory
symptoms and malaise usually precede these symptoms, and mild leukocytosis is usually present. Chest
radiography is essential for diagnosis, although radiographical appearance does not accurately predict the
etiology of the pneumonia. Pulmonary functions test are not indicated in the work up of uncomplicated
pneumonia.
An 18-year-old nulliparous African-American woman was admitted 4 days ago because of back pain, chills and
fever. She has been taking broad spectrum antibiotics but continues to have spiking fever ranging from 38.9 to
39.6°C. Work-up reveals a right ureteral obstruction secondary to calculi. Which of the following is the most
appropriate next step in the management of this patient?
a) aggressive hydration
b) change antibiotics
c) continue present antibiotics
d) pass a double-J ureteral stent
e) perform percutaneous nephrostomy
Correct!!! Renal infection is the most common serious medical complication of pregnancy. Initially
aggressive intravenous hydration is given to ensure adequate urinary output. Antimicrobials are begun
promptly after diagnosis. The majority of patients are afebrile by 72 hours. If there is no clinical
improvement by 72 hours, further evaluation is warranted including sonography to look for urinary tract
obstruction (abnormal ureteral or pyelocaliceal dilatation) or calculi. Obstruction can be relieved by
cystoscopic placement of a double-J ureteral stent unless long-term stenting is foreseen then percutaneous
nephrostomy is indicated.
A 24-year-old G1P0 woman at 12 weeks gestation presents for prenatal care. She is 5 ft 4” tall and weighs 220
pounds. She wants to know if there is an increased risk on her pregnancy because of her size. You correctly tell her
that the most common pregnancy problem identified in obese women is:
a) hypertension
b) preterm labor
c) post-term pregnancy
d) small for gestational age
e) shoulder dystocia
Incorrect!!! The Body mass index (BMI) is equal to a person's weight divided by their height. The National
Heart, Lung, and Blood Institute identify a normal BMI is 18.5 to 24.9 kg/m2; overweight is a BMI of 25 to
29.9 kg/m2; and obesity is a BMI of 30 kg/m2 or greater. Obesity is further categorized as class I (BMI: 30
to 34.9 kg/m2), class II (BMI: 35 to 39.9 kg/m2), and class III (BMI: 40-plus kg/m2). Increased maternal
morbidity results from obesity and includes chronic hypertension, gestational diabetes, preeclampsia, fetal
macrosomia, as well as higher rates of Cesarean delivery and postpartum complications. This patient’s BMI
is over 38 so she is a class II and has over a 7-fold increase risk for preeclampsia and a 3-fold risk for
hypertension.
A 16-year-old G1P0 African-American woman at 8 weeks gestation presents for prenatal care. She reports
occasional spotting but denies pain or fever. The laboratory reports hemoglobin of 8 g/dl and a peripheral smear
reveals hypochromia and microcytosis. Which of the following is the most likely diagnosis for this patient?
a) a. sickle cell anemia
b) b. folate deficiency
c) iron deficiency
d) β-thalassemia
e) acute blood less
Correct!!! The two most common causes of anemia during pregnancy and the puerperium are iron
deficiency and acute blood loss. Classical morphological evidence of iron-deficiency anemia is erythrocyte
hypochromia and microcytosis. Serum ferritin levels are lower than normal and there is no stainable bone
marrow iron on examination of a bone marrow aspirate.
A 27-year-old G1P0 woman at 22 weeks gestation with SLE presents complaining of malaise, some joint aches, and
fever. Physical examination reveals the following: Vital Signs – Pulse 88, Temperature 37.6°C, RR 22, and BP
150/110 (baseline is 100/70.) Laboratory analysis reviews 1 + proteinuria and AST/ALT 35 and 28 respectfully.
You suspect a lupus flare. Which of the following is the usual first-line therapy for severe manifestations of SLE in
pregnancy?
a) steroids
b) nonsteroidal anti-inflammatory drugs (NSAIDs)
c) azathioprine
d) cyclophosphamide
e) magnesium sulfate
Correct!!! Lupus is notoriously variable in its presentation, course, and outcome. Clinical manifestations
include malaise, fever, arthritis, rash, pleuropericarditis, photosensitivity, anemia, and cognitive
dysfunction. A significant number of patients have renal involvement. There is no cure, and complete
remissions are rare. Mild disease may be disabling because of pain and fatigue. Nonsteroidal antiinflammatory drugs are used to treat arthralgia and serositis. Severe disease is managed with
corticosteroids.
A 34-year-old G2P1 woman comes to see you at 18 weeks gestation because of a lump in her left breast. Fine needle
aspiration reveals adenocarcinoma. Which of the following is NOT a recommended therapy for breast cancer during
pregnancy?
a) Wide local excision biopsy
b) Modified radical mastectomy
c) Total mastectomy and node dissection
d) chemotherapy
e) radiotherapy
Incorrect!!! There is no doubt that breast cancer is more aggressive in younger women. Whether it is more
aggressive during pregnancy in young women is debatable. Slight delays (1 to 2 months) in clinical
assessment, diagnostic procedures, and treatment of pregnant women with breast tumors are common.
Approximately 30 percent of pregnant women with breast cancer have stage I disease, 30 percent have
stage II, and 40 percent stages III or IV. Many clinical reports maintain that when breast cancer is
diagnosed during pregnancy, the regional lymph nodes are more likely to contain microscopic metastases.
Surgical treatment may be definitive for breast carcinoma during pregnancy and in the absence of
metastatic disease a wide excisional biopsy, modified radical mastectomy, or total mastectomy with
axillary node staging can be performed. Non-pregnant women receive adjunctive radiotherapy with breastconserving surgery. However, this is not recommended during pregnancy due to sizeable abdominal scatter
placing the fetus at significant risk for excessive radiation.
A 27-year-old G1P0 woman at 14 weeks gestation presents with a 2-month history of insomnia, feeling down, and
unintentional weight loss. Symptoms began after the unexpected death of her father. She is not excited about this
pregnancy and reports no suicide ideation. Physical examination reveals a woman of stated age with a flat affect.
After evaluation, you decide to begin treatment. Which of the following is contraindicated in this patient?
a) Paroxetine (Paxil)
b) Sertraline (Zoloft)
c) Fluoxetine (Prozac)
d) Nortriptyline (Norpress)
e) Bupropion (Wellbutrin)
Incorrect!!! This patient has classic depression. The most commonly used antidepressants are the selective
serotonin reuptake inhibitors (SSRIs.) One SSRI, Paroxetine (Paxil) has recently been changed to a
category D drug because of the increased risk of fetal cardiac malformations and persistent pulmonary
hypertension. The older SSRI compounds, fluoxetine and sertraline, have not been reported to cause early
pregnancy loss or birth defects in animals or in humans. Because these agents have few side effects
compared with other antidepressants, they are a good choice for pregnant women. Tricyclic antidepressants
have a long record of use in pregnancy and there is no increase in the rate of fetal malformation. Bupropion
is not an MAO inhibitor, nor is it an SSRI and a report by the Bupropion Pregnancy Registry reports no
unusual effects in 90 exposed pregnancies.
An 18-year-old nulliparous woman presents for her 32 week visit. She complains of intense itching for the past 2
weeks and cannot stop scratching her arms, legs, and soles of her feet. She has tried over the counter lotions with no
relief. She also states that her family noticed she is slightly yellow. Her vital signs are normal and there are general
excoriations over her arms and legs. Which of the following is the best treatment in the management of this patient?
a) Aggressive hydration
b) Antivirals such as Acyclovir
c) Antihistamines
d) Cholestyramine
e) Ursodeoxycholic acid
Incorrect!!! This patient has pruritus gravidarum, a common pregnancy-related skin condition that is a mild
variant of intrahepatic cholestasis of pregnancy. There is retention of bile salt, and as serum levels increase
they are deposited in the dermis. This, in turn, causes pruritus. The skin lesions are secondary to scratching
and excoriation. Antihistamines and topical emollients may provide some relief and should be used
initially. Cholestyramine causes decreased absorption of fat-soluble vitamins and may be effective in 50 to
70 percent of patients. Ursodeoxycholic acid relieves pruritus and lowers serum enzyme levels but is not
the first line of therapy. Another agent reported to relieve the itching is the opioid antagonist naltrexon.
A 27-year-old G2P1 woman presents to the emergency room complaining of fever, nausea, vomiting, and midabdominal pain for the last 24 hours. For the last 12 hours, she has had no appetite. She has been healthy, but reports
that her 3-year-old son has had diarrhea for 2 days. Physical examination reveals a BP 100/60, P 88 beats/minute,
RR 18, and temperature 38.8°C degrees. Abdominal examination reveals decreased bowel sounds and tenderness
more pronounced on the right than the left. Which of the following is the next best step in the management of this
patient?
a) Check a complete blood count
b) Abdominal and pelvic ultrasound
c) Plain abdominal radiograph
d) Helical computed tomography
e) Graded compression ultrasound
Incorrect!!! Suspected appendicitis is one of the most common indications for surgical abdominal
exploration during pregnancy. The diagnosis of appendicitis is more difficult to make in pregnancy because
anorexia, nausea, and vomiting that accompany normal pregnancy are also common symptoms of
appendicitis. In addition, the enlarged uterus shifts the appendix upward and outward toward the flank, so
that pain and tenderness may not be located in the right lower quadrant. Appendicitis is easily confused
with preterm labor, pyelonephritis, renal colic, placental abruption, or degeneration of a uterine myoma.
Peritonitis and appendiceal rupture are more common during pregnancy. The diagnosis is made based on
clinical findings and graded compression ultrasonography that is sensitive and specific especially before 35
weeks gestation. This noninvasive procedure should be considered first in working up suspected acute
appendicitis. Laboratory evaluation may not be helpful since leukocytosis in pregnancy can be as high as
16,000 cells/mL with bandemia present, and still be considered a normal variant and not a clear indicator of
appendicitis or any other infection. Selective imaging of the appendix using helical computed tomography
may be a safe and potentially reliable tool to accurately identify appendiceal changes in appendicitis,
except that radiation exposure using this test is higher that graded compression ultrasonography. A plain
abdominal radiograph can be used to identify air fluid levels or free air but offers little diagnostic value for
appendicitis.
Obstetrics - Exam 18
A 17-year-old primigravida woman presents to her obstetrician for a routine visit at 37 weeks. She has no
complaints, but is found to have an initial blood pressure of 138/89 and 144/91 on a repeat reading. This reading is
noted to be a change from all of her previous visits, as her blood pressures have been in the 90/50 range since
initiating care at 8 weeks gestation. Her urinalysis is negative and her comprehensive metabolic panel and complete
blood count are normal. Which of the following is the most likely diagnosis in this patient?
a) Mild preeclampsia
b) Severe preeclampsia
c) Chronic hypertension
d) Gestational hypertension
e) Normotensive
Correct!!! Gestational hypertension is the correct diagnosis, since she lacks other defining criteria for
preeclampsia. She has normal labs, (specifically platelets, liver function tests) no proteinuria and no
symptoms. Since her blood pressures were normal prior to this visit, chronic hypertension can be excluded.
Up to one quarter of women with gestational hypertension will go on to develop preeclampsia.
A 17-year-old G1P0 woman at 34 weeks gestation with new onset hypertension was sent home to collect a 24-hour
urine sample. The laboratory reports a value of 440mg/dL/24hours as the result. The patient’s blood pressure is now
155/99 and she notes increased swelling in her face and hands over the last 2 days. Which of the following is the
most likely diagnosis in this patient?
a) Mild preeclampsia
b) Severe preeclampsia
c) Gestational hypertension
d) Pre-hypertension
e) Dietary non compliance with increased salt intake
Correct!!! This patient has met criteria for the diagnosis of mild preeclampsia based on her persistent
elevation of blood pressure and 24-hour urine results. The amount of protein excreted in the urine varies
throughout the day, therefore a sample is collected over a 24-hour time period. Twenty-four hour urine
protein values greater that 300mg are required for the diagnosis of mild preeclampsia. Values greater than
5000mg (or 5g) are required for the diagnosis of severe preeclampsia (assuming no other defining criteria
are present.) Blood pressure between 120/80 mmHg and 139/89 mmHg is termed prehypertension which is
indicative of developing hypertension in the future.
A 17-year-old G1P0 woman presents at 39 weeks with increased swelling in her face and hands over the last 2 days.
Her blood pressure is 155/99. A 24-hour urine collection shows 440mg of protein. What is the treatment for her
disease?
a) Magnesium sulfate
b) Management of fluids
c) Delivery
d) Furosemide
e) Hydralazine
Incorrect!!! Regardless of disease severity, the only definitive therapy for preeclampsia is delivery of the
fetus and placenta. This solution can occasionally be delayed in the setting of stable disease (mild or
severe) when it occurs at an extremely early gestational age. Fluid management must be monitored closely
in this person. Magnesium sulfate is the mainstay of therapy during labor and for 24 hours postpartum to
lower the seizure threshold. Low-dose aspirin may have some benefit in decreasing the risk of preeclampsia
in a subset of high-risk patients. Hydralazine is often the antihypertensive agent of choice for controlling
elevated blood pressures in the acute setting.
A 40-year-old G1P0 woman at 34 3/7 weeks gestation is brought in by ambulance after being found on the floor at
work having a grand mal seizure. Her co-workers explained that she should have been at home on bed rest for mild
preeclampsia. Her airway was secure. What is the initial treatment for her condition?
a) Magnesium sulfate
b) Valium
c) Hydantoin
d) Phenobarbital
e) Naloxone
Correct!!! Magnesium sulfate is the treatment of choice for eclampsia, which is her most likely diagnosis.
Valium, Hydantoin, Tiagabine, and barbiturates can also be used to treat seizures, but are not first-line
therapy for eclampsia. They can be added as second agents, or used if magnesium is contraindicated.
Naloxone (Narcan) is a drug used to counter the effects of opioid overdose, for example heroin or morphine
overdose, and is specifically used to counteract life-threatening depression of the central nervous system
and respiratory system.
A 42-year-old G5P2 woman at 36-weeks gestation is diagnosed with preeclampsia. Her previous pregnancy was
complicated by twins and preeclampsia at 36 weeks gestation. Which of the following conditions is not associated
with her increased risk for preeclampsia in this pregnancy?
a) Previous history of preeclampsia
b) Chronic hypertension
c) Multifetal pregnancy
d) Age
e) Previous spontaneous abortion
Incorrect!!! The incidence of preeclampsia is commonly cited to be about 5 percent and is markedly
influenced by parity. It is related to race and ethnicity and to genetic predisposition; environmental factors
likely play a role. Other risk factors for preeclampsia include a previous history of the disease, chronic
hypertension, multifetal pregnancy and molar pregnancy. In addition, patients at extremes of maternal age
or with diabetes, chronic renal disease, antiphospholipid antibody syndrome, vascular or connective tissue
disease or triploidy are at increased risk for developing preeclampsia. The previous history of spontaneous
abortion does not put the patient at increased risk.
A 22-year-old G1P1 woman is undergoing treatment with magnesium sulfate for severe preeclampsia. She was
delivered 10 hours previously via cesarean section for a non-reassuring fetal heart rate tracing. She has oliguria and
appears lethargic. On exam, no deep tendon reflexes can be appreciated. Her magnesium level is 11mEq/L. You
become concerned about magnesium toxicity at this level. What condition can magnesium toxicity at this level result
in?
a) Seizures
b) Paralysis
c) Respiratory depression
d) Pulmonary edema
e) Cardiac arrest
Incorrect!!! A therapeutic magnesium level is between 4-7 mEq/L. Loss of deep tendon reflexes occurs at a
level of 7-10 mEq/L. Respiratory depression may occur at levels above 12 mEq/L. Cardiac arrest may
occur at a level of 15 mEq/L. Seizures are prevented by the use of magnesium. Pulmonary edema can occur
with magnesium therapy, but is not related to toxicity from the drug.
A 31-year-old G3P0 woman at 27 2/7 weeks is undergoing expectant management for severe preeclampsia remote
from term. Her blood pressure is 155/100 on Aldomet 500mg three times a day. Her recent 24-hour urine had 6.6g
of protein. An ultrasound revealed a fetus with adequate growth, having an estimated fetal weight in the 10th
percentile. Her labs are normal, except for a uric acid of 8.0, hematocrit of 42% (increased from 37%,) and a platelet
count of 97,000. Which of these findings necessitates delivery at this time?
a) Elevated uric acid
b) Thrombocytopenia
c) Proteinuria
d) Poorly controlled blood pressures
e) Hemoconcentration
Incorrect!!! Thrombocytopenia <100,000 is a contraindication to expectant management of severe
preeclampsia remote from term (<32 weeks). Other contraindications include: inability to control blood
pressure with maximum doses of 2 antihypertensive medications, non-reassuring fetal surveillance, liver
function test elevated more than 2 times normal, eclampsia, persistent CNS (central nervous system)
symptoms and oliguria. Delivery should not be based on the degree of proteinuria. Although elevated uric
acid and hemoconcentration are markers of preeclampsia, they are not part of the diagnostic or
management criteria.
A 23-year-old G2P0 woman at 32 6/7 weeks presents to Labor and Delivery with acute nausea, vomiting and
epigastric pain. Her blood pressure is 145/90; she has 1+ protein on a urinalysis, and the following labs:
Hematocrit: 42%
White blood count: 11,000
Platelets: 42,000
Aspartate aminotransferase (AST): 291
Alanine aminotransferase (ALT): 344
Glucose: 100
Bilirubin: 0.4
Lipase: 11
Creatinine: 0.8
Uric acid: 7.7
Fibrinogen: 405.
Which of the following is the most likely diagnosis in this patient?
a) Mild preeclampsia
b) Hepatitis
c) HELLP syndrome
d) Cholecystitis
e) Acute fatty liver
Correct!!! HELLP syndrome is a disease process in the spectrum of severe preeclampsia. The acronym
stands for “hemolysis, elevated liver enzymes, low platelets” and can lead to swelling of the liver capsule
and possibly, liver rupture. It may or may not be accompanied by right upper quadrant pain. It is possible to
only have thrombocytopenia and elevated transaminases, without clear hemolysis (elevated bilirubin and
anemia) – especially if a diagnosis is made early. This patient does not have seizures and, therefore, does
not have eclampsia. The clinical scenario is not consistent with hepatitis or cholecystitis. Acute fatty liver
almost always manifests late in pregnancy. Symptoms develop over several days to weeks and include
malaise, anorexia, nausea and vomiting, epigastric pain, and progressive jaundice. In many women,
persistent vomiting in late pregnancy is the major symptom. About half of all women have hypertension,
proteinuria, and edema signs suggestive of preeclampsia. There is usually severe liver dysfunction with
hypofibrinogenemia, hypoalbuminemia, hypocholesterolemia, and prolonged clotting times. As AFL
worsens there is marked hypoglycemia.
A 36-year-old G5P4 woman with no prenatal care presented in active labor with a blood pressure of 170/105 and 3+
proteinuria. Fetal heart tones were found to be in the 170s with decreased variability and a sinusoidal pattern.
Resting uterine tone was noted to be increased and she was having frequent contractions (every 1-2 minutes.) The
patient complained of bright red vaginal bleeding for the past hour. Based on this history, what is the etiology of her
vaginal bleeding?
a) Uterine rupture
b) Placenta previa
c) Bloody show
d) Abruptio placenta
e) Cervical trauma
Incorrect!!! Although all the options above can result in third trimester vaginal bleeding, the most likely
cause in this patient is placental abruption. This diagnosis goes along with the tachysystole on tocometer
and evidence of fetal anemia (tachycardia and sinusoidal heart rate pattern) on the heart rate tracing.
Hypertension and preeclampsia are risk factors for abruption. She has no history of cervical trauma.
Your shift on Labor & Delivery has just begun and you have assumed the care of a 27-year-old P1 at 36 weeks with
severe preeclampsia. Her blood pressure is 200/105. She has so far received 2 doses of IV hydralazine to lower her
blood pressure. What diastolic blood pressure should you aim for in this patient?
a) 60-65 mm Hg
b) 66-69 mm Hg
c) 70-79 mm Hg
d) 80-89 mm Hg
e) 90-100 mm Hg
Incorrect!!! Treatment with an antihypertensive is indicated for diastolic blood pressures persistently
greater than 160 systolic and 105 diastolic. First-line agents include Hydralazine (a direct vasodilator) 5mg
IV followed by 5-10mg doses IV at 20-minute intervals (maximum dose = 40mg) or Labetalol (combined
alpha & beta-adrenergic antagonist) 10-20mg IV followed by 20mg, then 40mg, then 80mg IV every 10
minutes (maximum dose = 220mg). The goal is not a normal blood pressure, but to reduce the diastolic
blood pressure into a safe range of 90-100mm Hg to prevent maternal stroke or abruption, without
compromising uterine perfusion.
Obstetrics - Exam 19
A 28-year-old G2P1 woman presents at 20 weeks gestation transferred from her general practitioner. As an
obstetrician-gynecologist, you are asked to evaluate and care for this patient who has an anti-D titer of 1:64. This
pregnancy has been complicated by scant vaginal bleeding at 7 weeks and an abnormal maternal serum alpha
fetoprotein (MSAFP), with increased risk for Down syndrome, but had a normal amniocentesis: 46, XX. Her
previous obstetric history is significant for a Cesarean section at 34-weeks due to placental abruption and fetal
distress. Prenatal labs at 6 weeks: Rh-negative. Antibody screen positive: anti-D 1:64. Which is the most likely
source of the Rh sensitization?
a) ABO incompatibility
b) Placental abruption
c) Amniocentesis
d) Abnormal maternal serum alpha fetoprotein (MSAFP)
e) First trimester bleeding
Correct!!! This patient was sensitized during her first pregnancy that was complicated by abruption and
required Cesarean delivery. Transplacental hemorrhage of fetal Rh-positive red blood cells into the
circulation of the Rh-negative mother may occur following a number of obstetric procedures and
complications, such as amniocentesis, chorionic villus sampling, spontaneous/threatened abortion, ectopic
pregnancy, dilation and evacuation, placental abruption, antepartum hemorrhage, preeclampsia, cesarean
section, manual removal of the placenta and external version.
A 24-year-old G1P1 Rh-negative woman has just delivered a healthy term infant who is Rh-positive. She is
concerned about her risk for Rh sensitization. What is the most likely volume of feto-maternal hemorrhage that
occurred during this pregnancy?
a) <0.1cc
b) 5 cc
c) 10 cc
d) 20 cc
e) 50 cc
Incorrect!!! While 75% of all gravidas have evidence of transplacental hemorrhage during pregnancy or
immediately after delivery, 60% of these patients have <0.1 cc of fetal blood in the maternal circulation.
Less than 1% of women have >5 cc and less than 0.25% have >30cc of fetal blood in the circulation. The
incidence and size of transplacental hemorrhage increases as pregnancy advances. During the second
month of gestation, 5-15% of women will have evidence of feto-maternal hemorrhage. By the third
trimester, this number increases to 45% of patients. Less than 0.1 cc of fetal blood can sensitize a patient.
A 24-year-old G2P1 Rh-negative woman is found at 18 weeks gestational age to have anti-D antibodies. In
discussing the risks of Rh immunization with her, you tell her that her fetus may be at increased risk of significant
perinatal disease including fetal anemia. Which of the following non-invasive test can detect severe fetal anemia?
a) Serial fetal ultrasounds for rate of growth
b) Biophysical profile
c) Amniotic fluid index
d) Umbilical artery blood flow
e) Middle Cerebral Artery peak systolic velocity
Correct!!! Amniocentesis and cordocentesis have been used for many years to diagnose fetal anemia due to
red cell alloimmunization. These techniques, however, are invasive and many complications are associated
with their use. Noninvasive diagnosis of fetal anemia has been performed with Doppler ultrasonography.
The use of middle cerebral artery peak systolic velocity in the management of fetuses at risk for anemia
because of red cell alloimmunization has emerged as the best test for the noninvasive diagnosis of fetal
anemia. All the other listed tests are for assessment of fetal well being and non-specific to detect fetal
anemia.
A 24-year-old G2P1 Rh-negative female patient is found at 10 weeks gestation to have anti-D antibodies. You
follow her closely during this pregnancy and order serial ultrasound examinations. Which of the following fetal
ultrasound findings would be most explained by the presence of Rh disease?
a) Meconium
b) Fetal bladder obstruction
c) Oligohydramnios
d) Pericardial effusion
e) Placenta previa
Correct!!! Fetal hydrops is easily diagnosed on ultrasound. It develops in the presence of decreased hepatic
protein production. It is defined as a collection of fluid in two or more body cavities, such as ascites,
pericardial and/or pleural fluid and scalp edema. On occasion, when extramedullary hematopoiesis is
extensive, there will be evidence of hepatosplenomegaly. Placentomegaly (placental edema) and
polyhydramnios are also seen on ultrasound. Meconium, fetal bladder obstruction, oligohydramnios and
placenta previa do not fit the clinical scenario.
A 24-year-old G1P0 woman presents at 32 weeks gestational age with vaginal bleeding most likely caused by
placental abruption. She receives a standard dose of 300 micrograms of RhoGAM. What amount of fetal blood is
neutralized by this dose?
a) 10 cc
b) 20 cc
c) 30 cc
d) 40 cc
e) 50 cc
Correct!!! 30 cc of fetal blood is neutralized by the 300ug dose of RhoGAM. This is equivalent to 15 cc of
fetal red blood cells. At 28-weeks gestation, 300 micrograms of Rh-immune globulin is routinely
administered after testing for sensitization with an indirect Coombs’ test. Administration is given following
amniocentesis at any gestational age.
A 28-year-old G1P0 woman at 28-weeks gestation presents to the clinic for a routine prenatal visit. She is Rhnegative and you order RhoGAM. Which of the following is the current recommendation for preventing Rh
isoimmunization?
a) Routine administration for every Rh-sensitized woman at term
b) Administration for Rh-negative patients with no Rh antibodies at 28 weeks
c) Administration for every Rh-negative woman without evidence of Rh immunization who delivers an Rh-negative
infant
d) Routine administration for all Rh-negative patients during first trimester
e) Administration following all gynecologic procedures, such as endometrial biopsy
Correct!!! RhoGAM (Anti-D-immunoglobulin) is administered to Rh-negative women to prevent
isoimmunization. Each dose provides 300 mg of D-antibody and is given to the D-negative non-sensitized
mother to prevent sensitization after any pregnancy-related events that could result in fetal-maternal
hemorrhage. Up to 2 percent of women with a spontaneous abortion and 5 percent of those undergoing
elective termination become isoimmunized without D-immunoglobulin. The current recommendations for
Rh-negative women without evidence of Rh immunization is one prophylactic dose of RhoGAM
intramuscularly within 72 hours of delivering an Rh-positive baby, following spontaneous or induced
abortion, following antepartum hemorrhage, following amniocentesis or chorionic villus sampling, and
prophylactically at 28-weeks gestation (after an indirect Coombs’ test). If the father of the conceptus is
known to be Rh-negative, RhoGAM is not necessary, since the fetus will be Rh-negative and not at risk for
hemolytic disease. Rhogam is not indicated following gynecologic procedures.
A 24-year-old G1P0 woman presents at 32-weeks gestation with vaginal bleeding, most likely caused by placental
abruption. You want to give the patient an appropriate RhoGAM dose and determine the presence and degree of
fetal-maternal hemorrhage. How can you accurately and immediately estimate the degree of fetal-maternal
hemorrhage?
a) Delta OD 450 (optical density)
b) Fetal hemoglobin
c) Kleihauer-Betke test
d) Maternal hemoglobin
e) Fetal bilirubin
Incorrect!!! The Kleihauer-Betke test is an accurate and sensitive acid elution test. It has great value in
determining the incidence and size of fetal transplacental hemorrhage. In this test, using acid elution, the
mother’s red blood cells become very pale, while fetal cells, which contain a different form of hemoglobin,
remain stained. Simple comparative counts allow an estimate of whether a significant feto-maternal
transfusion has occurred.
A 24-year-old G2P1 woman has a fetus that is affected by Rh disease. At 28 weeks gestation, an amniocentesis was
performed. The amniotic fluid sample was sent for a delta OD450 (optical density deviation at 450 nm)
measurement. What does this value represent in the amniotic fluid?
a) Hemoglobin
b) Iron
c) Direct measure of the anti-D antibody titer
d) Ferritin
e) Bilirubin
Incorrect!!! In the presence of a severely erythroblastotic fetus, the amniotic fluid is stained yellow. The
yellow pigment is bilirubin, which can be quantified most accurately by spectrophotometric measurements
of the optical density between 420 and 460nm, the wavelength absorbed by bilirubin. The deviation from
linearity of the optical density reading at 450nm is due to the presence of heme pigment, an indicator of
severe hemolysis.
A 24-year-old G2P1 woman has a fetus that is affected by Rh disease. At 30 weeks gestation, the delta OD450
(optical density deviation at 450 nm) results plot on the Liley curve in Zone 3. Which of the following is the most
appropriate next step in the management of this patient?
a) Immediate cesarean delivery
b) Repeat amniocentesis in 3-5 days
c) Intrauterine intravascular fetal transfusion
d) Umbilical blood sampling
e) Maternal plasmapheresis
Incorrect!!! Values in Zone 3 of the Liley curve indicate the presence of severe hemolytic disease, with
hydrops and fetal death likely within 7-10 days, thus demand immediate delivery or fetal transfusion. At 30
weeks gestation, the fetus would benefit from more time in utero. Thus, an attempt should be made to
correct the underlying anemia. Intravascular transfusion into the umbilical vein is the preferred method.
Intraperitoneal transfusion is used when intravascular transfusion is technically impossible. If fetal hydrops
is present, the reversal of the fetal anemia occurs much more slowly via intraperitoneal transfusion.
Percutaneous umbilical blood sampling should not be used as a first-line method to evaluate fetal status.
Maternal plasmapheresis is used in severe disease when intrauterine transfusions are not possible.
A 24-year-old G2 P1 woman is sensitized to the D and C antigens despite receiving RhoGAM following her first
delivery. Which of the following statements best explains these findings?
a) The patient initiated her prenatal care late during the present gestation
b) The patient was sensitized during the previous pregnancy by receiving the RhoGAM
c) RhoGAM confers protection against the D and C antigens, but was inadequate in this case
d) The amount of feto-maternal hemorrhage was more than previously estimated
e) The cause is most likely idiopathic in this case
Incorrect!!! On rare occasion, an Rh-negative woman will subsequently be sensitized, despite prophylaxis.
The protection afforded by a standard RhoGAM administration is dose-dependent. One dose will prevent
Rh sensitization to an exposure of as much as 30 cc of Rh-positive red blood cells. With greater exposure,
there is only partial protection and Rh sensitization may occur as a result of failure to diagnose massive
transplacental hemorrhage. Alternatively, an Rh-negative woman may be sensitized in the latter part of
pregnancy or soon after delivery before the post-delivery prophylaxis dose is given. Inadvertent maternal
transfusion of Rh-positive blood may result in Rh sensitization to the D or another red blood cell antigen.
Patients may become sensitized if they did not receive RhoGAM following an episode of antenatal
bleeding or after an invasive procedure, such as amniocentesis or chorionic villus sampling. In addition,
RhoGAM only confers protection against the D antigen. Thereforee, despite administration of RhoGAM to
Rh-negative patients, they may still become sensitized to other red blood cell antigens.
Obstetrics - Exam 20
A 25-year-old G2P1 woman states her gestational age by a first trimester ultrasound is 16 weeks, 3 days. She reports
no complaints and is not yet feeling fetal movement. Her fundal height is just above the umbilicus and measures 22
cm. The MSAFP (maternal serum alpha fetoprotein) result is elevated. Which of the following is the most likely
cause for the abnormal MSAFP result?
a) Fetal trisomy
b) Dates in error
c) Twin gestation
d) Fetal gastroschisis
e) Fetal demise
Incorrect!!! Alpha fetoprotein (AFP) levels in twin gestations are elevated and should be roughly twice that
seen in singleton pregnancies. An additional clue to a possible diagnosis of twin gestation is the fundal
height exceeding gestational age in weeks. Other causes of elevated maternal serum AFP include errors in
dating, neural tube defects, pilonidal cysts, Cystic hygroma, Sacrococcygeal teratoma, fetal abdominal wall
defects, and fetal death.
A 35-year-old woman with a known twin gestation undergoes an ultrasound evaluation at 18 weeks. She would like
to know if her twins are identical or fraternal. Which ultrasound marker is suggestive of dizygotic (fraternal) twins?
a) Increased cerebral blood flow on Doppler velocimetry
b) Two separate placentas (anterior and posterior)
c) Dividing membrane thickness less than 1 mm
d) Different fetral weights
e) Twin-twin transfusion syndrome
Correct!!! Ultrasound markers suggestive of dizygotic (non-identical) twins include a dividing membrane
thickness greater than 2 mm, twin peak (lambda) sign, different fetal genders and two separate placentas
(anterior and posterior). The two different placental types in twin gestation are monochorionic and
dichorionic. Dizygotic conceptions always have dichorionic placentas. Monozygotic conceptions may have
either monochorionic or dichorionic placentation, depending upon the time of division of the zygote.
Diamniotic dichorionic placentation occurs with division prior to the morula state (within 3 days post
fertilization). Diamniotic monochorionic placentation occurs with division between days 4 and 8 postfertilization. Monoamniotic, monochorionic placentation occurs with division between days 8 and 12 post
fertilization. Division at or after day 13 results in conjoined twins. The ultrasound markers for
determination of chronicity described above have been used to assess risk for complications of pregnancy,
most notably the twin-twin transfusion syndrome.
A 29-year-old G0 woman presents to your office for an annual examination. She requests a referral to a reproductive
endocrinology specialist because she would like to conceive twins. She would like to expedite her childbearing by
having two infants in one pregnancy. What counseling do you tell her to help her make this decision?
a) The morbidity with multiple gestations is similar to singleton pregnancies
b) The twin infant death rate is much higher than that of singletons
c) The rate of cerebral palsy is only slightly increased in twin infants
d) The incidence of abnormal fetal growth is similar to singleton pregnancies
e) The incidence of prematurity is not a concern, since most twins do well
Correct!!! The twin infant death rate is five times higher than that of singletons. The epidemic of multiple
gestations resulting from assisted reproductive techniques is of great significance to individual parturients
and to society because of the major morbidities associated with twinning as well as with triplets and higher
order multiples. The risk for development of cerebral palsy in twin infants is five to six times higher than
that of singletons. One study, with dichorionic twins, monochorionic twins and singletons, showed that
twins had a higher incidence of IUGR (intrauterine growth restriction) than singletons. Fifty-eight percent
of twins deliver prematurely, with an average gestational age at delivery of 35 weeks. Twelve percent of
twins deliver very prematurely.
A 30-year-old G2P1 woman presents to your office for her first prenatal care visit after learning she is having twins.
She has surfed the Web and has a lot to discuss, especially about the risk of complications. Which of the following
obstetrical complications is more common in twins than in singleton gestations?
a) Postterm pregnancy
b) Congenital anomalies
c) Induction after 38 weeks
d) Macrosomia
e) Rh isoimmunization
Correct!!! The incidence of congenital anomalies is increased in twins, particularly monozygotic twins,
compared to singletons. The majority of twin pairs in which an anomaly is present will be discordant for
the anomaly. Twin gestations tend to deliver earlier than singleton gestations, with the average length of
twin gestation being 35-37 weeks. Postterm pregnancy (>41 weeks) is uncommon in twins. The optimal
length of twin gestation is a matter of some controversy. An observational study comparing perinatal
mortality among twin and singleton gestations showed that perinatal mortality reached a nadir at 37-38
weeks in twins and then increased. There have been no prospective studies to demonstrate that induction of
labor after 38 weeks in twin gestations improves perinatal outcome. Twins typically weigh less than
singletons of the same gestational age, but their weights usually remain within the normal range.
Macrosomia is, therefore, uncommon.
A 32-year-old G1P0 woman presents to your office at 10 weeks estimated gestational age, after an ultrasound
evaluation has revealed a diamniotic, dichorionic twin gestation. She is very concerned about the risk for preterm
delivery. Which intervention would you recommend as a possible means to reduce the risk of a preterm, lowbirthweight infant?
a) Bed rest
b) Cervical cerclage
c) Tocolytics starting at 24 weeks
d) Home uterine monitoring
e) Early, good weight gain
Incorrect!!! Although prematurity has been recognized as a major cause of morbidity and mortality among
twin gestations, interventions for prevention of prematurity have, in general, been unsuccessful. Studies
show that an adequate weight gain in the first 20 to 24 weeks of pregnancy is especially important for
women carrying multiples and may help to reduce the risk of having preterm and low-birth weight babies.
These pregnancies tend to be shorter than singleton pregnancies, and studies suggest that a good early
weight gain aids in development of the placenta, possibly improving its ability to pass along nutrients to the
babies. In a twin pregnancy, a gain of at least 24 pounds by the 24th week of pregnancy is often
recommended. Bed rest, long prescribed by obstetricians for the prevention of preterm birth, has never been
shown to be efficacious, and may be associated with thromboembolic complications. An observational
study of prophylactic cerclage for twin gestations failed to show any benefit. Tocolytic drugs for prevention
of preterm labor in asymptomatic women with twin gestations have not been shown to be effective. Home
uterine activity monitoring is another intervention that has been shown to be ineffective.
A 29-year-old G4P2 woman with no previous prenatal care presents to Labor and Delivery unit at 24 weeks 1 day
gestation with signs and symptoms of preterm labor. Her cervix is 3 cm dilated and 80% effaced. Fundal height is 30
cm and an ultrasound examination reveals a twin gestation. Estimated fetal weights on the twins are 850 gm and 430
gm respectively. The maximum vertical amniotic fluid pocket around the smaller twin is 1 cm; the maximum
vertical amniotic fluid pocket around the larger twin is 8 cm. Her last pregnancy was complicated by gestational
diabetes. Which diagnosis do you suspect?
a) Monozygotic dichorionic twinning
b) Twin-twin transfusion
c) Superfecundation
d) Dichorionic diamniotic twins
e) Gestational diabetes
Incorrect!!! Twin-twin transfusion syndrome is the result of an intrauterine blood transfusion from one twin
to the other. It occurs in monochorionic, monozygotic twins. Although monozygotic twins can have
dichorionic placentation, such twins are not at risk. Monozygotic twins with monochorionic, diamniotic
placentation or monochorionic, monoamniotic placentation are at risk. The donor twin is often smaller and
anemic at birth. The recipient twin is usually larger and plethoric at birth. Clues to the presence of the twintwin transfusion syndrome include the large weight discordance (although this is not necessary for
diagnosis), polyhydramnios around the larger (recipient) twin, and oligohydramnios around the smaller
(pump) twin. The two different placental types in twin gestation are monochorionic and dichorionic.
Monozygotic conceptions may have either monochorionic or dichorionic placentation, depending upon the
time of division of the zygote. Dizygotic conceptions always have dichorionic placentas. Diamniotic
dichorionic placentation occurs with division prior to the morula state (within 3 days post fertilization).
Diamniotic monochorionic placentation occurs with division between days 4 and 8 post fertilization.
Monoamniotic, monochorionic placentation occurs with division between days 8 and 12 post fertilization.
Division on or after day 13 results in conjoined twins. Acardiac twinning represents a syndrome
characterized by vascular anastomoses in combination with partial or complete lack of cardiac development
in one of the twins. Superfecundation is the fertilization of two or more ova from the same cycle by sperm
from separate acts of sexual intercourse.
A 29-year-old G4P2 woman was diagnosed with twin-twin transfusion syndrome when an ultrasound was
performed at 24 weeks gestational age. Which of the following is a complication of twin-twin transfusion
syndrome?
a) Hydrops in the surviving twin only
b) Neurologic sequelae in the survivors
c) Anemia in the recipient twin
d) Heart failure in the donor twin only
e) High perinatal mortality for donor twin only
Incorrect!!! Untreated severe twin-twin transfusion syndrome has a poor prognosis, with perinatal mortality
rates of 70-100%. Death in utero of either twin is common. Surviving infants have increased rates of
neurological morbidity, with increased risk of cerebral palsy for the surviving twin. Excessive volume can
lead to cardiomegaly, tricuspid regurgitation, ventricular hypertrophy and hydrops fetalis for the recipient
twin. The donor twin becomes anemic and hypovolemic, and growth is retarded. The recipient twin
becomes plethoric, hypervolemic and macrosomic. Either twin can develop hydrops fetalis. The donor twin
can become hydropic because of anemia and high-output heart failure.
A 29-year-old G4P2 woman was diagnosed with triplets when an ultrasound was performed at 24 weeks gestational
age. Which of the following is the most concerning complication for this multiple gestation?
a) Preeclampsia
b) Intrauterine growth restriction
c) Preterm birth
d) Gestational Diabetes
e) Abnormal placentation
Incorrect!!! Preterm delivery increases the risk of morbidity and mortality, increasing with higher orders of
multiples. Preterm birth occurs in over 50% of twin pregnancies, 90% of triplet pregnancies, and almost all
quadruplet pregnancies. While all the choices may occur with a multiple gestation, prematurity has the
most significant consequences as it is associated with an increased risk of respiratory distress syndrome
(RDS), intracranial hemorrhage, cerebral palsy, blindness, and low birth weight. Intrauterine growth
restriction, intrauterine death of one or more fetuses, miscarriage and congenital anomalies are all more
common with multiple gestations, as are the complications of preeclampsia, diabetes and placental
abnormalities.
A 34-year-old G4 woman presents at 36 weeks in labor. Her previous deliveries were normal spontaneous vaginal
deliveries at term, with the largest infant weighing 3400 grams. Twin A is breech with an estimated fetal weight of
2800 gm and twin B is vertex, with an estimated fetal weight of 3200 gm. Which of the following is an appropriate
delivery option for this patient?
a) Total breech extraction of twin A, vaginal delivery of twin B
b) External cephalic version for twin A, Vaginal delivery twin of B
c) Vaginal delivery of both, since twin B weighs more
d) Elective Cesarean Section
e) Vaginal delivery for Twin A and Cesarean delivery for Twin B
Incorrect!!! The optimal mode of delivery for twins in which the first twin is in the breech presentation is
by Cesarean section. Similar to singletons, if the first twin is breech problems can occur including head
entrapment and umbilical cord prolapse. When the presenting twin is vertex and twin B is not vertex,
controversy exists as to the optimal mode of delivery. A small randomized study comparing cesarean
delivery with vaginal delivery for vertex-non-vertex twins failed to show an advantage for Cesarean
delivery, but did not have statistical power to address rare neonatal morbidities. Some authors have
advocated external cephalic version for management of the second twin; however, observational studies
have not shown any advantage of this approach compared to total breech extraction.
A 34-year-old G1P0 woman presents to the office accompanied by her 32-year-old husband. She is thrilled that she
is now pregnant with twins. She has a history of polycystic ovarian syndrome and thought she would not be able to
have a baby. Her partner has a very large family, with twin second-cousins. She is aware that the rate of multiple
gestations in the United States has been increasing over the past 20 years and wants to discuss risk factors. Which of
the following is the most likely cause of twins in this patient?
a) Advanced maternal age
b) Obstetrical history
c) Assisted reproductive technology
d) Recent use of oral contraceptives
e) Paternal family history
Correct!!! Assisted reproduction has led to an increase number of multiple gestations. The rates of multiple
births after IVF (in-vitro fertilization) vary according to maternal age and the number of embryos
transferred. Transfer of multiple embryos is more likely to result in multiple gestations in younger women
than in older women. The frequency of multiple gestations increases with the use of ovulation inducing
drugs. The risk is in the 5-6% range, but varies depending on the drug and dosage regimen used. Dizygous
twinning results from the ovulation of multiple follicles and the rate of multiple gestation increases with
advancing maternal age. Elevated follicle-stimulating hormone correlates with dizygous multiple births. A
higher number of prior pregnancies and previous history of multiple births increases the chance of having a
multiple gestation. These do not apply here because the patient’s obstetrical history does not support this.
Dizygous twinning appears to have a genetic component and rates of dizygous twins vary according to
ethnicity, but are not related to paternal family history.
Obstetrics -Exam 21
A 24 year-old G4P0 presents to your office at 7 weeks gestation after two days of bleeding and cramping. She thinks
that she miscarried at home and did not bring in the tissue for pathologic evaluation. What is the karyotype most
likely to be found on chromosomal analysis?
a) Aneuploidy
b) Autosomal Trisomy
c) Down Syndrome (47, XY, +21)
d) Turner Syndrome (45, X)
e) Tetraploidy
Correct!!! Autosomal trisomy is the most common abnormal karyotype encountered in spontaneous
abortuses, accounting for approximately 40-50% of cases. The most common chromosomal aneuploidy
noted in abortuses is Trisomy 16. Triploidy accounts for approximately 15%, and tetraploidy for 5% of
cases. Monosomy X (45X, 0) is seen in 15-25% of losses.
A 34 year-old G1 is in a motor vehicle accident. While in the emergency department, the doctors order multiple xrays to evaluate her injuries. At what gestational age would the fetus be most susceptible to developing mental
retardation with sufficient doses of radiation?
a) 0-7 weeks
b) 8-15 weeks
c) 16-25 weeks
d) 26-30 weeks
e) 31-35 weeks
Incorrect!!! The risk of developing microcephaly and severe mental retardation is greatest between 8 and
15 weeks gestation. In 1990, The Committee on Biological Effects reported that no risk of mental
retardation has been documented with doses even exceeding 50 rad at less than 8 weeks or greater than 25weeks gestation.
A 21 year-old G1 presents to labor and delivery at 39 weeks gestation with a chief complaint of decreased fetal
movement over the last 2 days. An ultrasound shows a fetus with biometry consistent with 34 weeks gestation with
no cardiac activity. The head circumference and biparietal diameter are consistent with 37 weeks and the abdominal
circumference, femur and humerus lengths are all lagging by approximately 5 weeks. The amniotic fluid volume is
slightly decreased. No other abnormalities are identified. The patient’s medical history is notable for a deep venous
thrombosis which she had three years ago while she was using oral contraceptives. She had a reassuring quad
screen. She denies any history of fever or viral illnesses during the pregnancy. She works as a preschool teacher.
The patient had a fetal ultrasound at 20 weeks gestation. At that time all of the fetal anatomy was well-visualized
and no abnormalities were identified. Which of the following is the most likely explanation for the fetal demise in
this case?
a) Umbilical cord accident
b) Trisomy 18
c) Poorly controlled undiagnosed diabetes mellitus
d) Fetal parvovirus infection
e) Factor V Leiden mutation
Incorrect!!! This patient is most likely to have the autosomal dominant Factor V Leiden (FVL) mutation
based on her history. FVL is the most common inherited thrombophilic disorder affecting approximately
5% of Caucasian women in the United States. It is a point mutation which alters factor V making it
resistant to inactivation by protein C. The thrombophilic effect of a FVL mutation has been clearly
established. Heterozygosity for FVL is associated with a 5-10-fold increased risk of thrombosis, while
homozygosity is associated with an 80-fold increased risk. The FVL mutation is associated with obstetric
complications including stillbirth, preeclampsia, placental abruption and IUGR. Fetuses with trisomy 18 are
likely to have congenital anomalies that are detectable on prenatal ultrasound. Over 90% of cases of
trisomy 18 may be detected with the quad screen. A congenital parvovirus infection associated with a fetal
demise would likely cause hydrops in the fetus which would be identified on ultrasound. Although poorly
controlled diabetes mellitus and cord accidents are associated with fetal demise, they are not the most likely
etiologies in this patient whose presentation is classic for the FVL mutation.
A 39 year-old G4P1 at 36 weeks gestation presents to labor and delivery. Upon initial evaluation, no fetal heart
tones were noted on Doptone. Ultrasound confirms a stillbirth. Problems during the pregnancy include diagnosis of
an open neural tube defect, estimated fetal weight >90th percentile, polyhydramnios and a nonreactive NST (non
stress test) the week prior to admission. What is the most likely etiology of this stillbirth?
a) Uncontrolled hypertension
b) In-utero viral infection
c) Antiphospholipid antibody syndrome (APAS)
d) Uncontrolled diabetes
e) Untreated maternal hypothyroidism
Correct!!! Uncontrolled diabetes during organogenesis is associated with a high rate of birth defects. The
most common sites affected are the spine and the heart of the fetus although all birth defects are increased.
Fetuses in utero exposed to high levels of glucose transplacentally have increased growth and polyuria
resulting in an increase in the amniotic fluid. While some viral infections are also associated with
placentomegaly and polyhydramnios, the fetus will have normal or decreased growth depending on the
timing of the infection. Severe hypertension and active APAS is often associated with oligohydramnios and
intrauterine growth restriction. The risk of miscarriage and stillbirth is increased if hyperthyroidism goes
untreated, and the overall risks to mother and baby further increase if the disease persists or is first
recognized late in pregnancy.
A 20 year-old G1 presents to your office at 26-weeks gestation and is noted to have a fundal height of 20 cm. She
has had a normal pregnancy to date, with prenatal care starting at 7 weeks gestation. Her sonogram at 17 weeks was
normal. The patient does not have any pain and does not report any leakage of fluid or vaginal bleeding. What is the
most likely diagnosis to explain these findings?
a) Preterm spontaneous rupture of the membranes
b) Wrong gestational dating
c) Fetal demise
d) Uterine fibroids
e) Ovarian cyst
Incorrect!!! The fundal height is a measurement of the lower abdominal organs, including the uterus,
fallopian tubes and ovaries. A demise several weeks prior to the visit or with significant decrease in fetal
growth leading to demise would lead to a decrease in the fundal height measurement. Any significant
increase in the size of these structures (leiomyomata, tubal abscess, ovarian cysts) would lead to an increase
in the fundal height. Although rupture of membranes or an earlier gestational age can be associated with
size less than dates, the clinical history in this case does not support is as a possibility. It is unlikely she has
the wrong gestational age as her ultrasound at 17 weeks was normal.
A 26 year-old G1, with last menstrual period 10 weeks ago, presents to your office for her first prenatal visit. She
reports vaginal spotting for the last two days. You perform an ultrasound that shows an intrauterine pregnancy
consistent with 9 weeks gestation with no cardiac activity. She denies cramping or abdominal pain. What is the most
important laboratory test to check for this patient?
a) Quantitative beta-hCG
b) Maternal blood type
c) Hemoglobin and hematocrit
d) Platelet count
e) Progesterone
Incorrect!!! A maternal blood type should be checked on all women with vaginal bleeding during
pregnancy, unless it was documented earlier in the pregnancy. If the patient’s blood type is Rh-negative,
RhoGAM would be indicated to prevent Rh sensitization. Serial quantitative beta-hCGs can be useful in
confirming an ongoing pregnancy before fetal heart rate activity can be noted, but in this case there is a
non-viable intrauterine pregnancy. The patient is unlikely to have had significant blood loss making a blood
or platelet count unlikely to be necessary at this time. Although progesterone levels can be useful in
determining if a pregnancy is failing, the diagnosis is already clear in this case.
A 27 year-old G1 presents to labor and delivery and is found to have a fetal demise at 34-weeks gestation. She did
not have access to prenatal care during the pregnancy. Her vital signs are normal and she is not in labor. Her uterus
is non-tender and she does not have any vaginal bleeding or ruptured membranes on exam. Which untreated
condition is the most likely cause?
a) Diabetes
b) Anemia
c) Hypothyroidism
d) Herpes
e) Rh-isoimmunization
Incorrect!!! Uncontrolled glucose is associated with adverse fetal outcome. A patient with Type I diabetes
is at risk for many pregnancy complications, including fetal death and fetal macrosomia, although fetal
growth restriction may also occur. Diabetics also have increased risk for polyhydramnios, congenital
malformations (cardiovascular, neural tube defects, and caudal regression syndrome), preterm birth, and
hypertensive complications. The anemia most often seen in pregnancy is mild and would not be the most
likely cause. It is unlikely that she has an abruption causing the anemia, but this should be considered.
Hypothyroidism is usually associated with menstrual irregularities and infertility, and is a less likely cause.
Although chorioamnionitis is a possible cause of stillbirth, the clinical scenario does not support the
diagnosis, as she has no signs of infection. She is afebrile, has a nontender uterus, and does not have
ruptured membranes. Rh sensitization is unlikely since this is her first pregnancy and she did not have any
bleeding or procedures during the pregnancy.
A 34 year-old patient is diagnosed with a fetal demise at 37 weeks. She and her husband seem stunned. They cannot
believe the news. What is the next psychological response you would expect from this couple after the initial denial?
a) Ambivalence
b) Anger
c) Assessment
d) Annoyance
e) Acceptance
Correct!!! Couples who are presented with the news of a fetal birth defect or loss progress through a series
of coping responses. The response to “bad news” varies with the severity, treatability and the coping level
of the couple. As an individual starts to understand the situation, frustration or anger may be self-directed
or directed to the spouse, the affected child or the caregiver without a rational basis. This is important to
recognize to help the couple through these stages (Denial, Anger, Bargaining, Depression, Acceptance).
A 22 year-old G1 currently at 8 weeks gestation is noted to have a missed abortion on ultrasound, along with a
sharply retroverted uterus. She elects to undergo suction dilation and curettage. During the procedure, “fatty
appearing tissue” is noted to be coming through the curette. What is the next best step in the management of this
patient?
a) Continuing with the suction curettage
b) Remove the tissue from the curette and replace it into the uterus
c) Cut the tissue off at the cervical os
d) Proceed with laparoscopy
e) Stop the procedure and observe her the hospital for 48-hours
Incorrect!!! The tissue is consistent with omental tissue and may include segments of bowel. The suction
should be turned off and the tissue gently removed from the curette. Laparoscopy will allow closer
examination and should bowel appear to be involved, the surgeon should consider laparotomy for closer
evaluation of the bowel for damage. The other options would place the patient at increased risk of
complications and delay diagnosis.
A 33 year-old, G2P1 with a known twin gestation presents to your office at 24 weeks and notes that two days prior
she had a nosebleed. She has not been seen in your office for the last 7 weeks. Ultrasound today shows a demise of
one twin that has measurements consistent with 21 weeks gestation. What is the next step in the management of this
patient?
a) Immediate delivery of the surviving twin
b) Continued management as a singleton pregnancy
c) Maternal fibrinogen level
d) Abdominal x-ray to assess for Spalding's sign
e) Nonstress test of the surviving twin
Incorrect!!! The incidence of the death of one twin in-utero is 2-7%. When a dead fetus has been in utero
for 3-4 weeks, fibrinogen levels may decrease, leading to a coagulopathy. The patient’s nosebleed may be a
common pregnancy finding or be related to the demise, and a coagulopathy must be ruled out. Induction
should be considered, but may be delayed after the death of a twin in order to allow the viable twin to
mature. In these cases, fibrinogen levels should be monitored to detect a progressive coagulopathy.
Usually, this is performed weekly or biweekly, depending on the levels obtained. Spalding sign is an
overlapping of fetal skull bones suggesting a fetal demise. It is not necessary to do an x-ray, since the
diagnosis was confirmed on ultrasound. It would give unnecessary radiation exposure to the surviving twin.
Although an ultrasound to assess fetal well-being of the surviving twin would be very helpful, at this
gestational age and in a twin pregnancy, a non-stress test would be less so.
Obstetric - Exam 22
A 32-year-old G1P0 woman comes to your office for her first prenatal care visit. She has recently read an article
about the rising Cesarean section rate in the United States and asks you about the rate in your hospital. What do you
explain as the major cause of higher Cesarean delivery rates?
a) The rate of breech presentations has increased
b) Less women are having vaginal births after Cesarean
c) Obstetricians’ reluctance to perform forceps delivery
d) Increased rate of fetal macrosomia due to uncontrolled gestational diabetes
e) Rate of twins has increased
Incorrect!!! The rate of vaginal birth after Cesarean (VBAC) has decreased in recent years, due to studies
that showed an increased risk of complications, especially uterine rupture. This is one factor that has led to
the increased Cesarean section rate. In addition, although the rate of breech presentation is stable, there are
significantly fewer obstetricians who are willing to perform vaginal breech deliveries. Many obstetricians
do not perform instrumental vaginal deliveries, such as forceps and vacuum extractions, further
contributing to the rising rate. Gestational diabetes is a well-known pregnancy complication with clear
clinical guidelines.
A 15-year-old G1P0 woman at 40 weeks gestation presents to Labor and Delivery with contractions. At 10:00 am,
her cervical exam shows that she is 2 centimeters dilated, 70% effaced and the vertex is at 0 station. Clinical
pelvimetry reveals an adequate pelvis and membranes are intact. The fetus is in a cephalic presentation, with no
concern about macrosomia. Contractions are occurring every 3 to 4 minutes, based on the external monitor. Fetal
surveillance is reassuring and the patient is stable. Her labor slowly progresses and, at 1:00 pm, the patient has
spontaneous rupture of membranes. Fetal surveillance remains reassuring. Her cervical exam is 4 centimeters
dilated, 100% effaced, and 0 station. At 4:00 pm, the patient’s cervical exam is unchanged (4/100/0). Contractions
are occurring every 5 to 6 minutes. Which of the following is the most appropriate next step in the management of
this patient?
a) Consent the patient for a Cesarean section secondary to failure to progress
b) Continue fetal surveillance and reexamine the patient in two hours
c) Begin oxytocin augmentation
d) Perform a contraction stress test
e) Have the patient ambulate
Correct!!! The patient has an arrest of dilatation in the active phase of labor. She is only having
contractions every 5 to 6 minutes, so it is reasonable to start Pitocin to increase the frequency and strength
of this patient’s contractions. If the patient does not have cervical change once she is having more frequent
contractions on Pitocin, it would be reasonable to place an IUPC (intrauterine pressure catheter) to assess
the strength of the contractions. It is not yet necessary to perform a Cesarean section. Further observation
and having the patient ambulate do not facilitate delivery. An amnioinfusion is not indicated in this
situation.
A 34-year-old G2P1 woman at 40 weeks gestation, with a history of one prior vaginal delivery, strongly desires an
induction of labor, as she is unable to sleep secondary to severe back pain. Her cervical exam is closed, 20% effaced
and -2 station. The cervix is firm and posterior. Which of the following is the most appropriate next step in the
management of this patient?
a) Wait until 42 weeks for induction
b) Administer cytotec
c) Insert a foley bulb in the cervix
d) Perform artificial rupture of membranes
e) Perform a Cesarean section
Incorrect!!! The patient is multiparous at term and waiting until she reaches 42 weeks may increase the risk
of perinatal mortality. Since she is uncomfortable with back pain, she may be undergo labor induction. Her
cervix is unfavorable; therefore, cytotec administration is appropriate prior to pitocin induction. A foley
bulb or artificial rupture of membranes cannot be achieved in a patient with a closed cervix. There are no
indications to perform a Cesarean section in this patient.
A 22-year-old G1P0 woman at 39-weeks gestation presents to Labor and Delivery in active labor. She has had an
uneventful pregnancy. You suspect a breech presentation and ultrasound confirms your exam. Which of the
following is associated with breech presentation?
a) Gestational diabetes
b) Uterine fibroids
c) Oligohydramnios
d) Macrosomia
e) Mild preeclampsia
Correct!!! Prematurity, multiple pregnancy, genetic disorders, polyhydramnios, hydrocephaly,
anencephaly, placenta previa, uterine anomalies and uterine fibroids are all associated with breech
presentation.
A 20-year-old G1 woman at 40-weeks gestation presents to Labor and Delivery at 10:00 am, complaining of painful
contractions every 3 to 4 minutes since midnight. Her exam on admission was 2 centimeters dilated, 90% effaced
and 0 station. Three hours later, her exam is unchanged. The patient is still having contractions every 3 to 4 minutes.
She is discouraged about her lack of progress. Which of the following is the most appropriate next step in the
management of this patient?
a) Proceed with a Cesarean section
b) Artificial rupture of membranes
c) Counseling about latent phase of labor and rest
d) Cervical dilatation
e) Contraction stress test and laminaria placement
Incorrect!!! The patient is in the latent phase of labor and has not yet reached the active phase (more than 4
cm). A prolonged latent phase is defined as >20 hours for nulliparas and >14 hours for multiparas, and may
be treated with rest or augmentation of labor. It is premature to proceed with a C-section. Artificial rupture
of membranes is not recommended in the latent phase as it places the patient at increased risk of infection.
Cervical dilatation or laminaria placement are not indicated.
A 19-year-old G1 woman, who is the wife of basketball player, presents at 28 weeks. Her mother had a delivery
complicated by a shoulder dystocia and she is concerned about her own risk. Which of the following is her biggest
risk factor for shoulder dystocia?
a) Family history
b) Tall husband
c) Polyhydramnios
d) Gestational diabetes
e) Oligohydramnios
Correct!!! Fetal macrosomia, maternal obesity, diabetes mellitus, postterm pregnancy, a prior delivery
complicated by a shoulder dystocia, and a prolonged second stage of labor are all associated with an
increased incidence of shoulder dystocia. Although a family history can be indicative of large babies which
might place her at additional risk, her gestational diabetes presents her largest risk factor.
A 30-year-old G2P1 woman at 38 weeks gestation presents to Labor and Delivery with contractions every 2 to 3
minutes. Her membranes are intact. Her cervical exam is 5 centimeters dilated, 100% effaced, and -1 station. The
fetal heart rate is reassuring. Two hours later, she progresses to 7 cm dilated and 0 station. Two hours after that, her
exam is unchanged (7/100/0.) Fetal heart tones are reassuring. Which of the following is the most appropriate next
step in the management of this patient?
a) Allow her to ambulate and return when she is ready to push
b) Perform a contraction stress test
c) Perform an amniotomy
d) Perform a Cesarean Section
e) Place an internal fetal scalp electrode
Incorrect!!! This patient has secondary arrest of dilation, as she has not had any further cervical change in
the active phase for over 2 hours. A multiparous woman should have a rate of cervical dilation of at least
1.5 cm/hour in the active phase. Amniotomy is often recommended in this situation. After it is performed,
if the patient is still not in an adequate contraction pattern, augmentation with Pitocin can be attempted
after careful evaluation. Although the patient requires close following and requires active care, it is too
early to move to a Cesarean section. An internal scalp electrode is not necessary, since the fetal heart
monitoring is reassuring.
A 25-year-old G2P1 woman at term presents to Labor and Delivery in active labor. Her cervix rapidly dilates from 7
centimeters to complete in 1 hour. She has been pushing for 2 hours. The fetal station has changed from -1 to +1.
Fetal heart tones are reassuring. The patient is feeling strong contractions every 3 minutes. Which of the following is
the most appropriate next step in the management of this patient?
a) Cesarean Section
b) Forceps delivery
c) Continued monitoring of labor
d) Augmentation with oxytocin
e) Ultrasound for estimated fetal weight
Correct!!! Continued monitored labor is appropriate if clinical evaluation indicates that the fetus is not
macrosomic or there is no obvious fetopelvic disproportion. If either were the case, then a Cesarean section
would be indicated. At this time, there is no fetal or maternal indication to perform a forceps delivery
especially considering that the station is +1. Augmentation would be indicated if the contractions were
inadequate in intensity or frequency. An ultrasound at this stage of labor is inaccurate and one relies on
clinical estimates of weight.
A 35-year-old G3P2 woman is at 18 weeks gestation. Her obstetrical history is significant for two previous low
transverse cesarean sections. Her first one was performed secondary to arrest of dilation in the active phase. The
patient reports that she only dilated to 7 centimeters. She delivered a healthy 3500-gram male infant. Her second
cesarean section was an elective repeat. She delivered a healthy 3400-gram female infant. The patient strongly
desires to attempt a VBAC (vaginal birth after cesarean.) Which of the following statements is correct?
a) If she attempts a VBAC, the risk of uterine rupture is approximately 10%.
b) The likelihood of a successful VBAC is lower in patients with two previous Cesarean deliveries than in
women with one prior Cesarean delivery.
c) The likelihood of a successful VBAC does not appear to be affected by the indication of the previous Cesarean
delivery.
d) The success rate for having a successful VBAC after one previous low transverse Cesarean delivery is
approximately 65%.
e) She can safely undergo a prostaglandin induction of labor at term.
Correct!!! Women attempting a vaginal birth after Cesarean (VBAC) after one previous low transverse
Cesarean delivery have a 70-80% chance of having a successful VBAC. The risk of uterine rupture with a
history of one previous low transverse Cesarean section is approximately 1 percent or less. There are no
data to demonstrate the exact increased risk of uterine rupture with a history of two previous Cesarean
deliveries. The indication for the previous Cesarean section may affect the success rate of a future VBAC.
Patients who had a prior Cesarean delivery for a nonrecurring indication, such as placenta previa or breech
presentation are more likely to have a successful VBAC compared to patients whose previous Cesarean
section was performed secondary to cephalopelvic disproportion. Prostaglandin induction in this patient
would not be routinely performed.
A 25-year-old G1 woman at 41-weeks presents to Labor and Delivery with painful contractions every 4 minutes.
Her cervix is 5 cm dilated, 90% effaced. On cervical exam, you are able to feel a body part but it is not the head.
Which of the following is the most likely body part you were palplating?
a) A foot
b) A hand
c) The buttocks
d) The back
e) The shoulder
Incorrect!!! Breech presentation occurs in approximately 3-4% of women in labor overall, and occurs more
frequently in preterm deliveries. Frank breech is the most common type, occurring in 48-73% of cases and
the buttocks are the presenting part. Complete breech is found in approximately 5-12% of cases and
incomplete breech (footling breeches) occurring in approximately 12-38% of cases.
Obstetrics - Exam 23
A 19-year-old G1 woman presents to the hospital. She states she is 25-weeks pregnant and complains of vaginal
bleeding for the past hour. She had intercourse earlier without discomfort. Currently she denies cramping or pain
and feels the baby moving. Her prenatal course has been uncomplicated. She takes no medication besides prenatal
vitamins and denies smoking, alcohol or drug use. Her blood pressure is 110/70, pulse 68, and she is afebrile. Her
abdomen and uterus are soft and nontender. Fetal heart tones are in the 150s. Which of the following is the most
appropriate next step in the management of this patient?
a) Vaginal examination
b) Urgent Cesarean section
c) Pelvic ultrasound
d) Administer steroids
e) Recommend bed rest
Incorrect!!! An ultrasound should be performed to check for abnormal placentation. Placenta previa must
be ruled out before proceeding with vaginal examination because of the risk of injury to the placenta and
catastrophic bleeding. Bleeding from placenta previa often is without warning or pain. Cesarean section is
not indicated and there is no reason to place her on bed rest. It is not necessary to administer steroids at this
point unless it looks like a delivery is imminent.
A 23-year-old G2P1 woman at 36-weeks gestation presents with her second episode of heavy vaginal bleeding. She
has normal prenatal labs, but she has a known placenta previa. She denies uterine contractions or abdominal pain
and reports good fetal movement. Her blood pressure is 100/60, pulse 110, and she is afebrile. Her abdomen and
uterus are nontender. Fundal height measures 35 centimeters and fetal heart tones reveal a baseline of 140 and are
reassuring. Pelvic ultrasound confirms a placenta previa and the fetus is in the cephalic presentation. Which of the
following is the most appropriate next step in the management of this patient?
a) Tocolysis
b) Induction of labor
c) Cesarean delivery
d) Amniocentesis
e) Observation
Incorrect!!! This patient is near term with a second episode of active bleeding from a placenta previa. The
appropriate next step would be to move towards delivery via Cesarean section. The patient is not
experiencing contractions, so tocolysis is not necessary and would not be used with heavy vaginal bleeding.
Catastrophic bleeding could occur due to disruption of blood vessels as the cervix dilates if a vaginal
delivery is pursued, and induction of labor would therefore be contraindicated. An amniocentesis is not
indicated in this situation. Although the patient is not yet at term, delivery is indicated due to the second
episode of heaving bleeding. Conservative management with observation, so close to term, could lead to
severe complications and would not be appropriate in this patient.
A 38-year-old G5P4 woman, with a history of four Cesarean sections, is at 36-weeks gestation with a singleton
pregnancy. She presents to Labor and Delivery with complaints of vaginal bleeding for the last hour. Prenatal care
has been unremarkable except for a second trimester ultrasound discovering an anterior placenta, which partially
covers the cervical os. Follow up ultrasound exams have confirmed these findings. The patient denies uterine
contractions and abdominal pain. She feels the baby moving. Her blood pressure is 110/60, pulse 110, and she is
afebrile. Her abdomen and uterus are nontender and soft. Fetal heart tones have a baseline of 140 and are reassuring.
This patient is at greatest risk for which of the following complications?
a) Vasa previa
b) Placenta accreta
c) Placental abruption
d) Uterine dehiscence
e) Preterm labor
Incorrect!!! Placenta accreta occurs when the placenta grows into the myometrium. This patient is at risk
for this condition due to her history of four previous Cesarean sections, and the low anterior placenta. The
scar tissue from the previous surgery prevents proper implantation of the placenta and the placenta
subsequently grows into the muscle. Vasa previa is a rare condition where the umbilical cord inserts into
the membranes. Placental abruption is the premature separation of the normally implanted placenta. Uterine
dehiscence could be as high as 5% in this case, and the risk of placenta accreta with four prior c-sections
approaches 50%. The patient is not contracting at the present time, so preterm labor is unlikely.
A 24-year-old G2P1 woman is undergoing a Cesarean section for placental abruption. She presented to Labor and
Delivery with severe abdominal pain and heavy vaginal bleeding. The fetus was delivered uneventfully. The
placenta delivered with the fetus with a significant clot attached to the maternal surface. The patient continues to
bleed from the placental bed. Estimated blood loss is 1500 ml. The operative team decides to give her fresh frozen
plasma (FFP) because of which of the following components?
a) Platelets
b) Von Willebrand’s factor
c) Red blood cells
d) Fibrinogen
e) Factor X
Incorrect!!! Correcting coagulation deficiencies requires replacing all necessary components. Fresh frozen
plasma contains fibrinogen, as well as clotting factors V and VIII. Cryoprecipitate contains fibrinogen,
factor VIII and von Willebrand’s factor. Neither of these preparations contains red blood cells or platelets,
which must be given separately.
An 18-year-old G1 woman at 32-weeks gestation presents with severe abdominal pain and a small amount of
bleeding. She has received routine prenatal care, smokes one pack of cigarettes per day and admits to using crack
cocaine. On exam, her blood pressure is 140/80, pulse 100 and she is afebrile. Her uterus is tense and very tender.
Pelvic ultrasound reveals a fundal placenta, cephalic presentation of the fetus and no other abnormalities. Cervical
examination reveals blood coming through the os and is one centimeter dilated. Fetal heart tones have a baseline of
160s, with poor long-term variability and decelerations. Which of the following is the most likely diagnosis?
a) Placenta previa
b) Premature rupture of the membranes
c) Spontaneous labor
d) Placental abruption
e) Chorioamnionitis
Correct!!! This patient has a placental abruption. Common presenting signs of an abruption include
abdominal pain, bleeding, uterine hypertonus and fetal distress. Risk factors include smoking, cocaine use,
chronic hypertension, trauma and prolonged premature rupture of membranes, and history of prior
abruption. Treatment would involve an emergent Cesarean section with appropriate resuscitation, including
intravenous fluids and blood products as needed. A placenta previa is an abnormal location of the placenta.
A 21-year-old G1 at 36 weeks of pregnancy presents with sudden onset of abdominal pain and bleeding. She smokes
a pack of cigarettes a day, but otherwise her pregnancy has been uncomplicated. She takes no medications other than
prenatal vitamins. Her blood pressure is 150/90, pulse 90 and she is afebrile. Her uterus is tense and very tender.
Pelvic ultrasound shows the placenta to be posterior and fundal, with a cephalic presentation of the fetus. Cervical
examination reveals no lesions, blood coming through the os and is one centimeter dilated. Fetal heart tones have a
baseline of 150, with poor variability and late decelerations. Tocometer reveals contractions every 30-45 seconds.
Which of the following is the most appropriate next step in treatment?
a) Amniotomy
b) Cesarean delivery
c) Induction of labor
d) Tocolysis
e) A double set-up examination
Incorrect!!! This patient is undergoing a placental abruption, with a deteriorating fetal condition. An
emergent Cesarean delivery is necessary. The mother risks excessive blood loss, DIC and possible
hysterectomy. The fetus risks neurological injury from anoxia or death. Risk factors for abruption include
smoking, cocaine use, abdominal trauma, chronic hypertension, multiparity and prolonged premature
rupture of membranes. Since immediate delivery is needed, neither amniotomy, induction, nor tocolysis is
appropriate. A double set-up examination (performed in the operating room with a C/S team scrubbed and
ready) is not indicated, since the ultrasound determined the location of the placenta to be fundal.
A 25-year-old female patient presents to her doctor for preconception counseling. She is healthy without significant
medical problems. She takes no medications except for occasional Tylenol. She smokes one pack of cigarettes per
day since age 16 and drinks occasionally. She weighs 140 pounds and her vital signs and examination are normal.
The patient is at increased risk of which of the following during her pregnancy?
a) Fetal chromosomal abnormality
b) Breech presentation
c) Placental abruption
d) Macrosomia
e) Cerebral palsy
Incorrect!!! Smoking increases the risk of several serious complications of pregnancy, including placental
abruption, placental previa, fetal growth restriction, preeclampsia and infection. Women who smoke should
be counseled vigorously to quit smoking prior to conception and to resist restarting after the baby is born.
A 32-year-old G3P2 woman presents at 40 1/7 weeks gestation because of regular uterine contractions every 5
minutes for the last 2 hours. Her prenatal course was unremarkable. She states the baby is moving, but she has had a
bright red, bloody discharge for the last 30 minutes. She does not think she has ruptured her membranes. Her blood
pressure is 120/70, pulse 80 and she is afebrile. Her abdomen is soft and she has regular contractions of moderate
intensity. Fetal heart tones have a baseline of 130 and are reactive with no decelerations. Pelvic ultrasound reveals a
fundal placenta and cephalic presentation of the fetus. Cervical examination reveals a friable cervix that bleeds
easily and is 5 centimeters dilated and completely effaced. Membranes are confirmed to be intact. Which of the
following is the most likely source of bleeding?
a) Placental abruption
b) Placenta previa
c) Bloody show
d) Cervical cancer
e) Cervicitis
Incorrect!!! During pregnancy the cervix is extremely vascular, and with dilation a small amount of
bleeding may occur. This bloody show is not of clinical significance and often occurs with normal labor.
Serious causes of bleeding, such as placental abruption and placenta previa, need to be ruled out in order to
make the proper delivery plans. Cervical cancer and cervicitis are very unlikely causes for the bleeding in
this situation.
A 17-year-old G1P0 woman at 24 weeks gestation presents with vaginal bleeding. She denies any pain, cramping or
dysuria. She reports last having intercourse 3 weeks ago. Prenatal care and labs have been unremarkable. Her vital
signs are normal and she is afebrile. Pelvic ultrasound reveals a fundal placenta and viable fetus. Abdominal
examination is unremarkable. Vaginal examination reveals a uniformly friable cervix with a small amount of blood
in the vault. Digital examination reveals a firm, closed cervix. What is the most likely diagnosis that explains the
bleeding?
a) Trauma
b) Cervical cancer
c) Cervicitis
d) Bloody show
e) Threatened abortion
Incorrect!!! Cervicitis caused by chlamydia, gonorrhea, trichomonas or other infection can present with
vaginal bleeding. The cervix is much more vascular during pregnancy and inflammation can lead to
bleeding. Evaluation for other causes of bleeding must be completed and then treatment for the infection
should be initiated. The patient does not give any history of trauma and cancer is unlikely, since her routine
labs were normal. She is not in labor, and a bloody show associated with cervical dilatation is not
consistent with the history provided. Threatened abortion occurs during the first trimester.
A 45-year-old G4P3 woman presents with vaginal bleeding. Last week, she performed a home pregnancy test that
was positive. She thinks her last menstrual period was four months ago. The last time she saw her doctor was 8 years
ago, with the birth of her last child. She has no serious medical problems, has smoked a pack of cigarettes a day
since the age of 20, occasionally has a beer and does not exercise. Her vitals are normal. Abdominal examination
reveals a soft abdomen and the fundus palpable just below the umbilicus. Pelvic ultrasound reveals a fundal placenta
and a fetus measuring 18 weeks with normal cardiac activity. Vaginal examination reveals a 3-centimeter lesion
arising off the posterior lip of the cervix. It easily bleeds with palpation and is hard in consistency. Which of the
following is the most likely cause of the bleeding?
a) Trauma
b) Cervicitis
c) Threatened abortion
d) Cervical cancer
e) Nabothian cyst
Incorrect!!! Cervical cancer unfortunately can complicate pregnancies and presents with bleeding. Other
causes of bleeding need to be ruled out such as threatened abortion, infection or trauma. Treatment for
cervical cancer during pregnancy requires difficult decisions that consider the stage of cancer, appropriate
therapy, maternal welfare and fetal welfare.
Obstetrics - Exam 24
A 19-year-old G1 woman at 28 weeks gestation comes to Labor and Delivery because of the onset of contractions.
The patient describes the contractions as progressively becoming more painful, each lasting 40 seconds and now
occurring every 5 minutes. She reports good fetal movement and does not have any bleeding or leakage of fluid. On
evaluation in triage, it is noted that she is having regular contractions, approximately every 5 minutes, has intact
membranes and her cervical exam is 3 cm dilated and 50% effaced. What is the most frequent cause of this
condition?
a) Dehydration
b) Rigorous fetal movement
c) Idiopathic
d) Uterine Distortion
e) Iatrogenic
Correct!!! In most cases, preterm labor is idiopathic (i.e. no cause can be identified.) Dehydration and
uterine distortion (from uterine fibroids or structural malformations) can be associated with preterm labor.
In some cases, preterm labor is due to iatrogenic causes; for example, when a physician induces a preterm
patient who has severe preeclampsia. Fetal movement does not cause preterm labor.
A 20-year-old G1 woman presents to triage with uterine contractions every 4 minutes at 32 weeks gestation. On
exam, her cervix is long, closed and posterior. Her urinalysis is normal and she is not dehydrated. Which of the
following is the most appropriate next step in the management of this patient?
a) IV hydration
b) Tocolysis
c) Discharge home on bedrest
d) Continued observation
e) Treat with betamethasone
Correct!!! Approximately 50% of patients with preterm contractions have spontaneous resolution of
abnormal uterine activity. The patient should be observed until a correct diagnosis is made. There is no
indication to treat the patient with IV hydration. If there is evidence the patient is dehydrated and she is
unable to tolerate PO fluids, then IV hydration would be indicated. Preterm labor, which is defined as the
presence of regular uterine contractions leading to cervical change, needs to be promptly treated. Tocolysis
is not necessary in this case because a diagnosis of preterm labor has not been made (no cervical change).
The patient should not be sent home until diagnosis and treatment plans are determined. Cerclage is not
necessary, since she does not have an incompetent cervix. Treatment with betamethasone is not indicated
unless there is evidence that the patient is at increased risk of delivering preterm.
A 20-year-old G2P1 woman at 32 weeks gestation presents to Labor and Delivery with contractions every 4
minutes. On physical examination, her vital signs are: temperature 38.1; heart rate 120; respiratory rate 18, and
blood pressure 110/65. Her fundus is non-tender and the rest of the physical exam is benign. Her cervix is dilated 2
cm and is 50% effaced. Fetal heart tones are in the 140s and are reassuring, overall. Her white blood cell count
(WBC) is 18,000. Which of the following is the most appropriate next step in the management of this patient?
a) Continue close observation
b) Perform amniocentesis
c) Treat with steroids
d) Begin labor induction
e) Perform an urgent c-section
Incorrect!!! An amniocentesis should be performed to rule out an intra-amniotic infection. This patient has
an unexplained fever and elevated white blood cell count, which are concerning for an intra-amniotic
infection. Delivery is not warranted at this time, as it is possible that the fever and elevated white blood cell
count are from another source. Steroids should not be used until an intra-amniotic infection has been ruled
out. Conservative management with observation would delay diagnosis and would not be appropriate.
A 26-year-old G2P1 woman presents in preterm labor at 33 weeks gestation. She has a history of a prior preterm
birth at 32-weeks gestation. She has insulin dependent diabetes and has a history of myasthenia gravis. She has
regular contractions every 3 minutes and fetal heart tones are reassuring. Cervix is 3cm dilated and 0 station. Her
blood pressure is 140/90, which of the following is the most appropriate tocolytic to use in this patient?
a) Calcium channel blocker
b) Terbutaline
c) Magnesium sulfate
d) Indomethacin
e) Ritodrine
Incorrect!!! Calcium channel blocker is the best option for her as she has contraindications to the other
agents listed. Terbutaline and ritodrine are contraindicated in diabetic patients, magnesium sulfate is
contraindicated in myasthenia gravis, and indomethacin is contraindicated at 33 weeks due to risk of
premature ductus arteriosus closure.
A 19-year-old G2P1 woman at 28 weeks gestation has been diagnosed with preterm labor. Her physician has chosen
to treat her with magnesium sulfate. By what mechanism of action does magnesium sulfate work as a tocolytic?
a) Competes with calcium for entry into cells
b) Increases cAMP in the cell, which decreases free calcium
c) Decreases prostaglandin (PG) production by blocking conversion of free arachidonic acid to PG
d) Prevents calcium entry into muscle cells
e) Inhibits calcium transport
Correct!!! Magnesium sulfate works by competing with calcium entry into cells. Beta-adrenergic agents
work by increasing cAMP in the cell, thereby decreasing free calcium. Prostaglandin synthetase inhibitors,
such as Indomethacin, work by decreasing prostaglandin (PG) production by blocking conversion of free
arachidonic acid to PG. Calcium channel blockers prevent calcium entry into muscle cells by inhibiting
calcium transport.
A 35-year-old G1 woman presents at 30-weeks gestation. Her cervix is 3 cm dilated, 50% effaced and the vertex is
at 0 station. She is having contractions every 5 minutes and has no signs consistent with an intra-amniotic infection
(chorioamnionitis.) You make a diagnosis of preterm labor and choose to initially treat her with Terbutaline. Which
of the following side effects should you tell her about?
a) Premature constriction of the ductus arteriosus
b) Respiratory depression
c) Tachycardia
d) Tachypnea
e) Headache
Correct!!! Terbutaline is a beta-adrenergic agent. Side effects include tachycardia, hypotension, anxiety and
chest tightening or pain. Prostaglandin synthetase inhibitors, such as indomethacin, are associated with
premature constriction of the ductus arteriosus, usually after 34-weeks gestation. Magnesium sulfate may
cause respiratory depression when used in high doses. Other side effects of magnesium therapy include
flushing and headache. Tachypnea and headaches are not usual side effects.
You are asked to evaluate a 25-year-old G2P1 woman at 32 weeks gestation who is diagnosed with preterm labor.
She is currently receiving tocolytic therapy with magnesium sulfate. The patient’s nurse is concerned that the patient
may have magnesium sulfate toxicity. The patient is alert and has no complaints. Her contractions have stopped and
her vital signs are stable. Which of the following findings associated with magnesium sulfate treatment would this
patient experience before she develops respiratory depression?
a) Loss of deep tendon reflexes
b) Hyperreflexia
c) Tachycardia
d) Fetal bradycardia
e) Fetal tachycardia
Correct!!! High levels of magnesium sulfate may cause respiratory depression (12-15 mg/dl) or cardiac
depression (>15 mg/dl.) Prior to developing respiratory depression the patient should have diminished or
absent deep tendon reflexes. Beta-adrenergic agents, such as terbutaline, are associated with hypotension
and tachycardia. Fetal bradycardia has been reported with prostaglandin synthetase inhibitors, such as
Indomethacin, and fetal tachycardia is often noted with underlying infection or increased maternal
temperature.
Your 38-year-old sister is at 29-weeks gestation and has been diagnosed with preterm labor. Her perinatologist
recommended treatment with Indomethacin. Your sister is worried that this may harm her baby. What is a possible
adverse fetal effect associated with indomethacin treatment?
a) Skeletal anomalies involving the skeletal system
b) Premature constriction of the ductus arteriosus
c) Fetal growth restriction
d) Hypoxia
e) Anemia
Correct!!! Maternal indomethacin exposure can result in premature constriction of the ductus arteriosus,
especially if used after 34 weeks gestation. Skeletal anomalies and polyhydramnios are not associated with
Indomethacin. In fact, Indomethacin is associated with oligohydramnios. Fetal hypoxia and decreased
uteroplacental blood flow have been associated with the use of calcium channel blockers, such as
Nifedipine.
A 38-year-old G2P0 woman at 31-weeks gestation has been diagnosed with preterm labor and is currently stable on
magnesium sulfate. Her cervical exam has remained unchanged at 2 cm dilated, 75% effaced and -2 station. Her
vital signs are stable and fetal heart tones are reassuring. You recommend treatment with Betamethasone (a steroid).
What is a possible fetal effect associated with Betamethasone therapy?
a) Increased risk of infection in the newborn
b) Increased incidence of intracerebral hemorrhage
c) Enhancement of fetal growth
d) Increased incidence of necrotizing enterocolitis
e) Decreased incidence of intracerebral hemorrhage
Incorrect!!! Treatment with Betamethasone from 24 to 34-weeks gestation has been shown to increase
pulmonary maturity and reduce the incidence and severity of RDS (respiratory distress syndrome) in the
newborn. It is also associated with decreased intracerebral hemorrhage and necrotizing enterocolitis in the
newborn. It has not been associated with increased infection or enhanced growth.
A 37-year-old G3P0 woman at 29-weeks gestation presents with uterine contractions every 5 minutes. Her cervix is
1 cm dilated and 50% effaced. You perform a fetal fibronectin test that returns negative. The patient stops
contracting after treatment with bed rest and hydration. What is the strength of using a fetal fibronectin test in
patients with preterm contractions?
a) The positive predictive value
b) The negative predictive value
c) The high sensitivity
d) The low false positive rate
e) The high false positive rate
Incorrect!!! Fibronectin is an extracellular matrix protein that is thought to act as an adhesive between the
fetal membranes and underlying deciduus. It is normally found in cervical secretions in the first half of
pregnancy. Its presence in the cervical mucus between 22 and 34 weeks is thought to indicate a disruption
or injury to the maternal-fetal interface. Fetal fibronectin is FDA approved for use in women with
symptoms of preterm labor from 24 to 35 weeks and during routine screening of asymptomatic patients
from 22 to 30-weeks gestation. Fetal fibronectin has a negative predictive value of 99.2% in symptomatic
women — 198 out of every 200 patients with a single negative test result will not deliver in the next 14
days. The positive predictive value in symptomatic women is 16.7%. Therefore, 16.7% of women with a
positive test will deliver within 14 days. In asymptomatic women, a negative fetal fibronectin test has a
negative predictive value of 96.7% for delivery before 35 weeks.
Obstetrics - Exam 25
A 29-year-old G1P0 woman at 31 weeks gestation presents with watery discharge from the vagina commencing
several hours ago. Her prenatal course has been uncomplicated and she takes prenatal vitamins and iron. She denies
substance abuse, smoking or alcohol use. Her blood pressure is 110/70; pulse 84; temperature 98°F. Which of the
following is the most appropriate next step in the management of this patient?
a) Nitrazine testing of mucus swabbed from cervix
b) Microscopic examination of smear of vaginal fluid to evaluate for ferning
c) Digital examination of cervix to determine if palpable bag of water is present ahead of presenting part
d) Ultrasound examination to determine if oligohydramnios is present
e) Non-stress test to look for variable decelerations
Incorrect!!! Methods to confirm rupture of membranes include testing the vaginal fluid for ferning and
Nitrazine testing. It is important to test the fluid from the vagina and not to test cervical mucus because of
false positive ferning patterns. While a palpable bag of water would go against rupture (unless there was a
high leak) a digital exam should be avoided in a patient you suspect might have preterm rupture of
membranes because of the risk of introducing bacteria into the uterine cavity and, thus, chorioamnionitis.
Ultrasound may reveal oligohydramnios and support the diagnosis of rupture of membranes, but does not
confirm this diagnosis. Similarly, non-stress test may reveal variable decelerations, which may be present
in the setting of rupture of membranes.
A 29-year-old G1P0 woman presents at 31 weeks gestation with preterm rupture of membranes 6 hours ago. She
notes that for the last hour she has had an occasional contraction. Her prenatal course has been uncomplicated and
she takes prenatal vitamins and iron. She denies substance abuse, smoking or alcohol use. Her blood pressure is
110/70; pulse 84; temperature 98�F. What is the role of tocolysis in this patient?
a) To prevent delivery and prolong the pregnancy
b) To attempt to delay delivery until fetal lung maturity is reached
c) To attempt to delay delivery for one week
d) To attempt to delay delivery in order to administer steroids
e) Contraindicated
Incorrect!!! While the role of tocolysis in the setting of preterm rupture is controversial, it may be
appropriate in limited settings. Tocolysis may be administered in an attempt to prolong the interval to
delivery to gain time for steroids to obtain maximum benefit for the fetus. This may be reasonable in
women without evidence of infection or advanced preterm labor. Admittedly, the likelihood of success in
this setting is relatively poor, but the potential benefit to the fetus probably outweighs any maternal
complication from tocolysis.
A 19-year-old G2P1 woman at 30 weeks gestation presents with preterm rupture of membranes 6 hours ago. Her
prenatal course has been complicated by two episodes of bacterial vaginosis for which she was treated. She takes
prenatal vitamins and iron. She denies substance abuse or alcohol use, but admits to smoking five cigarettes each
day. Her prior pregnancy was delivered vaginally at 41 weeks after spontaneous rupture of membranes. Her blood
pressure is 110/70; pulse 84; temperature 98°F. Pertinent sonographic findings reveal oligohydramnios and a
cervical length of 34mm. Which of the following is the most likely cause of preterm premature rupture of
membranes in this patient?
a) Genital tract infections
b) Short cervical length
c) Presence of oligohydramnios
d) Smoking
e) Previous preterm rupture of membranes
Incorrect!!! The primary risk factor for preterm rupture of membranes is genital tract infection, especially
associated with bacterial vaginosis. All of the other listed options are risk factors, with the exception of
singleton pregnancy. Multiple gestations have a higher incidence of ruptured membranes. Smoking and
prior preterm premature rupture of membranes increases the likelihood of preterm rupture of membranes
two-fold. A shortened cervical length is also a risk factor, but her cervical length is normal.
A 19-year-old G2P1 woman at 30 weeks gestation presents with preterm rupture of membranes, 6 hours ago. She
denies labor. She takes prenatal vitamins and iron. She denies substance abuse, smoking or alcohol use. Her prior
pregnancy was delivered vaginally at 41 weeks after spontaneous rupture of membranes. Her blood pressure is
110/70; pulse 84; temperature 98°F. Pertinent sonographic findings reveal a cervical length of 34mm and an
amniotic fluid index of 3. What is the likelihood she will deliver within a week?
a) 80%
b) 40%
c) 20%
d) 10%
e) 5%
Incorrect!!! The time from premature rupture of membranes to labor is inversely related to gestational age.
While at term, 90% will go into labor within 24 hours of PROM, in those pregnancies at 28 weeks to 34
weeks; 50% labor within 24 hours and 80% within 48 hours.
A 33-year-old G2P1 woman at 29 weeks gestation presents with confirmed preterm rupture of membranes. She
denies labor. She takes prenatal vitamins and iron. She denies substance abuse, smoking or alcohol use. Her prior
pregnancy was delivered vaginally at 41 weeks after spontaneous rupture of membranes. Her blood pressure is
110/70; pulse 84; temperature 99°F. Which of the following will prolong the latency period by approximately 7
days?
a) Antibiotics
b) Corticosteroids
c) Calcium channel blocker
d) Beta mimetics
e) Magnesium sulfate
Incorrect!!! Antibiotic therapy given to patients with preterm premature rupture of the membranes has been
found to prolong the latency by up to 5-7 days, as well as reduce the incidence of maternal amnionitis and
neonatal sepsis. Corticosteroids and tocolytics may also prolong the pregnancy for various lengths of time,
but generally not 7 days.
A 24-year-old G1P0 woman at 32 weeks gestation presents with leaking watery fluid from the vagina. On
evaluation, preterm premature rupture of membranes is confirmed. She has occasional Braxton Hicks contractions
associated with fetal heart rate accelerations, no vaginal bleeding and vaginal fluid phosphatidylglycerol is absent.
Her blood pressure is 110/70; pulse 90; temperature 98°F. Fundal height is 30cm and her fundus is tender. Which of
the following findings is an indication for delivery?
a) Tender uterine fundus
b) Size less than dates
c) Fetal heart rate accelerations
d) Amniotic fluid index of less than 4
e) Absence of Vaginal fluid phosphatidylglycerol
Incorrect!!! Maternal signs of chorioamnionitis or other evidence of intra-amniotic infection are indications
for delivery. This patient has a tender fundus, which is a sign of chorioamnionitis. Labor at 32 weeks would
be allowed to progress and prolonged non-reassuring fetal testing would prompt delivery. There are no
criteria for amniotic fluid index or degree of oligohydramnios as an indication for delivery. Most authors
agree that the achievement of fetal lung maturity (i.e. positive phosphatidylglycerol or 34 weeks gestation)
is the threshold at which the risk of morbidity and mortality of maintaining the pregnancy in utero
outweighs the benefits of prolonging the pregnancy.
A 25-year-old G2P1 woman at 20 weeks gestation is diagnosed with preterm rupture of the membranes. She denies
labor. She takes prenatal vitamins and iron. She denies substance abuse, smoking or alcohol use. Her prior
pregnancy was delivered vaginally at 36 weeks after spontaneous rupture of membranes. Her blood pressure is
110/70; pulse 84; temperature 98°F. Of the following neonatal complication, for what is this patient’s fetus at
greatest risk?
a) Pulmonary hypoplasia
b) Cardiac defects
c) Urinary tract anomalies
d) Microcephaly
e) Compression fractures
Correct!!! Preterm premature rupture of the membranes that occurs before viability is associated with
significant risk of poor outcome. Neonatal survival when rupture occurs between 20 and 23 weeks is
approximately 25%. Complications that may be found in the developing fetus include structural
abnormalities that are primarily deformations (abnormalities that occur due to an insult after a structure has
already formed) rather than malformations (abnormal development of the structure itself.) Pulmonary
hypoplasia is seen when rupture of membranes occurs before 25 weeks gestation because the lack of
amniotic fluid interferes with the normal intrauterine breathing process. The result is failure of normal
development and growth of the respiratory tree.
A 28-year-old primigravid woman at 33 weeks gestation presents with complaints of fluid leaking from the vagina.
Preterm premature rupture of the membrane is diagnosed. The patient has mild uterine tenderness and you suspect
early chorioamnionitis. An amniocentesis is performed. Which of the following amniotic fluid results is indicative of
intra-amniotic infection?
a) Presence of leucocytes
b) Low Interleukin –6
c) Amniotic glucose less than 20 mg/dl
d) Presence of phosphatidylglycerol
e) Low Interleukin-12
Incorrect!!! In some cases of preterm rupture of the membranes, amniocentesis may be performed to detect
intra-amniotic infection. The presence of amniotic leucocytes has the lowest predictive value for the
diagnosis of chorioamnionitis. Interleukin-6 (if available) would be increased in the setting of
chorioamnionitis. A low amniotic fluid glucose is an indication of intra-amniotic infection.
Phosphatidylglycerol is a marker for fetal lung maturity and bacteria in a pooled specimen from the vagina
is contaminated with vaginal microflora and does not reflect the intra-amniotic fluid.
A 22-year-old G2P1 presents for prenatal care at approximately 10 weeks gestation. Her first pregnancy was
complicated by preterm premature rupture of the membranes at 28 weeks gestation. What is the recurrent risk of
preterm premature rupture of the membranes during this pregnancy?
a) 1%
b) 5%
c) 10%
d) 30%
e) 50%
Correct!!! Premature rupture of the membranes occurs in approximately 10 to 15 % of all pregnancies.
Preterm premature rupture of the membranes between 16 and 26 weeks gestational age is identified in 1%
of pregnancies. Preterm premature rupture of the membranes occurs in 1/3 of all preterm deliveries. The
reported recurrence rate for preterm premature rupture of the membranes is approximately 32% when it
occurred in the index pregnancy.
A 32-year-old G2P1 woman presents at 36 weeks gestation with premature rupture of the membranes. She denies
labor. She takes prenatal vitamins and iron. She denies substance abuse, smoking or alcohol use. Her prior
pregnancy delivered vaginally at 34 weeks after spontaneous rupture of membranes. Her blood pressure is 110/70;
pulse 84; temperature 98°F. The estimated fetal weight is 2700 grams. Which of the following is the most
appropriate next step in the management of this patient?
a) Observation until spontaneous onset of labor
b) Induction of labor
c) Magnesium sulfate
d) Terbutaline
e) Corticosteroids
Correct!!! In this patient, the benefits for delivery outweigh the risk of expectant management. The role of
tocolytics in the setting of preterm rupture is controversial, it may be appropriate in limited settings but not
after 34 weeks gestation. The current recommendations are that the benefits to the neonate outweigh the
potential risks of intra-amniotic infection prior to 32 weeks, but only in the absence of evidence of intraamniotic infection. Steroid administration after 32 weeks is controversial.
Obstetrics - Exam 30
A 19 year-old G1 at 40 weeks gestation reports that she is postterm and wants to be induced. She has had an
uncomplicated pregnancy. Her blood pressure is 124/76 and her pulse is 84. What gestational age would define
postterm pregnancy?
a) The due date
b) 40 completed weeks
c) 41 completed weeks
d) 42 completed weeks
e) 43 completed weeks
Incorrect!!! By definition, a postterm pregnancy is a pregnancy that has progressed past 42 completed
weeks or 294 days.
A 29 year-old G1 is at 42 weeks gestation, based on her last menstrual period and a first trimester ultrasound. Of the
following, what factor is most likely to be associated with postterm pregnancy?
a) Placental sulfatase deficiency
b) Fetal adrenal hyperplasia
c) Fetal alpha-fetoprotein deficiency
d) Fetal renal anomalies
e) Fetal chromosomal abnormalities
Incorrect!!! Postterm pregnancies are associated with placental sulfatase deficiency, fetal adrenal
hypoplasia, anencephaly, inaccurate or unknown dates and extrauterine pregnancy.
A 20 year-old G2P1 is 41 weeks gestation. Her prenatal course and past history are unremarkable. She has not had
any complications with her pregnancy and fetal surveillance is reassuring. Which of the following complications is
most likely to occur in this pregnancy?
a) Preeclampsia
b) Retained placenta
c) Postpartum hemorrhage
d) Macrosomia
e) Placenta abruption
Incorrect!!! Postterm pregnancies are associated with macrosomia, oligohydramnios, meconium aspiration,
uteroplacental insufficiency and dysmaturity. Gestational diabetes is associated with macrosomia, but is not
alone a risk factor for postterm pregnancies. There is no associated risk for preeclampsia in postterm
gestations.
A 32 year-old G2P1 is 41 weeks gestation. Her cervix is long and closed. She does not report contractions and states
there is good fetal movement. The patient strongly desires not to have an induction of labor. She would like to wait
until she goes into labor spontaneously. Which of the following treatment options is optimal at this time?
a) Perform a non-stress test (NST) and amniotic fluid index (AFI) twice a week with induction of labor for a
nonreactive nonstress test or oligohydramnios
b) Patient should perform daily fetal movement counts and proceed with induction for decreased fetal movement
c) Perform daily biophysical profiles and deliver if 4 or less
d) Immediate induction of labor
e) Immediate Cesarean section
Incorrect!!! Optimal management for the patient with an unfavorable cervix at 42 weeks gestation is
controversial. Induction of labor in a patient with a reactive tracing and an unfavorable cervix will
minimize any risk of antepartum fetal demise; however, the risk of Cesarean section is significantly
increased compared to a patient who goes into spontaneous labor. It is reasonable to follow a patient who is
41 weeks with antepartum fetal testing, such as twice weekly NSTs with amniotic fluid assessment. The
risk of fetal death is 1-2/1,000 high-risk pregnancies with a reassuring nonstress test, contraction stress test
or biophysical profile. The addition of amniotic fluid assessment may improve the predictive value of a
reactive NST and reduce the risk of antepartum fetal demise to even lower levels.
A 22 year-old G2P1 has a history of a previous postterm pregnancy. She delivered a 3500 g healthy male infant at
42-½ weeks gestation via a Cesarean section secondary to fetal distress. She is currently 15 weeks pregnant, based
on an irregular last menstrual period. What is the most appropriate management at this time?
a) Plan for a repeat Cesarean section at 38 completed weeks
b) Schedule for a repeat Cesarean section if she does not go into spontaneous labor by 40 weeks gestation.
c) Plan to admit the patient for an induction of labor (a VBAC) if she does not go into spontaneous labor by 41
weeks gestation.
d) Obtain a fetal ultrasound to date the pregnancy.
e) Start weekly non-stress tests and amniotic fluid indexes at 40 weeks gestation and proceed with either induction
of labor or Cesarean section for a nonreactive nonstress test or oligohydramnios or if patient has not gone into
spontaneous labor by 41 weeks gestation.
Incorrect!!! Approximately 50% of patients with a history of a postterm pregnancy will experience
prolonged pregnancy with the next gestation. The diagnosis of postterm pregnancy is based on the
establishment of an accurate gestational age. In a patient with irregular menses, it is important to obtain an
ultrasound prior to 20 weeks to accurately date the pregnancy. It is reasonable to allow a patient with
reassuring fetal surveillance to go past 41 weeks gestation. However, because of a prior Cesarean birth,
consideration should be given to delivery before 41 weeks.
A 24 year-old G2P1 at 42 weeks presents in early labor. At amniotomy, there is thick meconium. Her sister-in-law
had a procedure to dilute the meconium and she asks if this would be of benefit to her. Which of the following
statements is the most accurate regarding the benefits of amnioinfusion?
a) Decreases admissions to the neonatal intensive care unit
b) Decreases post maturity syndrome
c) Decreases repetitive variable decelerations
d) Decreases the risk for cesarean section
e) Decreases meconium below the vocal cords
Incorrect!!! Meconium staining of the amniotic fluid is 3-4 times more common in the postterm pregnancy.
This is likely due to two reasons: 1) greater length of time in utero allows for activation of a more mature
vagal system; and 2) fetal hypoxia. Amnioinfusion is a procedure where normal saline is infused into the
intrauterine cavity. Routine prophylactic amnioinfusion for thick meconium does not appear to decrease the
incidence of meconium aspiration syndrome or have an impact on neonatal outcomes. Based on current
literature, routine prophylactic amnioinfusion for meconium-stained amniotic fluid is not recommended.
Amnioinfusion remains a reasonable approach in the treatment of repetitive variable decelerations,
regardless of amniotic fluid meconium status.
A 32 year-old G2P1 is at 42 weeks gestation. Her prenatal course was uncomplicated and she had a first trimester
ultrasound confirming dates. Her cervix is 4 cm dilated and 100% effaced. She does not report contractions and
states there is good fetal movement. What is the next best step in the management of this patient?
a) Daily fetal movement counts
b) Twice weekly non-stress test (NST) and amniotic fluid index (AFI)
c) Daily biophysical profiles
d) Induction
e) Immediate cesarean section
Correct!!! Optimal management for the patient with a favorable cervix at greater than or equal to 41 weeks
gestation is delivery. Her dilation and effacement make it likely her induction will be successful. Induction
of labor in a patient with an unfavorable cervix increases the risk of Cesarean section significantly,
compared to a patient who goes into spontaneous labor. It is not advisable to follow a patient who is >42
weeks with antepartum fetal testing, such as twice weekly non-stress tests with amniotic fluid index, if the
gestational age is certain.
A 22 year-old G2P1 is at 42 weeks gestation dated by an ultrasound performed 5 weeks ago. Her cervix is long and
closed. She does not report contractions and states there is good fetal movement. She would like to wait until she
goes into labor spontaneously. Which of the following treatment options is optimal at this time?
a) Allow the patient to go into labor spontaneously
b) Perform an ultrasound to determine gestational age
c) Perform a non-stress test (NST) and amniotic fluid index (AFI) twice a week, with induction of labor for a
nonreactive NST or oligohydramnios.
d) Patient should perform daily fetal movement counts and proceed with induction for decreased fetal movement.
e) Perform daily biophysical profiles and deliver if 4 or less
Correct!!! Optimal management for the patient with an unfavorable cervix at an uncertain 42 weeks
gestation is arguable. Given the uncertainty of her dates, it is reasonable to follow this patient with
antepartum fetal testing, such as twice weekly nonstress tests with amniotic fluid index. The risk of fetal
death is 1-2/1000 high-risk pregnancies with a reassuring NST, contraction stress test or biophysical
profile. The addition of amniotic fluid assessment may improve the predictive value of a reactive NST and
reduce the risk of antepartum fetal demise to even lower levels. Ultrasound for gestational age
determination in the third trimester is not useful since the measurement error is +/- 3 weeks. Allowing
spontaneous onset of labor is OK, but not without some type of antepartum testing.
A 22 year-old G1P1 with no prenatal care delivered a 2100 g male infant who looks like an old man. The
pediatricians estimate the gestational age to be greater than 43 weeks. Of the following, which finding is not
associated with the dysmature postdate infant?
a) Peeling skin
b) Smooth skin
c) Meconium-stained
d) Long nails
e) Fragile
Incorrect!!! The incidence of infants with dysmaturity approaches 10% when the gestational age exceeds
43 weeks. Infants are described as withered, meconium stained, long-nailed, fragile and have an associated
small placenta. These infants are at great risk for stillbirth.
A 22 year-old G1 presents at 42 weeks gestation. Her cervix is long and closed. She does not report contractions and
states there is good fetal movement. You discuss the benefits of induction at this time versus waiting until she goes
into labor spontaneously. She agrees to proceed with an induction. Which of the following means should be used to
ripen the cervix in this patient?
a) Artificial rupture of membranes
b) Membrane stripping
c) Oxytocin infusion
d) Prostaglandin E1 tablet
e) RU486 (progesterone antagonist)
Incorrect!!! The American College of Obstetricians and Gynecologists (ACOG) recommendations for the
management of postterm pregnancy includes: patient records fetal kick counts, begin fetal surveillance at
42 weeks, using one of the following: NST, CST, biophysical profile and delivery for nonreassuring
testing. If the patient has a favorable cervix, induce at 42 weeks and, if the cervix is unfavorable, use
cervical ripening agents. Membrane stripping (digital separation of chorioamnion from lower uterine
segment) and artificial rupture of membranes cannot be performed in a patient with a closed cervix.
Prostaglandins applied locally are the most commonly-used cervical ripening agents. RU486 is not used for
cervical ripening.
Obstetrics - Exam 31
A 36 year-old G1 began prenatal care at 8 weeks gestation. At that time, the gestational age was confirmed by a
transvaginal ultrasound. She is now at 36 weeks gestation. Her previous medical history reveals hypertension for 8
years and Class F diabetes for 5 years (baseline proteinuria = 1 g). She smokes 1 pack of cigarettes per day. On
examination at 32 weeks gestation, her fundal height was 29 cm. At 33 weeks, biometry was consistent with 31-3/7,
EFW 1827g, 25th percentile. Today, ultrasound reveals limited fetal growth over the past 3 weeks. Biometry is
consistent with 31-5/7, EFW 1900 g, <10th percentile. What is the most likely etiology of the intrauterine growth
restriction in this case?
a) Genetic factors
b) Congenital anomaly
c) Tobacco use
d) Uteroplacental insufficiency
e) Perinatal infection
Correct!!! There is substantial evidence from experimental animal studies that suggests that alterations in
uteroplacental perfusion affect the growth and status of the fetus, as well as the placenta. This patient has
significant medical diseases that are affecting her vasculature and, ultimately, limiting the substrate
availability to the fetus. The vascular disease is evidenced by retinopathy and proteinuria. The other choices
above may all result in fetal growth restriction; however, they are not the most likely etiology in this
clinical scenario.
A 36 year-old female G1 presents for her prenatal care visit at 35 weeks gestation. She has good dating criteria that
were confirmed by a first trimester ultrasound. Her previous medical history is positive for hypertension and Type II
diabetes. You have been following fetal growth with serial ultrasounds. At this visit, ultrasound reveals limited fetal
growth over the past 3 weeks. Biometry is consistent with 32-5/7, EFW 2175 g, <10th percentile. What is the most
appropriate next test indicated in the management of this patient?
a) Amniotic fluid volume, umbilical artery Doppler systolic: diastolic ratio, nonstress test
b) Contraction stress test, percutaneous umbilical blood sampling
c) Amniocentesis for fetal lung maturity
d) None, observation
e) None, delivery is indicated
Correct!!! When a pregnancy is complicated by fetal growth restriction, various fetal physiologic
parameters require assessment. In growth-restricted pregnancies, oligohydramnios is frequently found. This
finding is presumably due to reduced fetal blood volume, renal blood flow and urinary output. Chronic
hypoxia is responsible for diverting blood flow from the kidney to organs that are more critical during fetal
life. The significance of the amniotic fluid volume with respect to fetal outcome has been well documented.
Ninety percent of patients with oligohydramnios delivered growth restricted infants. These infants
experienced a high rate of fetal compromise. The systolic/diastolic (S/D) ratio of the umbilical artery is
determined by Doppler ultrasound. An increase in the S/D ratio reflects increased vascular resistance. It is a
common finding in IUGR fetuses. A normal S/D ratio indicates fetal well-being. As vascular resistance
increases, the S/D ratio increases. With severe resistance, there is absence and ultimately reversal of enddiastolic flow. These findings are associated with an increased rate of perinatal morbidity and mortality,
and a higher likelihood of a long-term poor neurologic outcome. Options for antenatal testing include the
nonstress test, contraction stress test, and the biophysical profile. Any of these may be used in a growthrestricted fetus as a means of detecting possible or probable fetal asphyxia.
A 36 year-old G1 with type I diabetes, is diagnosed with intrauterine growth restriction at 33 weeks gestation. What
is the most appropriate next step in management?
a) Amniocentesis
b) Immediate delivery
c) Weekly ultrasounds to assess fetal growth
d) Antenatal testing of fetal well-being
e) Observation
Incorrect!!! Once intrauterine growth restriction is detected, the fetus needs to be evaluated periodically for
evidence of well-being until delivery is deemed necessary. This will result in once or twice weekly testing,
depending on the modality of assessment that is being used. Testing includes: Non-stress test (NST), where
the fetal heart beat is recorded over a period of at least 30 minutes, looking for accelerations with fetal
movement and the biophysical profile, which includes an ultrasound evaluation of fetal movement, fetal
tone, amniotic fluid and breathing, plus the nonstress test. NSTs should be performed twice weekly with at
least a weekly AFI. The BPP may be performed weekly. Ultrasound for fetal growth is not useful if more
frequent than every 2 weeks. An amniocentesis for fetal lung maturity can be considered at more advanced
gestational age.
A 28 year-old G1 at approximately 40 weeks gestation presents to triage with mild contractions. You measure her
fundal height at 34 cm. You are concerned about intrauterine growth restriction and you want to confirm her dates.
In reviewing her records, she reports first feeling fetal movements at 18 weeks gestation. The crown-rump length
measurements determined at 8 weeks and femur length at 20 weeks are consistent with 40 weeks gestation. Today’s
assessment reveals biometrics consistent with 34 weeks, amniotic fluid index of 1, and placental calcifications.
Which of the following is considered the most reliable method of determining the gestational age in this patient?
a) Crown-rump length measurement
b) Second trimester ultrasound
c) Quickening date
d) Third trimester composite biometry
e) Placental calcifications
Incorrect!!! In order to accurately confirm gestational age at term, one of the following criteria should be
met: Fetal heart tones have been documented for 20 weeks by a non-electronic fetoscope or for 30 weeks
by Doppler; it has been 36 weeks since a positive serum or urine HCG pregnancy test was performed by a
reliable laboratory; an ultrasound measurement of the crown-rump length, obtained at 6-12 weeks, supports
a gestational age of at least 39 weeks; and an ultrasound obtained at 13-20 weeks confirms the gestational
age of at least 39 weeks, determined by clinical history and physical examination.
A 34-year-old chronic hypertensive G1 comes to see you for a consultation at 34 weeks for size less than dates. Her
prenatal course has been uncomplicated and the genetic amniocentesis obtained at 15 weeks revealed a normal male.
Biometrics today reveal a biparietal diameter consistent with 33 weeks, abdominal circumference of 28 weeks, EFW
1600 g, less than 10th percentile, and an amniotic fluid index of 6. What is the most likely cause of fetal growth
restriction in this patient?
a) Chromosomal abnormality
b) Fetal infection with Rubella
c) Fetal infection with cytomegalovirus (CMV)
d) Uteroplacental insufficiency
e) Maternal infection with Varicella
Incorrect!!! Uteroplacental insufficiency can lead to asymmetric growth restriction. Asymmetric growth
restricted infants typically have a normal length, but their weight is below normal. On ultrasound, there is a
head-sparing effect, meaning that the head/brain is spared of the reduced blood flow that is a result of
uteroplacental insufficiency. Thus, the fetal abdomen measures below normal and the head remains very
close to normal. There is an asymmetrical growth pattern that is usually detected during the third trimester
and reflects uteroplacental insufficiency. Symmetric fetal growth restriction indicates that all fetal
measurements are below normal. As a general rule, such a finding indicates an intrinsic growth failure or an
“early event” secondary to one or more organ system anomalies, fetal aneuploidy or chronic intrauterine
infection. Infectious diseases are known to cause IUGR, but the number of organisms is poorly defined.
There is sufficient evidence to show a causal relationship between rubella and CMV infections and fetal
growth restriction. Other viruses to consider are syphilis and varicella. The protozoan toxoplasmosis results
in IUGR as well. There are no bacteria known to cause IUGR. Symmetrical growth restriction is usually
detected in the mid trimester of pregnancy.
A 32 year-old G1 is 36 weeks gestation. Ultrasound reveals limited fetal growth over the past 3 weeks. Biometry is
consistent with 30-5/7, EFW 1900 g, less than 10th percentile. Which of the following is LEAST likely to be
associated with this pregnancy?
a) Fetal demise
b) Perinatal demise
c) Polyhydramnios
d) Meconium aspiration
e) Polycythemia
Incorrect!!! This fetus had intrauterine growth restriction and with the exception of polyhydramnios, all of
the morbidities listed above may complicate intrauterine growth restriction. In general, the causes of
polyhydramnios relate to amniotic fluid production (abnormalities of the fetal urinary tract) and removal
(abnormalities of fetal swallowing and intestinal reabsorption of fluid). Some investigators report an
increase in fetal urinary output when there is hyperglycemia and increased renal osmotic load, thus
resulting in polyhydramnios. Abnormal fetal swallowing may be a result of a CNS or gastrointestinal tract
abnormalities, such as anencephaly, esophageal or duodenal atresia, diaphragmatic hernia or primary
muscular disease. Typically, polyhydramnios is not associated with asymmetric growth restriction (the
most common form of IUGR), since an asymmetric growth pattern reflects poor uterine blood flow and
limited substrate availability. In fact, oligohydramnios is frequently identified in pregnancies complicated
by fetal growth restriction.
7. A 32 year-old G1 is seeing you in consultation at 35 weeks gestation. Ultrasound reveals limited fetal growth over
the past 3 weeks. Biometry is consistent with 30-5/7, EFW 1900 g, less than 10th percentile. You counsel her about
short and long-term complications for her baby. This fetus is at increased risk for all of the following adult disorders
EXCEPT:
a) Cardiovascular disease
b) Chronic hypertension
c) Chronic obstructive lung disease
d) Diabetes
e) Osteoporosis
Correct!!! Epidemiologic studies indicate that fetal growth restriction is a significant risk factor for the
subsequent development of cardiovascular disease, chronic hypertension, chronic obstructive lung disease
and diabetes. Researchers suggest that the phenomenon of programming may be operable and that an
adverse fetal environment during a critical period of fetal growth helps to promote these adult diseases.
Osteoporosis risk factors include family history, slender body composition, prior history of osteoporosis,
Asian and Caucasian ethnicity, alcohol consumption, smoking, sedentary lifestyle, excess thyroid or
corticosteroids and use of anticonvulsant medications.
A 32 year-old G1 presents at 35 weeks gestation with decreased fetal movement. Her prenatal course has been
complicated by size less than dates. Serial ultrasounds show a decrease of the estimated fetal weight from 60th to
20th percentile. The nonstress test is reactive and the amniotic fluid index is 10. What is your next step in
management?
a) Continue with weekly non-stress tests
b) Obtain umbilical artery systolic diastolic ratio
c) Admission for daily fetal surveillance
d) Induction of labor today
e) Cesarean section today
Correct!!! Since the patient reported decreased fetal movement, a non-stress was performed and was
reassuring. The NST is based on the principle that when the fetus moves, its heartbeat normally accelerates.
The NST assesses fetal health through monitoring accelerations of the heart rate in response to the baby's
own movements. Amniotic fluid volume is important because a decreased amount raises the possibility that
the baby may be under stress. Since the fetus does not show growth restriction and fetal status was
reassuring, there are no indications for Doppler studies or delivery. In light of the dramatic decrease in
growth, it is reasonable to follow this patient with weekly non-stress tests.
Which of the following patients is most likely to deliver a macrosomic infant?
a) 23-year-old with Type 1 diabetes
b) 23-year-old with Type 2 diabetes with benign retinopathy
c) 23-year-old with gestational diabetes
d) 23-year-old with one abnormal value on 3 hr glucose tolerance test
e) 23-year-old a family history of diabetes
Incorrect!!! A patient with gestational diabetes is most likely to have a macrosomic infant. In diabetic
pregnancies, macrosomia is a result of high maternal blood sugars. In mothers with little or no end-organ
damage (i.e. vasculopathy), high blood sugars will also occur in the fetal system. As a result of the
increased glucose load, the fetus secretes more insulin. As a growth factor, increased insulin levels result in
increased fetal growth. In diabetic pregnancies, this extra fetal growth is seen as fat deposition in the malar
cheek pads, and in the paraspinal, abdominal and interscapular areas. This central deposition of fat is
characteristic of diabetic macrosomia and underlies the dangers associated with vaginal delivery in these
pregnancies. Maternal obesity – even outside the setting of diabetes – is also associated with an increased
rate of macrosomia. Pre-pregnancy weight and weight gain during pregnancy are each significant
predictors for fetal overgrowth. The longer a patient has diabetes, the more likely they have vascular endorgan disease. Thus, a patient who has had Type 2 diabetes for a very short period of time may have a
macrosomic infant if she has frequent hyperglycemia, yet a Type 2 diabetic patient with nephropathy and
retinopathy is more likely to have a growth restricted infant as a result of uteroplacental insufficiency. A
family history of diabetes is a risk factor for developing gestational diabetes.
A 33 year-old G2P1 presents at 34 weeks gestational age for consultation because ultrasound revealed a 3900 gm
fetus with biometrics consistent with 39 weeks. Her prior pregnancy was complicated by gestational diabetes and a
shoulder dystocia. Which of the following complications is this fetus at greatest risk?
a) Birth trauma
b) Hyperglycemia
c) Hypobilirubinemia
d) Hypothyroidism
e) Congenital anomalies
Correct!!! The fetus with enhanced general growth or macrosomia is defined by a birth weight at or above
the 90th percentile for gestational age. The condition can usually be ascribed to one of 3 etiologies:
enhanced growth potential (50-60%); abnormal maternal glucose homeostasis (35-40%); or
underestimation of fetal age (5%). Macrosomic newborns of diabetic mothers experience excessive rates of
neonatal morbidity, including birth trauma including shoulder dystocia and brachial plexus injury. These
infants have significantly higher rates of severe hypoglycemia and neonatal jaundice. Neonatal acidosis
occurs with poor glycemic control, thus increasing the incidence of fetal demise. While poorly controlled
pre-existing diabetes is associated with an increased risk of congenital anomalies, gestational diabetes is not
associated with increased risk of congenital anomalies.
Obstetrics - Exam 32
A 28 year-old G1 presents for prenatal care. Her periods have been irregular and she does not recall when the last
one occurred. She is healthy and denies any medical problems. The uterus is 10 weeks size and there are no adnexal
masses. At this point in time, what is the best way to date the pregnancy?
a) Qualitative Serum hCG
b) Quantitative Serum hCG
c) Ultrasound measurement, gestational sac
d) Ultrasound measurement, crown-rump length
e) Fetal heart tones heard with Doppler
Incorrect!!! All the above can potentially be used to help date a pregnancy; however, ultrasound
measurement of crown-rump length is considered the most reliable (+/- 4 to 5 days) in the first trimester.
Other means to date the pregnancy include: Fetal heart tones have been documented for 20 weeks by a nonelectronic fetoscope or for 30 weeks by Doppler; it has been 36 weeks since a positive serum or urine HCG
pregnancy test was performed by a reliable laboratory; an ultrasound measurement of the crown-rump
length obtained at 6-12 weeks supports a gestational age of at least 39 weeks; and an ultrasound obtained at
13-20 weeks confirms the gestational age of at least 39 weeks determined by clinical history and physical
examination. Clearly, these means are not as useful in early pregnancy, but in confirming the length of
pregnancy.
A 23 year-old G1 undergoes a vaginal delivery and an episiotomy is performed. Upon inspection, you notice that the
episiotomy has extended into the rectal sphincter and mucosa. What is the classification for this laceration?
a) 1st degree
b) 2nd degree
c) 3rd degree
d) 4th degree
e) Mediolateral episiotomy
Correct!!! A 4th degree laceration involves the rectal sphincter and rectal mucosa. Careful attention to
identification and repair of the disrupted ends of the sphincter is important to try to minimize the risk of
future fecal or flatus incontinence. 1st degree laceration involves only the vaginal mucosa, 2nd degree, the
vaginal fascia and perineum. A 3rd degree laceration involves the rectal partial or complete transection of
the rectal sphincter. A 4th degree laceration involves the external anal sphincter, the internal anal sphincter
and the rectal mucosa. A mediolateral episiotomy avoids the external anal sphincter.
A 30 year-old G2 P1 has an ultrasound at 42 weeks for size greater than dates. The fetus had an isolated enlarged
head measurement with a BPD of 11 cm, but otherwise appeared to have normal femur length and abdominal
circumference. Polyhydramnios is noted. The estimated fetal weight is 3900 g. There is a 10 cm lower uterine
segment fibroid protruding into the uterine cavity. The fetus is in the vertex presentation and the fetal head is above
the level of the uterine fibroid. Which of the following is an indication for primary Cesarean delivery in this patient?
a) Uterine fibroid
b) Fetal hydrocephalus
c) Polyhydramnios
d) Macrosomia
e) 42 weeks gestation
Correct!!! Uterine fibroids that are in the lower uterine segment may obstruct labor by preventing the fetal
head from entering the pelvis. A fetal head with measurements greater than 12 cm could benefit from
delivery by Cesarean. The fetus in the case presented does not necessarily have hydrocephalus. The fetus in
this case does not have macrosomia which may be defined as an estimated fetal weight greater than 4000
grams in a diabetic and greater than 4500 grams in a non-diabetic patient. Macrosomia defined as greater
than 4000 grams, as well as 42 weeks gestation, and polyhydramnios are not indications for primary
Cesarean section.
A 22 year-old G1, who is at 38 weeks gestation with an estimated fetal weight of 2500 g, presents in active labor.
She is completely dilated and effaced. The fetus is at plus 4 station and left occiput anterior with no molding. She
has an epidural and has been pushing effectively for 3 hours. She is exhausted. What is the next step in
management?
a) Allow to continue pushing until the baby delivers
b) Start Oxytocin to strengthen contractions
c) Discontinue the epidural
d) Forceps-assisted vaginal delivery
e) Cesarean section
Correct!!! This patient meets all the requirements for an operative vaginal delivery. Forceps application
requires complete cervical dilation, head engagement, vertex presentation, clinical assessment of fetal size
and maternal pelvis, known position of the fetal head, adequate maternal pain control and rupture of
membranes. Strict adherence to the guidelines suggested by the American College of Obstetricians and
Gynecologists (ACOG) for low forceps delivery does not increase the fetal or maternal risks when
performed by an experienced operator.
You are counseling a 22 year-old, G1, who is at 38 weeks gestation. She has been pushing for 4 hours and you
recommend a trial of forceps. She asks whether a vacuum-assisted delivery is associated with a lower incidence of
maternal and fetal trauma. Which of the following is less likely to occur during a vacuum delivery?
a) Maternal lacerations
b) Fetal cephalohematoma
c) Neonatal lateral rectus paralysis
d) Neonatal hyperbilirubinemia
e) Neonatal retinal hemorrhage
Incorrect!!! Newer forms of vacuum extractors cause less maternal discomfort as they are applied to the
vertex of the fetal head and do not take up additional space in the maternal pelvis. If properly applied, this
leads to a decreased rate of maternal lacerations. Fetal and neonatal complications related to vacuum use
include lacerations at the edges of the vacuum cup, particularly if torsion is applied. Torsion may also lead
to separation of the fetal scalp from the underlying structures can cause a cephalohematoma and places the
fetus at risk of jaundice. Transient neonatal lateral rectus paralysis has been found to occur more frequently
in vacuum-assisted deliveries, but because the paralysis resolves spontaneously, it is unlikely to be of
clinical importance.
A 19 year-old G1 presents at 41 weeks with a fever, spontaneous ruptured membranes and no contractions. Her
placenta slightly covering the os. Why should this patient be delivered by Cesarean section?
a) Chorioamnionitis
b) Post-term pregnancy
c) Oligohydramnios
d) Placenta covering the cervical os
e) Spontaneous ruptured membranes not in labor
Correct!!! Delivery is indicated in this patient because of chorioamnionitis. Her contraindication to
induction of labor is the placenta covering the os (placenta previa). Post-term pregnancies,
chorioamnionitis, oligohydramnios, and term premature rupture of membranes are all accepted indications
for induction and delivery if the patient and her baby are candidates for initiation of labor.
A 22 year-old G1 at 38 weeks gestation presents in labor. Her prenatal course and past history are uncomplicated.
She is having regular contractions and, on examination, the cervix is 5 cm, 100% effaced and fetal head at +1
station. The fetal heart rate tracing is shown below. What is the most likely interpretation?
a) Normal reassuring
b) Bradycardia
c) Early deceleration
d) Variable decelerations
e) Late decelerations
Incorrect!!! Early decelerations are thought to represent the fetal response to head compression during the
contraction and the fetal heart rate inversely mirrors the changes noted during the contraction. Variable contractions
are thought to be due to cord compression and can occur at any position in relation to a contraction. Generally, they
have an abrupt onset and return of the fetal heart rate deceleration to the baseline heart rate. Late decelerations are
thought to represent uteroplacental insufficiency. The deceleration of the fetal heart rate occurs at or after the peak
of the uterine contraction and returns to baseline after complication of the contraction. Bradycardia is defined as
fetal heart rate less than 110 beats per minute.
A 39 year-old G1 presents in labor at term. The estimated fetal weight is 3200 g. She is 10 cm dilated with left
sacrum anterior at +2 station. Which of the following is the most appropriate next step in the management of this
patient?
a) Attempt external version
b) Attempt internal version
c) Apply forceps
d) Apply a vacuum
e) Recommend a cesarean section
Incorrect!!! Most recent data suggests that breech infants delivered vaginally are at higher risk for neonatal
complications. Therefore, it would be recommended that this patient undergo a Cesarean section, especially
since this is her first pregnancy. External cephalic version is contraindicated in active labor.
A 39 year-old G5P4 is referred to you at 36 weeks for polyhydramnios. The estimated fetal weight is 1900 g, fetal
measurements are symmetric and the amniotic fluid index is 28. Her prenatal course was uneventful; however, she
declined genetic testing. She is not in labor, nor does she report any respiratory difficulty. Which of the following
would yield the most information to plan the delivery?
a) Maternal fibrinogen level
b) Contraction stress test
c) Kleihauer-Betke test
d) Therapeutic amniocentesis
e) Fetal karyotype
Incorrect!!! Amniocentesis is indicated to obtain the fetal karyotype. Symmetric fetal growth restriction in
the presence of polyhydramnios may be associated with Trisomy 18. Amniotic fluid should be obtained for
fluorescence in situ hybridization (FISH), which is useful for detecting aneuploid conditions. A FISH
result, which will provide information regarding a diagnosis of trisomy 13, 18, and 21, will generally be
available within 24 hours. The fluid should also be sent for a fetal karyotype which may take 10-14 days.
Therapeutic amniocentesis would be used if there were respiratory compromise or preterm labor, both
caused by polyhydramnios. Maternal fibrinogen level and Kleihauer-Betke levels are useful in assessing
abruption. A contraction stress test is useful for assessing fetal well-being.
A 36 year-old woman requests prenatal diagnosis. She is healthy and excited about finally getting pregnant. She is
interested in genetic counseling and asks about the advantages of chorionic villus sampling versus amniocentesis.
Which of the following is true when chorionic villus sampling is compared to amniocentesis?
a) Reduced post procedure loss rate
b) Performed earlier
c) More likely to obtain an adequate sample
d) Lower rate of procedure related birth defects
e) Performed in isoimmunized pregnancies
Correct!!! Chorionic villus sampling (CVS) is a prenatal test that can detect genetic and chromosomal
abnormalities of a fetus. The loss rate with amniocentesis is quoted as 0.5% vs. ~1 to 3% for chorionic
villus sampling. CVS is performed between 10 and 12 weeks gestation, while amniocentesis is performed
after 15 weeks. Early CVS (<10 weeks gestation) is associated with an increase in rare limb abnormalities.
It is more likely that a CVS will involve multiple attempts, a failure to obtain an adequate sample of cells
and the woman requiring a repeat test later on, when compared with amniocentesis. Pregnancies
complicated by isoimmunization can be followed by serial assessment of the amniotic fluid for bilirubin.
REI - Exam 45
A 32 year-old G0 woman presents with irregular menses occurring every 6-8 weeks for the past 8 months. The
bleeding alternates between light and heavy. Her irregular menses were treated successfully with
Medroxyprogesterone Acetate (MPA), 10 mg every day, taken for 10 days each month. By which mechanism does
the MPA control her periods?
a) Stimulates rapid endometrial growth and regeneration of glandular stumps
b) Converts endometrium from proliferative to secretory
c) Promotes release of Prostaglandin F2á
d) Regenerates functional layer of the endometrium
e) Decreases luteal phase inhibin production
Correct!!! Patients with anovulatory bleeding have predominantly proliferative endometrium from
unopposed stimulation by estrogen. Progestins inhibit further endometrial growth, converting the
proliferative to secretory endometrium. Withdrawal of the progestin then mimics the effect of the
involution of the corpus luteum, creating a normal sloughing of the endometrium. Stimulation of rapid
endometrial growth, conversion of proliferative to secretory endometrium, and regeneration of the
functional layer describe effects of estrogen on the endometrium. Inhibin is increased in the luteal phase.
A 41 year-old G3P3 woman reports heavy menstrual periods occurring every 26 days lasting 8 days. The periods
have been increasingly heavy over the last three months. She reports soaking through pads and tampons every 2
hours. She has a history of three uncomplicated spontaneous vaginal deliveries and a tubal ligation following the
birth of her last child. On pelvic examination, the cervix appears normal and the uterus is normal in size. Which of
the following tests or procedures would be most useful in further evaluation of this patient’s complaint?
a) Follicle stimulating hormone level
b) Prolactin level
c) Coagulation studies
d) Pelvic ultrasound
e) CT of the pelvis
Correct!!! A pelvic ultrasound would image the endometrium and rule in or rule out uterine, and possibly
endometrial, polyps. In the absence of menopausal symptoms, FSH is unlikely to be helpful. The patient is
unlikely to have a coagulation disorder, as she has had three spontaneous vaginal deliveries without
postpartum hemorrhage. CT is not as accurate a modality for evaluating the uterus as ultrasound.
Hyperprolactinemia is found with prolactin-secreting adenomas associated with amenorrhea.
A 14-year old G0 adolescent reports menarche six months ago, with increasingly heavy menstrual flow causing her
to miss several days of school. Three months ago, her pediatrician started her on oral contraceptives to control her
menstrual periods, but she continues to bleed heavily. Her previous medical history is unremarkable. The patient has
a normal body habitus for her age. Appropriate breast and pubic hair development is present. Her hemoglobin is 9.1
mg/dl, hematocrit 27.8%, urine pregnancy test negative. Which of the following etiologies for menorrhagia is most
likely the cause of her symptoms?
a) Uterine leiomyoma
b) Thyroid disorder
c) Coagulation disorder
d) Endometrial hyperplasia
e) Chronic Endometritis
Correct!!! Disorders of clotting may present with menstrual symptoms in young women, with Von
Willeberand disease being most common. Leiomyomas typically present in women in their 30s and 40s.
Endometrial hyperplasia can occur in younger anovulatory patients, but the short duration of this patient’s
symptoms makes this less likely. She does not have any signs of infection.
A 42 year-old G2P2 woman presents with menorrhagia. Her menses occur every 28 days. She noted that the
bleeding became worse after the birth of her last child and it has increased in severity over the last two years. She is
currently taking combination oral contraceptives, which were started six months ago by her family physician. On
pelvic examination, the uterus is normal in size and no adnexal masses are noted. Her hemoglobin is 11.9 mg/dl,
hematocrit 35.1%. Pelvic sonography reveals a normal appearing uterus and a normal appearing endometrial echo.
The ovaries appear normal without adnexal masses. Endometrial biopsy reveals a secretory endometrium. The
patient is unhappy with the current therapy and desires definitive treatment. Which of the following would be the
best long-term treatment?
a) Endometrial ablation
b) Dilatation and Curettage
c) Laparoscopic myomectomy
d) Leuprolide acetate
e) Cyclic Progestins
Correct!!! Endometrial ablation is an effective surgical procedure for dysfunctional uterine bleeding
(DUB). Dilation and curettage is not effective as a therapy for DUB in the long-term. Leuprolide should not
be used for more than 6 months because of the risk of osteoporosis. Cyclic progestins are unlikely to work
any better than oral contraceptives with ovulatory DUB. Pelvic exam and ultrasound did not reveal any
fibroids, making a myomectomy unnecessary.
A 35 year-old G0 woman presents with irregular menstrual periods occurring every 6-12 weeks with occasional
inter-menstrual bleeding. Currently, she has been bleeding daily for the last 4 weeks. She reports that her periods
have always been irregular, but have become more so with heavier flow and cramping in the last year. She is
sexually active with one partner. On physical exam, she is morbidly obese with no abnormalities detected on pelvic
exam. Which of the following is the most appropriate next step in the management of this patient?
a) Luteinizing hormone level (LH)
b) Pelvic CT
c) Testosterone level
d) Follicle stimulating hormone level (FSH)
e) Endometrial biopsy
Incorrect!!! Endometrial biopsy should be performed to rule out endometrial hyperplasia or carcinoma
given the history of irregular bleeding, coupled with the increased risk of these diagnoses in morbidly obese
patients. A pelvic CT would not evaluate for the presence of endometrial neoplasia. LH and FSH levels
would not aid in the diagnostic workup and testosterone levels would not be useful, unless signs of
hirsutism or virilization are present.
A 34 year-old G2P2 woman presents with inter-menstrual bleeding for one year. The bleeding typically occurs 2
weeks after her menses and last 2-3 days. The symptoms began 1 year ago and the bleeding has not changed
recently. She is currently taking oral contraceptives. On pelvic examination, the cervix appears normal and the
uterus is normal in size and shape. Her urine pregnancy test is negative; an endometrial biopsy is negative for
neoplasia. Which of the following tests or procedures would be indicated for further work-up?
a) Prolactin level
b) Progesterone level
c) Hysterosalpingogram (HSG)
d) Pelvic ultrasound
e) Colposcopy
Incorrect!!! Intermenstrual bleeding is frequently caused by structural abnormalities of the endometrial
cavity, such as myomas, polyps or malignancy. An ultrasound would be helpful as the next step in
diagnosis. Although an HSG might reveal structural abnormalities, it is too invasive for a next step. A
colposcopy would not be helpful in the diagnosis, nor would obtaining a Prolactin level, as it would be
indicated for the evaluation of anovulatory bleeding. Progesterone levels are not helpful in a patient on oral
contraceptives.
A 36 year-old G0 woman presents due to increasing facial hair growth and irregular menstrual cycles. She has
gained 40 pounds over the last 3 years. Her symptoms began 3 years ago and have gradually worsened. She has
never been pregnant and is not currently on any medications. On physical exam, she is overweight with dark hair
growth at the sideburns and upper lip. The pelvic exam is normal. Which of the following would you expect to find
in this patient?
a) Decreased luteinizing hormone levels
b) Elevated free testosterone
c) Decreased prolactin level
d) Increased ovarian estrogen production
e) Elevated 17-hydroxyprogesterone
Correct!!! This patient likely has polycystic ovarian syndrome (PCOS). PCOS patients have testosterone
levels at the upper limits of normal or slightly increased. Free testosterone (biologically active) is elevated
often because sex hormone binding globulin is decreased by elevated androgens. LH is increased in
response to increased circulating estrogens fed by an elevation of ovarian androgen production. Insulin
resistance and chronic anovulation are hallmarks of PCOS. Prolactin levels may be elevated in amenorrhea
but are not elevated in patients with PCOS.
An 18 year-old woman comes to the office due to vaginal spotting for the last two weeks. Her menstrual periods
were regular until last month, occurring every 28-32 days. Menarche was at age 13. She started oral contraceptives 3
months ago. On pelvic examination, the uterus is normal in size, slightly tender with a mass palpable in the right
adnexal region. No adnexal tenderness is noted. Which of the following tests is the most appropriate next step in the
management of this patient?
a) Endometrial biopsy
b) Coagulation studies
c) Pelvic sonography
d) Abdominal CT scan
e) Urine pregnancy test
Incorrect!!! It is vitally important to rule out pregnancy in the evaluation of abnormal uterine bleeding.
Although a bleeding disorder could be a possibility, it is less likely with her previous history of normal
cycles. Sonography could be considered as a next step if the pregnancy test is negative in order to evaluate
the adnexal finding. Abdominal CT would not be performed in this patient unless advanced adnexal
pathology was found on pelvic sonography. Endometrial biopsy would rarely be indicated in a teen with
abnormal bleeding, unless morbidly obese and anovulatory.
A 45 year-old G2P2 woman comes to the office because of heavy and irregular menstrual periods. The heavy
periods started three years ago and have gradually worsened in amount of flow over time. The periods are interfering
with her daily activities. The patient has had two spontaneous vaginal deliveries, followed by a tubal ligation 3 years
ago. On pelvic examination, the cervix appears normal and the uterus is normal in size without adnexal masses or
tenderness. A urine pregnancy test is negative. TSH and prolactin levels are normal. Hemoglobin is 12.5 mg/dl. On
pelvic sonography, she has a normal size uterus and a 2 cm simple cyst on the right ovary. Endometrial biopsy is
consistent with a secretory endometrium; no neoplasia is found. What is the most likely diagnosis in this patient?
a) Polycystic ovarian syndrome
b) Mid-cycle bleeding
c) Dysfunctional uterine bleeding
d) Benign cystic teratoma
e) Ovarian cancer
Correct!!! Dysfunctional uterine bleeding is defined as irregular or increased menstrual bleeding without
identified etiology. This patient had a complete workup, including TSH, Prolactin, pelvic ultrasound and
endometrial biopsy, which were all normal. Mid-cycle bleeding at the time of ovulation is due to the drop
in estrogen. Ovarian teratomas are unlikely to present with abnormal menses. They typically present with
abdominal or pelvic pain which may be associated with torsion. The 2 cm cyst is a functional cyst and is a
common finding in ovulatory patients.
A 35 year-old G2P2 woman comes to the office due to heavy menstrual periods. The heavy periods started three
years ago and have gradually worsened in amount of flow and duration. The periods are now interfering with her
daily activities. The patient had two spontaneous vaginal deliveries. She smokes 1 pack of cigarettes per day. On
pelvic examination, the cervix appears normal and the uterus is normal in size, without adnexal masses or
tenderness. A urine pregnancy test is negative. TSH and prolactin levels are normal. Hemoglobin is 12.5 mg/dl. On
pelvic sonography, a 2 cm submucosal leiomyoma is noted. An endometrial biopsy is consistent with a secretory
endometrium; no neoplasia is found. Which of the following would be the best therapeutic option for this patient if
she desires to have another child?
a) Hysteroscopy with myoma resection
b) Laparoscopic myomectomy
c) Endometrial ablation
d) Oral contraceptives
e) Dilatation and Curettage
Incorrect!!! Hysteroscopic myomectomy preserves the uterus, while removing the pathology causing the
patient’s symptoms. A laparoscopic approach is not indicated as the myoma is submucosal and not
accessible using a laparoscopic approach. Endometrial ablation destroys the endometrium and can create
Asherman’s syndrome, thus it is reserved for patients who have completed childbearing. Dilation and
curettage is unlikely to remove the myoma and is a blind procedure (carried out without direct
visualization). Oral contraceptives would typically help with heavy menses, but are contraindicated in this
patient, who is over 35 and smokes.
REI - Exam 43
A 24-year-old woman comes into the office because she has not had her menses for 6 months. She is in good health
and not taking any medications. She is not sexually active. She does well in graduate school, despite her demanding
new program. Her height is 5’ 6” and her weight is 104 pounds. Her blood pressure is 140/80, with other vital signs
being stable. Her physical examination, including a pelvic examination, is completely normal. What is the most
likely reason for her amenorrhea?
a) Ovarian dysfunction
b) Undiagnosed diabetes mellitus
c) Obstruction of the genital outflow tract at the level of the endocervical canal
d) Hypothalamic-Pituitary Dysfunction
e) Pregnancy
Correct!!! Anorexia nervosa or significant weight loss may cause hypothalamic-pituitary dysfunction that
can result in amenorrhea. A lack of the normal pulsatile secretion of gonadotropin releasing hormone
(GnRH) leads to a decreased stimulation of the pituitary gland to produce follicle stimulating hormone
(FSH) and luteinizing hormone (LH.) This leads to anovulation and amenorrhea. Diabetes mellitus does not
primarily result in amenorrhea. While ovarian dysfunction/failure, obstruction of the genital outflow tract
and pregnancy cause amenorrhea, they are unlikely in this case.
A 23-year-old woman presents to the office because she has not had any menses for 4 months. She has a long
history of irregular menstrual cycles since menarche at age 14. She is otherwise in good health and is not taking any
medications. She is thin and has chronic anxiety. Her beta-HCG is < 5, and her prolactin and TSH levels are normal.
You suspect hypothalamic-pituitary dysfunction as the cause of her amenorrhea. What would be the next best
diagnostic test to order?
a) Estrogen level
b) Transvaginal pelvic ultrasound
c) Gonadotropin releasing hormone level
d) Follicle stimulating hormone and luteinizing hormone levels
e) Progesterone level
Correct!!! The causes of hypothalamic-pituitary amenorrhea are functional, (weight loss, obesity, excessive
exercise) drugs, (marijuana and tranquilizers) neoplasia, (pituitary adenomas) psychogenic, (chronic
anxiety, anorexia nervosa) and certain other chronic medical conditions. In this case, the next step to make
a diagnosis is to obtain FSH and LH levels, which would be expected to be in the low range. You already
know that her prolactin level is normal, which is consistent with your diagnosis. Prolactin would be
elevated with a prolactin-secreting pituitary adenoma.
A 23-year-old woman presents to the office because she has not had any menses for 4 months. She has a long
history of irregular menstrual cycles since menarche at age 14. She is in good health and is not taking any
medications. She is sexually active with her partner of 6 months, and uses condoms for contraception. She is 5’4”
tall and weighs 170 pounds. On exam, she has noticeable hair growth on her upper lip and chin. The rest of her
examination including a pelvic exam is normal. Her beta-HCG is < 5, and her prolactin and TSH levels are normal.
In addition to recommending weight loss, what is the most appropriate next step in the management of this patient?
a) Initiate treatment with GnRH agonist
b) Initiate treatment with steroids
c) Initiate treatment with oral contraceptives
d) Check Progesterone levels
e) Check Cortisol levels
Correct!!! Oral contraceptives (OCPs) are the most appropriate treatment for this patient who most likely
has the diagnosis of polycystic ovarian syndrome (PCOS.) The constellation of findings support this
clinical diagnosis (irregular cycles, obesity, and hirsutism). Because she is using condoms for contraception
and is sexually active, OCP’s would help regulate her cycles and further provide effective contraception.
When she desires pregnancy, however, she will most likely need treatment for ovulation induction due to
the anovulatory cycles as the leading cause of her oligomenorrhea.
A 32-year-old G0 woman presents to the office because she has not had any menses for the last 3 months. She has a
long history of irregular cycles, 26 to 45 days apart, for the last two years. She is otherwise in good health and is not
taking any medications. She is sexually active with her husband and uses condoms for contraception. She is 5 feet 4
inches tall and weighs 140 pounds. On exam, she has a slightly enlarged, non-tender uterus. There are no adnexal
masses. What is the most appropriate next step in the management of this patient?
a) Perform a pelvic ultrasound
b) Check a TSH level
c) Check progesterone and estrogen levels
d) Perform a urine pregnancy test
e) Check FSH and LH levels
Correct!!! Pregnancy is the most common cause of amenorrhea. It is important to consider it early in the
workup to avoid unnecessary tests, procedures and treatments that may be contraindicated during
pregnancy. Although the patient has a history of irregular cycles and is using condoms for contraception, it
is important to first rule out pregnancy before initiating further work-up.
A 33-year-old woman presents to the office because she has not had any menses for the past 12 months. She also
reports a recent onset of dyspareunia, causing her to feel anxious about having intercourse. She had menarche at age
15. Her cycles were normal until two years ago when she began skipping menses. She is otherwise in good health.
She is 5’4”tall and weighs 130 pounds. Her physical examination is completely normal; TSH and prolactin levels
are normal. What is the most likely cause of this patient’s amenorrhea?
a) Psychogemic
b) Genital tract outflow obstruction
c) Pregnancy
d) Premature ovarian failure
e) Pituitary adenoma
Incorrect!!! The patient’s symptom of dyspareunia is likely caused by vaginal dryness, which is associated
with estrogen deficiency. Hypergonadotropic amenorrhea is the result of ovarian failure or follicular
resistance to gonadotropin stimulation. The history, physical exam and labs make the other possibilities less
likely: psychogenic disorder, (no chronic anxiety or anorexia nervosa) outflow obstruction, (previously had
periods) pregnancy, (duration too long and normal exam) or a pituitary tumor (normal labs.)
A 17-year-old woman is brought in by her mother because she has not yet had any menses. She is otherwise in good
health, but recently has been experiencing cyclical lower abdominal cramping. She reports never being sexually
active. She is 5’6” tall and weighs 120 pounds. On examination, her breasts are Tanner Stage IV. She has some
suprapubic tenderness on abdominal exam. Her pelvic exam reveals normal external genitalia, but there was
difficulty inserting a speculum due to patient’s discomfort. Beta-HCG < 5. What is the most likely diagnosis in this
patient?
a) Obstruction of the genital outflow tract
b) Műllerian agenesis
c) Hypothalamic-pituitary dysfunction
d) Psychogenic amenorrhea
e) Ovarian dysfunction
Correct!!! This patient’s primary amenorrhea, with normal secondary sexual characteristics development
and cyclical abdominal pain, points to an anatomical cause of amenorrhea, which is preventing menstrual
bleeding. An imperforate hymen commonly causes this and the treatment is surgical. In Mϋllerian agenesis,
or Mayer-Rokitansky-Kϋster-Hauser syndrome, there is congential absence of the vagina and usually an
absence of the uterus and fallopian tubes. Ovarian function is normal and all the secondary sexual
characteristics of puberty occur at the appropriate time.
A 28-year-old G0 female patient presents for an annual examination. She is in good health and not taking any
medications. She had a history of normal cycles until 6 months ago, when she stopped having menses after starting
an intense exercise regimen. She is 5’6” tall and weighs 120 pounds. Her examination is completely normal. Her
pregnancy test is negative. What is the underlying pathophysiology of the disease process in this patient?
a) Psychogenic amenorrhea
b) Premature ovarian failure
c) Hypothalamic amenorrhea
d) Androgen excess
e) Hyperthyroidism
Incorrect!!! Amenorrhea associated with exercise falls under the category of hypothalamic amenorrhea,
which causes chronic anovulation. Although it may be related to energy requirements, alterations in the
hypothalamic-pituitary-ovarian axis have been described in athletic women. The patient’s history and
physical exam make ovarian failure, androgen excess and hyperthyroidism less likely, although a TSH
level would still be recommended.
A 31-year-old G3P0 female patient comes to the office because she has not had any menses in the last 6 months. She
is otherwise in good health and is not taking any medications. She had a miscarriage 7 months ago, which was
complicated by an infection and required antibiotics and a dilation and curettage procedure. Her examination is
normal. Her laboratory results show a beta-HCG < 5 and normal TSH and Prolactin levels. What is the most likely
underlying cause of this patient’s amenorrhea?
a) Chronic endometritis
b) Multiple miscarriages
c) Hypothalamic pituitary amenorrhea
d) Asherman’s Syndrome
e) Sheehan’s Syndrome
Incorrect!!! Asherman’s syndrome can be caused by curettage or endometritis. The intrauterine synechiae
or adhesions result from trauma to the basal layer of the endometrium, which causes amenorrhea. Ovarian
failure does not contribute to this disease process. Sheehan’s syndrome is typically due to severe
postpartum hemorrhage leading to pituitary apoplexy.
A 23-year-old female college student presents to the office because she has not had any menses for 10 months. She
had menarche at age 14 and normal regular menses every 28 days until this year. She is in good health and not
taking any medications. She is 5’4” tall and weighs 130 pounds. Her examination, including a pelvic exam, is
normal. Beta-HCG is < 5, and TSH is 5.0 mU/L (normal 0.35-6.7 mU/L). What is the most appropriate next
diagnostic test to help determine the cause of amenorrhea in this patient?
a) Serum 17-Hydroxyprogesterone levels
b) Serum Prolactin levels
c) Pelvic ultrasound
d) Serum LH and FSH levels
e) Brain MRI
Incorrect!!! Measurement of serum prolactin level is part of the initial laboratory assessment for a patient
with amenorrhea and no other symptoms or findings on physical exam. A prolactinoma is the most
common pituitary tumor causing amenorrhea. Galactorrhea is often present with a hyperprolactinemic
cause of anovulation and amenorrhea. A pelvic ultrasound is not typically helpful in a young patient with a
normal pelvic examination. A brain MRI might be indicated if prolactin levels return elevated. 17Hydroxyprogesterone, LH and FSH levels might need to be obtained in the work-up of this patient, if other
tests return normal.
A 22-year-old G0 female student comes to the office because she has not had any menses since discontinuing her
oral contraceptive pills 5 months ago. She has been on the pill for the last 6 years and had normal menses every 28
days while taking them. She is in good health and not taking any medications. She is 5’4” tall and weighs 130
pounds. Her examination, including a pelvic exam, is normal. Which of the following history elements would be
most useful in determining the cause of amenorrhea in this patient?
a) Age at first intercourse
b) History of sexually transmitted diseases
c) Parity
d) History of oligo-ovulatory cycles
e) Recent history of weight loss
Incorrect!!! Since most women resume normal menstrual cycles after discontinuing oral contraceptive pills
(OCPs,) they are not usually considered the cause of the amenorrhea. A history of irregular cycles prior to
pill use may increase the risk of amenorrhea upon discontinuation. This is sometimes referred to as “post
pill amenorrhea.” A complete work-up should be performed to properly find the cause. Although the other
history elements are all important components of a complete gynecological history, they are not helpful to
find the etiology of amenorrhea in this patient. Significant weight loss might cause amenorrhea; however,
this patient still has normal body mass index, which makes it unlikely cause of amenorrhea.
REI - Exam 49
A 32 year-old G2P2 woman is concerned about symptoms associated with her menstrual cycle. During the second
half of her cycle, she feels anxious, sad and has difficulty sleeping. She has done research on the Internet and
believes she suffers from premenstrual dysphoric disorder (PMDD). Which of the following symptoms of the patient
is most consistent with this diagnosis?
a) Cyclic constellation of symptoms during the follicular phase
b) Cyclic occurrence of a minimum of described symptoms and interference in social functioning
c) Chronic, mild depressive symptoms that have been present for many years
d) Depressed mood or the loss of interest or pleasure in activities
e) Anxiety/nervousness interfering in social functioning
Incorrect!!! PMDD is a psychiatric diagnosis, describing a severe form of premenstrual syndrome in which
the diagnostic criteria include 5 out of 11 clearly defined symptoms, functional impairment and prospective
charting of symptoms. All three areas of symptoms need to be represented for the diagnosis of PMDD.
A 37 year-old G1P1 woman suffers from severe mood swings the week before her menstrual cycle. The mood
swings resolve after she stops bleeding. You diagnose her with premenstrual syndrome (PMS) after obtaining
further history and a normal examination. In addition to exercise, which of the following might be suggested to help
decrease this patient’s symptoms?
a) Folic acid
b) Ginkgo
c) Fish oil
d) Vitamin B6
e) Potassium
Correct!!! Vitamin deficiency of A, E and B6 have been associated with an increase in PMS. Replacement
of these vitamins might improve PMS symptoms and avoid further medical therapy.
A 42 year-old G2P2 woman complains of bloating, mood swings and irritability the week prior to her menses. She is
convinced that something is wrong with her hormone levels. In addition to a complete physical examination, which
of the following diagnostic tools would provide information to accurately determine the diagnosis?
a) Pelvic ultrasound
b) Estradiol level
c) CAGE questionnaire
d) Prospective symptom calendar
e) Mini mental status examination
Incorrect!!! A calendar of symptoms can clarify if there is a cyclic or constant nature of the symptoms.
Often women will mistakenly attribute their symptoms to their menstrual cycle. Different self-reporting
scales have been written to assist patients track their symptoms. Because she is menstruating regularly,
there is no role for obtaining serum hormone levels. The CAGE questionnaire is a screening test for alcohol
dependence.
A 22 year-old G0 college student returns for follow-up of mood swings and difficulty concentrating on her
schoolwork the week before her menses for the past 12 months. Her past medical history is unremarkable and
physical examination is normal. Which of the following would be an appropriate treatment option for this patient?
a) Oral contraceptive pills
b) Reassurance and observation
c) Methylphenidate (Ritalin)
d) Gabapentin
e) Ginkgo
Correct!!! This woman has premenstrual syndrome (PMS) with symptoms that warrant treatment. Patients
with PMS and premenstrual dysphoric disorder (PMDD) experience adverse physical, psychological and
behavioral symptoms during the luteal phase of the menstrual cycle. PMS is characterized by mild to
moderate symptoms, while PMDD is associated with severe symptoms that seriously impair usual daily
functioning and personal relationships. Mild symptoms of PMS often improve by suppressing the
hypothalamic-pituitary-ovarian axis with oral contraceptive pills. Ritalin and Ginkgo are not effective
treatments for PMS. Gabapentin is used for neuropathic pain and will not help alleviate her symptoms.
A 32 year-old G2P2 woman complains of depression, weight gain and premenstrual bloating. She has suffered from
these symptoms for 18 months and they have not responded to dietary changes and avoidance of alcohol and
caffeine. Her only medications are multivitamins and herbs to increase her energy. She is very concerned about
fatigue that often interferes with caring for her two children. A prospective symptom diary completed by the patient
indicates mood symptoms, fatigue and bloating almost every day of the past two months and regular menstrual
cycles, accompanied by breast tenderness. She denies feelings of wanting to hurt herself or others, and she has been
able to carry out her normal responsibilities. Physical examination is unremarkable. Which of the following
conditions is the most likely explanation for this patient’s symptoms?
a) Panic disorder
b) Anxiety disorder
c) Diabetes mellitus
d) Hypothyroidism
e) Premenstrual dysphoric disorder
Correct!!! Symptoms of hypothyroidism can mimic typical symptoms of PMS, but symptoms occur more
constantly throughout the cycle. Diagnosis involves complete work-up to rule out medical illnesses,
including hypothyroidism. Although fatigue can be associated with diabetes mellitus, this patient lacks
other common symptoms such as weight loss, thirst, frequent urination, or neurological symptoms, thereby
making this diagnosis less likely.
A 37 year-old G3P3 complains of severe premenstrual symptoms for the past two years. She finds her mood swings
and irritability troubling and requests a hysterectomy, as she thinks that this procedure will alleviate her symptoms.
Past medical history is only remarkable for high cholesterol and her physical examination, including pelvic
examination, is normal. The patient’s physician does not recommend a hysterectomy. Which of the following is the
most likely explanation for the physician’s recommendation not to perform a hysterectomy in this patient?
a) Surgical risks of hysterectomy are too high
b) Past medical history
c) Influence of thyroid hormone on symptoms
d) Influence of adrenal gland on symptoms
e) Influence of ovaries on symptoms
Incorrect!!! The patient’s mood swings are influenced by the hormonal shifts controlled by the
hypothalamic-pituitary-ovarian axis. A hysterectomy would only resolve the menstrual bleeding component
of this patient’s symptoms, and have no effect on the hormonal production of the ovaries.
A 27 year-old G1P0 woman complains of mood swings and fatigue in the week prior to her menstrual period. These
symptoms have worsened over the past 6 months. Some months, the symptoms are so severe she misses several days
of work. Her medical history is otherwise unremarkable and a physical examination is normal. Which of the
following is the most appropriate next step in this patient’s management?
a) Obtain a symptom diary for two months
b) Recommend dietary changes
c) Begin selective serotonin receptor inhibitor therapy
d) Refer for psychiatric consultation
e) Obtain a pelvic ultrasound
Correct!!! Obtaining further history with a menstrual calendar determines the cyclic nature of the PMS or
PMDD symptoms and helps guide appropriate therapy. While dietary changes may help, it is first important
to establish the diagnosis.
A 37 year-old G1P1 woman has experienced symptoms of depression and difficulty concentrating the week prior to
her menstrual period for the last three years, since her tubal ligation. She kept a symptom diary for three months
revealing symptoms clustered around her menstrual cycle. She was diagnosed with premenstrual syndrome and
began a regular exercise routine with dietary modifications, but only noticed mild relief in her symptoms. Work-up
is otherwise unremarkable. Which of the following will most likely alleviate her symptoms?
a) Evening primrose oil
b) Ginkgo
c) Progesterone cream
d) Fluoxetine hydrochloride
e) Levothyroxine sodium
Incorrect!!! Selective serotonin receptor inhibitors increase the amount of active serotonin in the brain and
have been found to be effective in alleviating PMS and PMDD symptoms. Patients can take the medication
either every day or for 10 days during the luteal phase. Progesterone cream will not help her symptoms.
A 37 year-old G0 woman complains that she experiences mood swings, irritability, bloating and headaches monthly
for 2-3 days prior to her menstrual cycle. Her medical history is unremarkable and physical examination is normal.
The physician advises her to keep a calendar of her symptoms. He also recommends a balanced diet, avoidance of
caffeine and alcohol, and daily regular exercise. The patient has never exercised regularly and wonders how this will
help her mood swings and bloating. Which of the following would provide the best explanation for the benefits of
exercise on her PMS symptoms?
a) Endorphins
b) Cortisol
c) Progesterone
d) Estrogen
e) Androgen
Correct!!! Exercise increases circulating endorphins in the brain which are “feel good” hormones and act
similar to serotonin. Therefore, in addition to being a benefit to cardiovascular health, regular exercise can
significantly decrease symptoms of PMS.
An 18 year-old G0 comes in for an annual exam with her mother. Her mother comments that she had severe PMS
symptoms in her twenties and thirties. She would like to know if her daughter will inherit this as well. Which of the
following has the strongest association with premenstrual syndrome?
a) Obesity
b) Positive family history
c) History of early menarche
d) Insulin dependent diabetes mellitus
e) Vitamin K deficiency
Correct!!! Risk factors for PMS include a family history of premenstrual syndrome (PMS), Vitamin B6,
calcium, or magnesium deficiency. PMS becomes increasingly common as women age through their 30s,
and symptoms sometimes get worse over time. Previous anxiety, depression or other mental health
problems are significant risk factor for developing premenstrual dysphoric disorder (PMDD). There is no
known association between premenstrual syndrome and obesity or insulin dependent diabetes mellitus.
Gynecology - Exam 33
A 36-year-old G2P2 woman comes to the office to discuss discontinuing contraception. Six weeks ago, she had her
first Depo-Provera injection and now she has unpredictable bleeding. She is frustrated and irritated by these
symptoms. She has a history of hypertension but is currently on no medications. Vital signs reveal a blood pressure
of 130/90; weight 188 pounds, height 65 inches; BMI 31.4kg/m2. Which of the following is the most appropriate
next step in the management of this patient?
a) Offer a different method of contraception
b) Reassurance that this is normal initially
c) Begin oral contraceptives as a backup method for contraception
d) Perform an endometrial biopsy to assess for endometrial cancer
e) Offer add-back estrogen therapy
Correct!!! She should be reassured since initially after Depo-Provera injection there may be unpredictable
bleeding. This usually resolves in 2 or 3 months. In general, after 1 year of using Depo-Provera, nearly
50% of users have amenorrhea.
An 18-year-old G0 woman with LMP 14 days ago presents to the office requesting contraception, as she had
unprotected intercourse the night before. She has taken oral contraceptives before and wants to know if she can just
start them again now. After counseling and a discussion of long-term contraception, as well as emergency
contraception, you advise her to take a low dose oral contraceptive. Of the following, which is the most appropriate
instruction to give her at this time?
a) If pregnancy occurs, termination is indicated
b) Begin taking the pills 72 hours after intercourse
c) Taking 5 pink/active tablets in two doses 12 hours apart has the same efficacy as taking them at the same time
d) She should not expect any changes in her next cycle since emergency contraception can be taken at any time
during a woman’s cycle
e) Insert the second dose per vagina or take an antiemetic 1-hour before dosing to decrease nausea
Incorrect!!! Emergency contraceptive pills are not an abortifacient, and they have not been shown to cause
any teratogenic effect if inadvertently administered during pregnancy; therefore, termination is not
indicated. They are more effective the sooner they are taken after unprotected intercourse, and it is
recommended that they be started within 72 hours, and no later than 120 hours. Plan B, the levonorgestrel
pills can be taken in one or two doses and cause few side effects. Oral contraceptives need to be taken 12
hours apart. Emergency contraceptive pills may be used anytime during a woman’s cycle, but may impact
the next cycle, which can be earlier or later with bleeding ranging from light, to normal, to heavy. Taking
an antiemetic one-hour before the dose will decrease her nausea. Bypassing the oral route by inserting the
medication per vagina will also have the same result and allow for appropriate absorption.
A 35-year-old G3P3 woman requests contraception. Her youngest child is 7 years old. Her periods have been regular
since she discontinued breast-feeding 5 years ago. Her past medical history includes depression that is controlled
with antidepressants, and a history of deep venous thrombosis. She denies smoking or alcohol use. In the past, oral
contraceptive pills have caused her to have severe gastrointestinal upset. What in her history makes her an ideal
candidate for progestin-only pills?
a) Depression
b) Smoking history
c) Severe nausea on combined oral contraceptives
d) Lactation history
e) Deep venous thrombosis
Incorrect!!! Ideal candidates for progestin-only pills include women who have contraindications to using
combined oral contraceptives (estrogen containing.) Contraindications include a history of thromboembolic
disease, women who are lactating, women over age 35 who smoke or women who develop severe nausea
with combined oral contraceptive pills. Progestins should be used with caution in women with a history of
depression.
A 24-year-old G1P1 woman comes to the office requesting contraception. Her past medical history is unremarkable,
except for a family history of endometrial cancer. She denies alcohol, smoking and recreational drug use. She is in a
monogamous relationship. She wants to significantly decrease her risk of having a gynecological malignancy. Of the
following, what is the best method of contraception for this patient?
a) Female condoms
b) Male condoms
c) Copper containing intrauterine device
d) Combined oral contraceptives
e) Cervical cap
Incorrect!!! Oral contraceptives will decrease a woman’s risk of developing ovarian and endometrial
cancer. The earlier, higher dose oral contraceptive pills have been linked to a slight increase in breast
cancer, but not the most recent pills. Women who use oral contraceptive pills have slightly higher risk of
developing cervical intraepithelial neoplasia, but their risk of developing PID, endometriosis, benign breast
changes and ectopic pregnancy are reduced. Both hypertension and thromboembolic disorders can be a
potential side effect from using oral contraceptive pills. Although the mechanism is unclear, an association
of tubal ligation and a lower incidence of ovarian cancer in BRCA-1 positive women has been reported.
Male condoms and intrauterine devices will not lower her risk of ovarian cancer.
A 35-year-old G3P3 woman comes to the office to discuss tubal ligation as she desires permanent sterilization. What
are the non-contraceptive health benefits of female sterilization?
a) Reduced risk of endometriosis
b) Reduced risk of ovarian cancer
c) Protection against endometrial cancer
d) Reduction in menstrual blood flow
e) Reduced risk of sexually transmitted diseases
Incorrect!!! Tubal ligation has not been shown to reduce the risk of endometriosis, sexually transmitted
diseases or endometrial cancer, nor is there a decrease in menstrual blood flow in women who have
undergone a tubal ligation. There is a slight reduction in the risk of ovarian cancer, but the mechanism is
not yet fully understood.
A 24-year-old G2P2 woman with a history of two prior Cesarean sections desires a tubal ligation for permanent
sterilization. She has two daughters, who are 3 and 1 years old. She is very sure she does not desire any more
children. She is happily married and a stay-at-home-mom. What is the strongest predictor of post-sterilization regret
for this patient?
a) Not working outside the home
b) Parity
c) Marital status
d) Age
e) Children’s gender
Correct!!! Approximately 10% of women who have been sterilized regret having had the procedure with
the strongest predictor of regret being undergoing the procedure at a young age. The percentage expressing
regret was 20% for women less than 30-years-old at the time of sterilization. For those under age 25, the
rate was as high as 40%. The regret rate was also high for women who were not married at the time of their
tubal ligation, when tubal ligation was performed less than a year after delivery, and if there was conflict
between the women and their partners.
A 32-year-old G3P3 woman comes to the office to discuss permanent sterilization. She has a history of hypertension
and asthma (on corticosteroids). She has been married for 10 years. Her blood pressure is 140/94; weight 280
pounds; height 69 inches; and BMI 41.4kg/m2. You discuss with her risks and benefits of contraception. Which of
the following would be the best form of permanent sterilization to recommend for this patient?
a) Laparoscopic bilateral tubal ligation
b) Mini laparotomy tubal ligation
c) Exploratory laparotomy with bilateral salpingectomy
d) Total abdominal hysterectomy
e) Vasectomy for her husband
Incorrect!!! Both vasectomy and tubal ligation are 99.8% effective. Vasectomies are performed as an
outpatient procedure under local anesthetic, while tubal ligations are typically performed in the operating
room under regional or general anesthesia; therefore carrying slightly more risk to the woman, assuming
both are healthy. She is morbidly obese, so the risk of anesthesia and surgery are increased. In addition, she
has chronic medical problems that make her at increase risk for surgery.
A 35-year-old G3P3 woman comes to the office because she desires contraception. Her past medical history is
significant for Wilson’s disease, chronic hypertension and anemia secondary to menorrhagia. She is currently on no
medications. Her vital signs reveal a blood pressure of 144/96. Of the following, what is the ideal contraceptive for
her?
a) Progestin-only pill
b) Low dose combination contraceptive
c) Continuous oral contraceptive
d) Copper containing intrauterine device
e) Levonorgestrel intrauterine device
Incorrect!!! The levonorgestrel intrauterine device has lower failure rates within the first year of use than
does the copper containing intrauterine device. It causes more disruption in menstrual bleeding, especially
during the first few months of use, although the overall volume of bleeding is decreased long-term and
many women become amenorrheic. The levonorgestrel intrauterine device is protective against endometrial
cancer due to release of progestin in the endometrial cavity. She is not a candidate for oral contraceptive
pills because of her poorly controlled chronic hypertension. The progestin only pills have a much higher
failure rate than the progestin intrauterine device. She is not a candidate for the copper-containing
intrauterine device because of her history of Wilson’s disease.
A 23-year-old G0 woman comes to the office to discuss contraception. Her past medical history is remarkable for
hypothyroidism and mild hypertension. She has a history of slightly irregular menses. Her best friend recently got a
“patch,” so she is also interested in using a transdermal system (patch.) Her vital signs are: blood pressure 130/84;
weight 210 pounds. What is the most compelling reason for her to use a different method of contraception?
a) Age
b) Hypothyroidism
c) Weight
d) Unpredictable periods
e) Hypertension
Correct!!! The patch has comparable efficiency to the pill in comparative clinical trials, although it has
more consistent use. It has a significantly higher failure rate when used in women who weigh more than
198 pounds. The patch (Ortho-Vera) is a transdermal system that is placed on a woman’s upper arm or
torso (except breasts). The patch slowly releases Ethinyl Estradiol and Norelgestromin, which establishes
steady serum levels for 7 days. A woman should apply one patch in a different area each week for 3 weeks,
then have a patch-free week, during which time she will have a withdrawal bleed. Patients should be
informed about the risks of using a transdermal delivery system including higher estrogen levels and
increased risk of thromboembolic events.
A 37-year-old G3P3 woman presents for a health maintenance exam. She is healthy with no history of medical
problems. She had a tubal ligation 2 years ago. She had used combined oral contraceptives previously for a total of
10 years. Her husband has a strong family history of cardiac disease and her grandmother passed away at age 87
from a stroke. She is worried about having a heart attack herself. She has smoked one pack of cigarettes a day for
the last 15 years and drinks alcohol 3 times a week. Which of the following is her strongest risk factor for cardiac
disease?
a) Past use of oral contraceptives
b) Alcohol consumption
c) Maternal family history
d) Paternal family history
e) Smoking history
Correct!!! Although oral contraceptives are contraindicated in women with coronary vascular disease, past
use of the pills does not increase current risk. This woman’s strongest risk is her smoking. Her husband’s
family history does not place her at risk. Her grandmother’s history can potentially place her at increased
risk but her own smoking is her biggest risk factor. Mild red wine consumption can potentially decrease her
risk, and other alcohol consumption does not pose a significant increased risk.
Gynecology -Exam 35
A 32-year-old P0 woman, with a last menstrual period three weeks ago, presents with a 3-month history of a
malodorous vaginal discharge. She reports no pruritus or irritation. She has been sexually active with a new partner
for the last 4 months. Her past medical history is unremarkable. Pelvic examination reveals normal external genitalia
without rash, ulcerations or lesions. Some discharge is noted on the perineum. The vagina reveals only a thin, gray
homogeneous discharge. The vaginal pH is greater than 4.5. Which medication is most likely to treat her symptoms
and the accompanying saline wet prep?
a) Ceftriaxone
b) Doxycycline
c) Metronidazole
d) Azithromycin
e) Penicillin
Correct!!! Bacterial vaginosis is the most common cause of vaginitis. The infection arises from a shift in
the vaginal flora from hydrogen peroxide-producing lactobacilli to non-hydrogen peroxide-producing
lactobacilli, which allows proliferation of anaerobic bacteria. The majority of women are asymptomatic;
however, patients may experience a thin, gray discharge with a characteristic fishy odor that is often worse
following menses and intercourse. Modified Amsel criteria for diagnosis include three out of four of the
following: 1) thin, gray homogenous vaginal discharge; 2) positive whiff test (addition of potassium
hydroxide releases characteristic amine odor;) 3) presence of clue cells on saline microscopy; and 4)
elevated vaginal pH >4.5. Treatment consists of Metronidazole 500mg orally BID for 7 days, or vaginal
Metronidazole 0.75% gel 5-grams QHS for 5 days.
A 64-year-old G2P2 woman presents with a 12-month history of severe vulvar pruritus. She has applied multiple
over-the-counter topical therapies without improvement. She has no significant vaginal discharge. She is currently
on oral estrogen therapy for osteoporosis. She has severe introital dyspareunia and has stopped having intercourse
because of the pain. Her past medical history is significant for allergic rhinitis and hypertension. On pelvic
examination, the external genitalia show loss of the labia minora with resorption of the clitoris (phimosis). The
vulvar skin appears thin and pale and involves the perianal area. No ulcerations are present. The vagina is mildly
atrophic, but appears uninvolved. Which of the following is the most likely diagnosis in this patient?
a) Squamous cell hyperplasia
b) Lichen sclerosus
c) Lichen planus
d) Candidiasis
e) Vulvar cancer
Correct!!! Lichen sclerosus is a chronic inflammatory skin condition that most commonly affects Caucasian
premenarchal girls and postmenopausal women. The exact etiology is unknown, but is most likely
multifactorial. Patients typically present with extreme vulvar pruritus and may also present with vulvar
burning, pain and introital dyspareunia. Early skin changes include polygonal ivory papules involving the
vulva and perianal areas, waxy sheen on the labia minora and clitoris, and hypopigmentation. The vagina is
not involved. More advanced skin changes may include fissures and erosions due to a chronic itch-scratchitch cycle, mucosal edema and surface vascular changes and, ultimately, scarring with loss of normal
architecture, such as introital stenosis, and resorption of the clitoris (phimosis) and labia minora. Treatment
involves use of high-potency topical steroids. There is less than a 5% risk of developing squamous cell
cancer within a field of lichen sclerosus.
A 22-year-old P0 woman, presents with a one-month history of profuse vaginal discharge with mild odor. She has a
new sexual partner with whom she has had unprotected intercourse. She reports mild to moderate irritation, pruritus
and pain. She thought she had a yeast infection, but had no improvement after using an over-the-counter antifungal
cream. She is concerned about sexually transmitted infections. Her medical history is significant for lupus and
chronic steroid use. Pelvic examination shows normal external genitalia, an erythematous vagina with a copious,
frothy yellow discharge and multiple petechiae on the cervix. Vaginal pH is 7. Saline wet mount reveals motile,
flagellated organisms and multiple white blood cells. Which of the following is the most appropriate treatment for
this patient?
a) Clindamycin
b) Azithromycin
c) Metronidazole
d) Ampicillin
e) Doxycycline
Correct!!! This patient has signs and symptoms of trichomoniasis, which is caused by the protozoan, T.
vaginalis. Many infected women have symptoms characterized by a diffuse, malodorous, yellow-green
discharge with vulvar irritation. However, some women have minimal or no symptoms. Diagnosis of
vaginal trichomoniasis is performed by saline microscopy of vaginal secretions, but this method has a
sensitivity of only 60% to 70%. The CDC recommended treatment is metronidazole 2 grams orally in a
single dose. An alternate regimen is metronidazole 500mg orally twice daily for seven days. The patient’s
sexual partner also should undergo treatment prior to resuming sexual relations.
A 42-year-old G2P2 woman presents with a 2-week history of a thick, curdish white vaginal discharge and pruritus.
She has not tried any over-the-counter medications. She is currently single and not sexually active. Her medical
history is remarkable for recent antibiotic use for bronchitis. On pelvic examination, the external genitalia show
marked erythema with satellite lesions. The vagina appears erythematous and edematous with a thick white
discharge. Saline wet prep reveals multiple white blood cells, but no clue cells or trichomonads. Potassium
hydroxide prep shows the following organisms, vaginal pH is 4.0. The cervix appears normal and the remainder of
the exam is unremarkable except for mild vaginal wall tenderness. Which of the following is the most appropriate
treatment for this patient?
a) Clindamycin
b) Azole cream
c) Metronidazole
d) Doxycycline
e) Ciprofloxacin
Correct!!! Vulvovaginal candidiasis (VVC) usually is caused by C. albicans, but is occasionally caused by
other Candida species or yeasts. Typical symptoms include pruritus and vaginal discharge. Other symptoms
include vaginal soreness, vulvar burning, dyspareunia and external dysuria. None of these symptoms are
specific for VVC. The diagnosis is suggested clinically by vulvovaginal pruritus and erythema with or
without associated vaginal discharge. The diagnosis can be made in a woman who has signs and symptoms
of vaginitis when either: a) a wet preparation (saline or 10% KOH) or Gram stain of vaginal discharge
demonstrates yeasts or pseudohyphae; or b) a vaginal culture or other test yields a positive result for a yeast
species. Treatment for uncomplicated VVC consists of short-course topical Azole formulations (1-3 days,)
which results in relief of symptoms and negative cultures in 80%-90% of patients who complete therapy.
A 52-year-old nulliparous woman presents with long-standing vulvar and vaginal pain and burning. She has been
unable to tolerate intercourse with her husband because of introital pain. She had difficulty sitting for prolonged
periods of time or wearing restrictive clothing because of worsening vulvar pain. She recently noticed that her gums
bleed more frequently. She avoids any topical over-the-counter therapies because they intensify her pain. Her
physical examination is remarkable for inflamed gingiva and a whitish reticular skin change on her buccal mucosa.
A fine papular rash is present around her wrists bilaterally. Pelvic examination reveals white plaques with
intervening red erosions on the labia minora as noted below. A speculum cannot be inserted into her vagina because
of extensive adhesions. The cervix cannot be visualized. Which of the following is the most likely diagnosis in this
patient?
a) Squamous cell hyperplasia
b) Lichen sclerosus
c) Lichen planus
d) Genital psoriasis
e) Vulvar cancer
Incorrect!!! Lichen planus is a chronic dermatologic disorder involving the hair-bearing skin and scalp,
nails, oral mucous membranes and vulva. This disease manifests as inflammatory mucocutaneous eruptions
characterized by remissions and flares. The exact etiology is unknown, but is felt to be multifactorial.
Vulvar symptoms include irritation, burning, pruritus, contact bleeding, pain and dyspareunia. Clinical
findings vary with a lacy, reticulated pattern of the labia and perineum, with or without scarring and
erosions as well. With progressive adhesion formation and loss of normal architecture, the vagina can
become obliterated. Patients may also experience oral lesions, alopecia and extragenital rashes. Treatment
is challenging, since no single agent is universally effective and consists of multiple supportive therapies
and topical superpotent corticosteroids.
A 27-year-old P0 woman presents with a 3-year history of dyspareunia. She reports a history of always having
painful intercourse, but she is now unable to tolerate intercourse at all. She has avoided sex for the last 6 months.
She describes severe pain with penile insertion. On further questioning, she reports an inability to use tampons
because of painful insertion. She also notes a remote history of frequent yeast infections while she was on antibiotics
for recurrent sinusitis that occurred “years” ago. Her medical history is unremarkable, and she is on no medications.
Pelvic examination is remarkable for normal appearing external genitalia. Palpation of the vestibule with a Q-tip
elicits marked tenderness and slight erythema. A normal-appearing discharge is noted. Saline wet prep shows only a
few white blood cells, and potassium hydroxide testing is negative. Vaginal pH is 4.0. The cervix and uterus are
unremarkable. Which of the following is the most likely diagnosis in this patient?
a) Vaginal cancer
b) Genital herpes infection
c) Vulvar vestibulitis
d) Contact dermatitis
e) Chlamydia infection
Incorrect!!! Vulvar vestibulitis syndrome consists of a constellation of symptoms and findings limited to
the vulvar vestibule, which include severe pain on vestibular touch or attempted vaginal entry, tenderness
to pressure and erythema of various degrees. Symptoms often have an abrupt onset and are described as a
sharp, burning and rawness sensation. Women may experience pain with tampon insertion, biking or
wearing tight pants, and avoid intercourse because of marked introital dyspareunia. Vestibular findings
include exquisite tenderness to light touch of variable intensity with or without focal or diffuse
erythematous macules. Often, a primary or inciting event cannot be determined. Treatment includes use of
tricyclic antidepressants to block sympathetic afferent pain loops, pelvic floor rehabilitation, biofeedback,
and topical anesthetics. Surgery with vestibulectomy is recommended for patients who do not respond to
standard therapies and are unable to tolerate intercourse.
A 30-year-old G1P1 woman presents with a history of chronic vulvar pruritus. The itching is so severe that she
scratches constantly and is unable to sleep at night. She reports no significant vaginal discharge or dyspareunia. She
does not take antibiotics. Her medical history is unremarkable. Pelvic examination reveals normal external genitalia
with marked lichenification (increased skin markings) and diffuse vulvar edema and erythema. Saline microscopy is
negative. Potassium hydroxide testing is negative. Vaginal pH is less than 4.5. The vaginal mucosa is normal. Which
of the following is the most likely diagnosis in this patient?
a) Lichen simplex chronicus
b) Lichen sclerosus
c) Lichen planus
d) Candidiasis
e) Vulvar cancer
Incorrect!!! Lichen simplex chronicus, a common vulvar non-neoplastic disorder results from chronic
scratching and rubbing, which damages the skin and leads to loss of its protective barrier. Over time, a
perpetual itch-scratch-itch cycle develops, and the result is susceptibility to infection, ease of irritation and
more itching. Symptoms consist of severe vulvar pruritus, which can be worse at night. Clinical findings
include thick, lichenified, enlarged and rugose labia, with or without edema. The skin changes can be
localized or generalized. Diagnosis is based on clinical history and findings, as well as vulvar biopsy.
Treatment involves a short-course of high-potency topical corticosteroids and antihistamines to control
pruritus.
A 20-year-old nulliparous woman college student presents with a one-month history of profuse vaginal discharge
and mid-cycle vaginal spotting. She uses oral contraceptives and she feels her irregular bleeding is due to the pill.
She is sexually active and has had a new partner within the past 3 months. She reports no fevers or lower abdominal
pain. She has otherwise been healthy. On pelvic examination, a thick yellow endocervical discharge is noted. Saline
microscopy reveals multiple white blood cells, but no clue cells or trichomonads. Potassium hydroxide testing is
negative. Vaginal pH is 4.0. No cervical motion tenderness or uterine/adnexal tenderness is present. Testing for
Gonorrhea and Chlamydia is performed, but those results will not be available for several days and the student will
be leaving for Europe tomorrow. Which of the following is the most appropriate treatment for this patient?
a) Metronidazole and Erythromycin
b) Ceftriaxone and Azithromycin
c) Ampicillin
d) Doxycycline
e) No treatment is necessary until all tests results are known
Correct!!! Mucopurulent cervicitis (MPC) is characterized by a mucopurulent exudate visible in the
endocervical canal or in an endocervical swab specimen. MPC is typically asymptomatic, but some women
have an abnormal discharge or abnormal vaginal bleeding. MPC can be caused by chlamydia trachomatis
or Neisseria gonorrhoeae; however, in most cases neither organism can be isolated. Patients with MPC
should be tested for both of these organisms. The results of sensitive tests for C. trachomatis or N.
gonorrhoeae (e.g. culture or nucleic acid amplification tests) should determine the need for treatment,
unless the likelihood of infection with either organism is high or the patient is unlikely to return for
treatment. Antimicrobial therapy should include coverage for both organisms, such as azithromycin or
doxycycline for chlamydia and a cephalosporin or quinolone for gonorrhea. Uncomplicated cervicitis, as in
this patient, would require only 125 mg of Ceftriaxone in a single does. Ceftriaxone 250 mg is necessary
for the treatment of upper genital tract infection, or pelvic inflammatory disease (PID).
A 37-year-old nulliparous woman, a CEO for a Fortune 500 company, presents with a one-week history of a painful
vulvar ulcer. She reports no fevers, malaise or other systemic symptoms. She recently started use of a topical steroid
ointment for a vulvar contact dermatitis. She is married and has no prior history of sexually transmitted infections.
She reports no travel outside the United States by her husband or herself. Her last Pap smear, 6 months ago, was
normal. A vulvar herpes culture later returns positive for herpes simplex virus type 2. A VDRL is nonreactive, and
HIV testing is negative. Which of the following is the most likely diagnosis in this patient?
a) Primary HSV episode
b) Recurrent HSV-1 episode
c) Recurrent HSV-2 episode
d) Atypical HSV episode
e) Contact dermatitis
Correct!!! Two serotypes of HSV have been identified: HSV-1 and HSV-2. Most cases of recurrent genital
herpes are caused by HSV-2. Up to 30% of first-episode cases of genital herpes are caused by HSV-1, but
recurrences are much less frequent for genital HSV-1 infection than genital HSV-2 infection. Genital HSV
infections are classified as initial primary, initial nonprimary, recurrent and asymptomatic. Initial, or firstepisode primary genital herpes is a true primary infection (i.e. no history of previous genital herpetic
lesions, seronegative for HSV antibodies.) Systemic symptoms of a primary infection include fever,
headache, malaise and myalgias, and usually precede the onset of genital lesions. Vulvar lesions begin as
tender grouped vesicles that progress into exquisitely tender, superficial, small ulcerations on an
erythematous base. Initial, nonprimary genital herpes is the first recognized episode of genital herpes in
individuals who are seropositive for HSV antibodies. Prior HSV-1 infection confers partial immunity to
HSV-2 infection and thereby lessens the severity of type 2 infection. The severity and duration of
symptoms are intermediate between primary and recurrent disease, with individuals experiencing less pain,
fewer lesions, more rapid resolution of clinical lesions and shorter duration of viral shedding. Systemic
symptoms are rare. Recurrent episodes involve reactivation of latent genital infection, most commonly with
HSV-2, and are marked by episodic prodromal symptoms and outbreaks of lesions at varying intervals and
of variable severity. Clinical diagnosis of genital herpes should be confirmed by viral culture, antigen
detection or serologic tests. Treatment consists of antiviral therapy with Acyclovir, Famciclovir or
Valacyclovir.
A 45-year-old G2P2 woman presents with a 2 month history of vulvar pruritus. She does not have a significant
vaginal discharge. Her medical history is significant for HIV diagnosis for 2 years, and a pack of cigarettes a day
smoking habit for the last 20 years. Her last Pap smear, 2 years ago, showed LGSIL (low-grade squamous
intraepithelial lesion). She underwent colposcopy at that time, which was adequate and consistent with CIN-I/mild
dysplasia, and she was subsequently lost to follow-up. Her menstrual history is unremarkable. Pelvic examination is
remarkable for a 3cm non-ulcerated, hyperkeratotic lesion on the right labia minora, as well as multiple papillary
growths on the posterior fourchette and perineum. The vagina and cervix are grossly normal. In addition to
performing a Pap smear, which of the following procedures is most appropriate for this patient?
a) Vaginal biopsy
b) Vulvar biopsy
c) Endometrial biopsy
d) Cervical biopsy
e) Colposcopy
Correct!!! Genital condylomata, or warts, are typically caused by human papillomavirus (HPV) types 6 or
11. Other HPV types in the anogenital area (e.g. types 16, 18, 31, 33 and 35) have been strongly associated
with cervical neoplasia and cancer. Vulvar intraepithelial neoplasia (VIN) is a possibility in this patient,
given her history of cervical dysplasia, tobacco use and HIV status. Genital condylomata that do not
respond to topical therapies should be biopsied. Likewise, vulvar biopsy is indicated to evaluate the
hyperkeratotic lesion in this patient and rule out the possibility of vulvar neoplasia.
Approach to the patient - Exam 3
A 52-year-old G3P2 woman reports irregular vaginal spotting and bleeding after intercourse for the past 18 months.
She stopped having menses at the age of 48 and has never been on hormone replacement therapy. She also notes
new onset low back pain. She has been a two pack a day smoker for the past thirty years. Her last gynecologic exam
was 10 years ago. On physical exam, she is a thin female who appears older than her stated age. She weighs 120 lbs.
and is 5’6” tall. Her pelvic examination reveals atrophy of the external genitalia and vagina, and a minimal amount
of dark brown blood in the vault and a large parous cervix with a friable lesion on the anterior lip of the cervix. The
uterus is normal size, immobile and fixed in a retroverted position. There are no palpable adnexal masses, but there
is firm nodularity in the posterior cul-de-sac on rectal exam. Which of the following is the most appropriate next
step in the management of this patient?
a) Computerized tomography of the lower spine and pelvis
b) A Pap smear
c) A colposcopy
d) A cervical biopsy
e) A pelvic ultrasound
Incorrect!!! This patient is at high-risk for cervical cancer. Her risk factors include tobacco use and a poor
screening history. The symptoms of postmenopausal and postcoital bleeding should be taken seriously, and
a cervical biopsy of the suspicious cervical lesion performed. Her physical exam with fixation of the uterus
and thickening of the rectovaginal septum and back pain suggests involvement of the parametria (Stage II)
and possible extension to the sidewall (Stage III.) A Pap smear should not be used to exclude cervical
cancer, as it is a screening test, not a diagnostic test, and colposcopy would not be useful since a clinically
visible lesion is already present. Although a CT scan may ultimately be needed as part of the evaluation of
cervical cancer, a diagnosis must first be made by biopsy. Ultrasonography may be helpful in the diagnostic
evaluation of post-menopausal bleeding, but not in the setting of an obvious cervical lesion.
A 17-year-old female high school student is brought in by her mother for her first gynecologic exam. She began her
menses at age 12 and has had regular periods for the past three years. For privacy, you ask to examine the patient
without her mother. Further history is obtained in the examination room. She admits that she has been sexually
active with her boyfriend for the past three years. She uses condoms occasionally and is fearful about possible
pregnancy. She requests that her mother not be informed about her sexual activity. On physical exam, she is
anxious, but normally developed. Her pelvic examination reveals no vulvar lesions, minimal non-malodorous
vaginal discharge, and a nulliparous appearing cervix. The bimanual examination reveals a slightly enlarged uterus,
and her adnexa are non-tender and not enlarged. Urine pregnancy test is negative. Which of the following
interventions do you recommend at this point?
a) Pelvic transvaginal ultrasound
b) Obtaining a Pap smear
c) Empiric treatment with doxycycline and ceftriaxone
d) Initiation of oral contraceptives
e) DNA probes for gonorrhea and chlamydia
Incorrect!!! Screening for sexually transmitted diseases is important in this 17-year-old patient. Guidelines
for initiation of cervical cancer screening recommend 3 years after onset of sexual activity or age 21. A
pregnancy test is also certainly indicated given the patient’s concerns and sexual/contraceptive history,
especially given the enlarged uterus. A pelvic ultrasound, however, would not be indicated at this time
especially if the pregnancy test is negative and given her lack of menstrual or pelvic symptoms. Counseling
about and screening for sexually transmitted infections is advisable, but this patient does not require
treatment due to a lack of diagnostic criteria. Discussions about various contraceptive methods are always
recommended in this setting.
A 68-year-old grandmother who has recently moved in with her daughter (a long-standing patient of yours) comes in
for an annual examination. A vaginal hysterectomy was done in her fifties for uterine prolapse. She is not sure if her
ovaries were removed. She has never had an abnormal mammogram or Pap and has had yearly exams. She stopped
hormone replacement therapy 10 years ago. She was recently widowed after being married for 50 years. She does
not smoke or drink. Her diabetes is well-controlled with Metformin; she takes a daily baby aspirin and is on a lipidlowering agent, but cannot recall the name. On examination, she is a thin elderly woman with a dowager’s hump.
Her breast exam is unremarkable. Her lower genital tract is remarkable for marked atrophy. No masses are noted on
bimanual and recto-vaginal exam. A fecal occult blood test is negative. Which of the following tests is not
necessary?
a) Bone density
b) Colonoscopy
c) Pap smear
d) Mammogram
e) Annual bimanual and recto-vaginal exam
Incorrect!!! Pap smear screening is not indicated in patients who have had a total hysterectomy with
removal of the cervix, unless it was done for cervical cancer or a high-grade cervical cancer precursor.
Patients with a uterus can discontinue cervical cancer screening between the ages of 65 – 70 if they have
had three consecutive negative smears and no history of high-grade cervical intraepithelial neoplasia or
cancer. Patients still need yearly bimanual and rectovaginal exam. Mammograms are done annually, as
breast cancer increases with age. Colon cancer screening is recommended at age fifty. The patient has an
exaggerated thoracic spine curvature, termed a dowager’s hump, likely secondary to thoracic compression
fractures secondary to osteoporosis. If this is confirmed on a bone density test, she may benefit from the
addition of bisphosphonates.
A 49-year-old mother of two presents for her annual examination and reports irregular periods for the past year with
associated hot flashes. Her husband has had a vasectomy. She notes erratic bleeding every two to three weeks, with
just a day of spotting or several days of moderate flow. She reports no significant pain, fever or discharge. She had
two vaginal deliveries and one early miscarriage. A Pap smear several years ago had to be repeated, but Pap smears
have been negative for the past three years. Review of systems is positive for insomnia and new onset anxiety. She
has borderline hypertension and is followed closely by her family practitioner. Her mother died of breast cancer and
her brother has multiple sclerosis. She has smoked a pack of cigarettes a day and for the past 22 years. On
examination, she is thin, pulse is 110, blood pressure is 160/100 mmHg. She weighs 120 lbs. and is 5’6” tall. Her
pelvic exam reveals blood flowing from the cervical os and a moderate amount of bright red blood in the vault. No
cervical lesions are apparent. Bimanual examination reveals an upper limit of normal size, non-tender uterus. The
adnexa are not palpable. Her recto-vaginal exam is unremarkable. Which of the following studies are indicated at
this visit?
a) Schedule for a dilatation and curettage
b) Thyroid Stimulating Hormone and BHCG
c) Pelvic ultrasound
d) Pap smear
e) Complete blood count coagulation studies
Incorrect!!! This patient has signs and symptoms very suspicious for hyperthyroidism; therefore, a thyroid
stimulating hormone test and a pregnancy test would be the most appropriate choice. She is thin,
tachycardic with frequent irregular menses, temperature instability, and anxiety and sleep disturbance.
Another possible explanation would be perimenopause. Since she has active heavy vaginal bleeding, the
Pap smear should be rescheduled for a time when she is not bleeding. The Pap is a screening test which
samples exfoliated cells from the transformation zone, and has a high false negative rate. Obscuring blood,
inflammation and drying artifact can hamper the cytopathologist’s ability to interpret the smear. The
remaining tests may be necessary later, but certainly not at this visit.
A 22-year-old G0 female college student is scheduled to discuss her recent abnormal Pap smear which showed
atypical squamous cells of undetermined significance (ASCUS.) She has had negative Pap smears on a yearly basis
since age 18. Her only significant gynecologic history is genital warts that have not responded to treatment with
local application of trichloroacetic acid. She has had eight sexual partners. She uses condoms and oral
contraceptives. She has smoked a pack a day for the past two years. Which of the following is the most appropriate
next step in the management of this patient?
a) HPV typing
b) Repeat Pap smear in 1 year
c) Cone biopsy
d) Cryotherapy
e) Loop Electrosurgical Excision Procedure (LEEP)
Correct!!! HPV typing is indicated in the initial triage of the finding of atypical squamous cells of
undetermined significance (ASCUS) on a Pap smear. If a high-risk HPV type is detected, then the patient
needs a colposcopy with biopsies. An alternative approach can be close surveillance with repeat Pap smears
in 6 months, with referral to colposcopy for any cytological abnormality more than ASCUS. Two
consecutive negative Paps are required before she may return to annual screening. Initiation of treatment by
way of cone biopsy, LEEP, or cryotherapy is not indicated at this time without a biopsy-confirmed
diagnosis of cervical dysplasia.
A 19-year-old woman presents, complaining of lower abdominal cramping. The pain started with her menses and
has persisted, despite resolution of the bleeding. She thinks she may have a fever, but has not taken her temperature.
No urinary frequency or dysuria is present. Her bowel habits are regular. She denies vomiting, but has mild nausea.
A yellow blood-tinged vaginal discharge preceded her menses. No pruritus or odor was noted. She is sexually
active, uses oral contraceptives and states that her partner does not like condoms. On examination, her temperature
is 38 degrees centigrade, pulse 90, blood pressure 110/60 mmHg; she is well-developed and nourished, and appears
acutely ill. No flank pain is elicited. Her abdomen is soft with normal bowel sounds. She is very tender with
guarding in the lower quadrants. No rebound is present. Pelvic examination reveals a moderate amount of thick
yellow discharge. The cervix is friable with yellow mucoid discharge at the os. Cervical motion tenderness is present
and the adnexa are tender without masses. The uterus is of normal size and is tender. Urine dip is negative for
nitrates. Urine HCG test is negative. A cotton swab is placed in the endocervical canal and rotated for ten seconds. A
wet prep is also obtained. What is the most likely diagnosis?
a) Vulvovaginal Candidiasis
b) Acute salpingitis
c) Trichomonas vaginitis
d) Gonorrhea cervicitis
e) Bacterial vaginosis
Incorrect!!! This patient has symptoms suggestive of acute salpingitis (pelvic inflammatory disease.)
Mucopurulent cervicitis with peak in the symptoms during and after menstruation is classically gonorrhea.
Chlamydia is frequently associated with gonorrhea and also causes cervicitis and pelvic inflammatory
disease. Gonorrhea cultures require specialized media, specifically chocolate agar (Thayer-Martin.) A
quick diagnosis of gonorrhea can be made on gram stain. Although not very sensitive, the finding of gramnegative intracellular diplococci is highly specific for gonorrhea. Trichomonas may cause a yellow frothy
discharge, and Candida may cause a thick white cottage cheese like discharge, but neither would cause
fever and abdominal pain.
A 39-year-old female homeless patient presents to the free clinic reporting nontender plaques on her vulva for about
a week. She noticed these plaques when she wiped. No pruritus or pain is present. She also notes a brownish rash on
the palms of her hands. She has lived on the streets for five years. She admits to IV drug abuse. She was diagnosed
as HIV-positive two years ago, but has not been compliant with suggested treatment. On examination, three elevated
plaques with rolled edges are noted on the vulva. They are nontender. A brown macular rash is noted on the palms
of her hands and the soles of her feet. What is the most appropriate next step in the management of this patient?
a) Dark-field examination of the vulvar lesions
b) Biopsy of the lesion
c) Colposcopic evaluation of the vulvar lesions
d) Culture the base of the lesion
e) Perform a wet prep
Correct!!! The classic coiled spirochete is easily seen with dark-field microscopy. Contact the laboratory as
to availability of this technique and rapidly transport the slide for evaluation. Condyloma lata are
characteristically found as secondary lesions of syphilis after the primary chancre has resolved. Another
characteristic finding is a macular rash on the palms and soles that are often described as copper penny
lesions. Both lesions are teaming with spirochetes. Immunofluorescent tests and cultures are not available
clinically. Colposcopy would not be diagnostic, but certainly is helpful to evaluate for any vulvar lesions
thought to be dysplastic. Biopsies can be stained for spirochetes and may show a necrotizing vasculitis, but
certainly would not be the most expedient way to make the diagnosis.
A 24-year-old female law student presents with multiple painful ulcers involving the vulva. The sores initially were
fluid filled, but are now open, weeping and crusted. She has a fever to 101°F and is having difficulty voiding due to
pain. She uses a vaginal ring for contraception. She has multiple sexual partners and uses condoms for vaginal
intercourse. She is distraught that she may have a sexually transmitted disease. She is healthy and does not smoke or
use drugs. On physical exam, she is in obvious distress. Temperature is 100.2°F, Pulse 100. Examination of the
genital tract is limited due to her discomfort. Multiple ulcers and erosions of variable size are localized to the
perineum, labia minora and vestibule. Swelling is diffuse. The lesions are eroded, some with a purulent eschar.
There is exquisite tenderness to touch. What further testing should be offered to this patient?
a) RPR, rapid plasma reagin
b) HIV, human immunodeficiency virus
c) Herpes culture
d) Cervical DNA probe for gonorrhea and chlamydia
e) All of the above
Incorrect!!! This unfortunate patient has classic primary herpes with painful genital ulcerations, fever and
dysuria. Given the presence of one sexually transmitted disease, screening should be offered for other
STDs. Resolution of the acute episode is required before a speculum can be inserted to allow endocervical
sampling for GC and chlamydia. If it was a high-risk exposure, prophylactic empiric treatment could be
offered to cover GC and chlamydia. The patient should be counseled that primary herpes can be acquired
despite condoms and even by oral-genital inoculation. Hepatitis B vaccination should be offered to protect
her against any future exposures. She should be encouraged to discuss her diagnosis with all sexual partners
and to continue to reliably use latex condoms.
A 38-year-old woman comes to the office because she noted a persistent yellow, frothy discharge associated with
mild external vulvar irritation. She denies any odor. She tried over the counter anti-fungal medication without
success. The discharge has been present for over three months, gradually increasing in amount. Douching has
resulted in temporary relief, but the symptoms always recur. She has well-controlled hypertension treated with ACE
inhibitors. Her partner of two years has a vasectomy. Pelvic examination reveals mild erythema at the introitus and a
copious yellow frothy discharge fills the vagina. The cervix has erythematous patches on the ectocervix. A sample
of the discharge is examined under the microscope. What is the most likely finding?
a) Strong amine “fishy” odor when KOH applied to sample
b) Marked polymorphonuclear cells with multi-nucleate giant cells
c) Motile ovoid protozoa with flagella
d) Budding yeast and pseudo-hyphae
e) Clue cells
Incorrect!!! This patient most likely has trichomoniasis. The erythematous patches on the cervix are
characteristic of “strawberry cervicitis.” Trichomonads are unicellular protozoans, which are easily seen
moving across the slide with beating flagella. The slide must be examined immediately. The discharge is
mixed with saline and placed on the slide and covered with a cover slip. Clue cells are characterized by
adherent coccobacillary bacteria that obscure the edges of the cells. A drop of KOH releases amines from
the cells and a fishy or putrid odor is noted if bacterial vaginosis is present. Yeast vaginitis is characterized
by a thick white clumpy discharge which results in erythema, swelling and intense pruritus. Multinucleate
giant cells and inflammation may be herpes.
A 23-year-old woman reports having a solitary, painful vulvar lesion that has been present for three days. This
lesion has occurred twice in the past. She states that herpes culture was done by her doctor during her last exam and
was negative. She is getting frustrated in that she does not know her diagnosis. She has no significant previous
medical history. She uses oral contraceptives and condoms. She has had four sexual partners in her lifetime. On
physical examination, a cluster of three irregular erosions with a superficial crust is noted on the posterior
fourchette. Urine HCG is negative. You suspect recurrent genital herpes. How do you explain the negative culture?
a) Cultures were taken too early in the course of the lesion
b) The more definitive test would be serum herpes antibody testing
c) The cultures were refrigerated prior to transport to the lab
d) Herpes cultures have a 10-20% false negative rate
e) The herpes virus cannot be recovered with recurrent infections
Incorrect!!! Culture is the gold standard of diagnosis. They are highly specific, yet sensitivity is limited. It
is best to culture the lesion very early in the course. The roof of the blister is unroofed and the base is
vigorously scraped. The herpes virus can theoretically be isolated from both primary and recurrent
infections. The culture must be refrigerated if any delay occurs in transport to the lab. This patient very
likely presented too late in the course for a useful culture. Serum antibody screening only indicates lifetime
exposure and would not answer the question as to the etiology of the specific lesion. Alternatively, DNA
studies such as the polymerase chain reaction can be done, if available.
Gynecology - Exam 36
A 49-year-old G5P5 woman presents for her first annual exam since she had her last child 10 years ago. She has no
health complaints. She has had 2 sexual partners. She smokes 3-5 cigarettes per day, and has been smoking for the
past 15 years. She uses marijuana occasionally. Last month, her mother underwent a radical hysterectomy for Stage
1B cervical carcinoma. On exam, she has a normal pelvic exam, except for mucopurulent discharge and vaginal
condyloma. Which of the following places the patient at greatest risk for developing cervical cancer?
a) Family history of cervical cancer
b) Smoking history
c) Vaginal condylomata
d) Multiparity
e) Marijuana use
Correct!!! Risk factors for cervical cancer are all connected to HPV exposure and include early-onset
sexual activity, multiple sexual partners, a sexual partner with multiple partners, history of HPV or other
sexually transmitted diseases, immunosuppression, smoking, low socioeconomic status and a lack of
regular Pap smears.
An 18-year-old G0 woman comes in for her first pelvic exam. She is completely asymptomatic, healthy, and reports
having only one sexual partner. She uses a diaphragm for contraception. On examination, the patient has a normal
appearing cervix except for minimal, non-malodorous vaginal discharge. You obtain chlamydia and gonorrhea
cultures, and perform a Pap smear. The Pap smear is read as ASCUS, and her cultures are negative. Which of the
following is the most appropriate management strategy for this patient?
a) Annual exam and a Pap smear in one year
b) Colposcopy with endocervical curettage and directed biopsies
c) Cryotherapy
d) Repeat the Pap smear in 4-6 weeks after antibiotic treatment for bacterial vaginosis
e) Cervical conization or LEEP
Correct!!! If a patient with an ASCUS Pap smear has presence of infection/severe inflammation, it is
appropriate to treat the underlying infection and repeat a Pap in 4-6 weeks. In this patient, she is entirely
asymptomatic and has no convincing evidence of an infection. Her age puts her in the category of
adolescents (20 years and younger), in which the ASCCP (American Society for Colposcopy and Cervical
Pathology) recommends repeat cytology in 12 months for an ASCUS result. In older women, management
of ASCUS can be to perform HPV DNA testing, repeat cytology at 6 and 12 months following the
abnormal Pap test result, or colposcopy. If the HPV testing is negative, then cytology can be repeated in 12
months. If HPV is positive, or if repeat cytology at 6 or 12 months using the other algorithm reveals
ASCUS or higher, then colposcopy should be performed.
A 28-year-old G0 woman was recently diagnosed with HIV, with a CD4 count of 365. She was started on triple
antiviral therapy. Her last Pap smear was 2 years ago and showed a low-grade lesion. She was encouraged by her
infectious disease specialist to see a gynecologist. She has no complaints. On exam, she has a normal pelvic exam
except for a clumpy white discharge. A normal saline wet prep shows normal squamous cells. Whiff test is negative.
KOH preparation shows budding hyphae. Assuming her current Pap smear turns out normal, which of the following
would you recommend?
a) Treatment with Flagyl
b) Repeat Pap smear in 2 months
c) Perform a loop electrosurgical excisional procedure (LEEP)
d) Colposcopy
e) Repeat Pap Smear in 6 months
Correct!!! Women with HIV have higher rates of cervical dysplasia and invasive cancer than do women
without HIV. Treatment failures and recurrences are more frequent, and disease severity correlates with
decreasing CD4 counts. A Pap smear should be obtained twice in the first year after diagnosis of HIV
infection and, if the results are normal, annually surveillance can resume. If the Pap results are abnormal,
care should be provided according to the ASCCP Consensus Guidelines for Management of Abnormal
Cervical Cytology. Women with cytological reports of ASC-US, low or high-grade SIL or squamous cell
carcinoma, regardless of CD4+ count or antiretroviral treatment status, should undergo colposcopy and
directed biopsy. Colposcopy and biopsy are not indicated in HIV-positive women with negative Pap
smears. The patient has a yeast infection. First-line treatments for a yeast infection include topical or oral
antifungal (Imidazole) agents. The patient’s saline wet prep was negative, thus she likely does not have
trichomoniasis or bacterial vaginosis. Both of these conditions can be treated with metronidazole (Flagyl).
Since the patient had a normal Pap smear, she does not need to have colposcopy performed. There is no
indication for a LEEP.
A 34-year-old G3P3 woman presents for annual exam and has no specific complaints. She has not seen a
gynecologist since she had a tubal ligation 6 years ago. She has been married for 12 years. She has no history of
abnormal Pap smears or sexually transmitted diseases. The patient’s physical examination is normal, except for a
small white plaque noted at the 12:00 position on her external cervical os. In addition to obtaining a Pap smear,
which of the following is the most appropriate step in the management of this patient?
a) Recommend annual Pap smears if this one returns normal
b) Repeat Pap smear in 3 months
c) Biopsy the lesion
d) Colposcopy with endocervical curetlage
e) HPV testing/hyping
Correct!!! A white plaque found on the cervix is called leukoplakia and should be biopsied directly or
under colposcopic guidance as soon as possible, regardless of Pap smear outcome. Endocervical curettage
is often done as part of evaluation of an abnormal pap smear, along with colposcopy and biopsies. Pap
smears have a false-negative rate as high has 20-30%. HPV testing is most commonly done in the initial
triage of an ASCUS smear, but otherwise has no role in the management of an obvious cervical lesion.
A 28-year-old G0 woman has her first abnormal Pap, which was read as high-grade squamous intraepithelial lesion
(HSIL.) She has no complaints. She smokes one pack of cigarettes per day. Her pelvic exam is normal. Colposcopy
is performed. The cervix is noted to have an ectropion and there is abundant aceto-white epithelium. Mosaicism,
punctations and several disorderly, atypical vessels are noted. Several biopsies are obtained and sent to pathology.
Which of the findings on this patient’s colposcopy is most concerning?
a) Ectropion
b) Acetol-white epithelium
c) Mosaicism
d) Punctations
e) Disorderly, atypical vessels
Correct!!! Punctations and mosaicism represent new blood vessels on end and on their sides, respectively.
Atypical vessels usually represent a greater degree of angiogenesis and, thus, usually a more concerning
lesion. An ectropion is an area of columnar epithelium that has not yet undergone squamous metaplasia. It
appears as a reddish ring of tissue surrounding the external os. Acetowhite epithelium can represent
dysplasia but, in most cases, is less concerning than the above vascular changes.
A 32-year-old G3P3 woman had a Pap smear which showed high-grade squamous intraepithelial lesion (HSIL.) She
smokes one pack of cigarettes per day. She has a history of 3 vaginal deliveries and a tubal ligation. On colposcopic
examination, at 12:00, there is an acetowhite lesion with punctations that extends into the endocervical canal.
Endocervical speculum is unsuccessful at visualizing the entire lesion. Endocervical curettage is negative. Which of
the following would be the most appropriate next step in the management of this patient?
a) Repeat colposcopy in two months
b) Cryotherapy ablation of the transformation zone
c) Cervical conization
d) HPV typing to rule out high risk types
e) Repeat Pap smear in three to six months
Incorrect!!! Because the entire lesion cannot be visualized, this colposcopy is inadequate and severe
dysplasia and even invasive cancer cannot be confidently ruled out. Endocervical curettage has a relatively
low sensitivity (i.e. a high amount of false negatives) and, therefore, you cannot rule out endocervical
disease. The endocervical canal must be histologically examined. A cold-knife cone biopsy should be
performed to obtain a pathology specimen. Cryotherapy may serve to ablate part of the canal, but will not
confirm or rule out a diagnosis.
A 21-year-old G0 woman has her first Pap smear, and it shows a high-grade squamous intraepithelial lesion (HSIL.)
Colposcopy is performed, and three biopsies and an endocervical curettage are obtained. The biopsies and
endocervical curettage were read as normal. Which of the following would be the most appropriate next step in the
management of this patient?
a) Pap smear in 6 months
b) Colposcopy with directed biopsies in 3-6 months
c) Cryotherapy
d) Cervical conization
e) Treat for presumed infection and repeat Pap in 4-6 weeks
Incorrect!!! Indications for cervical conization include unsatisfactory colposcopy, including inability to
visualize the entire squamocolumnar junction, positive endocervical curettage; Pap smear indicates
adenocarcinoma in situ, and a substantial discrepancy between Pap smear and biopsy results. The other
answers are not aggressive enough. Since there was a significant discrepancy between the HSIL Pap smear
result and the normal reading on the biopsies and the ECC, the patient requires a cervical conization.
A 48-year-old G3P3 woman recently had an abnormal Pap smear with high grade squamous intraepithelial lesion
(HSIL.) Colposcopy with a biopsy showed CIN 3. You did a LEEP and are reviewing the specimen with the
pathologist. The specimen shows abnormal squamous cells that extend 2mm beyond the basement membrane. You
agree that her final diagnosis is:
a) CIN 2
b) CIN 3
c) Microinvasive cancer
d) Paget's disease
e) Carcinoma in situ
Correct!!! Both CIN 3 and CIS represent abnormal cells extending up to the full length of the squamous
surface to the basement membrane, but not beyond the basement membrane. In cancer, the cells invade
beyond the basement membrane. In microinvasive cancer, they invade less than 3mm. Paget’s disease is a
vulvar condition.
A 28-year-old female patient with a low grade squamous intraepithelial lesion (LSIL) on a Pap smear comes in for
colposcopy. She has no complaints. Colposcopy is adequate and shows a lesion at 3:00 that turns white with acetic
acid, has punctations and mosaicism, and is friable. You take a biopsy of that lesion, which is read by the pathologist
as CIN 1. The patient’s endocervical curettage (ECC) is positive for a high-grade lesion. Which of the following
would be the most appropriate next step in the management of this patient?
a) Follow up Pap smear in 3-6 months
b) Cryotherapy
c) Repeat colposcopy with biopsies and ECC in 3-6 months
d) Cervical conization
e) Hysterectomy
Incorrect!!! Cervical conization is indicated if cervical biopsies show severe dysplasia, carcinoma in situ or
if a patient has a positive ECC. Hysterectomy is the treatment for invasive cancer. Waiting 6 months can
potentially be harmful, as the lesion can progress or a higher-grade lesion might already be present.
Cryotherapy will not provide a pathologic specimen to rule out invasive cancer.
A 20-year-old woman presents requesting OCP’s. She received the HPV vaccine series last year, and had her first
sexual encounter last month. She is wondering if she needs a pap smear. Otherwise, she is in good health and is a
non-smoker. Her pelvic examination reveals normal external genitalia, and a nulliparous cervix without discharge or
mucosal lesions. A UPT is negative. Which of the following guidelines regarding pap smear screening are
recommended?
a) Return next year for a pap smear
b) Return in 4-6 weeks for pap smear and cultures
c) Perform a pap smear now
d) Perform HPV testing
e) Defer pap smear screening until age 30
Correct!!! The American College of Obstetrics and Gynecology (ACOG) makes the following
recommendations for cervical cancer screening: • Annual cervical cytology screening should begin
approximately 3 years after initiation of sexual intercourse, but no later than age 21 years. • Women
younger than 30 years should undergo annual cervical cytology screening. • Women aged 30 years and
older who have had three consecutive negative cervical cytology screening test results and who have no
history of CIN 2 or CIN 3, are not immunocompromised and are not HIV infected, and were not exposed to
diethylstilbestrol in utero may extend the interval between cervical cytology examinations to every 2–3
years. • Evidence-based data indicate both liquid-based and conventional methods of cervical cytology are
acceptable for screening. • Women who have undergone hysterectomy with removal of the cervix for
benign indications and who have no prior history of CIN 2 or CIN 3 or worse may discontinue routine
cytology testing. • The use of a combination of cervical cytology and HPV DNA screening is appropriate
for women aged 30 years and older. If this combination is used, women who receive negative results on
both tests should be rescreened no more frequently than every 3 years.
Gynecology - Exams 38
A 27-year-old G0 woman presents due to a one-year history of painful periods and intercourse. Pain, when present is
recently 7/10 in strength and occasionally requires that she miss work. She now tries to avoid intercourse and no
longer finds it pleasurable. She is otherwise in good health. Her last menstrual period was 17 days ago and her
menses are typically 28 days apart. She had chlamydia once, at age 19. Physical exam is notable for some tenderness
on abdominal and pelvic exams in the lower quadrants. Uterus is normal in size, but there is some tender nodularity
on the back of the uterus. What is the most likely diagnosis in this patient?
a) Chronic pelvic inflammatory disease
b) Endometriosis
c) Adenomyosis
d) Uterine retroversion
e) Endometritis
Incorrect!!! This patient has typical symptoms of endometriosis, including dysmenorrhea and dyspareunia.
In addition, the nodularity on the back of the uterus is suggestive of endometriosis. Chronic pelvic
inflammatory disease would not present this far out from a known infection. Adenomyosis is endometrial
glands embedded in the wall of the uterus. Endometritis is an infection of the endometrium. The patient
does not have a retroverted uterus on exam.
A-29-year old G0 woman presents, due to the inability to conceive for the last year. Her cycles are regular every 28
days, but she has very painful periods, occasionally requiring that she miss work despite the use of non-steroidal
anti-inflammatory medicine (NSAIDs). She also reports painful intercourse, which is becoming a problem as she
now tries to avoid intercourse, even though she would like to conceive. She is otherwise in good health and has been
married for 5 years. She is 5 feet 4 inches tall and weighs 130 pounds. She has a history of pelvic inflammatory
disease at age 19, for which she was hospitalized. Her mother had a history of ovarian cancer at age 49. Physical
exam is notable for some lower quadrant tenderness on abdominal and pelvic exams. Uterus is normal in size, but
there is a slightly tender palpable left adnexal mass. A pelvic ultrasound shows a 5 cm left complex ovarian cyst and
2 simple cysts measuring 2 cm in the right ovary. What is the underlying pathophysiology of the disease process in
this patient?
a) Chronic pelvic inflammatory disease
b) Her family history of ovarian cancer
c) Endometrial glands outside of uterine cavity
d) Polycystic ovarian syndrome
e) Functional hemorrhagic cysts
Incorrect!!! The patient has typical signs of endometriosis, which is characterized by the presence of
endometrial glands and stroma outside of the uterus. Endometriosis is present in about 30% of infertile
woman. She does not have the signs and symptoms of chronic pelvic inflammatory disease. She also does
not have the signs and symptoms of polycystic ovarian syndrome, which typically presents with
oligomenorrhea in overweight patients. The complex ovarian cyst is most likely an endometrioma. The
duration of her symptoms makes functional hemorrhagic cyst a less likely option.
A 63-year-old G0 woman comes to the office for a health maintenance exam. She is healthy and not taking any
medications. She has no history of abnormal Pap smears or sexually transmitted diseases. She has a history of
endometriosis and infertility in the past. She has been postmenopausal for 10 years and is not on hormone
replacement therapy. She is 5 feet 4 inches tall and weighs 130 pounds. On pelvic examination, the patient has a
palpable left adnexal mass. An ultrasound was obtained, which showed a 5 cm complex left ovarian cyst. What is
the most appropriate next step in the management of this patient?
a) CAT scan of the abdomen and pelvis
b) MRI of the pelvis
c) Exploratory surgery
d) Repeat ultrasound in 3 months
e) Observation
Incorrect!!! A complex ovarian mass in a postmenopausal patient needs to be surgically explored. Although
this could be an old endometrioma which never resolved, this cannot be assumed. If this is ovarian cancer,
waiting 3 months can change this patient’s prognosis and, most likely, it will not resolve, since this is not a
physiological cyst. A CAT scan or MRI will not add more information and ultrasounds are typically the
best imaging studies for the uterus and adnexa.
A 26-year-old G0 woman returns for a follow-up visit regarding endometriosis. She has been using NSAIDs to
manage her pelvic pain, but it has been getting worse recently and she has had to miss 4 days of work in the last two
months. She is sexually active with her husband of 2 years, although it has been more painful recently. She has
regular menstrual cycles and is using condoms for contraception. On pelvic exam, she had localized tenderness in
the cul de sac and there were no palpable masses. What is the most appropriate next step in the management of this
patient?
a) GnRH agonist
b) Observation
c) Oral contraceptives
d) Laparoscopy and ablation of endometriosis
e) Progesterone intrauterine device
Incorrect!!! Oral contraceptives will be the next best choice for this patient. They provide negative
feedback to the pituitary-hypothalamic axis which, in turn, stops stimulating the ovary to produce sex
hormones, such as estrogen which, in turn, stimulates endometrial tissue located outside of the
endometrium and uterus. GnRH agonists also exert negative feedback, but can be used short term only and
have more side effects. Observation is not the next best step, as the patient has worsening symptoms. It is
appropriate for patients with limited disease and mild or no symptoms, or for those trying to conceive.
Laparoscopy is indicated in the patient who fails medical treatment and/or is planning pregnancy in the
near future. A progesterone intrauterine device might potentially help alleviate some of her symptoms but
is not the best management for endometriosis.
A 26-year-old G0 woman returns for a follow-up visit regarding suspected endometriosis. She has been using
NSAIDs and birth control pills to help manage her pelvic pain, which has been getting worse. While discussing
further treatment options, she asks if there is any test or procedure you can perform to confirm her diagnosis. Which
of the following tests or procedures would you recommend?
a) Pelvic ultrasound
b) Blood FSH/LH ratio and estradiol level
c) MRI of the pelvis
d) Exploratory laparoscopy
e) History and physical exam confirm the diagnosis
Correct!!! Definitive diagnosis is based on exploratory surgery and biopsies, although endometriosis is
usually initially treated based on the clinical presentation. In addition, this patient can benefit from
laparoscopy, since she has failed the two most common treatments for endometriosis, NSAIDs and OCPs.
There is no imaging study or blood test that can confirm the diagnosis of endometriosis. Although often
used when implementing initial treatment, history and physical exam are not sufficient to confirm the
diagnosis as patients present in diverse ways.
A 48-year-old G0 woman comes to the office for a health maintenance exam. She is healthy and not taking any
medications. She has no history of abnormal Pap smears or sexually transmitted infections. Her menstrual cycles are
normal and her last cycle was three weeks ago. Her mother was diagnosed with endometriosis and had a
hysterectomy and removal of the ovaries at age 38. She is 5 feet 4 inches tall and weighs 130 pounds. On pelvic
examination, the patient had a palpable left adnexal mass. An ultrasound was obtained, which showed a 4 cm
complex left ovarian cyst and a 2 cm simple cyst on the right ovary. What is the most likely diagnosis in this
patient?
a) An endometrioma
b) A hemorrhagic cyst
c) Ovarian carcinoma
d) A mature teratoma
e) Polycystic ovaries
Correct!!! This patient most likely has a hemorrhagic cyst, considering her history and where she is in her
menstrual cycle. Her mother’s history of endometriosis does increase her risk; however, it is unlikely since
she has never had any symptoms herself. Ovarian carcinoma would need to be ruled out, but it is unlikely
in an otherwise asymptomatic premenopausal patient. A mature teratoma would have more pathognomonic
findings on ultrasound. This patient does not have typical symptoms, body habitus or ultrasound findings
for patients with polycystic ovaries.
A 48-year-old G0 woman comes to the office for a health maintenance exam. She is healthy and not taking any
medications. She has no history of abnormal Pap smears or sexually transmitted diseases. She is not currently
sexually active. Her menstrual cycles are normal and her last cycle was three weeks ago. She smokes one pack of
cigarettes per day. Her mother was diagnosed with endometriosis and had a hysterectomy and removal of the ovaries
at age 38. She is 5 feet 4 inches tall and weighs 130 pounds. On pelvic examination, the patient had a palpable left
adnexal mass. An ultrasound was obtained, which showed a 4 cm complex left ovarian cyst and a 2 cm simple cyst
on the right ovary. What is the most appropriate next step in the management of this patient?
a) CAT scan of the abdomen and pelvis
b) MRI of the pelvis
c) Abdominal hysterectomy and bilateral salpingo-oophorectomy (TAH/BSO)
d) Repeat ultrasound in 2 months
e) Oral contraceptives
Incorrect!!! A repeat ultrasound is the most appropriate next step, as this is most likely a hemorrhagic cyst
which will resolve on its own. A CAT scan or MRI of the pelvis will not add any more information. There
is no indication to proceed with a TAH/BSO. Oral contraceptives are contraindicated in this patient, as she
is older than 35 and smokes.
A 26-year-old G0 woman presents to the emergency room with severe right lower quadrant pain associated with
nausea for the last 6 hours, which began shortly after she finished her aerobic exercises. She has a history of
suspected endometriosis, which was diagnosed 2 years ago, based on her severe dysmenorrhea. She has been using
NSAIDs with her menses to control the pain. She is not sexually active, and is otherwise in good health. Her
menstrual cycles are normal every 28 days and her last menstrual period was 3 weeks ago. She has no history of
sexually transmitted infections. Her blood pressure is 145/70; pulse is 100; temperature is 99.2°F. She appears
uncomfortable. On abdominal exam, she has moderate tenderness to palpation in the right lower quadrant. On pelvic
exam, she has no lesions or discharge. A complete bimanual exam was difficult to perform due to her discomfort.
BHCG <5 and WBC is upper limit normal. A pelvic ultrasound showed a 6cm right ovarian mass. The uterus and
left ovary appeared normal. There was a moderate amount of free fluid in the pelvis. What is the most likely
diagnosis in this patient?
a) Appendicitis
b) Exacerbation of the endometriosis
c) Ovarian carcinoma
d) Ovarian torsion
e) Ovarian ectopic pregnancy
Incorrect!!! The sudden onset of pain and nausea, as well as the presence of a cyst on ultrasound suggest
ovarian torsion. Although appendicitis is on the differential, it is unlikely to have such a sudden onset of
pain and a normal white count. Her endometriosis can get worse but it would be unlikely to be of such
sudden onset. Although she has an ovarian mass, the BHCG is negative, which rules out pregnancy.
A 26-year-old G0 woman presents to the emergency room with 8 hours of severe right lower quadrant pain
associated with nausea. She has a history of suspected endometriosis, which was diagnosed 2 years ago, based on
severe dysmenorrhea. She has been using NSAIDs with her menses to control the pain. She is not sexually active.
She is otherwise in good health. Her menstrual cycles are normal every 28 days and her last menstrual period was 3
weeks ago. She has no history of sexually transmitted infections. Her blood pressure is 145/70; pulse is 100;
temperature is 99.2°F. She appears uncomfortable. On abdominal exam, she has moderate tenderness to palpation in
the right lower quadrant. On pelvic exam, she has no lesions or discharge. A complete bimanual exam was difficult
to perform due to her discomfort. Labs: BHCG <5, hematocrit 29%. A pelvic ultrasound showed a 6cm right ovarian
mass. The uterus and left ovary appeared normal. There was a moderate amount of free fluid in the pelvis. What is
the most appropriate next step in the management of this patient?
a) MRI of the pelvis
b) Doppler pelvic ultrasound
c) CAT scan of the pelvis
d) Begin oral contraceptives
e) Surgical exploration
Correct!!! This patient most likely has ovarian torsion and needs to be surgically explored. Further imaging
studies will not help beyond the information obtained on the ultrasound. A Doppler ultrasound to check the
blood flow to the ovaries is controversial, as normal flow does not rule out ovarian torsion. Although oral
contraceptives can help decrease the development of further cyst formation and control the pain associated
with endometriosis, this patient needs immediate surgical attention due to suspected ovarian torsion.
A 29-year-old G0 woman presents due to the inability to conceive for the last two years. She has a known history of
endometriosis, which was diagnosed by laparoscopy 3 years ago. She has pelvic pain, which is controlled with nonsteroidal anti-inflammatory drugs. Her cycles are regular every 28 days. She is otherwise in good health and has
been married for 5 years. Her husband had a semen analysis, which was normal. She had a hysterosalpingogram,
which showed patent tubes bilaterally. She is getting frustrated that she has not yet gotten pregnant and asks to
proceed with fertility treatments. What is the most appropriate next step in the management of this patient?
a) Perform a laparoscopy
b) Administer a GnRH agonist
c) Ovarian stimulation with Clomiphene Citrate
d) Proceed with in vitro fertilization
e) Intrauterine insemination
Incorrect!!! A patient with a known history of endometriosis who is unable to conceive and has an
otherwise negative workup for infertility, benefits from ovarian stimulation with Clomiphene Citrate, with
or without intrauterine insemination. A GnRH agonist is used to control pelvic pain in endometriosis
patients unresponsive to other hormonal treatments. A laparoscopy with treatment of endometriosis can
increase fertility rates but, in this patient, is better reserved if she fails medical treatment. Intrauterine
insemination can be added if ovarian stimulation alone does not result in pregnancy. In vitro fertilization
and adoption can be offered if other treatments fail.
REI - Exam 46
A 19 year-old G0 presents with severe menstrual pain which causes her to miss school. She takes 600 mg of
ibuprofen every 4-6 hours to control the pain, but this does not relieve the discomfort. She is sexually active, with
one present partner (two lifetime partners) and uses condoms for contraception. Examination is normal. What is the
most appropriate next step in the management of this patient?
a) Change NSAID to a COX-2 inhibitor
b) GnRH agonist
c) Oral contraceptives
d) Continuous medroxyprogesterone
e) Laparoscopy
Correct!!! Dysmenorrhea or painful menstrual cramps is often incapacitating. Oral contraceptives will not
only relieve primary dysmenorrhea, but also provide more reliable contraception. COX-2 inhibitors have
targeted action but have significant side effects, and are no longer routinely prescribed. Continuous oral
Medroxyprogesterone may be effective, but will not provide contraception. Depo-Provera would be a better
choice. GnRH agonists are too expensive and have too high a side effect profile to be used for this purpose.
A 15 year-old G0 presents with severe menstrual pain for the past 12 months. The pain is severe enough for her to
miss school. The pain is not relieved with ibuprofen 600 mg every 4 hours. She is not sexually active and the
workup reveals no pathology. The most appropriate next step in the management of this patient is to begin
combination oral contraceptives. How do oral contraceptives relieve primary dysmenorrhea?
a) Creating endometrial atrophy
b) Decreasing inflammation
c) Increasing prolactin levels
d) Decreasing inhibin levels
e) Thickening cervical mucous
Incorrect!!! The progestin in oral contraceptives creates endometrial atrophy. Since prostaglandins are
produced in the endometrium, there would be less produced. Dysmenorrhea should be improved.
A 21 year-old G0 presents with severe menstrual pain. She takes 600 mg of ibuprofen every 4-6 hours to control the
pain, but this does not relieve the discomfort. She is sexually active with one present partner and has four lifetime
partners. She uses condoms for contraception. Past medical history is unremarkable, except for breast cancer in her
father’s sister. Examination is normal. In addition to a Pap smear, what is the most appropriate additional test needed
for this patient?
a) Baseline mammography
b) Chlamydia testing
c) HIV testing
d) HPV DNA typing
e) Lipid profile
Correct!!! The U.S. Preventive Services Task Force recommends chlamydia and gonorrhea screening for
all sexually active patients, age 25 and younger. Since pelvic inflammatory disease is a cause of secondary
dysmenorrhea, it needs to be evaluated as a potential cause of her symptoms. Although HPV screening is
becoming more common, it can be used as an adjunct to cytology in primary screening in women 30 years
or older, and is not indicated in a 21 year-old unless her Pap smear shows ASCUS. Lipid screening is not
necessary in a 21 year-old otherwise healthy woman with no risk factors.
A 19 year-old G0 woman presents with severe menstrual pain that causes her to miss school. She takes 600 mg of
ibuprofen every 4-6 hours to control the pain, but this does not relieve the discomfort. You started oral
contraceptives, but her symptoms persisted. She also tried Depo-Provera for 3 months without much improvement.
She still has menstrual pain and continues to miss some classes. What is the most appropriate next step in the
management?
a) Transdermal narcotic for pain relief
b) Diagnostic laparoscopy
c) Presacral neurectomy
d) Prescribe a selective serotonin reuptake inhibitor
e) Prescribe GnRH agonists
Incorrect!!! Laparoscopy is recommended to confirm the diagnosis of endometriosis and exclude other
causes of secondary dysmenorrhea. Some authors suggest that a course of GnRH agonists are appropriate,
with laparoscopy reserved for those women who have pain during or after completion of a 3-month course.
A 23 year-old G0 woman with severe dysmenorrhea that is unresponsive to non-steroidal anti-inflammatory agents
and oral contraceptives is taken to the operating room for a laparoscopy. Blue-black powder burn lesions are seen in
the pelvis. A biopsy is performed and sent to pathology. Which of the following pathologic lesions would you
expect to see in this patient?
a) Blue-domed cysts greater than 3 mm
b) Decidual effect in the endometrium
c) Endometrial glands or stroma and hemosiderin-laden macrophages
d) Invasion of endometrial glands into the myometrium
e) Well-circumscribed, non-encapsulated myometrium
Correct!!! The lesions described are classic for endometriosis. One would therefore expect to see
endometrial glands/stroma with hemosiderin-laden macrophages.
A 42 year-old G4P4 woman presents with a history of progressively worsening severe menstrual pain. Menses are
regular, but she complains of very heavy flow requiring both a menstrual pad and tampon with frequent bleeds
through this protection on heavy days. She takes Oxycodone that her husband used for back pain to relieve her
dysmenorrhea. She had a tubal ligation four years ago. Pelvic examination shows an enlarged, soft, boggy uterus.
No masses are palpated. Pregnancy test is negative, hemoglobin 9.8 and hematocrit 28.3%. What is the most likely
diagnosis?
a) Adenomyosis
b) Endometrial carcinoma
c) Endometriosis
d) Primary dysmenorrhea
e) Endometrial hyperplasia
Incorrect!!! This is a typical presentation of adenomyosis (presence of endometrial glands and supporting
tissues in the muscle of the uterus). The gland tissue grows during the menstrual cycle and, at menses, tries
to slough, but cannot escape the uterine muscle and flow out of the cervix as part of normal menses. This
trapping of the blood and tissue causes uterine pain in the form of monthly menstrual cramps. Endometrial
hyperplasia and carcinoma are less likely in a woman with regular menses. Endometriosis would most
likely have presented earlier in life and would not explain the enlarged uterus.
A 42 year-old G4P4 woman presents for management of suspected adenomyosis. She had a tubal ligation four years
ago. A pelvic examination shows an enlarged, soft, boggy uterus. A pregnancy test is negative and she is mildly
anemic. An ultrasound shows an enlarged uterus with no fibroids. The patient desires definitive treatment for this
condition. What is the most appropriate next step in her management?
a) Continuous estrogen/progestin therapy
b) Endometrial ablation
c) GNRH agonists
d) Hysterectomy
e) Insertion of a levonorgestrel containing intrauterine system
Incorrect!!! Hysterectomy is nearly 80% effective in eliminating pain and abnormal bleeding, if she is
willing to undergo surgery. Gonadotropin releasing agents are the first choice for medical therapy for the
pain, but the problem is that the adenomyosis seems to recur after discontinuing the therapy. Endometrial
ablation and insertion of a levonorgestrel-containing intrauterine system are options in women who decline
hysterectomy or desire to maintain fertility. For abnormal bleeding problems and desire for uterine
conservation, a progesterone intrauterine contraceptive device can also be used to improve irregular
bleeding. Hysteroscopic endometrial ablation can be a treatment for adenomyosis.
A 43 year-old G4P4 woman presents with a history of progressively worsening, severe menstrual pain. Her workup
is consistent with adenomyosis, and she selects hysterectomy for definitive treatment. What is the pathologic lesion
to confirm the diagnosis?
a) Blue-domed cysts greater than 3 mm.
b) Decidual effect in the endometrium
c) Endometrial glands/stroma and hemosiderin-laden macrophages
d) Invasion of endometrial glands into the myometrium
e) Well-circumscribed, non-encapsulated myometrium
Incorrect!!! Invasion of endometrial glands into the myometrium is the pathological hallmark of
adenomyosis.
A 41 year-old G2P2 woman presents with menstrual pain, menorrhagia, irregular periods and intermenstrual
bleeding. She describes the pain as pressure and cramps. Ibuprofen improves the pain, but does not entirely
eliminate the discomfort. Pelvic examination reveals a 14-week size uterus with irregular masses within the uterus.
Pelvic ultrasound confirms the diagnosis of fibroids. What is the most appropriate next step in the management of
this patient?
a) CA125 assay
b) CT scan of the pelvis
c) Endometrial biopsy
d) GnRH agonists
e) Hysterectomy
Incorrect!!! This patient has classic symptoms of leiomyomata, including menorrhagia. An endometrial
biopsy should be performed on all women over age 40 with irregular bleeding to rule out endometrial
carcinoma. The CA125 assay measures the level of CA125 in the blood and is increased in some types of
cancer, including ovarian cancer or other conditions. This non-specific marker is not indicated in this
patient. A CT scan of the pelvis is also not indicated. A simple pelvic ultrasound could be used to help
confirm the clinical diagnosis. GnRH agonist and hysterectomy are not used until the diagnosis of
leiomyomata uteri is confirmed. Her desire for future fertility should be discussed.
A 42 year-old G2P2 woman undergoes a hysterectomy for definitive treatment of her dysmenorrhea and large
uterine fibroids. The uterus is sent to pathology. Which of the following would confirm the diagnosis of fibroids?
a) Blue-domed cysts greater than 3 mm
b) Decidual effect in the endometrium
c) Endometrial glands/stroma and hemosiderin-laden macrophages
d) Invasion of endometrial glands into the myometrium
e) Well-circumscribed, non-encapsulated myometrium
Incorrect!!! Well-circumscribed, non-encapsulated myometrium confirms the diagnosis of fibroids.
Leiomyosarcomas will have > than 10 mitotic figures per high power field.
Gynecology - Exam 39
A 17-year-old G0 woman presents with a 3-year history of severe dysmenorrhea shortly after menarche at age 14.
Her menstrual cycles occur every 27-29 days and last 5-7 days, with heavy flow. She has been treated with NSAIDs
and cyclic oral contraceptives for the last year without significant improvement. She misses 2-3 days of school each
month due to her menses. She has never been sexually active. Physical exam is remarkable for breasts, Tanner Stage
IV, and pubic hair, Tanner Stage IV. Pelvic examination is normal. Both the patient and her mother are concerned.
What is the next best step in the management of this patient?
a) Empiric treatment with GnRH agonist
b) CT of the pelvis
c) Diagnostic laparoscopy
d) Pelvic ultrasound
e) A hysterosalpingogram
Incorrect!!! Chronic pelvic pain is the indication for at least 40% of all gynecologic laparoscopies.
Endometriosis and adhesions account for more than 90% of the diagnoses in women with discernible
laparoscopic abnormalities, and laparoscopy is indicated in women thought to have either of these
conditions. Often, adolescents are excluded from laparoscopic evaluation on the basis of their age, but
several series show that endometriosis is as common in adolescents with chronic pelvic pain as in the
general population. Therefore, laparoscopic evaluation of chronic pelvic pain in adolescents should not be
deferred based on age. A GnRH agonist should not be used in a patient without confirmed diagnosis of
endometriosis at such a young age.
A 24-year-old G0 woman presents with a one-year history of introital and deep thrust dyspareunia. She also has a 2year history of severe dysmenorrhea, despite the use of oral contraceptives. She underwent a diagnostic laparoscopy
6 months ago that showed minimal endometriosis with small implants in the posterior cul de sac only, which were
ablated with a CO2 laser. On further questioning, she reports significant urinary frequency, urgency and nocturia. A
recent urine culture was negative. The most likely additional diagnosis for this patient is:
a) Acute cystitis
b) Interstitial cystitis
c) Acute urethral syndrome
d) Acute urethritis
e) Salpingitis
Incorrect!!! Interstitial cystitis (IC) is a chronic inflammatory condition of the bladder, which is clinically
characterized by recurrent irritative voiding symptoms of urgency and frequency, in the absence of
objective evidence of another disease that could cause the symptoms. Pelvic pain is reported by up to 70%
of women with IC and, occasionally, it is the presenting symptom or chief complaint. Women may also
experience dyspareunia. The specific etiology is unknown, but IC may have an autoimmune and even
hereditary component.
A 32-year-old G3P2, with a last menstrual period 2 weeks ago, presents with a 6-month history of abdominal pain.
She has noncyclic intermittent pain, which she describes as crampy and diffuse across the lower abdomen. Her pain
is typically relieved with defecation and is associated with loose, watery stools. Onset of the symptoms is associated
with a change in stool frequency from once daily to multiple times daily. She also experiences bloating and
abdominal distention several times a week. Her medical history is significant for chronic migraines and she denies
previous surgery. Her gynecological history is unremarkable. Her abdominal and pelvic examinations are both
normal, except for mild tenderness in the left lower quadrant to palpation. The most likely diagnosis in this patient
is:
a) Pelvic adhesions
b) Diverticulosis
c) Gastroenteritis
d) Irritable bowel syndrome
e) Mechanical gastrointestinal cause
Correct!!! Irritable bowel syndrome (IBS) is a common functional bowel disorder of uncertain etiology. It
is characterized by a chronic, relapsing pattern of abdominal and pelvic pain, and bowel dysfunction with
constipation or diarrhea. IBS is one of the most common disorders associated with chronic pelvic pain. IBS
appears to occur more commonly in women with chronic pelvic pain than in the general population.
Diagnosis is based on the Rome II Criteria for IBS, which includes at least 12 weeks (need not be
consecutive) in the preceding 12 months of abdominal discomfort or pain that has 2 of 3 features: 1) relief
with defecation; 2) onset associated with a change in frequency of stool; or 3) onset associated with a
change in stool form or appearance. The patient’s history does not support pelvic adhesions, and
diverticulosis (although very common) typically may be asymptomatic unless inflammation/infection
develops. In this case, the symptoms for IBS may be indistinguishable from diverticulitis or severe
diverticular disease.
A 22-year-old G0 woman graduate student, presents with worsening pelvic pain. She previously underwent a
laparoscopic ablation of endometriosis followed by continuous oral-contraceptive pills. She had short-term relief
from this approach, but now has failed this treatment and is seeking additional medical management. Which of the
following mechanisms best explains how a gonadotropin releasing hormone (GnRH) agonist would help alleviate
her symptoms?
a) "Down-regulation” of the hypothalamic-pituitary gland production and release of LH and FSH leading to
reduction in estradiol levels
b) “Up-regulation” of the hypothalamic-pituitary gland production and release of LH and FSH leading to elevation
in estradiol levels
c) Suppression of both LH and FSH mid-cycle surges, resulting in the loss of ovarian production of estrogen
d) Induction of a “pseudopregnancy” state, causing a decidual reaction in the functioning endometrial tissue
e) Competitive inhibitor for estrogen receptors
Correct!!! Gonadotropin-releasing hormone (GnRH) agonists are analogues of naturally occurring
gonadotropin-releasing hormones that down-regulate hypothalamic-pituitary gland production and the
release of luteinizing hormone and follicle-stimulating hormone leading to dramatic reductions in estradiol
level. The GnRH agonists available in the United States are Nafarelin, Goserelin and Leuprolide.
Numerous clinical trials show GnRH agonists are more effective than placebo and as effective as Danazol
in relieving endometriosis-associated pelvic pain. Danazol, a 17-alpha-ethinyl testosterone derivative,
suppresses the mid-cycle surges of LH and FSH. Combined estrogen and progestin therapy in oral
contraceptives produces the ‘pseudopregnancy’ state.
A 35-year-old G1P0 woman business executive with last menstrual period one week ago presents with an 8-month
history of pelvic pain. She reports regular menstrual cycles with moderate flow and dysmenorrhea, relieved with
NSAIDs. She describes her pain as a deep, achy sensation with frequent sharp exacerbations. She has not been
sexually active for the last several months because of dyspareunia and some arguments with her new partner of 1
year. She uses no steroid contraception. She has no history of sexually transmitted infections. Her medical history is
significant for irritable bowel syndrome, managed with a fiber supplement. She has smoked one pack of cigarettes a
day since age 25, and drinks a glass of wine three times a week. She tries to exercise regularly by running three to
four times a week. This new pain is distinctly different from her IBS symptoms. Which of the following risk factors
can contribute to increased incidence of pelvic pain in this patient?
a) Her alcohol use
b) Her smoking habit
c) Her job
d) Her new partner
e) Her age
Incorrect!!! Most published evidence suggests a significant association of physical and sexual abuse with
various chronic pain disorders. The arguments with the new partner allude to possible abuse issues. Studies
have found that 40-50% of women with chronic pelvic pain have a history of abuse. Whether abuse
(physical or sexual) specifically causes chronic pelvic pain is not clear, nor is a mechanism established by
which abuse might lead to the development of chronic pelvic pain. Women with a history of sexual abuse
and high somatization scores have been found to be more likely to have non-somatic pelvic pain,
suggesting the link between abuse and chronic pelvic pain may be psychologic or neurologic. However,
studies also suggest that trauma or abuse may also result in biophysical changes, by literally heightening a
person's physical sensitivity to pain.
A 48-year-old G4P4 woman with her last menstrual period 4 weeks ago presents with a 1 year history of noncyclical pelvic pain, dysmenorrhea and dyspareunia. She has a past history of endometriosis, diagnosed 10-years ago
by laparoscopy. She had previously been on cyclic oral contraceptives for both birth control and menstrual cycle
control, but elected for permanent laparoscopic sterilization 14 months ago. Minimal endometriosis was noted at the
time of laparoscopy. She now has recurrent symptoms, but wants to avoid medication. The most appropriate surgical
option for this patient is:
a) Hysteroscopy and dilation and curettage
b) Diagnostic laparoscopy
c) Hysterectomy with bilateral salpingo-oophorectomy
d) Endometrial ablation
e) Wedge resection of the ovaries
Correct!!! It is estimated that chronic pelvic pain is the principal preoperative indication for 10-12% of
hysterectomies. Since the patient had a tubal ligation, and does not desire any more children, the best
option is removal of ovaries with or without a hysterectomy. Repeat laparoscopy with treatment of
endometriosis and adhesions can be helpful; however, the patient will continue to be at increased risk of
recurrent disease. An endometrial ablation or removal of ovaries alone would not be very helpful in the
setting of non-cyclical pain.
A 62-year-old G5P5 grandmother, menopausal for 12 years, presents with a 7-month history of pelvic pain and
pressure, as well as abdominal distention and bloating. She experiences occasional constipation, but no melena or
hematochezia. She also has mild to moderate urinary frequency without dysuria, hematuria or flank pain. She
describes no exacerbating or alleviating factors. Her medical history is significant for hypertension and obesity. She
has never been on hormone therapy. She notes one episode of light vaginal bleeding several months ago. Her family
history is significant for postmenopausal ovarian cancer in her mother and maternal aunt, but is otherwise negative
for breast, endometrial or colon carcinoma. Pelvic examination is remarkable for vaginal atrophy, cervical stenosis
and difficult uterine and adnexal assessment due to her body habitus. What is the most appropriate next step in the
management of this patient?
a) Performing a transvaginal ultrasound
b) Diagnostic laparoscopy
c) Computed Axial Tomography (CT) scan of the abdomen and pelvis
d) Colonoscopy
e) Hysteroscopy
Correct!!! Given the patient’s age, nonspecific abdomino-pelvic symptoms, recent postmenopausal
bleeding episode and family history of ovarian cancer, a transvaginal ultrasound is the next best step as it is
more sensitive than CT for evaluation of the uterus and adnexa. Endometrial thickness of 4 mm or less on
transvaginal ultrasound in postmenopausal women is rarely associated with endometrial malignancy.
Colonoscopy is useful for colorectal cancer screening, as well as evaluation of the patient’s gastrointestinal
symptoms, but would not provide information regarding pelvic anatomy. Diagnostic laparoscopy would be
a more invasive procedure that could be performed as indicated, after these other diagnostic studies.
Hysteroscopy might be useful based on the ultrasound results, since it might be difficult to perform an
endometrial biopsy in the office.
A 29-year-old G0 woman, presents for follow up after hospitalization for pelvic inflammatory disease and right
tubo-ovarian abscess (TOA) 14-months ago. She was treated with parenteral antibiotics. She now has a 10-month
history of pelvic pain and deep thrust dyspareunia, as well as a persistent right adnexal tubular mass on ultrasound.
She desires future fertility. Pelvic examination reveals a retroverted, normal-sized uterus with limited mobility and
marked tenderness and fullness in the right adnexa. Findings at the time of laparoscopy include multiple filmy and
dense adhesions between the posterior uterus and cul-de-sac, normal left fallopian tube and ovary, and large right
hydrosalpinx, with a few filmy adhesions between the right ovary and distended fallopian tube. What is the most
appropriate treatment for this patient?
a) Exploratory laparotomy, right salpingo-oophorectomy and lysis of adhesions
b) Exploratory laparotomy, right salpingectomy and lysis of adhesions
c) Laparoscopic right salpingo-oophorectomy and lysis of adhesions
d) Laparoscopic right salpingectomy and lysis of adhesions
e) Total abdominal hysterectomy and bilateral salpingo-oophorectomy
Incorrect!!! Approximately 18-35% of all women with acute pelvic inflammatory disease (PID) develop
chronic pelvic pain. The actual mechanisms by which chronic pelvic pain results from PID are not known
and not all women with reproductive organ damage secondary to acute PID develop chronic pelvic pain.
Chronic pelvic pain is the indication for at least 40% of all gynecologic laparoscopies. Endometriosis and
adhesions account for more than 90% of the diagnoses in women with discernible laparoscopic
abnormalities. Given this patient’s desire for future fertility, conservative surgical intervention is indicated
with lysis of adhesions. Retention of the patient’s ovaries is also possible, given the limited involvement
with adhesions. Since the patient has a persistent hydrosalpinx and pelvic pain, the right fallopian tube
should be removed with conservation of the right ovary. A salpingostomy (an incision in the tube) would
not be adequate treatment for this patient.
A 33-year-old G2P2 woman reports a 2-year history of severe dysmenorrhea, menorrhagia and pelvic pain
immediately following the forceps-assisted delivery of her last child, which was complicated by postpartum
hemorrhage and right vaginal sidewall laceration. She describes her pelvic pain as primarily in the right lower
quadrant, radiating into the vagina. Her pain worsens throughout the day with standing and is associated with pelvic
pressure and fullness. Her pelvic examination reveals a mildly enlarged uterus with marked tenderness to palpation
of the right adnexa, and no other significant findings. A vaginal ultrasound with color-flow Doppler reveals multiple
dilated vessels traversing the right broad ligament to the lower uterus and cervix. The uterus shows no fibroids or
other significant changes. Endometrial thickness appears normal. Which of the following is the most likely diagnosis
in this patient?
a) Endometriosis
b) Endometritis
c) Adenomyosis
d) Pelvic congestion
e) Pelvic floor relaxation
Incorrect!!! The pelvic congestion syndrome is a cause of chronic pelvic pain occurring in the setting of
pelvic varicosities. The unique characteristics of the pelvic veins make them vulnerable to chronic
dilatation with stasis leading to vascular congestion. These veins are thin walled and unsupported, with
relatively weak attachments between the supporting connective tissue. The cause of pelvic vein congestion
is unknown. Hormonal factors contribute to vasodilatation when pelvic veins are exposed to high
concentration of Estradiol, which inhibits reflex vasoconstriction of vessels, induces uterine enlargement
with selective dilatation of ovarian and uterine veins. This pain may be of variable intensity and duration, is
worse premenstrually and during pregnancy, and is aggravated by standing, fatigue and coitus. The pain is
often described as a pelvic “fullness” or “heaviness,” which may extend to the vulvar area and legs.
Associated symptoms include vaginal discharge, backache and urinary frequency. Menstrual cycle defects
and dysmenorrhea are common. No signs of pelvic floor relaxation were noted on exam.
A 45-year-old G2P2 woman underwent an abdominal hysterectomy for a large fibroid uterus via a low transverse
skin incision. Her postoperative course was significant for new onset right lower quadrant pain and numbness,
radiating into the right inguinal area and medial thigh. Her pain was exacerbated by adduction of her right thigh. On
abdominal examination, there is a well-healed low transverse incision. Her pain is reproduced with adduction of the
right thigh. There is decreased sensation to light touch and pinprick over the right inguinal area and right medial
thigh. Patellar reflexes are 2+ and symmetric. Entrapment of which of the following nerves is the most likely cause
of her pain?
a) Obturator nerve
b) Ilioinguinal nerve
c) Lateral femoral cutaneous nerve
d) Femoral nerve
e) Iliohypogastric
Incorrect!!! “Nerve entrapment syndrome” is a commonly misdiagnosed neuropathy that can complicate
pelvic surgical procedures performed through a low transverse incision. The nerves at risk are the
iliohypogastric nerve (T-12, L-1) and the ilioinguinal (T-12, L-1) nerve. These two nerves exit the spinal
column at the 12th vertebral body and pass laterally through the psoas muscle before piercing the
transversus abdominus muscle to the anterior abdominal wall. Once at the anterior superior iliac spine, the
iliohypogastric nerve courses medially between the internal and external oblique muscles, becoming
cutaneous 1cm superior to the superficial inguinal ring. The iliohypogastric nerve provides cutaneous
sensation to the groin and the skin overlying the pubis. The ilioinguinal nerve follows a similar, although
slightly lower, course as the iliohypogastric nerve where it provides cutaneous sensation to the groin,
symphysis, labium and upper inner thigh. These nerves may become susceptible to injury when a low
transverse incision is extended beyond the lateral border of the rectus abdominus muscle, into the internal
oblique muscle. Symptoms are attributed to suture incorporation of the nerve during fascial closure, direct
nerve trauma with subsequent neuroma formation, or neural constriction due to normal scarring and
healing.
**Gynecology - Exam 40
A 68 year-old G3P3 woman comes to the office due to breast tenderness. She is in good health and not taking any
medications. Family history is significant for her 70 year-old sister being diagnosed with breast cancer. On breast
examination, her breasts have no lesions; there are no palpable masses, nodules or lymphadenopathy. Her last
mammogram was 4 months ago and was normal. What is the most appropriate next step in the management of this
patient?
a) Order a mammogram
b) Order a breast ultrasound
c) Obtain genetic testing (BRCA-1 and BRCA-2 mutations)
d) Reassurance
e) Order a breast MRI
Correct!!! Age and gender are the greatest risk factors for developing breast cancer. Having one firstdegree relative with breast cancer does increase the risk, but genetic mutations occur in a low percentage of
the general population. There is no indication for a mammogram since the patient’s last mammogram was
normal 4 months ago. Ultrasound and MRI would not add valuable information especially in the setting of
a normal mammogram and no masses on physical examination. Genetic testing is not indicated in this case
as there is no strong family history and the sister with breast cancer was postmenopausal at time of diagnos
A 31 year-old woman presents at the office for an annual exam. She has no prior history of breast disease and has
large, pendulous breasts. There is no family history of breast cancer. Which of the following positions enhances the
visualization of asymmetry?
a) Upright position of the patient
b) Leaning forward
c) Pressing her hands on her hips
d) Left lateral decubitus position
e) Right lateral decubitus position
Incorrect!!! A properly performed clinical breast exam includes inspection and palpation of the breasts.
Leaning forward facilitates the inspection for asymmetry even if the patient has large and pendulous breasts
(this position allows the breasts to hang from the chest wall). A clinical breast exam should include
inspection of the patient in an upright, sitting position, while pressing her hands on her hips, placing hands
above the head and leaning forward. Moving the pectoralis muscle elicits skin changes that can be
associated with invasive breast cancer, such as skin puckering or dimpling. When the patient is supine with
her hands above her head, the breast tissue rests on the chest wall and is easier to palpate for masses. If the
patient has large breasts, placing a pillow under her shoulder can assist with maintaining an even
distribution of the breast tissue on the chest wall. Differences between the two breasts, such as differences
in breast contour or sizes, nipple inversion or retraction, may be other physical findings suggestive of an
underlying mass. Inspection of the patient in the lateral decubitus position is not usually helpful.
A 48 year-old woman presents with complaints of a white, watery nipple discharge for 4 months. She has been told
in the past she had fibrocystic breast changes, but otherwise has no significant medical problems or surgical history.
The white nipple discharge is noted on manual expression, but the exam is otherwise normal. Her serum prolactin
level was 45 ng/ml (normal below 40 ng/ml). What is the most appropriate next step in the management of this
patient?
a) Obtain a brain MRI
b) Obtain a β- HCG
c) Begin Bromocriptine
d) Obtain a fasting prolactin level
e) Order a ductogram
Incorrect!!! Stimulation of the breast during the physical examination may give rise to an elevated prolactin
level. Accurate prolactin levels are best obtained with patients fasting. If these are still elevated, then a
brain MRI would be indicated to rule out a pituitary tumor. Although pathologic factors, such as
hypothyroidism, hypothalamic disorders, pituitary disorders (adenomas, empty sella syndrome), chest
lesions (breast implants, thoracotomy scars, and herpes zoster) and renal failure can elevate prolactin levels,
a non-significant benign elevation needs to be ruled out first. A ductogram is usually indicated in patients
who have bloody discharge from the nipple.
42 year-old G3P3 patient comes to the office after noticing a breast mass while performing a breast self-exam. She
is in good health and has normal menstrual cycles. Family history is significant for multiple first and second degree
relatives having breast cancer. Physical exam reveals a 2-centimeter dominant breast mass. The remainder of the
exam is normal. A mammogram obtained today shows no abnormalities. What is the most appropriate next step in
the management of this patient?
a) Reassurance and observation
b) Obtain genetic testing
c) MRI of the breast
d) Fine needle aspiration
e) Repeat mammogram in 2 months
Correct!!! Any solid dominant breast mass on exam should be evaluated cytologically, with a fine needle
aspiration (FNA) or histologically, with an excisional biopsy. In this scenario, an MRI should not be part of
the initial work-up for the patient’s palpable breast mass. Testing for genetic mutations is indicated for
patients with a strong family history of breast cancer, but diagnosis is the most important next step in the
management of this patient. A normal mammogram does not rule out the presence of cancer, and there is
no reason to repeat the mammogram in two months, especially considering that the first one was normal.
A 24 year-old patient complains of cyclic mastalgia since the onset of her period at age 12. The symptoms have
increased over the years and were less troublesome when she took oral contraceptives a few years ago. Currently,
she takes no medications and is not sexually active. She is a strict vegetarian and eats soy products. She does not
smoke and reports she drinks a glass of wine three times a week, and several diet cokes every day. Her mother was
diagnosed with breast cancer at age 55. Her breast exam is normal, except for some fibrocystic changes. Which of
the following elements in her history contributes to her increasing pain?
a) Alcohol intake
b) Vegetarian diet
c) Family history of breast cancer
d) Caffeine intake
e) Age at menarche
Incorrect!!! Fibrocystic breast changes are the most common type of benign breast conditions and occur
most often during the reproductive years. There is an increased risk of breast cancer when atypia is present.
The changes do not appear distinct histologically (3 stages) or mammographically. Fibrocystic disease is
often associated with cyclic mastalgia, possibly related to a pronounced hormonal response. Caffeine intake
can increase the pain associated with fibrocystic breast changes, so recommending that she decrease her
caffeine intake might be helpful.
A 54 year-old woman presents with a breast mass she noticed two months ago. She has no family history of breast
cancer. On exam, there is a 2 cm mass palpable in the upper outer quadrant of the left breast. There are no other
masses noted and no lymphadenopathy. A fine needle aspiration returns bloody fluid and reduces the size of the
mass to 1 cm. In addition to obtaining a mammogram, what is the most appropriate next step in the management of
this patient?
a) Repeat exam in 2 months
b) Excisional biopsy of the mass
c) Obtain a breast MRI
d) Perform a lumpectomy and lymph node dissection
e) Follow-up in 1 year if mammogram is normal
Correct!!! The first noticeable symptom of breast cancer is typically a lump that feels different from the rest
of the breast tissue. More breast cancer cases are discovered when the woman feels a lump. . Breast cancer
can also present with a spontaneous bloody nipple discharge. Even though the mass decreased in size after
aspiration, the bloody discharge obtained obligates an excisional biopsy to be performed to rule out breast
cancer. If clear discharge is obtained on aspiration and the mass resolves, reexamination in 2 months is
appropriate to check that the cyst has not recurred. An MRI is not the appropriate next step and
lumpectomy with lymph node dissection is not yet indicated in this case. A normal mammogram does not
rule out breast cancer, especially in the presence of bloody discharge.
A 23 year-old nulliparous woman presents with a painful mass in her axilla for three days. She has no personal or
family history of breast disease. On exam, no abnormalities are seen on inspection and no breast mass is palpated. A
tender, enlarged lymph node is noted in her left axilla. What is the most likely underlying etiology for the lymph
node enlargement?
a) Early stage carcinoma
b) Fibroadenoma
c) Fibrocystic change
d) Infection
e) Duct obstruction
Incorrect!!! Although axillary lymphadenopathy can be a presenting sign for breast cancer, a tender mobile
axillary lymph node is more typically associated with an infectious process. Adenopathy associated with
carcinoma is usually firm, non-tender and fixed. Fibroadenomas are common, but are usually firm, painless
and freely movable. A clogged milk duct can be present in the axillary region, but it is typically present in a
woman who is breast feeding.
A 25 year-old G1P1 comes to the office due to left breast pain and fever. She is breast feeding her 2 1/2-week old
infant. The symptoms began earlier in the day and are not relieved by acetaminophen. Blood pressure 120/60; pulse
64; temperature 99.9° F, 37.7° C. On exam, she has erythema on the upper outer quadrant of the left breast, which is
tender to touch; there are no palpable masses. In addition to starting oral antibiotics, what is the most appropriate
next step in the management of this patient?
a) Discontinue breastfeeding
b) Add ibuprofen for pain relief
c) Obtain a breast ultrasound
d) Use a topical antifungal
e) Admission to the hospital
Correct!!! Mastitis that accompanies pregnancy or nursing is the most common breast infection. Puerperal
mastitis most commonly occurs during the second to fourth week after delivery. Patients are treated with
oral or IV antibiotics, depending on the severity of infection. Patients may use ibuprofen in addition to
acetaminophen for pain relief, and are encouraged to continue breast feeding or expressing their milk
during treatment. Mastitis is usually treated as an outpatient. Patients may be admitted to the hospital in
severe cases and will be administered intravenous antibiotics. A breast ultrasound is not indicated if there is
no suspicion of a breast abscess.
A 25 year-old G1P1 who is breast feeding her 2 ½ week old comes to the office with left breast pain and fever. The
symptoms began earlier today and are not relieved by acetaminophen. She has no known drug allergies. Blood
pressure 120/60; pulse 64; temperature 99.9° F, 37.7° C. On exam, there is erythema on the upper outer quadrant of
the left breast, which is tender to touch; there are no palpable masses. What is the most appropriate antibiotic
therapy for this patient?
a) Dicloxacillin
b) Erythromycin
c) Doxycycline
d) Gentamicin
e) Cefotetan
Incorrect!!! Most postpartum mastitis is caused by staphylococcus aureus, so a penicillin-type drug is the
first line of treatment. Dicloxacillin is used due to the large prevalence of penicillin resistant staphylococci.
Erythromycin may be used in penicillin-resistant patients.
A 42 year-old G3P3 patient comes to the office after noticing a breast mass while performing a breast self-exam.
She is in good health and has normal menstrual cycles. Physical exam is significant for a 2 cm dominant breast
mass. The remainder of the exam is normal. A mammogram obtained today shows no abnormalities. A fine needle
aspiration was negative, and the mass persisted. What is the most appropriate next step in the management of this
patient?
a) Reassurance and observation
b) Obtain CAT scan of the chest
c) Breast ultrasound
d) Perform an excisional biopsy
e) Repeat mammogram in 2 months
Incorrect!!! A specimen obtained on fine-needle aspiration is examined both histologically and
cytologically. An excisional biopsy should be performed when the results are negative, due to the
possibility of a false-negative result. It can, however, prevent the need for other diagnostic testing and is the
appropriate next step. Breast ultrasound can be used to distinguish between a cyst and a solid mass. Fine
needle aspiration under ultrasound guidance can help distinguish a fibroadenoma from a cyst and exclude
cancer in certain situations. A normal mammogram does not rule out breast cancer and there is no need to
repeat it in 2 months. There are no indications for obtaining a CAT scan of the chest in the initial diagnosis
of this patient.
REI - Exam 48
A 27 year-old G0 woman presents to the clinic because of concerns that she has not been able to get pregnant for the
last 3 months. She married a year ago and was using condoms for contraception, which she stopped 3 months ago
when she decided to start a family. She is in good health and her only medication is a prenatal vitamin. Her periods
are regular every 28 days with normal flow; her last period was 2 weeks ago. She has no history of sexually
transmitted infections and no abnormal Pap smears. Her husband is also healthy with no medical problems. She is
5’4” tall and weighs 130 pounds. Her examination, including a pelvic exam, is completely normal. What is the most
appropriate next step in the management of this patient?
a) Reassurance and observation
b) Order a hysterosalpingogram
c) Order a transvaginal pelvic ultrasound
d) Recommend husband gets a semen analysis
e) Order mid-cycle blood LH and FSH levels
Incorrect!!! Reassurance and observation is most appropriate as the patient has only been trying to conceive
for 3 months. After 1 month, 20% of couples will conceive; after 3 months, 50%; after 6 months, 75%; and
after 12 months, 90% will conceive. Primary infertility is defined as inability to conceive for 1 year without
contraception. The patient is young and healthy, with no obvious reasons for infertility, so at this point,
reassurance and observation is the proper management. There is no reason to order any studies now,
especially since she has normal cycles.
A 27 year-old G0 woman presents to the clinic because of concerns that she has not been able to get pregnant for the
last year. She has been married for two years and was using oral contraceptives, which she stopped a year ago to
start a family. She is in good health and her only medication is a prenatal vitamin. She was hospitalized at age 19 for
a “pelvic infection.” Her periods are regular, every 28 days with normal flow; her last period was 2 weeks ago. She
has no history of abnormal Pap smears. Her husband is also healthy with no medical problems. She is 5’4” tall and
weighs 130 pounds. Her examination, including a pelvic exam, is completely normal. Which of the following is the
most likely diagnostic test to find out the cause of her infertility?
a) Hysteroscopy
b) A hysterosalpingogram
c) Progesterone level mid-cycle
d) Clomiphene citrate challenge test
e) Cervical mucous monitoring
Correct!!! This patient is having difficulty conceiving after trying for one year. Based on her history, the
most likely underlying factor is tubal disease, as she has a history of being hospitalized for a pelvic
infection – most likely pelvic inflammatory disease. This can cause adhesions and blockage of the tubes,
which is best assessed with a hysterosalpingogram to evaluate the uterine cavity and tubes. After a single
episode of salpingitis, 15% of patients experience infertility. A hysteroscopy will only assess the uterine
cavity and no information on tubal patency. Progesterone levels, a Clomiphene challenge test or cervical
mucous monitoring are used at times with infertility workups but, in a young patient of normal BMI and
with normal cycles, it is unlikely to find major ovulatory dysfunction.
A 32 year-old G0 woman comes to the office due to the inability to conceive for last two years. She reports having
been on oral contraceptives for 8 years prior. She had menarche at age 14 and has had irregular cycles about every 3
months until she started oral contraceptives, which regulated her cycles. In the last year, she has had about 5 cycles
in total; her last menstrual period was 6 weeks ago. She is otherwise in good health and has not had any surgeries.
She has no history of abnormal Pap smears or sexually transmitted infections. She is 5’4” tall and weighs 165
pounds. On general appearance, she seems to be hirsute on the face and the abdomen. The rest of her exam is
otherwise normal. Which of the following is most likely to help identify the underlying cause of this woman’s
infertility?
a) Lutenizing hormone levels
b) Testosterone levels
c) Follicle stimulating hormone levels
d) Thyroid function tests
e) Progesterone levels
Incorrect!!! This patient most likely has PCOS (polycystic ovarian syndrome), based on her history of
irregular cycles, her body habitus and hirsutism. Having normal cycles on the birth control pills (OCPs)
supports the diagnosis as other causes, such as hypothyroidism, will not normalize the cycles on OCPs.
Testosterone levels will be helpful to confirm the diagnosis, especially in the presence of hirsutism. Once a
diagnosis is established, progesterone levels are helpful during medical treatment to check if the woman is
ovulating. An increased LH/FSH ratio is observed to be elevated in PCOS patients but each test separately
will not aid in the diagnosis.
A 32 year-old G0 woman comes to the office due to the inability to conceive for the last two years. She reports
having been on oral contraceptives for 8 years prior. She had menarche at age 14 and had irregular cycles about
every 3 months apart until started birth control pills, which made her cycles regular. In the last year, she has had
about 5 cycles in total; her last menstrual period was 6 weeks ago. She is otherwise in good health and has not had
any surgeries. She has no history of abnormal Pap smears or sexually transmitted infections. Her husband of 4 years
is 35 years-old, and has a 10 year-old son from a previous marriage. She is 5’4” tall and weighs 165 pounds. On
general appearance, she is hirsute on the face and the abdomen. The rest of her exam is otherwise normal. In
addition to weight loss and starting Metformin, what is the most appropriate treatment for this patient’s infertility
problem?
a) Laparoscopy and ovarian biopsy
b) Ovulation induction agents
c) Intrauterine insemination
d) In vitro fertilization with ICSI (Intracytoplasmic sperm injection)
e) In vitro fertilization without ICSI
Correct!!! This patient has PCOS (polycystic ovarian syndrome), based on her history, and signs and
symptoms. It is most important for her to try to lose weight. Metformin and ovulation induction agents are
the first-line of treatment for ovulatory dysfunction in PCOS patients. Since there is known ovulatory
dysfunction and there is no reason to believe there are problems with semen analysis, IVF with ICSI is not
justified. Intrauterine insemination will not help with this problem. There is no role for the laparoscopy in
this patient.
A 37 year-old G2P1 woman comes to the clinic with her husband due to the inability to conceive for the last year.
She reports being in good health and not having problems with her prior pregnancy two years ago, except for some
postpartum depression for which she was placed on Imipramine and which she continues to take. She took birth
control pills after her pregnancy and stopped 1 year ago, when she began trying to conceive. Her periods were
regular on the pills, but have been irregular since she stopped taking them. She has no history of sexually transmitted
infections or abnormal Pap smears. Her husband is also healthy and he fathered their first child. Her physical
examination is completely normal. Laboratory tests show:
Results Normal Values
TSH 2.1 mIU/ml 0.5-4.0 mIU/ml
Free T4 1.1 ng/dl 0.8-1.8 ng/dl
Prolactin 60 ng/ml <20 ng/ml
FSH 6 mIU/ml 5-25 mIU/ml
LH 4 mIU/ml 5-25 mIU/ml
What is the most appropriate next step in the management of this patient’s subfertility?
a) Begin Bromocriptine
b) Ovulation induction with Clomiphene Citrate (Clomid)
c) Wean off Imipramine
d) Perform a visual field examination
e) Obtain a brain MRI
Incorrect!!! This patient has hyperprolactinemia due to imipramine. The patient has to be weaned off
imipramine and be placed on a more appropriate medication. Once she is off imipramine and the cause of
her elevated prolactin levels is confirmed, her normal menses should resume. Although MRI of the brain
would be a reasonable step, it would be premature, and visual field examination does not aid in the
diagnostic work-up. It would be premature to obtain an MRI or begin bromocriptine without this
intermediate step. An endometrial biopsy is not indicated at this point, especially since the patient had
normal cycles on OCPs. Although Clomid is used to help with ovulatory dysfunction, the
hyperprolactinemia must be addressed first.
A 27 year-old G0 woman comes to the clinic as she has been unable to conceive for the last year. She is in good
health and has not used any hormonal contraception in the past. She had normal cycles in the past every 28 days
until about 6 months ago. At that time, she began to have irregular menses every 2-3 months, with some spotting in
between. She is not taking any medications. She has no history of abnormal Pap smears or sexually transmitted
infections. Her physical examination is normal. Laboratory tests show:
Results Normal Values
TSH 10 mIU/ml 0.5-4.0 mIU/ml
Free T40.2 ng/dl 0.8-1.8 ng/dl
Prolactin 40 ng/ml <20 ng/ml
FSH 6 mIU/ml 5-25 mIU/ml
LH 4 mIU/ml 5-25 mIU/ml
What is the most appropriate step in the management of this patient?
a) Begin Synthroid
b) Begin bromocriptine
c) Order a Clomid ovulation challenge test
d) Obtain a brain MRI
e) Order a thyroid gland ultrasound
Correct!!! This patient is having abnormal cycles due to hypothyroidism, which is also the most likely
cause of her hyperprolactinemia. The best treatment at this point is to correct the hypothyroidism. It is not
necessary to treat the hyperprolactinemia with bromocriptine or order a brain MRI until the hypothyroidism
is first corrected. There is no reason to obtain a thyroid ultrasound, as the diagnosis of hypothyroidism is
clear from the laboratory results.
A 23 year-old G0 woman comes to the clinic because she is interested in becoming pregnant. She is in good health;
however, she has not had any menses for the last two years. She had menarche at age 15, had normal periods until 3
years ago, when she started having periods irregularly every 3 months and then stopped 2 years ago. She has no
history of pelvic infections or abnormal Pap smears. She exercises every day by running and has run 4 marathons in
the last 3 years. She is 5’10” tall and weighs 115 pounds. Her examination including a pelvic exam is normal.
Laboratory results show:
Results Normal Values
TSH 3.5 mIU/ml 0.5-4.0 mIU/ml
Free T4 0.9 ng/dl 0.8-1.8 ng/dl
Prolactin 10 ng/ml <20 ng/ml
FSH 6 mIU/ml 5-25 mIU/ml
LH 4 mIU/ml 5-25 mIU/ml
BHCG 2 mIU/ml <5 mIU/ml
What is the most appropriate next step in the management of this patient?
a) Check cortisol levels
b) Order a brain MRI
c) Obtain a pelvic ultrasound
d) Check testosterone levels
e) Check estrogen levels
Correct!!! This patient most likely has exercise-induced hypothalamic amenorrhea, which is characterized
by normal FSH and low estrogen levels. The other studies will not help determine the diagnosis. The best
treatment is to encourage the patient to gain weight by decreasing exercise and increasing caloric intake. If
her menses fail to resume, she may be treated with exogenous gonadotropins (LH and FSH) to help her
conceive. Clomiphene citrate tends not to work as well, due to the baseline hypoestrogenic state.
A 45 year-old G3P3 woman comes to the office because she has been unable to conceive for the last two years. She
is healthy and has 3 children ages 10, 12 and 14, whom she conceived with her husband. She used a copper IUD
after the birth of her last child and had it removed two years ago, hoping to have another child. She has no history of
sexually transmitted infections or abnormal Pap smears. Her cycles are regular every 28 to 32 days. She is not taking
any medications. She has been married for the last 16 years, and her husband is 52 years-old and in good health. Her
physical examination, including a pelvic exam, is completely normal. What is the most appropriate next step in the
management of this patient?
a) Perform a hysteroscopy
b) Order a hysterosalpingogram
c) Order clomiphene challenge test
d) Order a semen analysis
e) Basal body temperatures for 6 months
Incorrect!!! This patient, most likely, has decreased ovarian reserve due to her age. A clomiphene challenge
test, which consists of giving clomiphene citrate days 5 to 9 of the menstrual cycle and checking FSH
levels on day 3 and day 10, will help determine ovarian reserve. This will help counsel the patient on
appropriate venues to have a child, as most women will not be able to conceive at this age and would not be
good candidates for ovarian stimulation or IVF. Even though this patient had an IUD in the past, there is no
reason to believe that this contributed to her inability to conceive, as IUDs do not cause infertility and she
has no risk factors for tubal disease. Having her keep basal body temperatures for 6 months would be a
waste of time for this patient, who is already 45. A semen analysis is not necessary as a first step as she was
able to conceive from her husband previously without problems. Most likely this patient will have to use a
donor egg, if she wants to carry the pregnancy herself.
A 28 year-old G0 woman comes to the office for preconception counseling and the inability to conceive for one
year. She and her husband of 3 years are both in good health. She has normal cycles every 28-33 days. She has
intercourse about once a month, depending on her schedule. She is an airline pilot and travels a lot. Her examination
is normal. She asks about when to best have intercourse during her cycle to maximize her chances of pregnancy.
What is the most appropriate advice to give her?
a) Keep basal body temperatures and try to attempt intercourse immediately after the rise in body temperature
b) Best to attempt intercourse after she is done with her menses
c) Use ovulation predictor kits and attempt intercourse after it turns positive
d) Take a leave from her work so she can have intercourse 3 times a week until she gets pregnant
e) Attempt intercourse on day 18 of her cycle
Incorrect!!! Women are most fertile during the middle of their cycle when they are ovulating. Assuming
normal cycles every 28 days, a woman is most likely to ovulate on day 14. Since sperm can live for up to
three days, intercourse up to three days before ovulation can still result in pregnancy. Since this patient has
cycles that vary in length, she can best tell when she is ovulating by using an ovulation predictor kit. The
basal body temperature charts tell when a patient ovulated retrospectively, so it cannot be used to time
intercourse to conceive, as the egg is only viable for about 24 hours. Although having intercourse more
frequently will increase her likelihood of conceiving, it is not a practical solution for a working person to
stop their work in order to conceive.
A 27 year-old G0 woman presents to the clinic because of concerns that she has not been able to get pregnant for the
last year. She has been married for two years ago and was using birth control pills for contraception. She stopped
using birth control pills when she decided to start a family one year ago. She is in good health and her only
medication is a prenatal vitamin. Her periods are regular, every 28 days, with normal flow; her last period was 2
weeks ago. She has no history of sexually transmitted infections and no abnormal Pap smears. Her husband is also
healthy with no medical problems. She is 5’4” tall and weighs 130 pounds. Her examination, including a pelvic
exam, is completely normal. Laboratory results show normal thyroid function tests and normal prolactin level. What
is the most appropriate next step in the management of this patient?
a) Reassurance and observation
b) Perform a pelvic ultrasound
c) Order a hysterosalpingogram
d) Order a semen analysis
e) Recommend a diagnostic laparoscopy
Correct!!! This patient has primary infertility, since she has not been able to conceive for one year. She
does not appear to have underlying pathology to explain why she has not conceived, and her husband’s
semen has not yet been examined. The male factor plays a role in about 35% of infertility cases. A pelvic
ultrasound is unlikely to add any information, as the patient has normal cycles and normal exam. Although
a hysterosalpingogram might be ordered in the future, the male factor needs to be ruled out first, as it is less
invasive to perform. Even though this patient had been on birth control pills previously, this should not be
affecting her fertility a year later. In patients who use OCPs for prolonged periods, there might be a few
months delay in returning to normal fertility.
Gynecology Exam 37
A 76-year-old G3P3 woman presents to your office with worsening urinary incontinence for the past three months.
She reports increase in urinary frequency, urgency and nocturia. Her exam shows mild cystocele and rectocele. A
urine culture is negative. A post-void residual is 400 cc. Which of the following is the most likely diagnosis in this
patient?
a) Genuine stress incontinence
b) Detrusor instability
c) Overflow incontinence
d) Functional incontinence
e) Mixed incontinence
Incorrect!!! Overflow incontinence is characterized by failure to empty the bladder adequately. This is due
to an underactive detrusor muscle (neurologic disorders, diabetes or multiple sclerosis) or obstruction
(postoperative or severe prolapse). A normal post-void residual (PVR) is 50-60 cc. An elevated PVR,
usually >300 cc, is found in overflow incontinence. Stress incontinence occurs when the bladder pressure is
greater than the intraurethral pressure. Overactive detrusor contractions can override the urethral pressure
resulting in urine leakage. The mixed variety includes symptoms related to stress incontinence and urge
incontinence.
A 76-year-old G3P3 presents to the office with worsening stress urinary incontinence for the last 3 months. She
reports an increase in urinary frequency, urgency and nocturia. On exam, she has a moderate size cystocele and
rectocele. A urine culture is negative. A post-void residual is 50 cc. A cystometrogram shows two bladder
contractions while filling. Which of the following is the most likely diagnosis in this patient?
a) Genuine stress incontinence
b) Urge incontinence
c) Overflow incontinence
d) Functional incontinence
e) Continuous incontinence
Incorrect!!! Detrusor overactivity incontinence is also known as urge incontinence. Detrusor instability is
due to the overactivity of the bladder muscle. Though the testing may be simple (using a Foley catheter and
attached large syringe without the plunger, filling with 50-60 cc of water at a time) or complex (using
computers and electronic catheters,) the uninhibited contraction of the bladder with filling makes the
diagnosis.
A 60-year-old G4P4 woman presents with a two-year history of leakage with activity such as coughing, sneezing
and lifting. Her past medical history is significant for vaginal deliveries of infants over 9 pounds. She had a previous
abdominal hysterectomy and bilateral salpingo-oophorectomy for uterine fibroids. She is on vaginal estrogen for
atrophic vaginitis. Physical exam shows no anterior, apical or posterior wall vaginal prolapse. Vagina is wellestrogenized. Postvoid residual was normal. Q-tip test showed a straining angle of 60 degrees from the horizon.
Cough stress test showed leakage of urine synchronous with the cough. Cystometrogram revealed the absence of
detrusor instability. The patient failed pelvic muscle exercises and was not interested in an incontinence pessary.
Which of the following is the best surgical option for this patient?
a) Retropubic urethropexy
b) Needle suspension
c) Anterior repair
d) Urethral bulking procedure
e) Colpocleisis
Incorrect!!! Genuine stress incontinence (GSI) is the loss of urine due to increased abdominal pressure in
the absence of a detrusor contraction. The majority of GSI is due to urethral hypermobility (straining Q-tip
angle >30 degrees from horizon.) Some (<10%) of GSI is due to intrinsic sphincteric deficiency (ISD) of
the urethra. Patients can have both hypermobility and ISD. Retropubic urethropexies (Burch and Marshall
Marchetti) and sling procedures have the best 5-year success rates for patients with GSI due to
hypermobility. Needle suspensions and anterior repairs have lower 5-year success rates for GSI. Urethral
bulking procedures are best for patients with ISD, but with little to no mobility of the urethra. Colpocleisis
is one option to treat uterine prolapse, and is not indicated for urinary incontinence.
A 70-year-old G3P3 woman presents with a 4-year history of constant leakage. Her history is significant for
abdominal hysterectomy and bilateral salpingo-oophorectomy for endometriosis. She has had four anterior repairs in
the past for recurrent cystocele. The leakage started six months after her last anterior repair. Pelvic exam showed no
evidence of pelvic relaxation. The vagina was well-estrogenized. Q-tip test revealed a fixed, immobile urethra.
Cystometrogram showed no evidence of detrusor instability. Cystourethroscopy showed no evidence of any fistula
and revealed a “drain pipe” urethra. Which of the following is the best first treatment for this patient?
a) Retropubic urethropexy
b) Needle suspension
c) Artificial urethral sphincter
d) Urethral bulking procedure
e) Sling procedure
Incorrect!!! This is a classic example of intrinsic sphincteric deficiency. The success rates for retropubic
urethropexies, needle suspension and slings are less than 50%. An “obstructive or tight” sling can be
performed to increase the success rate, but the voiding difficulties are significant, even requiring prolonged
or lifelong self-catheterization. Urethral bulking procedures are minimally invasive and have a success rate
of 80% in these specific patients. Artificial sphincters should be used in patients as a last resort.
A 60-year-old woman presents with complaints of urinary frequency and urge incontinence. Past medical history is
unremarkable. She is on no medications. Pelvic exam shows no evidence of pelvic relaxation. Post void residual is
normal. Urine analysis is negative. A cystometrogram revealed uninhibited detrusor contractions upon filling.
Which of the following is the best non-surgical treatment for this patient?
a) Amitriptyline
b) Oxybutynin
c) Topical (vaginal) estrogen
d) Pseudoephedrine
e) Kegel exercises
Correct!!! The patient has the diagnosis of detrusor instability. The parasympathetic system is involved in
bladder emptying and acetylcholine is the transmitter that stimulates the bladder to contract through
muscarinic receptors. Thus, anticholinergics are the mainstay of pharmacologic treatment. Oxybutynin is
one example. Although the tricyclic antidepressant, amitriptyline, has anticholinergic properties, its side
effects do not make it an ideal choice. Vaginal estrogen has been shown to help with urgency, but not urge
incontinence. Pseudoephedrine has been shown to have alpha adrenergic properties and may improve
urethral tone in the treatment of stress incontinence. Kegel exercises or pelvic muscle training are used to
strengthen the pelvic floor and decrease urethral hypermobility for the treatment of stress urinary
incontinence.
A 67-year-old G3P3 woman presents to the office due to severe pelvic protrusion status post hysterectomy. She
denies any incontinence. She failed conservative management with a pessary. She underwent a vaginal surgical
repair where the pubocervical fascia was plicated in the midline as well as laterally to the arcus tendineus fascia
(white line.) This patient underwent the repair of what defect?
a) Cystocele
b) Rectocele
c) Uterine prolapse
d) Enterocele
e) Urethral diverticulum
Incorrect!!! Central and lateral cystoceles are repaired by fixing defects in the pubocervical fascia or
reattaching it to the sidewall, if separated from the white line. Defects in the rectovaginal fascia are repaired
in rectoceles. Uterine prolapse is surgically treated by a vaginal hysterectomy, but this patient already had a
hysterectomy. Enteroceles are repaired by either vaginal or abdominal enterocele repairs. Vaginal vault
prolapse is treated either by supporting the vaginal cuff to the uterosacral ligaments, sacrospinous ligament
or sacrocolpopexy. Urethral diverticulum does not present with severe pelvic protrusion.
A 57-year-old G2P2 woman presents to the office with a six-month history of leaking urine, urgency, and nocturia.
She describes the amount of urine loss as large and lasting for several seconds. The urine loss occurs when she is
standing or sitting and is not associated with any specific activity. What is the most likely cause of this patient’s
symptoms?
a) Stress incontinence
b) Overflow incontinence
c) Urge incontinence
d) Mixed incontinence
e) Vesicovaginal fistula
Incorrect!!! This patient has urge incontinence, which is caused by overactivity of the detrusor muscle
resulting in uninhibited contractions, which cause an increase in the bladder pressure over urethral pressure
resulting in urine leakage. Stress incontinence is caused by an increase in intra-abdominal pressure
(coughing, sneezing) when the patient is in the upright position. This increase in pressure is transmitted to
the bladder that then rises above the intra-urethral pressure causing urine loss. Associated structural defects
are cystocele or urethrocele. Overflow incontinence is associated with symptoms of pressure, fullness, and
frequency, and is usually a small amount of continuous leaking. It is not associated with any positional
changes or associated events. Mixed incontinence occurs when increased intra-abdominal pressure causes
the urethral-vesical junction to descend causing the detrusor muscle to contract. A vesicovaginal fistula
typically results in continuous loss of urine.
A 56-year-old G3P3 woman presents to the office for her annual health maintenance exam. She is in good health
and is not taking any medications. She has been postmenopausal for 3 years. She had an abnormal Pap smear 10
years ago, but results have been normal every year since. She is sexually active with her husband and uses lubricant
during intercourse due to mild vaginal dryness. On examination, her cervix was 1 cm above the vaginal introitus and
she was noted to have a moderate bladder prolapse. Her uterus is normal in size and she has no adnexal masses, and
she was non-tender. In addition to performing a Pap smear and recommending a mammogram, what is the most
appropriate next step in the management of this patient?
a) Cystocele repair
b) Perform a pelvic ultrasound
c) Total hysterectomy
d) Observation
e) Topical estrogen
Correct!!! This patient is asymptomatic from her prolapse; therefore, no intervention is necessary at this
point. Cystocele repairs and hysterectomies are invasive procedures which are not indicated in this
asymptomatic patient. It is not necessary to obtain a pelvic ultrasound, as her uterus is normal in size and
she has no adnexal masses. Topical estrogen would not help improve the prolapse, although it might help
with her vaginal dryness. She seems to be doing well with the lubricants and it is not necessary to expose
her to the estrogen, especially since she still has her uterus, and estrogen treatment alone may increase her
risk of endometrial cancer.
A 90-year-old G7P7 woman presents with severe vaginal prolapse. The entire apex, anterior and posterior wall are
prolapsed beyond the introitus. She cannot urinate without reduction of the prolapse. Hydronephrosis was noted on
ultrasound of the kidneys and it is thought to be related to the prolapse. She has a long-standing history of diabetes
and cardiac disease. She is not a candidate for general or regional anesthesia. She has failed a trial of pessaries.
Which of the following is the next best step in the management of this patient?
a) Do nothing and observe
b) Anterior and posterior repair
c) Colpocleisis
d) Sacrospinous fixation
e) Sacrocolpopexy
Incorrect!!! Because of the hydronephrosis due to obstruction, intervention is required. Anterior and
posterior repairs provide no apical support of the vagina. She will be at high risk of recurrent prolapse. The
sacrospinous fixation (cuff to sacrospinous-coccygeus complex) or sacrocolpopexy (cuff to sacral
promontory using interposed mesh) require regional or general anesthesia. Colpocleisis is a procedure
where the vagina is surgically obliterated and can be performed under local anesthesia. Recurrence is
minimal.
A 65-year-old G3P3 woman presents to the office with symptoms of vaginal pressure and heaviness, which seems to
worsen towards the end of the day. She has a history of three vaginal deliveries. Her surgical history is significant
for history of hysterectomy for abnormal vaginal bleeding at age 45. On exam, she is found to have a large pelvic
prolapse. Which of the following is the most appropriate first step for the treatment of this patient’s prolapse?
a) Sacrospinous ligament suspension
b) Transvaginal tape
c) Pessary fitting
d) Anterior repair
e) Topical vaginal estrogen
Correct!!! Pessary fitting is the least invasive intervention for this patient’s symptomatic prolapse.
Although a sacrospinous ligament suspension would be an appropriate procedure for this patient, it is
invasive and not an appropriate first step. Transvaginal tape is used for urinary incontinence and has no role
in the management of this patient. An anterior repair can potentially help with her symptoms, depending on
what is contributing most to her prolapse but, again, it is invasive. Topical estrogen is unlikely to properly
treat her prolapse and related symptoms.
Gynecology - Exam 47
A 53 year-old G2P2 woman comes to your office complaining of 6 months of worsening hot flashes, vaginal
dryness, night sweats and sleep disturbances. Her last normal menstrual period was 6 months ago and she has been
experiencing intermittent small amounts of vaginal bleeding. Her medical history is significant for hypertension,
which is well-controlled by a calcium-channel blocker, adult onset diabetes, for which she takes Metformin, and
hyperthyroidism, for which she takes Propylthiouracil. The patient is 5’7” tall and weighs 140 pounds. Blood
pressure is 120/70. Physical examination is unremarkable. Which of the following medical conditions in this patient
is a contraindication to treatment of menopausal symptoms with hormone therapy?
a) Vaginal bleeding
b) Hypertension
c) Diabetes
d) Osteoporosis
e) Hyperthyroidism
Correct!!! The principal symptom of endometrial cancer is abnormal vaginal bleeding. Although the
patient’s worsening symptoms makes treatment an important consideration, the specific organic cause(s) of
abnormal bleeding must be ruled out prior to initiating therapy. A tissue diagnosis consistent with normal
endometrium or a pelvic ultrasound with an endometrial stripe of <4 mm ought to be documented. In
addition, risks and benefits of hormone replacement therapy must be discussed with this patient at length
prior to beginning treatment.
A 47 year-old G2P2 woman comes to your office because she has skipped her menstrual period for the past 3
months. She denies any menopausal symptoms. Review of symptoms and physical exam are unremarkable.
Quantitative BHCG <5 mIU/ml, TSH= 1.2 mIU/L (normal). What is the most likely diagnosis in this patient?
a) Hypothyroidism
b) Early pregnancy
c) Perimenopause
d) Premature ovarian failure
e) Autoimmune disorder
Correct!!! Although there has been a decline in the average age of menarche with the improvement in
health and living conditions, the average age of menopause has remained stable. The Massachusetts
Women’s Health Study reports that the average age of menopause is 51.3. This patient is most likely
perimenopausal and will probably have more menstrual periods in the future. She has normal BHCG and
TSH. Premature ovarian failure occurs before age 35. Her normal TSH rules out hypothyroidism and she
lacks signs and symptoms of other autoimmune disorders.
A 58 year-old G3P3 woman has been postmenopausal for 5 years and is concerned about osteoporosis. She has
declined hormone therapy in the past. Her mother has a history of a hip fracture at age 82. A physical exam is
unremarkable. In addition to weight bearing exercise and vitamin D supplementation, what optimal daily calcium
intake should she take?
a) 500 mg
b) 750 mg
c) 1000 mg
d) 1200 mg
e) 2000 mg
Incorrect!!! Calcium absorption decreases with age because of a decrease in biologically active vitamin D.
A positive calcium balance is necessary to prevent osteoporosis. Calcium supplementation reduces bone
loss and decreases fractures in individuals with low dietary intakes. In order to remain in zero calcium
balance, postmenopausal women require a total of 1200 to 1500 mg of elemental calcium per day.
A 58 year-old G3P1 woman presents to your office for her annual exam. She became menopausal at age 54. Her
past medical history is significant for angina. She experienced a Colles’ fracture 14 months ago when she tripped
and fell while running after her grandson. She has not had any surgeries. She takes no medications and has no
known drug allergies. She smokes 10 cigarettes a day and drinks a glass of red wine at dinner. Her father was
diagnosed with colon cancer at the age of 72. Physical exam revealed a BP =120/68, P=64, BMI= 22. Her heart,
lung, breast and abdominal exams were normal. Pelvic exam was consistent with vaginal atrophy and a small uterus.
There was no adnexal tenderness and no masses were palpated. What is the next step in the management plan for
this patient?
a) Begin screening for osteoporosis at age 65
b) DEXA scan now and treat if T score is < -1.5
c) Begin hormone replacement therapy
d) Begin treatment with bisphosphonates
e) Test for the presence of biochemical bone markers in the blood
Correct!!! This patient has many of the major risk factors for osteoporosis including history of fracture as
an adult, low body weight and being a current smoker. General recommendations for the prevention of
osteoporosis include eating a balanced diet that includes adequate intake of calcium and vitamin D, regular
physical activity, avoidance of heavy alcohol consumption, and smoking cessation. DEXA is the test of
choice for measuring bone mineral density (BMD). Bone markers are used in research but are not yet a
reliable predictor of BMD. Patients who already have had an osteoporotic fracture may be treated on this
basis alone, although a DEXA scan may be useful for other reasons, such as ruling out a pathologic fracture
from metastatic disease. Hormone replacement therapy is not recommended long term for disease
prevention especially in patients with cardiovascular disease. Prior to beginning treatment with
bisphosphonates, a BMD should be documented and repeated at two -year intervals to monitor treatment.
A 54 year-old G2P2 presents to your office for an annual visit. Her last menstrual period was 8 months ago. She
complains of severe vasomotor symptoms, vaginal dryness, and dyspareunia and she desires treatment for her
symptoms. She has otherwise been in good health. She has no significant past medical or past surgical history. Her
family history is significant for a mother who has severe osteoporosis at the age of 75 and a grandmother who died
of breast cancer at the age of 79. She does not report any smoking, alcohol or drug use. On physical exam her BP is
130/78, Pulse 84, BMI is 26. The remainder of her exam is within normal limits except for severe vaginal atrophy
noted on the pelvic examination. The best recommendation for this patient would include which of the following?
a) Lowest effective dose of combination hormone replacement therapy for the shortest duration possible
b) Long term hormone replacement therapy to treat her vasomotor symptoms and prevention of osteoporosis
c) Testosterone cream
d) Progesterone cream
e) Biosphophonates
Correct!!! The ACOG 2004 report on hormone replacement therapy considers hormone replacement
therapy (HRT) the most effective treatment for severe menopausal symptoms that include hot flashes, night
sweats and vaginal dryness. The physician should counsel the woman about the risks and benefits before
initiating treatment. ACOG recommends “the smallest effective dose for the shortest possible time and
annual reviews of the decision to take hormones.” HRT should not be used to prevent cardiovascular
disease due to the slight increase in risk of breast cancer, myocardial infarction, cerebrovascular accident,
and thromoboembolic events. A woman with an intact uterus should not use estrogen-only therapy because
of the increased risk of endometrial cancer. In addition to the same risks as FDA approved treatments,
bioidentical hormones such as testosterone and progesterone cream may have additional associated risks.
Bisphosphonates are used to treat osteoporosis and will not relieve her symptoms.
A 52 year-old G3P3 woman presents to your office with severe hot flashes and vaginal dryness for 6 months. Her
last menstrual period was 15 months ago. After discussing the risks and benefits of hormone therapy with this
patient, she decides to begin treatment. This patient is most likely to stop hormone therapy secondary to what side
effect?
a) Vaginal bleeding
b) Development of breast cancer
c) Hirsutism
d) Nausea
e) Relief of menopausal symptoms
Incorrect!!! Most irregular bleeding due to initiation of hormone therapy occurs in the first 6 months. It has
been cited as the most common reason as to why women stop hormone therapy. Women who are
amenorrheic for some period of time are often disturbed by the resumption of any vaginal bleeding/spotting
and find it intolerable.
49 year-old G2P2 woman s/p hysterectomy at age 45 for fibroids presents to your office complaining of severe
vasomotor symptoms for 3 months. Hot flashes are affecting her quality of life and she would like to discuss options
for treatment. What treatment option for hot flushes associated with menopause do you recommend as the most
effective?
a) Lifestyle modifications such as dressing in layers
b) Estrogen
c) Selective estrogen receptor modulator (SERMs)
d) Selective serotonin reuptake inhibitors (SSRIs)
e) Treatment with Phytoestrogen (soy)
Incorrect!!! Except for estrogen receptor modulator therapy, all of the above treatment options will improve
hot flash symptoms. Treatment with estrogen is most effective, and the current recommendation is for the
lowest dose for the shortest duration of time. Hot flashes will resolve completely in 90% of patients
receiving this therapy. Raloxifene, a selective estrogen receptor modulator, may actually cause hot flashes
to worsen in a patient who has not stopped having these symptoms completely. SSRI antidepressants, some
anti-seizure medications and alternative treatments, such as soy products and herbs, have not been shown to
be as effective as estrogen.
A 49 year-old G1P1 woman comes to your office for menopause counseling. She has been experiencing severe sleep
disturbances and night sweats for the past 4 months. She would like to begin hormone therapy, but is concerned
because she has elevated cholesterol levels for which she takes medication. You explain to her that hormone therapy
has the following effect on a lipid/cholesterol profile:
a) Both LDL and HDL levels increase
b) Both HDL and LDL levels decrease
c) HDL and LDL levels are unaffected
d) HDL levels increase and LDL levels are unaffected
e) HDL levels increase and LDL levels decrease
Correct!!! Recent data have confirmed the overall positive effects of hormone therapy on serum lipid
profiles. The most important lipid effects of postmenopausal hormone treatment are the reduction in LDL
cholesterol and the increase in HDL cholesterol. Estrogen increases triglycerides and increases LDL
catabolism, as well as lipoprotein receptor numbers and activity, therefore causing decreased LDL levels.
Hormones inhibit hepatic lipase activity, which prevents conversion of HDL2 to HDL3, thus increasing
HDL levels. Hormone therapy is not currently recommended for the primary prevention of heart disease.
A 59 year-old G0 woman comes to your office for an annual exam. The patient has been postmenopausal for 5
years. She is complaining of vaginal dryness and pain with intercourse for the past 3 years. If you were to check,
what would be this patient’s circulating estradiol level?
a) 10-20 pg/ml
b) 50-60 pg/ml
c) 100-150 pg/ml
d) 200-250 pg/ml
e) 400-450 pg/ml
Incorrect!!! The circulating estradiol level after menopause is approximately 10-20 pg/ml. This is derived
from the peripheral conversion of estrone, which is mainly a result of the peripheral conversion of
androstenedione.
A 54 year-old G4P4 woman who has been menopausal for 4 years comes to you for an annual exam. She is in good
health, eats a balanced diet, exercises regularly, and has never had any menopausal symptoms and wants to know
why. You explain to her that some untreated postmenopausal women will have circulating estrogen levels that are
adequate to prevent them from experiencing the symptoms of menopause. What is the most likely source of these
circulating estrogens?
a) Exogenous dietary intake
b) Limited adrenal estrogen secretion
c) Lingering ovarian estrogen
d) Aromatization of circulating androgens
e) Continued low-level ovarian production
Correct!!! Estrogen production by the ovaries does not continue beyond the menopause. Estrogen levels in
postmenopausal women can be significant, due to the extraglandular conversion of androstenedione and
testosterone to estrogen. This conversion occurs in peripheral fat cells and thus, body weight has been
directly correlated with circulating levels of estrone and estradiol.
REI - Exam 34
A 23-year-old woman with 6 weeks amenorrhea presents with lower abdominal pain and vaginal bleeding. Her
-week size and tender. There are no adnexal masses.
Urine pregnancy test is positive. What is the most likely diagnosis?
a) Threatened abortion
b) Missed abortion
c) Normal pregnancy
d) Septic abortion
e) Ectopic Pregnancy
Incorrect!!! The patient has a septic abortion. She has fever, bleeding, cervix is open and exam findings
consistent with septic abortion. Threatened abortions clinically have vaginal bleeding, positive pregnancy
test and a cervical os closed or uneffaced, while missed abortions have retention of a nonviable intrauterine
pregnancy for an extended period of time (i.e. dead fetus or blighted ovum.) A normal pregnancy would
have a closed cervix. Ectopic pregnancy would likely present with bleeding, abdominal pain and possibly
have an adnexal mass; the cervix would typically be closed.
A 23-year-old woman with 6 weeks amenorrhea presents with lower abdominal pain and vaginal bleeding. Her
-week size, tender and there are no adnexal masses.
Urine pregnancy test is positive. Which of the following is the most appropriate next step in the management of this
patient?
a) Single-agent antibiotics
b) Observation
c) Laparoscopy plus antibiotics
d) Uterine evacuation plus antibiotics
e) Medical termination of pregnancy plus antibiotics
Correct!!! The management of septic abortion is broad spectrum antibiotics and uterine evacuation. Single
agent antimicrobials do not provide coverage for the array of organisms that may be involved and therefore
are not indicated. A laparoscopy can be indicated if ectopic pregnancy is suspected, but it is unlikely in this
case. Medical termination is not the best option since prompt evacuation of the uterus is indicated in this
case.
A 29-year-old G3P0 woman presents for evaluation and treatment of pregnancy loss. Her past medical history is
remarkable for 3 early (<14 weeks gestation) pregnancy losses and a deep vein thrombosis 2 years ago. Parental
karyotype was normal. Which of the following is the most appropriate next step in the management of this patient?
a) Place a prophylactic cerclage
b) Obtain cervical length
c) Refer for in vitro fertilization
d) Obtain a semen analysis
e) Check antiphospholipid antibodies
Correct!!! Antiphosphospholipid antibodies are associated with recurrent pregnancy loss. The workup for
antiphospholipid syndrome includes assessment of anticardiolipin antibody status, PTT, and Russell viper
venom time. There are multiple etiologies for recurrent pregnancy loss defined as >2 consecutive or >3
spontaneous losses before 20 weeks gestation. Etiologies include anatomic causes (uterine abnormalities
either acquired or inherited,) endocrine abnormalities such as hyper or hypothyroidism and luteal phase
deficiency, parental chromosomal anomalies, immune factors such as lupus anticoagulant and idiopathic
factors. Cervical incompetence is diagnosed by history, physical exam and other diagnostic tests, such as
ultrasound. The treatment is placement of a cerclage. Semen analysis would not be useful in this workup.
A 29-year-old G3P0 woman presents for evaluation and treatment of pregnancy loss. Her past medical history is
remarkable for 3 early (<16 weeks gestation) pregnancy losses and a deep vein thrombosis 2 years ago. Her work up
includes: MRI of pelvis normal, prolonged dilute Russell viper venom test and elevated anticardiolipin antibodies,
normal thyroid function and prolactin. She wishes to get pregnant soon. In addition to aspirin, which of the
following treatments is appropriate for this patient?
a) No additional treatment
b) Corticosteroid
c) Heparin
d) Levothroxine
e) Bromocriptine
Incorrect!!! The prolonged dilute Russell viper venom time leads one to suspect that the etiology of
recurrent pregnancy loss is due to antiphospholipid antibody syndrome. The treatment is aspirin plus
heparin. There is roughly a 75% success rate with combinations therapy versus aspirin alone. There is
conflicting evidence regarding steroid use for treatment and heparin is more effective.
A 25-year-old G1P0 woman at 6 weeks gestational age comes to the office because of undesired pregnancy. You
discuss with her the risks and benefits of surgical versus medical abortion using misoprostol and mifepristone.
Compared to surgical abortion, which of the following is increased in a woman undergoing a medical abortion?
a) Post abortion pain
b) Lower failure rate
c) Long-term psychologic sequelae
d) Blood loss
e) Future fertility
Incorrect!!! In early in pregnancy (less than 49 days) both medical and surgical procedures can be offered.
Mifepristone (an antiprogestin) can be administered, followed by misoprostol (a prostaglandin) to induce
uterine contractions to expel the products of conception. This approach has proven to be effective (96%)
and safe. A surgical termination is required in the event of failure or excessive blood loss. It seems to be
more desirable by some patients as they do not have to undergo a surgical procedure; however, it is
associated with higher blood loss than surgical abortion. It does not affect future fertility. Any termination
of pregnancy, whether medical or surgical, can have psychological sequelae.
A 36-year-old G2P0 woman at 11 weeks gestational age requests a surgical termination of pregnancy. She had a
manual vacuum aspiration last year and would like to undergo the same procedure again. She has chronic
hypertension and diabetes well controlled on medications. Vital signs reveal a blood pressure of 120/80 and fasting
blood glucose of 100. Which of the following is a contraindication for manual vacuum aspiration of this patient?
a) Age
b) Parity
c) Gestational age
d) Chronic hypertension
e) Diabetes
Correct!!! Manual vacuum aspiration is more than 99% effective in early pregnancy (less than 8 weeks.)
Age, parity and medical illnesses are not contraindications for manual vacuum aspiration. Although the risk
of Asherman’s syndrome increases with each subsequent pregnancy termination, this patient may still
undergo surgical termination as long as she understands risks and benefits. Complications of pregnancy
termination increase with increasing gestational age.
A 25-year-old G1 woman at 20 weeks gestation desires termination of the pregnancy. Her prenatal course has been
unremarkable except for a chromosomal analysis positive for Trisomy 18. She desires an autopsy of the fetus.
Which of the following is the most appropriate next step in the management of this patient?
a) Perform a dilatation and curettage
b) Perform a dilatation and evacuation
c) Await fetal demise then start induction of labor
d) Perform an induction with intraamniotic hypertonic saline
e) Perform an induction with intravaginal prostaglandins
Correct!!! Both medical and surgical abortions are options for this patient, depending on her personal
preferences. However, if she desires an autopsy, she must undergo a medical abortion in order to have an
intact fetus. Abortion is legal until viability is achieved (24 weeks gestation) unless a fetal anomaly
inconsistent with extrauterine life is identified. A dilatation and curettage is performed if the fetus is less
than 16 weeks, while dilatation and evacuation can be performed after 16 weeks by those trained in the
procedure. Inductions with hypertonic saline have a high morbidity, so are no longer performed.
A 23-year-old G1P0 woman at 6 weeks gestation undergoes a medical termination of pregnancy. One day later, she
presents to the emergency room with bleeding and soaking more than a pad per hour for the last 5 hours. Her blood
pressure on arrival is 110/60; heart rate 86. On exam, her cervix is 1cm dilated with active bleeding. Hematocrit on
arrival is 29%. Which of the following is the most appropriate next step in the management of this patient?
a) Admit for observation
b) Repeat hematocrit in 6 hours
c) Begin transfusion with O-negative blood
d) Give an additional dose of prostaglandins
e) Prepare her for a dilatation and curettage
Incorrect!!! This patient is having heavy bleeding as a complication of medical termination of pregnancy.
This is managed best by performing a dilatation and curettage. It is not appropriate to wait 6 hours before
making a decision regarding next step in management, or to just admit her for observation. Since the patient
is not symptomatic from her anemia, it is not necessary to transfuse her at this time.
A 22-year-old G1P0 woman with LMP 6 weeks ago presents for elective termination of her pregnancy. She is
healthy with no medical problems. Her pregnancy thus far has been uncomplicated. An ultrasound performed in the
office shows an 8mm endometrial stripe with no intrauterine gestational sac and no adnexal masses. Which of the
following is the most appropriate next step in the management of this patient?
a) Obtain a beta-hCG level
b) Perform dilation and curettage
c) Give patient methotrexate
d) Perform a diagnostic laparoscopy
e) Repeat ultrasound in 2 weeks
Incorrect!!! Even though the patient reports being pregnant, she is asymptomatic with no gestational sac in
the uterus. First step in her management is to establish pregnancy by obtaining a beta-hCG level. One
should not assume she has an intrauterine pregnancy and perform a D&C or assume that she has an ectopic
pregnancy and treat her with methotrexate or surgery until the pregnancy is confirmed.
A 24-year-old G2P1 woman who underwent an elective termination two days ago presents to the emergency room
with abdominal and pelvic pain. She has been feeling nauseated and reports a fever at home to 101.3°F. On
presentation, her blood pressure is 100/60; pulse 100; respiration 16; temperature 102°F. Physical examination
reveals diffuse abdominal tenderness and, on pelvic exam, she has marked cervical motion tenderness. In addition to
sending a CBC and cultures, which of the following is the most appropriate next step in the management of this
patient?
a) Obtain beta-hCG level
b) Proceed with a laparoscopy
c) Proceed with a dilatation and curettage
d) Begin IV antibiotics
e) Order a hysterosonogram
Incorrect!!! This patient has postoperative endometritis that could be due to introduction of bacteria into the
uterine cavity at the time of dilatation and curettage. It is important to begin antibiotics immediately. After
starting antibiotics, an ultrasound should be obtained to look for products of conception. If found, then the
patient would require a repeat dilatation and curettage. An hCG level would not be helpful 2 days after the
termination. Hysterosonogram is contraindicated when infection is present. There are no indications for
laparoscopy in this patient.
REI - Exam 44
A 35 year-old Asian woman presents with irregular menses and hirsutism of 3 months duration. The patient has no
family history of hirsutism. On exam, the patient was noted to have terminal hair growth on her chest and recently
had laser treatment to remove similar hair on her chin. Her total testosterone is 76 ng/dl (normal) and her DHEAS is
1500µg/dl (elevated). Which of the following is the most likely diagnosis in this patient?
a) Pituitary adenoma
b) Ovarian tumor
c) Cushing’s syndrome
d) Adrenal tumor
e) Idiopathic
Incorrect!!! The short duration of symptoms and the significantly elevated DHEAS support the diagnosis of
an adrenal tumor as the etiology of the patient’s symptoms. In addition, the patient is Asian and is less
likely to have a predisposition to idiopathic hirsutism.
An 18 year-old G0 woman presents with a 1-year history of hirsutism and acne. She had menarche at age 14 and her
menses have been irregular every 26-60 days. Her sister has a similar pattern of hair growth. The patient is 5’4” tall
and weighs 180 pounds. On exam, a few terminal hairs were identified on her chin and upper lip. TSH, prolactin,
total testosterone, and DHEAS levels are normal. Which of the following is the most appropriate next test to
evaluate this patient’s condition?
a) Estradiol levels
b) Serum cortisol levels
c) Urinary cortisol levels
d) Random blood glucose
e) 17-hydroxyprogesterone
Incorrect!!! Checking 17-hydroxyprogesterone would rule out late onset 21-hydroxylase deficiency.
Normal TSH, Prolactin, total testosterone and DHEAS levels rule out pituitary or adrenal tumors. The
patient could have polycystic ovarian syndrome; however, normal serum testosterone levels make it less
likely.
A 22 year-old G0 woman presents with hirsutism, which has been present since menarche. She states that she has
laser treatments done to remove the hair on her chin every couple of months, and was wondering if there are
additional treatments which might help her. She is otherwise in good health. She has normal menstrual cycles every
28 days. She is sexually active and uses birth control pills for contraception. The patient is adopted and has no
information about family history. She is 5’4” tall and weighs 125 pounds. On examination, the patient was noted to
have terminal hair growth on her chest. Her TSH, Prolactin, total testosterone, DHEAS, 17-Hydorxyprogesterone
levels are normal. Which of the following is the most likely underlying etiology for the hirsutism in this patient?
a) Polycystic ovarian syndrome
b) Side effects of the oral contraceptives
c) Cushing’s syndrome
d) Adrenal tumor
e) Idiopathic
Correct!!! This patient most likely has idiopathic hirsutism. She has no other clinical signs of polycystic
ovaries, such as irregular cycles or obesity. Normal laboratory values rule out other pathogenic causes of
hirsutism, such as Cushing’s syndrome or adrenal tumor. Oral contraceptives are actually used for the
treatment of hirsutism.
A 17 year-old patient presents with hirsutism, irregular menses and obesity. Her mother is moderately obese with
mild hirsutism. Recently the patient’s hirsutism has worsened and she has been depressed. She has also gained 20
pounds in the past two months and has noticed stretch marks on her abdomen. At the time of your examination, you
note that she has terminal hair growth on her chin and hair growth on the back of her hands. Her cheeks appear
flushed. Her stretch marks are purplish in color. The rest of her exam is normal. Which of the following is the most
appropriate first test to order for this patient?
a) Overnight dexamethasone suppression test
b) 17-hydroxyprogesterone
c) Fasting insulin
d) TSH
e) Pelvic ultrasound
Correct!!! Since Cushing's syndrome is suspected, either a dexamethasone suppression test or a 24-hour
urinary measurement for cortisol can be performed. Elevated cortisol would be indicative of Cushing's
syndrome. The other tests listed would be reasonable, but only after Cushing’s syndrome had been
excluded.
A 26 year-old woman presents with hirsutism and irregular menses. Her mother, who is diabetic, had similar
complaints prior to menopause. On physical exam, this patient is noted to have terminal hair on her chin and a graybrown velvety discoloration on the back of her neck. This lesion is acanthosis nigricans. Which of the following is
the most appropriate first test to order for this patient?
a) Fasting insulin
b) TSH
c) 17-hydroxyprogesterone level
d) Cortisol level
e) Pelvic ultrasound
Correct!!! Acanthosis nigricans is associated with elevated androgen levels and hyperinsulinemia. Since
this woman has a family history of diabetes and also has acanthosis nigricans, the most appropriate test of
those listed would be the fasting insulin. The other tests would also be reasonable, but hyperinsulinemia is
most likely in this patient.
A 36 year-old woman comes to the office due to hair loss. She delivered a healthy infant girl three months ago. She
is currently on a progestin-only oral contraceptive pill since she is breast feeding. In the last month, she has noticed
a large amount of hair on her brush each morning. Her father has male pattern baldness and her mother, who is
postmenopausal, has also had some thinning of her hair, as well. Testosterone and TSH levels are within the normal
range. Which of the following is the most likely underlying cause for alopecia in this patient?
a) Genetic predisposition
b) Progesterone only pills
c) High estrogen levels during pregnancy
d) Stress during pregnancy and delivery
e) Breastfeeding
Incorrect!!! High estrogen levels in pregnancy increase the synchrony of hair growth. Therefore, hair grows
in the same phase and is shed at the same time. Occasionally, this can result in significant postpartum hair
loss. In the non-pregnant state, asynchronous hair growth occurs such that a portion of hair is in one of the
three hair growth cycles at all times.
A 34 year-old G2P2 presents with concerns of hormonal changes. She is worried about facial hair growth,
worsening acne, and deepening of her voice. She also realized that she has missed her period for 2 months, and has
been sexually active and had tubal ligation for contraception. On examination, she is moderately obese and noted to
have severe acne, upper lip and chin terminal hair. Her abdomen is obese with moderate hair growth. Pelvic
examination is most notable for an enlarged clitoris, and pelvic exam reveals an enlarged right sided adnexal mass.
Which of the following is the most likely diagnosis in this patient?
a) Sertoli-Leydig cell tumors
b) Granulosa cell tumor
c) Benign cystic teratoma
d) Choriocarcinoma
e) Cystadenoma
Correct!!! The most likely diagnosis in this patient is a testosterone-secreting ovarian tumor. Sertoli-Leydig
cell tumors are commonly diagnosed in women between the ages of 20-40, and are most often unilateral.
Rapid onset of hirsutism and virilizing signs are hallmarks of this disease, and include many of the findings
in this patient including acne, hirsutism, amenorrhea, clitoral hypertrophy, and deepening of the voice.
Abnormal laboratory findings include suppression of FSH and LH, marked elevation of testosterone, and
presence of an ovarian mass. The constellation of findings is most consistent with a testosterone-secreting
tumor, and a pelvic ultrasound will confirm the presence of an ovarian mass. The other tumors do not cause
virilization. Granulosa cell tumors are estrogen secreting and choriocarcinoma is notable for elevated
BHCG.
A 34 year-old woman comes to you for a chief complaint of hirsutism. She states that this has been present since
menarche, but has gotten worse in the past two years. Her menses have become more irregular, now every 28-45
days apart. She states that she quit smoking and gained approximately thirty pounds in the past three years. Her
mother is obese, diabetic and has hirsutism. There is no hair seen on her chin, but she shaves every few days. TSH,
prolactin, 17-hydroxyprogesterone and DHEAS are normal. Testosterone is mildly elevated. Which of the following
is the most likely etiology of her hirsutism?
a) Polycystic ovarian syndrome
b) Ovarian neoplasm
c) Diabetes
d) Cushing’s syndrome
e) Adrenal tumor
Correct!!! Based on her clinical symptoms and her mildly elevated testosterone this patient has PCOS. The
laboratory data does not support an ovarian neoplasm or diabetes as a cause of her symptoms. The normal
laboratory data above rule out the other causes of hirsutism.
A 26 year-old woman comes to the office due to irregular menses since menarche, worsening for the last 6 months.
The patient has noted increasing hair growth on her chin and most recently hair growth on her chest, requiring that
she shave periodically. No one in her family has hirsutism. On exam, you also notice acne on her chin, acanthosis
nigricans and temporal balding. Her serum testosterone is elevated. You suspect hyperthecosis. Which of the
following might also be associated with this condition?
a) Hyperthyroidism
b) Hyperprolactinemia
c) Atrophic changes of external genitalia
d) Deepening of the voice
e) Hyperparathyroidism
Correct!!! Hyperthecosis is a more severe form of polycystic ovarian syndrome (PCOS). It is associated
with virilization due to the high androstenedione production and testosterone levels. In addition to temporal
balding, other signs of virilization include clitoral enlargement and deepening of the voice. Hyperthecosis
is more difficult to treat with oral contraceptive therapy. It is also more challenging to achieve successful
ovulation induction. Hyperthyroidism and hyperparathyroidism are not typically associated with
hyperthecosis. Hyperprolactinemia is typically associated with amenorrhea and does not cause hirsutism.
A 21 year-old woman comes to the office because of acne, irregular menses and hirsutism. She initially was
evaluated 6 months ago. At that time, she was diagnosed with idiopathic hirsutism. She was started on oral
contraceptive pills to improve her symptoms. Menstrual periods now occur every month, but her hirsutism has not
significantly improved. In addition to the oral contraceptives, which of the following would be an appropriate
treatment for hirsutism?
a) Spironolactone
b) Lupron
c) Danazol
d) Depo-Provera
e) Steroids
Correct!!! Spironolactone, an aldosterone antagonist diuretic, can also be used in addition to the oral
contraceptives for hirsutism. Danazol is primarily used for the treatment of endometriosis and may actually
worsen hirsutism and acne. Lupron and Depo-Provera are also reasonable as second-line treatments of
hirsutism, had the patient not already been on oral contraceptives. Steroids will not help.
Neoplasia - Exam 51
An 88 year-old G2P2 nursing home resident is brought in for evaluation of blood found in her diapers. She has a
long-standing history of incontinence. This is the first time that her caregivers have noted blood. They describe it as
“quarter size.” Her nurses think that she may have been itching, as they frequently find her scratching through the
diaper. On review of her medical record, biopsy documented lichen sclerosus of the vulva was diagnosed fifteen
years ago. She has not been on any therapy for this condition for years. Examination of the external genitalia reveals
an elevated, firm irregular lesion arising from the left labia. The lesion measures 2.5 cm in greatest dimension. The
remainder of the external genitalia shows evidence of excoriation of thin, white skin with a wrinkled parchment
appearance. The vagina and cervix are atrophic. No masses are noted on bimanual or rectovaginal exam and a
sample of her stool is negative for blood. No nodularity is noted in her groin. Which of the following is the most
appropriate next step in the management of this patient?
a) Begin steroids for her lichen sclerosis
b) Benadryl and xylocaine jelly for symptomatic relief
c) Biopsy the lesion
d) Exam under anesthesia with colposcopy and CO2 laser ablation
e) Complete vulvectomy and lymph node dissection
Correct!!! The most important step is to first biopsy the lesion. It would be inappropriate to treat the lichen
sclerosus first, as the lesion is suspicious for malignancy. Prescribing medications that would provide
symptomatic relief for the itching is reasonable, but would not be the single most appropriate for evaluation
of the lesion. A biopsy should be performed to make a definitive diagnosis and rule out malignancy. It
would also be inappropriate to treat the patient with a vulvectomy and lymph node dissection before
obtaining a clear diagnosis.
A 72 year-old woman presents to the office reporting a history of vulvar itching that has been worsening over the
last 6 months. She has a long history of lichen sclerosus, for which she has not been receiving treatment. On exam,
you find an irregular- shaped lesion which measures 3.5 cm in greatest dimension, suspicious for malignancy. You
perform a 4 mm punch biopsy at the edge of the lesion and send it for pathologic evaluation. The pathologist reports
moderately differentiated squamous cell carcinoma. Which of the following is the most appropriate treatment for
this patient?
a) Steroid treatment
b) Radical vulvectomy and groin node dissection
c) Excisional biopsy
d) Laser vaporization of the lesion
e) Cryotherapy
Incorrect!!! Given the findings of obvious, moderately differentiated carcinoma, definitive treatment can be
recommended with radical vulvectomy and groin node dissection. Only microinvasive squamous cell
carcinoma of the vulva can be treated by wide local excision, but it is a diagnosis that is only made after
pathology evaluation of a small (<2 cm), well-differentiated lesion, with invasion <1.0 mm. Excisional
biopsy is not indicated given the larger lesion and confirmed finding of cancer. It would be inappropriate to
laser a malignant lesion. Squamous cell carcinoma is the most common vulvar malignancy and may arise in
the setting of chronic irritation from lichen sclerosus. Steroids would treat the lichen sclerosus, but would
only result in needless delay in treatment of the malignancy. Cryotherapy is not an acceptable treatment for
squamous cancer of the vulva.
An 88 year-old nursing home resident is brought in for evaluation of blood found in her diapers. She has a history
lichen sclerosis of the vulva, which was diagnosed fifteen years ago. She quit smoking in her fifties. Examination of
the external genitalia reveals an elevated, firm, erythematous, ulcerated lesion arising from the left labia, measuring
2.5 cm in greatest dimension. What is the most likely diagnosis in this patient?
a) Malignant melanoma
b) Squamous cell carcinoma
c) Lichen sclerosus
d) Paget’s disease
e) Verrucous carcinoma
Correct!!! Squamous cell carcinoma accounts for approximately 90% of vulvar cancers. Patients commonly
present with a lump and they commonly have a long-standing history of pruritus. The chronic itch-scratch
cycle of untreated lichen sclerosus, or any other chronic pruritic vulvar disease, is thought to stimulate the
development of squamous carcinoma. The mean age of squamous cell carcinoma is 65 years and smoking
is known to increase the risk of development of vulvar cancer, especially in the setting of HPV infection. .
With lichen sclerosus, the skin appears thin, inelastic and white, with a “crinkled tissue paper” appearance.
Paget’s disease of the vulva is associated with white plaquelike lesions, not a discrete mass. Verrucous
carcinoma has cauliflower-like lesions. Melanoma typically presents as a pigmented lesion.
A 42 year-old G2P2 woman presents to the office because she recently noticed a pigmented lesion on her vulva. She
does not know how long it has been there and it doesn’t bother her except that she is worried that she may have
warts. Her yearly Paps have been negative. Her prior exams have been reported as normal. She is a nonsmoker.
Examination of her vulva reveals a pigmented, flat lesion, approximately 1.5 cm in largest diameter. It is nontender.
No induration is present. Her groin nodes are not enlarged. Her vagina and cervix are well estrogenized and without
obvious lesions. Which of the following is the most likely diagnosis?
a) Vulvar condyloma
b) Squamous cell carcinoma
c) Melanoma in situ
d) Paget’s disease
e) Lichen sclerosis
Incorrect!!! This lesion may be melanoma and a biopsy must be done to exclude this diagnosis. The
concerning features are the size and irregularity of the lesion. Melanoma represents 5% of vulvar cancer,
which is not insignificant given the lack of sun exposure and the relatively small surface area. There is no
variability in the coloration, ulceration or thickening of the lesion to suggest malignancy at this time.
Squamous cell carcinoma is typically not pigmented. High-grade vulvar intraepithelial neoplasia may be
flat and pigmented. Paget’s disease is usually erythematous with a lacy white mottling of the surface.
Condyloma lesions have a characteristic verrucous appearance. With lichen sclerosus, the skin appears thin,
inelastic and white, with a “crinkled tissue paper” appearance.
A 38 year-old G3P3 woman presents to the office because she has noted dark spots on her vulva. She states that the
lesions have been present for at least two years and are occasionally itchy. She has a history of laser therapy for
cervical intraepithelial neoplasia ten years ago, and has not had a pelvic exam since then. She has had multiple
partners and uses condoms. Her menses are regular and she had a tubal ligation for birth control. She has smoked
since age 14. She has a history of genital herpes, but has only one or two recurrences a year. On examination,
multicentric brown-pigmented papules are noted on the perineum, perianal and labia minora. No induration or groin
nodularity is noted. The vagina and cervix are normal in appearance. Which of the following is the most likely
diagnosis?
a) Hidradenitis suppurativa
b) Molluscum contagiosum
c) Vulvar intraepithelial neoplasia
d) Melanoma
e) Paget’s disease
Correct!!! This presentation is classic for human papilloma virus (HPV) related VIN 3. Melanoma would
be unlikely to be multifocal and warts have a characteristic verrucous appearance, although pigmentation
can occur. Molluscum, a pox virus, is characterized by multiple shiny non-pigmented papules with a central
umbilication. Paget’s disease, although multicentric, does not have brown pigmentation. Hidradenitis is a
chronic, unrelenting skin infection causing deep, painful scars and foul discharge
A 43 year-old G5P5 woman presents for biopsy results of a vulvar lesion. Her pathology showed vulvar
intraepithelial neoplasia grade 3 (VIN 3). She has a history of chronic hypertension, type II diabetes mellitus, and
her gynecologic history is notable for chlamydia, genital herpes and prior treatment for cervical dysplasia 5 years
ago. She smokes one pack per day. Which of the following is the patient’s greatest risk factor for developing VIN 3?
a) Parity
b) Diabetes mellitus
c) Smoking
d) Genital herpes
e) Human papillomavirus (HPV)
Incorrect!!! VIN is associated with human papillomavirus infection and smoking. The patient’s known
history of cervical dysplasia treatment implies prior exposure to HPV. Parity is not a significant risk factor
for VIN. Chlamydia and genital herpes are not directly associated with VIN, although patients with a
history of sexually transmitted infections are more likely to have concurrent human papillomavirus
infection. Diabetes is not associated with an increased risk of VIN.
A 45 year-old G3P3 woman presents to the office because of a large dark spot on her vulva. She states that the
lesion has been present for at least two years and is occasionally itchy. She has smoked since age 20. She has a
history of genital herpes, but only has one or two recurrences a year. On examination, a 2.5 cm lesion is noted. No
induration or groin nodularity is noted. The vagina and cervix appear normal. There are no additional lesions noted
on colposcopic examination of the vulva. A biopsy of the lesion returns as vulvar intraepithelial neoplasia grade III
(VIN III). What is the most appropriate next step in the management of this patient?
a) Imiquimod (Aldara) treatment
b) Trichloroacetic acid treatment
c) Wide local excision
d) Cryotherapy
e) Radical vulvectomy
Incorrect!!! VIN III should be treated with local superficial excision. Even with complete removal of all
disease, the likelihood is very high for recurrence and the patient will need close surveillance. It is
inappropriate to do radical surgery in this setting as cancer has not been diagnosed. Treatment with TCA
and Aldara are reserved for condyloma, although some studies have shown utility in the use of Aldara in
treating low grade VIN. Cryotherapy is primarily used to treat cervical dysplasia.
A 44 year-old G2P2 woman presents with 6 months of intermittent vulvar itching. She denies any bleeding, but does
have a whitish discharge. She has not felt any obvious lumps or sores. She was diagnosed with lupus over 10 years
ago and is on prednisone, mycophenolate mofetil (CellCept), and hydroxychloroquine (Plaquenil). Her periods are
irregular and her Pap smears have been normal, but infrequently done (last one was 2-3 years ago). She reports
treatment in the past for warts when she was first diagnosed with lupus, but denies any other sexually transmitted
infections. Examination of the vulva is notable for diffuse, erythematous labia, with a thin white filmy discharge.
There appears to be subtle, but multi-focal, flat, whitish lesions measuring 0.5 – 1 cm on the labia bilaterally. Her
remaining pelvic exam is unremarkable. A wet prep is performed and is negative. The next appropriate intervention
would be:
a) Treatment with oral fluconazole and a topical imidazole
b) Colposcopy and directed biopsies of the vulva
c) Treatment of the lesions with topical trichloroacetic acid
d) Wide local excision of the vulva
e) Adjustment of her immunosuppressive therapies
Incorrect!!! These lesions most likely represent an HPV-related condition such as condyloma or vulvar
dysplasia. Women who are on immunosuppressive therapy are at higher risk of such conditions and require
close surveillance. Although, her history of prior treatment for warts suggests these could be condyloma
again, their flat subtle appearance raises a concern that they may be dysplastic lesions, and further
investigation with colposcopy and biopsy are indicated. Treatment for presumed yeast infection is
reasonable, given her susceptibility, but would not be the sole treatment in this setting of new clinicallyevident lesions especially in light of a negative wet prep. Wide excision is not indicated at this time without
a diagnosis.
A 34 year-old G2P2 woman presents with biopsy-proven vulvar intraepithelial neoplasia, grade 2 (VIN 2). She had
undergone routine examination by her primary physician, who performed a Pap smear (normal) and noted multiple
warty-type lesions on the labia. The patient describes some mild itching that she self-treated for a yeast infection,
with minimal relief. Otherwise, she is completely healthy, except for smoking ½-pack of cigarettes per day. She is
sexually active, and is concerned about the impact this will have on her sex life. Examination confirms multiple,
whitish raised 0.5 – 1.5 cm papules throughout her labial minora, majora, clitoral hood and perineum. Which of the
following is the most appropriate treatment option for this patient?
a) Trichloroacetic acid (TCA)
b) Skinning vulvectomy
c) Observation and expectant management
d) Co2 laser ablation of the lesions
e) Smoking cessation
Correct!!! Given the multifocality of the vulvar dysplasia, laser treatment is the best choice. In order to
adequately treat these lesions, a complete (skinning) vulvectomy would be the other choice, but would be
disfiguring and require removal of the clitoris which would have detrimental effects on her sexual function.
Treatment with TCA is recommended for treatment of warts and not VIN. Smoking cessation is strongly
recommended regardless, but would not be the sole means of addressing these lesions. Observation is not
ideal, given her mild symptoms, moderate grade, and diffuse nature of the lesions.
A 58 year-old G2P2 woman presents to your office complaining of two years of a vulvar rash. She has seen multiple
physicians and no one has made a definitive diagnosis. The patient has experienced intermittent pruritus for one
year. She has been prescribed every yeast medication known and has also used multiple over-the-counter products.
She was recently given topical steroid cream, which did not alleviate her symptoms. She is a breast cancer survivor
and was diagnosed and treated one year ago. She is presently on Tamoxifen. No vaginal bleeding has occurred since
her menopause. On examination, her vulva is fiery red background mottled with whitish hyperkeratotic areas
without a distinct lesion. No nodularity or tenderness is noted. The rest of her exam, with the exception of atrophy,
is normal. What is the most likely diagnosis in this patient?
a) Lichen sclerosus of the vulva
b) Contact dermatitis
c) Yeast vulvitis
d) Psoriasis of the vulva
e) Paget’s disease of the vulva
Correct!!! This is a typical description of Paget’s disease of the vulva. Paget’s is an in situ carcinoma of the
vulva. The association with breast cancer is significant, but not as high as Paget’s disease of the nipple. It
would be unlikely for psoriasis to present this late in life. Contact dermatitis is unlikely to last for years and
this woman has had therapy for yeast. Lichen sclerosus is possible and more common, but does not have
the hyperkeratotic overlay and would have more likely responded to steroid use.
Neoplasia - Exam 54
A 57 year-old G0, postmenopausal woman presents to her gynecologist with a complaint of vaginal bleeding for 1
week. The patient reports the cessation of normal menses approximately four years ago. She has had no previous
episodes of irregular bleeding except when she took hormonal replacement therapy for 6 months. She saw her nurse
practitioner 5 months ago and reports having a normal gynecologic evaluation and a normal pap smear. Her past
medical history is significant for hypercholesterolemia, and diet-controlled diabetes mellitus. Physical exam reveals
a 5’3” tall, 275-pound woman in no acute distress. Pelvic exam demonstrated a normal vulva, urethra, vagina and
cervix. Bimanual exam was normal. An endometrial biopsy was obtained and demonstrated complex atypical
hyperplasia. Which of the following is this patient’s greatest risk factor for developing endometrial cancer?
a) Nulliparity
b) Obesity
c) Post menopausal status
d) Use of hormone replacement therapy
e) Complex atypical hyperplasia
Incorrect!!! Endometrial cancer is a gynecologic malignancy that has easily identifiable risk factors and
typically presents with symptoms that lead to an early diagnosis. Risk factors include nulliparity, obesity,
late menopause, hypertension and exposure to unopposed estrogens. Of these risk factors, obesity confers
the greatest risk of developing endometrial carcinoma, especially when the patient is more than 50 pounds
over ideal body weight (10-fold increase). However, in this case, the patient’s greatest risk for developing
an endometrial cancer is the presence of complex atypical hyperplasia (CAH) on endometrial biopsy. If left
untreated, this process has approximately a 28% chance of progressing to an invasive cancer. More
importantly, approximately 30% of women with a diagnosis of CAH will be found to have an invasive
endometrial cancer on final pathology. Most women who develop endometrial cancer are post-menopausal,
but this is less of an issue because of the finding of CAH.
A 69 year-old G3P3 woman comes in for an annual exam. Her friend was recently diagnosed with endometrial
cancer and she is concerned about her risk. Your patient experienced her last menstrual period at age 49. She has not
had any bleeding since. Her medications include only a multivitamin and supplemental calcium. She has no
significant family history. Her physical examination including a pelvic examination is normal. She is 5 ‘5” tall and
weighs 120 pounds. What is the most appropriate management for this patient?
a) Endometrial biopsy
b) Serum estradiol levels
c) Ultrasound with measurement of the endometrial lining
d) Annual exams
e) Dilation and curettage
Correct!!! Less than 5% of women diagnosed with endometrial cancer are asymptomatic. Approximately
80-90% of women with endometrial carcinoma present with vaginal bleeding or discharge as their only
presenting symptom. Since this patient does not have any symptoms or risk factors for endometrial cancer,
she does not need to have any diagnostic testing. Risk factors for endometrial cancer include late
menopause, unopposed estrogen therapy, nulliparity, obesity, Tamoxifen therapy and diabetes mellitus.
Endometrial cancer ranks as the fourth most common cancer detected in women in the U.S. In 2010,
according to the American Cancer Society, there will be an estimated 43,470 new endometrial cancer cases.
It is the most common gynecologic malignancy. Top Five Cancers Detected in Women: • Breast 28% •
Lung 14% • Colon 10% • Uterine 6% • Ovary 3% Gynecologic Cancers: • Uterine 52% • Ovary 26% •
Cervix 14% • Vulva 5% • Vagina 3%
A 72 year-old G3P3 woman presents to the emergency room complaining of abnormal vaginal discharge for the past
two months. She has had two episodes of vaginal bleeding over the last year. She used combination hormone
replacement therapy for 10 years when she went through menopause at age 58, but stopped once the Women’s
Health Initiative report came out. Her last gynecologic exam and Pap smear were two years ago and were normal.
She has tried several over-the-counter antifungal creams for what she presumed was a yeast infection; however, she
reports no change in the nature of the discharge. She does note that she has some mild lower abdominal discomfort.
The only significant finding on exam is a mucopurulent discharge from a multiparous cervical os. She has a 10-week
sized globular uterus. Which of the following findings is most concerning for presence of endometrial cancer?
a) Vaginal bleeding
b) Late menopause
c) Abnormal vaginal discharge
d) Enlarged uterus
e) Hormone replacement therapy
Correct!!! Endometrial cancer is a disease that typically presents with symptoms and clinical findings that
lead to an early diagnosis. The most common symptom is abnormal postmenopausal bleeding. However,
other symptoms or clinical findings that may be seen include abnormal vaginal discharge and lower
abdominal discomfort. Endometrial cancer can increase the size of the uterus as it grows, but is usually not
the most common finding given the early diagnosis of this cancer. Unopposed estrogen replacement
therapy does increase the risk, but not when taken in combination. Early menarche and late menopause are
additional risk factors that may be related to endometrial cancer development.
A 65 year-old G2P1 woman has been referred to you for further evaluation of postmenopausal bleeding. She
initially was seen by her internist after 2 weeks of intermittent vaginal spotting. She reports a similar episode
approximately 2 months ago. A recent exam and Pap smear were normal. A transvaginal ultrasound showed a
homogeneous endometrial lining measuring 3.5 mm. A subsequent office endometrial sample was obtained and
returned scant tissue with rare atypical cells. What is the most appropriate next step in the management of this
patient?
a) Repeat office endometrial sample
b) Follow-up ultrasound in 8 to 12 weeks
c) Return visit in 3 to 6 months
d) Abdominopelvic CT scan
e) Dilatation and curettage
Correct!!! The incidence of endometrial cancer in postmenopausal women with irregular bleeding is 4% to
25%. More importantly, of the women ultimately diagnosed with endometrial cancer, 90% report a history
of vaginal bleeding. Though the diagnostic step of choice is the office endometrial sampling, which has a
detection rate as high as 99% in postmenopausal women, in a patient who has persistent symptoms
suggestive of an underlying endometrial malignancy, this patient has atypical cells on endometrial biopsy
which warrants further investigation with dilation and curettage.
A 62 year-old G0 postmenopausal woman is being referred to your gynecologic oncology colleague after an office
endometrial sample demonstrated a FIGO grade 1 endometrioid adenocarcinoma. The patient has no significant
medical, surgical or other gynecologic history. She does not smoke and drinks only occasionally at social events.
She takes a multivitamin. Her physical exam is unremarkable. Which of the following additional tests is indicated
for this patient?
a) PET/CT scan
b) Chest x-ray
c) Pelvic MRI
d) CA-125
e) Serum estrogen level
Incorrect!!! Once a pathologic diagnosis is confirmed by biopsy, a basic clinical assessment should ensue
in all patients to help define the extent of the disease. If a careful history and clinical gynecologic exam
suggests that the carcinoma is likely of an early stage, minimal pre-treatment evaluation is necessary.
Routine evaluation in this setting should include a chest x-ray as the lungs are the most common site of
distant spread. When there is a low suspicion for advanced disease, a CT scan, MRI, PET scan, and other
invasive and costly tests are not indicated. A CA-125 may be helpful in predicting those patients that may
have extrauterine spread, but is not absolutely necessary.
A 45 year-old G5P5 premenopausal woman was initially seen in your office for work-up and evaluation of a FIGO
grade 2 endometrial cancer that was diagnosed by her gynecologist. Which of the following is the most appropriate
treatment for this patient?
a) a. Total laparoscopic hysterectomy with bilateral salpingo-oophorectomy
b) b. Vaginal hysterectomy with bilateral salpingo-oophorectomy
c) c. Total abdominal hysterectomy, bilateral salpingo-oophorectomy, bilateral pelvic and para-aortic
lymphadenectomy, pelvic washings
d) d. Supracervical abdominal hysterectomy with ovarian preservation
e) Megace
Correct!!! The recommended components of the surgical approach to an early endometrial cancer are the
extrafascial total abdominal hysterectomy, bilateral salpingo-oophorectomy, collection of abdominal and
pelvic washings, and pelvic and para-aortic lymphadenectomy. An omentectomy is indicated in patients
with obvious metastatic disease or a high grade lesion on initial biopsy. When surgically treated in the
appropriate manner, 5-year survival for women found to have stage I disease is over 90%. Alternative
surgical approaches to early endometrial cancer include a total vaginal hysterectomy with or without a
bilateral salpingo-oophorectomy in women who are medically unstable or have contraindications to major
abdominal surgery. Ideally, this approach would only be utilized in patients with well-differentiated
endometrioid adenocarcinomas and avoided in patients with high grade lesions or aggressive cell types,
such as clear cell or papillary serous carcinomas. A total laparoscopic hysterectomy, BSO, +/- staging is
being utilized more and more in lieu of the traditional open approach for select patients in some centers,
and may be a reasonable alternative.
A 48 year-old G0 perimenopausal woman is seen in your office for evaluation of heavy irregular vaginal bleeding.
She has never been sexually active and has never had a pelvic examination or Pap smear. She reports menarche at
age 12, and recalls a life-time of irregular menstruation. She had been on oral contraceptives in her 20s for
regulation of her periods, but stopped them because of the development of hypertension. She also has type II
diabetes mellitus and has always struggled with being overweight. On physical exam, she has a BMI of 32, is mildly
hirsute and has centripetal obesity. A pelvic exam is not tolerated well, although she appears to have normal female
genitalia development. A speculum and bimanual exam cannot be performed. Which of the following do you
recommend in the evaluation process?
a) Transvaginal ultrasound
b) Cyclic medroxyprogesterone (Provera) therapy
c) Office endometrial sampling
d) Dilation and curettage
e) Pelvic MRI
Incorrect!!! This patient has multiple risk factors for endometrial hyperplasia/carcinoma, including obesity,
diabetes, hypertension, and nulliparity. Her history of irregular menstruation is suggestive of anovulatory
cycles which also increase the risk of endometrial neoplasia. Tissue sampling of the endometrium is
critical, and given her intolerance to an office procedure, a dilation and curettage is the next step in the
work-up of her bleeding. A pelvic ultrasound and MRI may provide additional information, but would not
render a pathologic diagnosis. Treatment with Provera may be indicated in the setting of endometrial
hyperplasia, but not as empiric therapy without a confirmed diagnosis.
An 81 year-old G3P3 woman presents to your office with a history of light vaginal spotting. She states this has
occurred recently and in association with a thin yellow discharge. She never experienced any vaginal bleeding since
menopause at the age of 52, and denies ever having been on hormone replacement therapy. She is otherwise
reasonably healthy, except for osteoporosis, well-controlled hypertension, and diabetes. She is physically active and
still drives to all her appointments. She is no longer sexually active since the death of her husband 2 years ago. On
examination, she is noted to have severe atrophic changes affecting her vulva and vagina. A small Pederson
speculum allows for visualization of a normal multiparous cervix, and the bimanual examination is notable for a
small, mobile uterus. Rectovaginal exam confirms no suspicious adnexal masses or nodularity. Which of the
following is the most appropriate management for this patient?
a) Pelvic transvaginal ultrasound
b) Office endometrial biopsy
c) Reassurance and observation for further bleeding
d) Vaginal estrogen therapy
e) Dilation and curettage
Incorrect!!! Postmenopausal bleeding or discharge accounts for the presenting symptom in 80-90% of
women with endometrial cancer. However, the most common causes of post-menopausal bleeding are
atrophy of the endometrium (60-80%), hormone replacement therapy (15-25%), endometrial cancer (1015%), polyps (2-12%), and hyperplasia (5-10%). Any history of vaginal bleeding requires a thorough
history, physical/pelvic examination, and assessment of the endometrium. This is ideally done via office
endometrial sampling as part of the initial work-up. The use of pelvic transvaginal ultrasound can provide
useful information as to the presence of any structural changes (polyps, myomas, endometrial thickening),
and for which a diagnosis of endometrial cancer would be less likely if the endometrial thickness is < 5
mm. Although this patient is likely to have atrophy as the cause of her spotting, a thin endometrial stripe
does not exclude the possibility of a non-estrogen dependent carcinoma of the atrophic endometrium.
A 65 year-old G2P2 postmenopausal woman with a prior history of stage I endometrial cancer returns to your office
for a routine cancer surveillance visit. During a review of systems, the patient complains of several months of a dry
cough. She saw her family physician, who treated her with a short course of antibiotics; however, her symptoms
have persisted. Concerned about a possible recurrence, you order a chest x-ray and CA-125. The CA-125 level is
mildly elevated to 75 and the chest x-ray shows multiple pulmonary nodules. What is the most appropriate next step
in the management of this patient?
a) Give another course of antibiotics
b) Refer to pulmonologist
c) Refer to palliative care
d) Refer to oncologist
e) Repeat chest x-ray in 3 months
A 68 year-old woman with a history of breast cancer presents for evaluation of endometrial cancer risk. She was
treated with lumpectomy and axillary node dissection and radiation therapy. She has been on tamoxifen therapy for
the past year. She denies any vaginal bleeding or discharge. She is 5’3” tall and weighs 140 pounds. Her pelvic
examination is notable only for severe vulvovaginal atrophy. What is the next best step in the management of this
patient?
a) Endometrial biopsy now to get a baseline
b) Annual endometrial biopsy
c) Annual exams
d) Annual pelvic ultrasound
e) Obtain an endometrial biopsy upon completion of 5 years of Tamoxifen use
Incorrect!!! Tamoxifen is known to increase the risk of endometrial cancer. However, diagnostic studies,
such as endometrial biopsy, are reserved for when the patient develops symptoms of bleeding or abnormal
vaginal discharge. Ultrasound is not helpful because Tamoxifen is known to cause changes to the
endometrium, including thickening. Endometrial biopsy is not indicated as a screening tool for endometrial
cancer.
Neoplasia - Exam 55
A 35-year-old African-American G0 woman has a family history of ovarian cancer. Her mother was diagnosed with
ovarian cancer at age 50 and is in remission. The patient had onset of menarche at age 14. She has used oral
contraceptives for a total of 10 years. She smokes one pack of cigarettes per week. The patient has a history of
gonorrhea and herpes and has had a LEEP for treatment of cervical dysplasia. Which of the following places the
patient at greatest risk for developing ovarian cancer?
a) African American race
b) Nulligravidy
c) Late age at menarche
d) Smoking
e) Family history of ovarian cancer
Correct!!! The events leading to the development of ovarian cancer are unknown. Epidemiologic studies,
however, have identified endocrine, environmental and genetic factors as important in the carcinogenesis of
ovarian cancer. The established risk factors include nulliparity, family history, early menarche and late
menopause, white race, increasing age and residence in North America and Northern Europe. Smoking has
not been demonstrated to be associated with an increased risk of ovarian cancer.
An 18-year-old G0 woman presents to discuss contraception. Her best friend’s mother was just diagnosed with
ovarian cancer. The patient is healthy and does not have any significant medical history. She does not have a family
history of ovarian, breast or any other cancer. She uses condoms for birth control. She would like to know what she
can do to minimize her risk for developing ovarian cancer. Which of the following recommendations is the most
appropriate for this patient?
a) Begin childbearing now
b) Use an intrauterine device
c) Use oral contraceptives until she is ready to have children
d) Have a prophylactic oophorectomy once childbearing is complete
e) There are no proven means to reduce the risk of ovarian cancer
Incorrect!!! A woman’s risk for development of ovarian cancer during her lifetime is approximately 1%.
Factors associated with development of ovarian cancer include low parity and delayed childbearing. Longterm suppression of ovulation appears to be protective against the development of ovarian cancer. Oral
contraceptives that cause anovulation appear to provide protection against the development of ovarian
cancer. Five years’ cumulative use decreases the lifetime risk by one-half. Prophylactic oophorectomy is
not a practical choice for this patient with no family history, even once she completes childbearing. This
option might be considered for a woman with a strong family history and the BRCA 1 mutation.
A 25-year-old G0 woman comes in to renew her prescription for oral contraceptives. She has no significant medical
or surgical history. Her grandmother was recently diagnosed with ovarian cancer and her mother is undergoing
treatment for metastatic breast cancer. The patient is interested in assessing her risk for ovarian cancer susceptibility.
Which of the following is the most appropriate next step in the management of this patient?
a) Annual CA125 levels
b) Annual pelvic ultrasound
c) Genetic testing of BRCA 1 and 2 mutations on the patient’s mother
d) Genetic testing of BRCA 1 and 2 mutations on the patient’s grandmother
e) Check the patient for p53 mutation
Correct!!! BRCA-1 and BRCA-2 mutations are typically seen in cases of hereditary ovarian cancers.
However, this represents only about 6-8% of all ovarian cancers. Given this family history, it is highly
likely that a mutation is present, and the affected individual (proband) should be tested if still alive.
Because breast cancers are part of the BRCA mutation, the affected mother should be tested. Routine
“screening” for ovarian cancer has not been established.
A 25-year-old G1P1 woman comes in for her annual exam. She has intermittent left lower quadrant discomfort. She
has regular menses every 30 days and uses a diaphragm for birth control. Her last menstrual period was
approximately three weeks ago. Her physical exam is notable for a 3 x 5 cm left adnexal mass. Ultrasound shows a
unilocular simple cyst. Which of the following is the most likely diagnosis in this patient?
a) Endometrioma
b) Functional ovarian cyst
c) Mucinous cystadenoma
d) Serous cystadenoma
e) Dermoid
Correct!!! Functional ovarian cysts are a result of normal ovulation. They may present as an asymptomatic
adnexal mass or become symptomatic. Ultrasound characteristics include a unilocular simple cyst without
evidence of blood, soft tissue elements or excrescences. An endometrioma is an isolated collection of
endometriosis involving an ovary. This would not classically appear as a simple cyst on ultrasound. Serous
cystadenomas are generally larger than functional cysts and patients may present with increasing abdominal
girth. Mucinous cystadenomas tend to be multilocular and quite large. Dermoid tumors usually have solid
components or appear echogenic on ultrasound, as they may contain teeth, cartilage, bone, fat and hair.
A 72-year-old G3P2 postmenopausal woman is referred to your office by her internist after work-up for abdominal
bloating revealed a large pelvic mass on transvaginal ultrasound, and an elevated CA-125. The patient had a normal
colonoscopy and mammography two months ago. The physician states that the patient’s greatest complaint is that of
early satiety and upper abdominal discomfort and that her physical exam shows a significant fluid wave. Which of
the following tests would be most helpful in assessing the extent of disease?
a) Barium enema
b) PET Scan
c) CT scan of abdomen and pelvis
d) Chest X-ray
e) Intravenous pyelogram
Incorrect!!! The most useful radiologic tool for evaluating the entire peritoneal cavity and the
retroperitoneum is computerized tomography. Specifically in this patient, it would be important to look for
significant involvement of the omentum. A chest x-ray provides adequate evaluation of the chest, unless it
is abnormal. If there is a suspicion for chest involvement on the chest film, then a chest CT is necessary.
With a normal colonoscopy and no symptoms suggestive of colonic obstruction, a barium enema would not
be useful. PET scan, to date, has not been shown to play a role in the initial evaluation of women with a
suspected ovarian malignancy. However, PET scan may play a role in evaluating women with a known
diagnosis of ovarian cancer who have a suspected recurrence. An IVP would only be indicated if there was
suspected renal obstruction.
A 72-year-old G3P2 postmenopausal woman is referred to your office by her internist after a work-up for abdominal
bloating revealed a large pelvic mass on transvaginal ultrasound and an elevated CA-125. Reviewing her chart, you
see that she has no significant medical history and only a prior appendectomy. The CT scan showed a large
heterogenous pelvic mass measuring 20x13x10 cm. There was a moderate amount of intra-abdominal ascites and
likely “omental caking.” There was no significant pelvic or abdominal lymphadenopathy, and the chest x-ray
showed only a small right pleural effusion. On pelvic exam, there is minimal cul-de-sac nodularity and the mass is
readily palpable and somewhat mobile. There is an obvious fluid wave. Which of the following is the most
appropriate next step in the management of this patient?
a) Thoracentesis for staging
b) Exploratory laparotomy and ovarian cancer staging
c) Paracentesis for cytologic diagnosis
d) Diagnostic laparoscopy for tissue biopsies
e) Neoadjuvant chemotherapy
Incorrect!!! Based on this patient’s initial evaluation, it seems highly likely that she has an advanced stage
ovarian carcinoma. Given her lack of significant medical co-morbidities, and a gynecologic exam
suggestive that the tumor would be potentially respectable, it would be appropriate to offer her a surgical
intervention. The surgical staging of an apparent ovarian cancer should include a vertical skin incision,
sampling ascites for cytology, inspection of the entire peritoneal cavity, total abdominal hysterectomy,
bilateral salpingo-oophorectomy, omentectomy, cytoreduction to microscopic disease or to disease <1 cm
in maximum diameter, and bilateral pelvic and para-aortic lymph node dissection. If omental disease is less
than 2 cm or if bulky, lymph node disease is present. Thoracentesis to rule out malignant pleural effusion is
part of the staging criteria, but may not be justified if the effusion is too small, and would not impact on the
need to perform surgery. Neoadjuvant chemotherapy is generally considered when a patient has
unresectable disease or is a poor surgical candidate, at which point paracentesis for cytologic confirmation
would be needed.
A 17-year-old G0 sexually active female presents to the emergency room with acute right lower quadrant pain and
nausea for 12 hours. Her periods have always been irregular, with her last one 6 weeks ago. She is otherwise
completely healthy. Physical examination is notable for an uncomfortable teenager: Temperature 99.6°F; blood
pressure 110/60; heart rate 108. She has moderate abdominal tenderness with right greater than left pelvic
tenderness. Pelvic exam reveals normal genitalia; speculum exam with mild pink-tinged yellow discharge,
bimanual/rectovaginal exam confirms mild cervical motion tenderness and fullness in the right adnexa with
moderate tenderness and some voluntary guarding. What is the single most important test to obtain?
a) Pelvic ultrasound
b) Culdocentesis
c) GC and chlamydia DNA probe
d) Beta HCG
e) CBC with differential
Incorrect!!! Although all of the tests listed above may be considered, it is imperative to obtain a beta-HCG
to rule out an ectopic pregnancy.
A 61-year-old woman is diagnosed with stage IIIA papillary adenocarcinoma of the ovary. She is concerned about
her long-term prognosis. Which of the following factors would be most helpful in determining this patient’s
prognosis?
a) Volume of residual disease
b) Tumor stage
c) Presence of non-malignant ascites
d) Tumor grade
e) Ovarian tumor diameter
Correct!!! The 5-year survival of patients with epithelial ovarian cancer is directly correlated with the
tumor stage. The volume of residual disease following cytoreductive surgery is also directly correlated with
survival. Patients who have been optimally debulked (generally <2 cm or <1 cm maximal tumor diameter)
have a significant improvement in there median survival. Histologic grade of tumor is important. Women
harboring poorly differentiated tumors or clear-cell carcinomas typically have a worse survival than those
with well to moderately differentiated tumors. This is especially important in early-stage disease. Tumor
size, bilaterality and ascites without cytologically positive cells, are not considered to be of prognostic
importance.
A 44-year-old G0 woman was found to have a stage IIIB endometrioid adenocarcinoma of the ovary. She reports
that her surgical procedure was three weeks ago and that the surgeon said he got all of the cancer. Her medical
history is significant for diabetes, hypertension, obesity, hypercholesterolemia and major depression. Which of the
following is the most appropriate treatment for this patient?
a) Hospice
b) Chemotherapy
c) Surveillance
d) Pelvic radiation
e) Exploratory laparotomy for residual disease in one year
Incorrect!!! In all patients with advanced ovarian cancer, post-operative chemotherapy with a combination
of a taxane and platinum adjunct is considered standard of care in the United States. Women who undergo
surgical cytoreduction, followed by chemotherapy, have a better overall survival than those who undergo
surgery alone. The overall response rate in women with advanced ovarian cancer following surgery and 4
to 6 cycles of combination chemotherapy with a taxane and platinum adjunct is 60-80%. The overall 5-year
survival for women with Stage III and IV disease is approximately 30%.
A 30-year-old G1P1 woman presents to the emergency room with left-sided abdominal pain. Physical examination
is notable for a 5x6 cm mobile left adnexal mass. An ultrasound is performed, which shows a left ovarian mass with
cystic and solid components. Which of the following is the most likely diagnosis for this patient?
a) Serous cystadenoma
b) Mucinous cystadenoma
c) Endometrioid tumor
d) Dermoid tumor
e) Brenner tumor
Incorrect!!! The most common tumor found in women of all ages is the dermoid. 80% occur during the
reproductive years, with a median age of occurrence of 30 years. Dermoids may contain differentiated
tissue from all three embryonic germ layers. Dermoid tumors can contain teeth, hair, sweat and sebaceous
glands, cartilage, bone, and fat.
Neoplasia - Exam 50
An obese 30-year-old G3P10 Asian female undergoes an uncomplicated D&C for a first-trimester miscarriage.
Pathology reveals a molar pregnancy. The patient’s medical history is significant for chronic hypertension. She has a
history of a previous uncomplicated term pregnancy, and termination of a pregnancy at 16 weeks gestation of a
pregnancy diagnosed with trisomy 18. What aspect of the patient’s history places her at increased risk for a molar
pregnancy?
a) Obesity
b) Previous history of fetal aneuploidy
c) Asian race
d) Chronic hypertension
e) Prior term pregnancy
Incorrect!!! The incidence of molar pregnancy is approximately 1 per 1,500-2,000 pregnancies among
Caucasians in the United States. There is a much higher incidence among Asian women in the United
States (1/800.) Molar pregnancy occurs more frequently in women less than 20 or older than 40 years old.
The incidence is higher in areas where people consume less beta-carotene and folic acid. There is no known
association between molar pregnancy and obesity, a previous history of fetal aneuploidy, chronic
hypertension and parity. The risk of having a molar pregnancy is increased in women with two or more
miscarriages.
An obese 30-year-old G3P11 Asian female undergoes an uncomplicated D&C for a first-trimester miscarriage.
Pathology reveals a molar pregnancy. The patient’s medical history is significant for chronic hypertension. She has a
history of a previous uncomplicated term pregnancy and a termination of a pregnancy at 16 weeks gestation of a
pregnancy diagnosed with trisomy 18. Which of the following is most likely to decrease this woman’s chance of
having another molar pregnancy in the future?
a) Achieve an ideal body weight prior to conception
b) Folic acid supplementation before and during pregnancy
c) Use donor sperm
d) Wait a minimum of two years before trying to conceive again
e) Initiate a vegetarian diet
Correct!!! Molar pregnancy has been associated with folic acid deficiency. The etiology of this disorder is
unknown. No factors have been specifically identified that may decrease the recurrence risk of a molar
pregnancy. The interval between a molar pregnancy and a subsequent pregnancy is not associated with
recurrence of a future molar pregnancy.
A 20-year-old previously healthy nulliparous female presents to the emergency room with painless vaginal bleeding.
Her last menstrual period was 16 weeks prior. On physical exam, her temperature is 98.3°F, heart rate is 120 bpm,
and blood pressure is 140/90. Abdominal and pelvic exam confirms a 20-week sized uterus with a small amount of
blood in the vagina. Beta-HCG is 68,000. Fetal doppler tones are not auscultated. Which of the following findings
would you expect to see on a pelvic ultrasound of this patient?
a) A fetus with no cardiac activity
b) A fetal pole in one of the fallopian tubes
c) A multifetal gestation
d) A uterus with a snowstorm appearance
e) An empty uterus with an enlarged, complex adnexal mass
Correct!!! A complete mole has a characteristic “snowstorm” appearance on ultrasound. This is due to the
presence of multiple hydropic villi. This patient has a classic presentation for a molar pregnancy. Vaginal
bleeding is universal in molar pregnancies. Uterine size greater than dates (weeks from LMP) can be seen
in 25-50% of moles, although size less than dates can be seen in 14-33% of moles. There is no fetus seen in
cases of a complete mole. There can be a fetus, which is usually grossly abnormal, in cases of a partial
mole. There is detectable beta-hCG in molar pregnancies. The beta-hCG values are generally higher than
the values observed in normal pregnancy. Caution should be used against “single-value” diagnosis of betaHCG to rule in or out a molar pregnancy. However, when combined with the findings of an enlarged uterus
and vaginal bleeding, a beta-HCG value >1,000,000 may be diagnostic. Tachycardia from hyperthyroidism
(10% serum diagnosis; 1% clinical diagnosis) and hypertension from preeclampsia (12-25%) can occur in
molar pregnancy.
A 15-year-old female presents to your office with a positive home urine pregnancy test. She has never been pregnant
before and thought she was just putting on weight. Her LMP was 10 weeks ago. Pelvic examination confirms a 16week sized uterus. Which of the following is most appropriate to rule out a molar pregnancy in this patient?
a) Qualitative beta-HCG
b) Quantitative beta-HCG
c) Pelvic ultrasound
d) Pathologic assessment of the uterine contents
e) Repeat examination in 48 hours
Incorrect!!! There is no single beta-HCG value that is diagnostic for a molar pregnancy. A quantitative
beta-HCG, though, is crucial at determining whether or not a pelvic (transvaginal) ultrasound will confirm
a very early gestation. For a gestation at or beyond 10 weeks (with a 16-week uterus) as in this case, a
pelvic ultrasound should readily confirm an intrauterine pregnancy (IUP.) If an IUP is not seen, the
ultrasound findings (in conjunction with the beta-HCG level) should identify a mole (multiple internal
echoes) or an ectopic (absence of intra-uterine gestation.) Suction curettage will provide a pathologic
specimen that can distinguish between a normal and molar pregnancy, but it is used only as a therapeutic
intervention. Although this patient is asymptomatic, a repeat examination in 48 hours would not be useful,
and further diagnostic evaluation is warranted.
A 39-year-old female with an ultrasound report suggestive of a molar pregnancy comes to your office. She is
asymptomatic. She has three children and was not planning any more pregnancies. Her uterus is 16 weeks size and
her beta-HCG is >200,000. What is the recommended treatment for this patient?
a) Suction curettage
b) Hysterectomy
c) Induction with Pitocin
d) Expectant management (await spontaneous miscarriage)
e) Methotrexate
Correct!!! Suction curettage is the standard management for molar pregnancies. Hysterectomy would be a
reasonable option as this particular patient has completed childbearing. However, the morbidity of
abdominal hysterectomy is still considered greater than suction curettage (D&C). Induction with Pitocin
would result in severe bleeding once cervical dilation and contractions developed, and expectant
management would risk increased growth and progression of the mole (not to mention the similar
unnecessary risk of bleeding.) Methotrexate may become necessary if she develops post-molar GTD, but
not as a sole means of primary treatment.
A 22-year-old G1P0 woman who underwent a D&C for a presumed missed abortion has a pathology report
revealing a partial molar pregnancy. Compared to a complete mole, which of the following is true about a partial
mole?
a) Karyotype 69XXY, fetus present, higher risk of developing post-molar GTD
b) Karyotype 69XXY, fetus present, lower risk of developing post-molar GTD
c) Karyotype 46XX, fetus present, higher risk of developing post-molar GTD
d) Karyotype 46XX, fetus present, lower risk of developing post-molar GTD
e) Karyotype 46XX, fetus absent, lower risk of developing post-molar GTD
Incorrect!!! Molar pregnancies are classified as either complete or partial, depending on several histologic,
pathologic and genetic characteristics. Partial moles may contain fetus/fetal parts, placenta/cord; complete
moles do not. Partial moles are triploid karyotype (usually 69XXY, 69XXX, or 69XYY) resulting from
fertilization of egg by dispermy; complete moles are diploid resulting from fertilization of “empty egg” by
single sperm (46XX, 90%) or by 2 sperm (X & Y = 46XY 6-10%.) Partial moles show marked villi
swelling; complete moles show trophoblastic proliferation with hydropic degeneration. Clinically, partial
moles present with lower beta-HCG levels, affect older patients, have longer gestations, and are often
diagnosed as missed or incomplete abortions. Complete moles usually present with larger uteri,
preeclampsia and higher likelihood of developing into post-molar GTD.
A 38-year-old G1P0 woman undergoes a D&C for a partial molar pregnancy. The patient and her husband are very
devastated by the loss of this much-desired pregnancy. Because she feels that her “reproductive clock” is ticking
away, the patient would like to get pregnant as soon as possible. Which of the following recommendations is
appropriate for this patient?
a) Attempt pregnancy once she recovers from the D&C
b) Attempt pregnancy once beta-HCG normalizes
c) Contraception until 6 months after negative beta-HCG levels
d) Avoid a future pregnancy
e) Avoid oral contraceptives
Correct!!! Once evacuation has been accomplished, patients must be followed regularly with serial betaHCG levels to insure spontaneous regression. Pregnancy should be avoided during this follow-up period,
and for the following 6 months. Effective contraception (OCP or other hormonal contraception) is strongly
recommended to prevent confusion in interpreting a rising beta-HCG as a post-molar
recurrence/progression versus a new, spontaneous pregnancy. OCP’s have been proven safe to use and are
a highly effective means of contraception among this patient population.
A 28-year-old G1P0 woman has recovered from treatment (suction curettage) for a complete molar pregnancy. She
understands that her beta-HCG will be monitored closely, but is eager to have a baby. Which of the following most
accurately describes this patient’s chances of successfully conceiving and delivering a viable infant?
a) Poor, because of the high risk of developing another mole
b) Excellent, because of the low risk of developing another mole
c) Low, because of the poor success rates in curing gestational trophoblastic disease
d) High, because of the effective use of chemotherapy in treating molar pregnancies
e) Low, because molar pregnancies are seen more frequently in women who have difficulty conceiving.
Incorrect!!! The risk of developing another molar pregnancy is approximately 1-2% (higher than compared
to women who have never had a molar pregnancy). Therefore, full-term pregnancy is considered very
plausible, even in women with repeated molar pregnancies. GTD is considered the most curable
gynecologic malignancy and preservation of fertility allows for successful future pregnancy. Although
chemotherapy is very effective in treating the higher risk GTD diagnoses (choriocarcinoma, invasive
moles, post-molar GTD), it is not indicated for treatment of molar pregnancy. Even with chemotherapy
treatment, successful pregnancy can ensue. There is no known association between molar pregnancy and
infertility.
A 33-year-old G3P1 woman presents to your office with a positive home pregnancy test. Her last menstrual period
was 12 weeks ago. Obstetrical history is notable for a prior full term vaginal delivery and a miscarriage. Ultrasound
reveals multiple internal echoes consistent with a “snow storm” appearance within the 20-week sized uterus, as well
as bilateral 6 cm ovarian theca lutein cysts. Beta hCG level is >200,000. A D&C is performed and final pathology
reveals a complete molar pregnancy. Her risk of developing persistent (post-molar) gestational trophoblastic disease
is:
a) Non existent, given the complete evacuation
b) Lower than that of a partial molar pregnancy
c) Higher than that of a partial molar pregnancy
d) Same as that of a partial molar pregnancy
e) High enough to mandate prophylactic treatment with methotrexate
Incorrect!!! Although very effective in evacuating both complete and partial molar pregnancies, suction
curettage (D&C) provides definitive therapy in the vast majority of partial moles (>95%.) For complete
molar pregnancies, although beta hCG levels initially do drop following D&C, they can plateau and
eventually rise in approximately 20% of cases. The risk following partial moles is much less (5%.) The
development of this post-molar GTD may be due to persistent (retained or invasive) disease in the uterus or
metastatic disease (often to the lungs.) The constellation of findings described in this patient (large uterus,
theca lutein cysts, high beta hCG) increase the risk that this molar pregnancy will persist despite complete
evacuation, hence the need for close follow-up with serial beta-HCG levels. Persistent disease can easily be
cured with chemotherapy, if it develops, and is therefore not routinely given prophylactically, except in
high-risk situations (e.g. non-compliant patient who will be lost to follow-up.)
A 28-year-old previously healthy woman presents to the emergency room obtunded. Her husband reports that she
has been feeling more and more fatigued since the delivery of their child 8 months ago. Her physical examination is
unrevealing and her pelvic exam reveals a normal appearing cervix, normal-sized uterus and no adnexal masses. Her
work-up ultimately reveals multiple nodules on chest X-ray and within the brain and liver, suspicious for metastasis.
Which diagnostic evaluation would aid in making a diagnosis of choriocarcinoma in this patient?
a) Quantitative beta-HCG
b) Serum CA-125
c) Transvaginal ultrasound
d) Fine needle aspiration of the liver lesions
e) Biopsy of a chest nodule
Correct!!! A diagnosis of choriocarcinoma is made once the presence of beta hCG is confirmed. Certainly,
intrauterine pregnancy and ectopic pregnancy must be excluded, but this can easily be done depending on
the quantitative level. In the presence of metastatic disease of unclear primary, the diagnosis of GTD
(choriocarcinoma) must be considered. Ultrasound is useful in ruling out an intrauterine or ectopic
pregnancy, but provides no information if the beta-HCG is negative or below the discriminatory zone.
Serum CA-125 is a tumor marker for most epithelial ovarian cancers but, because it is non-specific, its
possible elevation in this case is not diagnostic. Because metastatic choriocarcinoma is quite vascular,
suspicious lesions should never be biopsied. Tissue diagnosis is the standard in establishing a diagnosis of
most all malignancies, with the exception of choriocarcinoma. Only a positive beta-HCG in a reproductiveaged woman who has a history of a recent pregnancy (term, miscarriage, termination, mole) is necessary to
establish the diagnosis
Domestic Violence - Exams 57 and 58
An 18 year-old college student is seen in the emergency room, claiming she was raped by a 29 year-old janitor in
her dorm 4 hours ago. He threatened her with a knife and she did not resist. She appears calm and has a flattened
affect when the history is taken. Currently, she is sexually active with a fellow student and is taking birth control
pills. The alleged attacker used a condom, which she helped him put on. The student is 5’2” tall and weighs 110
pounds. She is a cheerleader for the college football team. Examination reveals no bruising and gynecologic
examination reveals no apparent injuries. Which of the following statements is the most likely explanation for this
patient’s presentation?
a) The attack was not emotionally traumatic
b) The student is in shock
c) The attack never happened
d) The student cooperated with the attacker
e) The student suffers from chronic depression
Incorrect!!! Women who are raped are often in denial or shock. A woman may shower after an attack,
destroying evidence in an attempt to deny what happened or to “clean herself.” She may blame herself for
the attack and feel she should have resisted more.
A 10 year-old girl has been drawing sexually explicit pictures during art class. Her behavior in class has ranged from
being shy to being aggressive with her fellow students. She won’t go to the bathroom unless someone goes with her.
Physical examination shows a normally developed girl. Breast budding is occurring and she has wisps of axillary
and pubic hair. She will not let you “look at her bottom.” Which of the following is the most likely cause of this
condition?
a) Normal adolescent development
b) Attention deficit disorder
c) Precocious puberty
d) Child abuse
e) Hypothyroidism
Incorrect!!! Children who are sexually abused will often “act out” and behave inappropriately. There is
often no evidence of physical injury and a careful history must be taken.
A 6 year-old girl has begun to bed-wet after having been toilet trained for 3 years. She has also been found “playing
with herself” for the past several weeks. In a private conversation the child states, “My daddy has been playing with
me down there.” Which of the following is the most appropriate course of action at this time?
a) Suggest family counseling
b) Notify police
c) Treat for urinary tract infection
d) Speak with both parents and child together
e) Refer to a pediatric urologist
Correct!!! Child abuse can happen in any circumstance. It is not always the “boyfriend” who commits such
a crime. Often the perpetrator is a “pillar of the community.” The child must be able to talk in a safe
environment, certainly not in the presence of the person accused of the deed.
A mother brings in her 6 year-old daughter for an examination. She states that she thinks her boyfriend may be
“fooling around” with the child. The girl runs and hides when the boyfriend comes to their home. Examination
reveals a normal 6 year-old girl. There is no evidence of sexual abuse. The mother wants nothing done as she is
dependent on her boyfriend for support. Which of the following is the most appropriate course of action?
a) Advise return visit in one month
b) Notify police
c) Advise family counseling
d) Inform mother that the child may be jealous
e) Perform colposcopic examination
Incorrect!!! In all suspected cases of child abuse, the proper authorities must be notified. Often victims of
child abuse have no physical findings. Colposcopic examination is often inconclusive. The child may be
acting out, but it is better to err on the side of safety.
A 3 year-old girl is taken to see her primary care physician by her father, secondary to vaginal discharge and
swelling of the vulvar area for the past week. She has been scratching the area and says it hurts. She was on an
antibiotic 2 weeks ago for an ear infection. The child has had normal growth and development. She has 2 male
siblings ages 8 and 6. Her father is a stay-at-home dad and her mother is an attorney. Examination reveals a red and
swollen vulva with linear abrasions over the area. Which of the following is the most appropriate next step in
treatment?
a) Check a complete blood count
b) Begin antifungal treatment
c) Perform nasal speculum vaginal examination
d) Refuse to treat until mother is present
e) Notify police
Incorrect!!! Yeast infections are common after antibiotic therapy. Scratching can appear like abuse is taking
place. To examine a child, even with a nasal speculum is traumatic. If a foreign body is suspected, an exam
under anesthesia may be necessary, if ultrasound is not successful or indicated.
A 4 year-old girl is taken to see her pediatrician by her father. He states that he has noticed a foul odor in the area of
the child’s “private parts.” He brings in a pair of panties with a brown-yellow stain that has a “funny smell.” The girl
is otherwise healthy. Which is the most likely cause of this condition?
a) Child abuse
b) Foreign body
c) Yeast infection
d) Pinworm infection
e) Urinary tract infection
Incorrect!!! Children will often place foreign objects in any body orifice. Toilet paper is the most common
foreign body found.
A 3 year-old girl is seen in the emergency room because her mother saw her putting something inside of her vagina.
The mother had placed an earring stud on the bed and the child was playing with it. Ultrasound examination reveals
a metal object deep in the child’s vagina. Which of the following is the most appropriate course of action at this
time?
a) Vaginal extraction using a nasal speculum for visualization
b) Removal under anesthesia
c) Provide instructions to the girl and mother on how to remove the object
d) Prescribe broad speculum antibiotic until object expelled
e) Order gentle vaginal douching to assist in expelling object
Correct!!! An attempt to remove an object without anesthesia is almost impossible in such a young child.
Under anesthesia, a gentle rectal exam using the smallest finger can often “milk-out” a foreign body.
An 18 year-old college student is brought to your office by her roommate who found her sobbing in their room. The
student states that she was sitting on a couch with her new boyfriend. He gave her something to drink. She thinks
that she passed out and awoke 2 hours later and was undressed. She is afraid that she may have had intercourse. She
is not on any form of contraceptive. Her last menstrual period was 2 weeks ago. Examination reveals no obvious
injuries and a vaginal swab shows motile sperm. In addition to providing counseling and screening for sexually
transmitted infections, which of the following is the most appropriate next step in management?
a) Discuss her responsibility to report this to her college
b) Admit to the hospital for intravenous antibiotics
c) Recommend emergency contraception
d) Notify her college advisor
e) Obtain more information from her roomate
Correct!!! In a case of date rape, the best course of action is to ensure that the patient does not get pregnant.
The patient should also have screening for sexually transmitted diseases, with consideration to being
offered antibiotic prophylaxis. Although the risk of infection is unknown among victims of sexual assault,
it may be higher compared to consensual sexual encounters.
A 17 year-old G0 was kissing her 18 year-old boyfriend in a parked car. She refused to have sexual intercourse and
he overpowered her, hit her on the face and raped her. He did not use a condom and ejaculated inside her. Which of
the following made this case a rape?
a) Failure to use effective barrier contraception
b) Sexual intercourse
c) Lack of consent
d) Legal age of woman
e) Legal age of the boyfriend
Correct!!! Rape is characterized by lack of consent or inability to give consent. Most definitions include the
use of physical force, deception, intimidation or the threat of bodily harm.
A 30 year-old G2P2 was raped by a stranger while she was walking through a park alone at 2:00 am. The assailant
did not use a condom. An intern evaluated the patient in the emergency room. He took a detailed history, performed
a complete physical exam and collected forensic specimens. He obtained cultures for gonorrhea and chlamydia, and
obtained an RPR, hepatitis antigens, an HIV test, a urinalysis and culture, and a pregnancy test. He provided the
patient with postcoital contraceptive medication. Which of the following additional actions is most appropriate?
a) Notify police
b) The patient’s parents or her closest relative or friend should be notified
c) The patient should be offered antibiotic prophylaxis for sexually transmitted infections
d) The patient should be counseled regarding the practice of “safe behavior,” including not walking in deserted areas
alone
e) The patient should have testing for herpes simplex virus (HSV)
Incorrect!!! Antibiotic prophylaxis should be offered to all adult rape victims. Although patients are often
reticent to do so, they should be gently encouraged to work with the police. This has been associated with
improved emotional outcomes for victims. Counseling the patient regarding the practice of “safe behavior”
at this time may make the patient feel blame, when the blame should be placed on the rapist. HSV antibody
testing is not indicated.
A representative from a domestic violence outreach program asks for your help in distributing information, to
include a hot line to call, if needed. Where is the best place for this information to be made available?
a) Office waiting room
b) Distribution to all patients at time of check in
c) Office rest room
d) Letters sent to all your patients
e) Posters in front of your office
Incorrect!!! The best place to have literature is where there is the most privacy. In the other areas cited, an
abuser may see the information and prevent his spouse or partner from obtaining it.
An 84 year-old widow is seen in the emergency room because of a pain in her right arm. Her daughter, with whom
she lives, states that her mother is “always falling down” and is getting forgetful. She would put her in a nursing
home, but her mother’s social security check is needed for family expenses. The patient is on Zolpidem 10 mg for
sleep. She has been unable to afford the bisphosphonates needed for bone loss. Examination reveals a poorly
nourished white woman, 5’2” tall, weighing 94 pounds. She has tenderness in her right forearm with swelling. She is
also noted to have old and new bruises on her chest and arms. An x-ray of the right arm reveals a spiral fracture.
Which of the following is the most likely cause of the bone fracture?
a) Osteoporosis
b) Domestic violence
c) Bone metastasis from occult breast cancer
d) Malnutrition
e) Hypocalcemia
Correct!!! Spiral fractures rarely occur unless violence has been practiced. It is due to a twisting of the arm
causing the almost pathognomonic finding. Care for the elderly is difficult, but there is no excuse for
violence against the elderly.
A 79 year-old woman is seen in the emergency room in a comatose state. Her daughter states that her mother is
always wandering away from the home. She has tried to tie her to her bed, but has found that if she gives her
Zolpidem every 4 hours, it keeps her sedated. The daughter works during the day and would put her mother in a
nursing home, but she needs the mother’s social security check to maintain the household. Examination reveals an
unresponsive, thin, dehydrated woman with acute and chronic bedsores. After hydration and treatment of her skin
condition in the hospital, which of the following is the most appropriate next step?
a) Notify social services
b) Notify social security
c) Discharge patient home after giving daughter appropriate instructions on how to care for her elderly mother
d) Advise the daughter to change her work schedule
e) Decrease the Zolpidem dose
Incorrect!!! In a case of obvious elderly abuse, social services must be notified. The patient should not be
taking Zolpidem, and this should also be discontinued. Physicians must be aware of the social services
available for their patients. Although the patient may ultimately be discharged home, this needs to be done
in consultation with social services.
A 24 year-old woman comes to her physician for help with her premenstrual syndrome symptoms. She complains of
“not being herself” for 3 to 4 days before her period and has episodes of crying and irritability. She denies
depressive symptoms and notes she is a stay-at-home mother for her three children. After a complete history and
physical examination, the patient is prescribed a selective serotonin reuptake inhibitor but, after three months, she
returns as there is no change in her symptoms. Upon further discussion, the patient admits that her husband has a
bad temper at times. Physical examination is normal with the exception of some bruising on the patient’s arms. She
claims that she fell and that she is often clumsy. Which of the following is the most appropriate next step in the
management of this patient?
a) Report the injuries to the police
b) Offer domestic violence resources to the patient
c) Refer the patient for a psychiatric consultation
d) Arrange for a home visit by a women’s shelter counselor
e) Arrange for security to escort her to a women’s shelter
Incorrect!!! Victims of domestic violence frequently present with vague physical and emotional complaints
that are consistent with PMS. Victims will rarely volunteer information on first physician encounters, but
subsequent visits give opportunities to offer resources for assistance. It is the patient’s choice as to whether
she would like to involve the police.